Carpe Diem 2 Flashcards

Agosto-Septiembre

1
Q
  1. Masculino de 67 años de edad se encuentra programado para resección de un adenocarcinoma de colon en 1 semana. Refiere haber perdido 7 kg en los últimos 3 meses. Tiene antecedente de cardiopatía isquemia, asma leve e hipertensión arterial por lo que toma aspirina, metoprolol, amlodipino, hidroclorotiazida. Dejó de fumar hace 2 años. No usa el salbutamol de rescate desde hace 3 meses. Mide 178 cm, pesa 56,7 kg y su IMC es de 18 kg/m2. Sus signos vitales son normales. El examen físico evidencia cabello delgado, frágil y pérdida de grasa subcutánea en mejillas. Aparenta bocio tiroideo. Dentro de los paraclínicos destaca hipoproteinemia e hipoalbuminemia. ¿Cuál es el abordaje preoparatorio apropiado?
    A. Realizar pruebas de función pulmonar.
    B. Suspender metoprolol hasta la cirugía.
    C. Soporte nutricional enteral.
    D. Medición de T4 libre y T3 total.
A

Respuesta correcta: C

Tema e ítem: Anestesiología – Valoración preoperatoria.

Argumento: Este paciente tiene desnutrición: niveles bajos de albúmina y proteínas totales, pérdida de peso reciente, perdida de grasa subcutánea y bocio tiroideo (Probablemente por déficit de yodo). Debido a los posibles resultados negativos asociados a la desnutrición (Resultados posoperatorio deficientes, mayor morbimortalidad, estancias hospitalarias prolongadas, mayor riesgo de infección, retraso en la cicatrización de heridas y mayor tasa de ingreso a UCI) a todo paciente se le debe evaluar el estado nutricional previo a una cirugía. Los pacientes desnutridos deben recibir apoyo nutricional por vía enteral o parenteral. Sin embargo, la nutrición enteral preserva la fisiología normal y función inmunitaria del sistema digestivo por lo que se debe escoger este tipo de nutrición siempre que sea posible. Otras pruebas preoparatorias incluyen una radiografía de tórax por ser >60 años, antecedente de enfermedad cardiaca y consumo de cigarrillo, así como un electrocardiograma por ser >65 años y con antecedente de cardiopatía isquémica.

  • Las pruebas de función pulmonar se usa en el preoperatorio de pacientes que se someterán a cirugía torácica o en planificación de resección pulmonar. También en pacientes con enfermedad pulmonar con un flujo de aire deteriorado. Este paciente se someterá a una cirugía intraabdominal en lugar de torácica. Además, la última vez que usó un inhalador de rescate fue hace 3 meses y no se nos hace referencia a hallazgos anormales en el examen pulmonar del paciente por lo que su asma está controlada y no tiene evidencia de alteración significativa en el flujo de aire.
  • Los betabloqueadores pueden tener efectos nocivos si se suspenden abruptamente: exacerbación de cadiopatía isquémica e incluso infarto de miocardio. Los pacientes que toman metoprolol deben continuar tomándolo en el período perioperatorio, ya sea por vía enteral o parenteral. El día del procedimiento, el metoprolol y cualquier otro medicamento necesario se puede tomar de forma segura 2 horas antes de la anestesia.
  • La medición de pruebas tiroideas ayudan a determinar la funcionalidad de la glándula tiroides. Este paciente tiene manifestaciones que caracterizan al hipertiroidismo. Sin embargo, la hipoalbuminemia y la hipoproteinemia sugieren otra causa de la pérdida de peso. Además, este paciente no tiene otros síntomas de hipertiroidismo como taquicardia, intolerancia al calor, diarrea o aumento del apetito.
How well did you know this?
1
Not at all
2
3
4
5
Perfectly
2
Q
  1. Masculino de 4 meses es llevado a consulta por episodios recurrentes de regurgitación. Recibe lactancia materna exclusiva. La madre refiere que en las últimas 3 semanas el paciente vomita minutos después de cada toma, sin embargo, está ansioso por comer constantemente. Niega cambios de color, disnea o irritabilidad con las tomas. No tiene antecedentes de importancia. Sus signos vitales y examen físico son normales. Su peso y talla se encuentran en el percentil 50 para la edad. ¿Cuál es el manejo adecuado para este paciente?
    A. Iniciar manejo con inhibidor de bomba de protones.
    B. Alimentar con menos frecuencia y con mayor volumen.
    C. Iniciar alimentación complementaria temprana.
    D. Tranquilizar y educar a los padres
A

Respuesta correcta: D

Tema e ítem: Reflujo gastroesofágico fisiológico vs patológico en la edad pediátrica.

Argumento: El crecimiento de este paciente es normal lo que indica que sus síntomas se deben a un reflujo gastroesofágico fisiológico. El reflujo gastroesofágico es causado por un esfínter esofágico inferior inmaduro, que permite el flujo retrógrado de contenido gástrico hacia el esófago. Esta condición se suele presentar en los primeros 6 meses de vida con regurgitación o vómitos de leche materna o de formula posterior a la alimentación. En contraste con el reflujo patológico, el reflujo fisiológico no causa molestias al regurgitar y los lactantes continúan con deseo de alimentarse y peso normal (Como este paciente). Prácticamente el diagnostico se basa en la clínica y un examen normal. El manejo incluye educación sobre medidas preventivas (p. ej., posición vertical después de las tomas), así como brindar tranquilidad a los padres de que los síntomas generalmente se resuelven por sí solos entre los 12 y los 18 meses de edad.

  • Los signos de reflujo gastroesofágico patológico incluyen irritabilidad, mala alimentación y/o pérdida de peso. En dicho caso el manejo inicial incluye manejo farmacológico e incluso cambios en la alimentación. Sin embargo, este paciente no presenta características de reflujo patológico por lo que no se deben tomar estas medidas.
  • Se recomienda brindar alimentos con mayor frecuencia y menor volumen (no de menor frecuencia ni mayor volumen) en pacientes con regurgitación para reducir la sobrealimentación y la distensión gástrica que podrían contribuir al reflujo gastroesofágico.
How well did you know this?
1
Not at all
2
3
4
5
Perfectly
3
Q
  1. Femenina de 34 años consulta por amenorrea desde hace 5 meses. Refiere que esta condición es nueva y que anteriormente sus menstruaciones eran regulares. Una prueba de embarazo es negativa. Refiere además cefalea y ha notado que cuando va a cine no ve los bordes exteriores de la pantalla sin girar la cabeza. La paciente le pregunta a que se deben estos síntomas. Usted responde que se debe al crecimiento de la siguiente estructura:
    A. Adenohipófisis.
    B. Glándula pineal.
    C. Células ependimarias.
    D. Células de Schwann.
A

Respuesta correcta: A

Tema e ítem: Alteraciones visuales relacionadas al crecimiento hipofisiario.

Argumento: El crecimiento anormal de la adenohipófisis es la causa subyacente más probable de los síntomas de esta paciente, probablemente por un adenoma hipofisario. Los adenomas de la glándula pituitaria se deben comúnmente a la proliferación aberrante de células productoras de prolactina en la adenohipófisis. La hiperprolactinemia resultante finalmente provoca supresión de las gonadotropinas y subsiguiente amenorrea secundaria, como en esta paciente. La hemianopsia bitemporal es el hallazgo oftalmológico más importante a reconocer y es secundario a la compresión del quiasma óptico.

  • El crecimiento de la glándula pineal conduce al desarrollo de un pinealoma, que suele manifestarse durante la infancia. Los síntomas de los pinealomas incluyen alteraciones visuales, mareos y cefalea (Por hidrocefalia). Sin embargo, se asocian con parálisis vertical de la mirada más hemianopsia bitemporal (Este último hallazgo se observa en este paciente). Además, en el pinealoma no se esperarían trastornos menstruales.
  • El crecimiento de las células ependimales se denominan ependimomas, la mayoría de los cuales surge cerca del cuarto ventrículo. Los ependimomas comúnmente causan hidrocefalia obstructiva, que puede manifestarse como cefalea recurrente. Sin embargo, estos se desarrollan en la niñez y no se asocian a irregularidades menstruales o trastornos visuales.
  • El crecimiento de las células de Schwann se denomina scwannoma que generalmente surge cerca del ángulo pontocerebeloso. Este tipo de lesiones producen síntomas secundarios a hipertensión endocraneal. No se asocian a irregularidades menstruales o hemianopsia bitemporal.
How well did you know this?
1
Not at all
2
3
4
5
Perfectly
4
Q
  1. Paciente masculino consulta por dolor lumbar. Refiere que el dolor se irradia a la cara lateroposterior del muslo y presenta paresia para extender el hallux ipsilateral. Al examen neurológico se evidencia marcha de trendelemburg. ¿Cuál de las siguientes condiciones es la más probable?
    A. Compromiso de la raíz L5.
    B. Compromiso de la raíz S1.
    C. Compromiso de la raíz L1.
    D. Compromiso de la raíz L3
A

Respuesta correcta: A

Tema e ítem: Radiculopatía lumbar.

Comentario: El lumbago es un síndrome clínico que es frecuente, con prevalencia a lo largo de la vida que se ha reportado entre 51% - 84% de la población general. Dada la frecuencia de presentación, esta patología representa una causa frecuente de discapacidad y ya que la mayoría de casos no requieren intervenciones más allá de la analgesia, cuidados posturales, cambios en el estilo de vida y terapia física, buena parte de los casos serán tratados de forma inicial o definitiva por médicos de atención primaria. Este paciente tiene un subtipo de dolor neuropático que se caracteriza por el compromiso de una raíz nerviosa específica por lo que el dolor se irradia a través del territorio inervado por dicha raíz, permitiendo su caracterización clínica desde el examen físico. Este paciente probablemente tiene compromiso de la raíz nerviosa L5 caracterizada por dolor lumbar irradiado a la región posterolateral del muslo, paresia al extender el hallux (Primer dedo del pie) y marcha de trendelemburg por debilidad del musculo glúteo medio (Inervado principalmente por la raíz L5).

  • En la radiculopatia S1 el dolor lumbar se distribuye en la región posterior de la pierna y del pie. Hay cambios sensitivos a nivel de la región posterolateral de la pierna, cara lateral del pie y la planta. Suele haber debilidad para la platiflexión, flexión de la rodilla y extensión de la cadera.
  • El compromiso de la raíz nerviosa L1 se caracteriza por dolor distribuido en la región inguinal con cambios sensitivos a este mismo nivel. Suele asociarse a debilidad en la flexión de la cadera y anomalías en el reflejo cremastérico.
  • El compromiso de la raíz nerviosa L3 se caracteriza por dolor en la región anterior del mulo y rodilla con cambios en la sensibilidad en estas mismas regiones. La extensión de rodilla, flexión y aducción de cadera se ven comprometidas.
How well did you know this?
1
Not at all
2
3
4
5
Perfectly
5
Q
  1. A un primer nivel de atención llega un paciente masculino de 30 años de edad refiriendo secreción uretral clara desde este mañana asociado a disuria. Refiere ser sexualmente activo con múltiples parejas y no usa condones. Sus signos vitales y examen físico son normales. Usted decide realizar una tinción de Gram de un hisopado uretral en el que se evidencian neutrófilos, pero no se logra visualizar ningún microorganismo. ¿Cuál de los siguientes es el patógeno causal más probable en este caso?
    A. Neisseria gonorrhoeae.
    B. Gardnerella vaginalis.
    C. Chlamydia trachomatis.
    D. Virus herpes simplex.
A

Respuesta correcta: C

Tema e ítem: Infecciones de transmisión sexual: uretritis/diagnostico.

Comentario: La disuria y secreción uretral clara en ausencia de signos sistémicos de inflamación como fiebre y escalofríos sugieren el diagnóstico de uretritis. La tinción de Gram es una prueba que puede ser de gran ayuda en el primer nivel para suponer el agente causal de la uretritis en pacientes que se acercan a un primer nivel de atención. En este caso usted debe sospechar que el agente causal es la Chlamydia trachomatis. Esta bacteria es un microorganismo intracelular que es claramente difícil de teñir, lo que explicaría que se vean neutrófilos en la tinción de gram en ausencia de bacterias visibles. La prueba de elección para la detección es la amplificación de ADN, sin embargo, recuerde que está en un primer nivel en donde no se dispone de esta prueba.

  • Neisseria gonorrhea es una causa común de uretritis. Sin embargo, la secreción uretral en la uretritis gonocócica suele ser purulenta y la tinción de Gram habría mostrado diplococos gramnegativos intracelulares (dentro de los neutrófilos) o extracelulares.
  • Gardnerella vaginalis es una causa rara de uretritis, sin embargo, puede causarla. Como se mencionó la tinción de Gram es de mucha ayuda ya que si el agente etiológico fuese este se podrían observar células Clue en el microscopio.
  • El virus del herpes simple puede causar infecciones de las mucosas, incluida la uretritis sin organismos visibles en una tinción de Gram. Sin embargo, esta infección típicamente se manifiesta con lesiones genitales ulcerativas y linfadenopatía local asociada.
How well did you know this?
1
Not at all
2
3
4
5
Perfectly
6
Q
  1. Camilo, es un paciente de cinco meses. Nació a término, sin complicaciones durante su gestación y parto. Es alimentado con lactancia materna exclusiva. Su madre lo lleva a urgencias porque hace 3 días está presentando deposiciones con pintas de sangre, sin cambios en frecuencia o en la consistencia. Al examen físico, buen peso y talla para la edad. ¿Cuál sería el paso a seguir?
    A. Necesito más datos
    B. HLG
    C. Colonoscopia
    D. Rx de abdomen
A

Respuesta correcta: A.

Argumento: Es muy importante que a la hora de responder preguntas con selección múltiple, leamos bien pero la interpretación es igual de importante. No debemos suponer información que no esté escrita, si no lo mencionan muy posiblemente no es un elemento relevante para llegar a la respuesta correcta. Sin embargo, en este caso, estamos hablando de un caso clínico donde no se describe un paso sumamente importante en el examen físico de un paciente con hemorragia digestiva baja: La inspección anal. Una de las etiologías más frecuentes de hematoquezia en lactantes son las fisuras anales, por lo tanto en este paciente necesitamos más datos para poder definir qué exámenes adicionales realizaremos.

La hemorragia digestiva baja es el sangrado intestinal cuyo origen se encuentra desde la válvula ileocecal hasta el ano. Se puede presentar como rectorragia, enterorragia, hematoquezia o sangre oculta. Es mucho más frecuente que la hemorragia del tracto digestivo superior.

Es importante realizar un examen físico detallado y minucioso, iniciando con el estado general y el triángulo de valoración pediátrica. Se deben buscar lesiones vasculares cutáneas o mucosas como angiomas, manchas melánicas, cicatrices, circulación colateral, lesiones traumáticas. Las petequias y púrpura son indicativas de trombocitopenia y coagulopatía. En la púrpura de Henoch Schonlein se muestran lesiones características en miembros inferiores y glúteos. En el abdomen, debemos descartar organomegalias, masas y dolor.

En cuanto a la exploración perianal y ano-rectal, es importante realizar una inspección y realizar tacto rectal para evaluar fisuras, fístulas, pólipos y en niñas es fundamental descartar hemorragia vaginal. Las hemorroides internas o externas son raras en la infancia, por lo general se presentan en casos de hipertensión portal.

En un paciente con deposiciones diarréicas con sangre, lo primero que debemos descartar es una colitis, sea inflamatoria, infecciosa o pseudomembranosa.

How well did you know this?
1
Not at all
2
3
4
5
Perfectly
7
Q
  1. Paciente femenina de 12 meses de edad. Llevada por su madre a la consulta debido a brote en el área del pañal desde hace una semana, acompañado de disminución en la micción e irritabilidad. Al examen físico, con signos vitales estables. Se observan placas eritematosas, brillantes y descamativas localizadas en región púbica, labios mayores, nalgas y cara interna de los muslos, con compromiso de los pliegues y pápulas eritematosas satélite (se adjunta la imagen). ¿Cuál considera que es el manejo más apropiado?
    A. Solicitar HLG y uroanálisis
    B. Brindar educación sobre higiene y uso de crema dermoprotectora
    C. Brindar educación sobre higiene y uso de crema dermoprotectora + clotrimazol
    D. Brindar educación sobre higiene y uso de crema dermoprotectora + mupirocina
A

Respuesta correcta: C

Tema-Ítem: Enfermedades de la piel en pediatría-Dermatitis del área del pañal

Argumento: La descripción de las lesiones de esta paciente es propia de una dermatitis del área del pañal, especialmente porque se limita solo a este sitio (no hay descripción de lesiones en otros sitios corporales). Mencionan además pápulas satélite, que indica sobreinfección por cándida, la cual es más frecuente que la sobreinfección bacteriana.

La dermatitis del área del pañal se define como un proceso inflamatorio de la piel que se encuentra en contacto con el pañal (incluye genitales, nalgas, cara interna de los muslos y cintura). Es un trastorno frecuente en la infancia, con un pico de presentación entre los 9 y 12 meses.

La dermatitis del área del pañal tiene un espectro clínico que va desde máculas con leve eritema, hasta placas, pápulas o nódulos eritematosos que se pueden asociar a descamación, erosión o exudado. Todas estas lesiones están localizadas exclusivamente en áreas de contacto directo con el pañal, de predominio en las superficies convexas de la piel que semejan la forma de una W. Además, pueden presentarse síntomas secundarios a dolor, como irritabilidad, alteración en el patrón de sueño y disminución en la frecuencia de la micción y la deposición. Sin embargo, en caso de presentar síntomas sistémicos como fiebre, se debe sospechar sobreinfección bacteriana, sobre todo, en los casos de dermatitis que llevan más de 3 días sin tratamiento. Dicho brote se caracteriza por eritema, erosión y maceración en los pliegues, sin lesiones satélite.

La candidiasis en el área del pañal también se trata de una sobreinfección más frecuente que la bacteriana y aporta gravedad al cuadro clínico, generalmente a expensas de mayor erosión. Se manifiesta con placas rojas, brillantes y de manera muy característica aparecen pápulas satélites, usualmente con compromiso de los pliegues.

El pilar del tratamiento se basa en reparar la piel dañada y evitar las recurrencias. Es importante recomendar el uso de pañales superabsorbentes en lugar de pañales de tela para reducir la humedad local. También se recomienda aumentar la frecuencia del cambio de pañal y en lo posible permitir periodos libres de pañal. La piel de la zona debe ser cubierta con una capa generosa de ungüento o pasta dermoprotectora preferiblemente que contenga óxido de zinc, aunque en su defecto podría usarse petrolato (vaselina). Además, es importante realizar la limpieza del área genital suavemente. En caso de usar toallas húmedas, deben ser libres de alcohol y fragancias. Las heces secas se pueden remover con el uso de un algodón humedecido con aceite mineral. No se requiere remover completamente el dermoprotector en cada cambio de pañal. Otra opción es realizar lavado del área con agua tibia, si es necesario aplicar jabón de pH neutro o ácido y secar la piel sin friccionar.

Cuando hay sobreinfección por cándida, debe utilizarse antimicótico tópico (clotrimazol o nistatina) en las zonas afectadas, con cada cambio de pañal, por 7 días o hasta que las lesiones mejoren. Si la sobreinfección es bacteriana leve puede usarse mupirocina tópica dos veces al día por 5 a 7 días. Pero los casos graves requieren manejo antibiótico oral.

How well did you know this?
1
Not at all
2
3
4
5
Perfectly
8
Q
  1. Laura, 5 años, con antecedentes de varicela a los 10 meses de edad, sin complicaciones. Sin otros antecedentes patológicos. Hace 4 días inicia con una erupción vesicular eritematosa en cinturón en región lateral derecha del tórax. Refiere prurito. Al examen físico, se evidencian vesículas en cinturón, comprometiendo dermatomas T7-T8. De acuerdo a la patología que sospecha, ¿Cuál de las siguientes afirmaciones es cierta?
    A. Se trata de un herpes zóster complicado por la edad
    B. Su presentación en la infancia se asocia a una alta incidencia de neuralgia postherpética
    C. Se debe iniciar tratamiento con antivirales
    D. Requiere solo manejo sintomático
A

Respuesta correcta: D

Tema-Ítem: Enfermedades infecciosas en la infancia-Herpes Zóster

Argumento: Esta paciente tiene un antecedente de varicela. Recordemos que ña primera infección con el virus varicela zoster causa la varicela. Luego el virus permanece latente en los ganglios sensoriales de las raíces dorsales de la médula espinal o de los pares craneales, y su reactivación desde los ganglios resulta en una distribución en dermatoma de vesículas llamada zóster.

El principal factor de riesgo para desarrollar zoster es haber tenido varicela antes del año de
vida. El herpes zoster en niños causa mínimo dolor y fiebre y más frecuentemente compromete los dermatomas torácicos. Está raramente asociado a neuralgia post herpética y usualmente se resuelve espontáneamente en menos de 8 días. Generalmente no se le inician antivirales a los pacientes inmunocompetentes y cuando deciden iniciarse, se prefiere en aquellos en los que la evolución es ≤72 horas, puesto que se desconoce la utilidad clínica de iniciar la terapia antiviral en pacientes con lesiones de ≥72 horas de evolución. Sin embargo, en pacientes inmunocomprometidos se recomienda iniciar manejo, incluso en un cuadro clínico de ≥72 horas. El aciclovir es el agente de elección.

Ciertos pacientes inmunocompetentes con herpes zoster presentarán manifestaciones oculares, óticas o neurológicas, lo que se define como herpes zóster complicado. Dichos pacientes pueden requerir terapia intravenosa y/o prolongada.

How well did you know this?
1
Not at all
2
3
4
5
Perfectly
9
Q
  1. Lactante de 3 meses de edad, llevado a su consulta porque la madre refiere que en los últimos días ha observado que el paciente se cansa, se pone cianótico y presenta diaforesis profusa durante las tomas de lactancia materna. Niega antecedentes patológicos. Al examen físico, taquipnea con leve tiraje subcostal, adecuada saturación al aire ambiente y un soplo pansistólico grado 3/6 audible en múltiples focos, con máxima intensidad a nivel del tercer-cuarto espacio intercostal izquierdo. Se realizó un ECG con evidencia de patrón de bloqueo incompleto de rama derecha. De acuerdo al caso clínico, ¿Cuál considera que es la patología más probable en este paciente?
    A. Comunicación interauricular
    B. Transposición de grandes vasos
    C. Comunicación interventricular
    D. Atresia pulmonar
A

Respuesta correcta: C

Tema-Ítem: Cardiología pediátrica-Cardiopatías congénitas

Argumento: En este paciente hay varios factores importantes, tales como la fatiga y sudoración durante las tomas, la taquipnea y el soplo cardíaco que es patológico (un soplo inocente es de bajo grado). Además, encontramos un ECG patológico. Todo esto sugiere una cardiopatía congénita con repercusión hemodinámica. En este paciente, la sospecha sería una cardiopatía congénita con cortocircuito izquierda-derecha, siendo las más probables el ductus arterioso persistente (DAP) y la comunicación interventricular (CIV).

La CIV es un orificio en el tabique interventricular que puede hallarse en cualquier parte del mismo, ser único o múltiple y puede presentarse de forma aislada o asociado a otras cardiopatías congénitas. Representa en su forma aislada el 20% del total de cardiopatías congénitas, siendo la segunda más frecuente (tras la válvula aórtica bicúspide).

Se produce un cortocircuito de izquierda a derecha, pues al ser la presión ventricular izquierda superior a la derecha, la sangre pasa del ventrículo izquierdo (VI) al ventrículo derecho (VD), lo que implica exceso de sangre en la arteria pulmonar (el denominado hiperflujo pulmonar) y aumento del retorno venoso, que tiene que ser manejado por la aurícula y el ventrículo izquierdos. Esta sobrecarga de volumen producirá crecimiento de las cavidades izquierdas y finalmente insuficiencia cardiaca congestiva. Al estar la presión pulmonar aumentada de forma fisiológica en los recién nacidos, inicialmente puede no haber (o ser mínimo) el cortocircuito entre ambos ventrículos, con lo que la CIV puede pasar inadvertida (sin producir clínica ni soplo audible) durante los primeros días de vida, hasta que disminuyan las presiones pulmonares. Puede darse el caso de que un niño con una CIV amplia presente una evolución aparentemente benigna por persistencia de las presiones pulmonares elevadas, que enmascare la anomalía subyacente con el riesgo de desarrollar una enfermedad vascular pulmonar obstructiva que puede ser muy grave.

Las manifestaciones clínicas consisten inicialmente en taquipnea y trabajo respiratorio, en relación con el hiperflujo pulmonar. Posteriormente puede presentarse sudoración excesiva, fatiga con las tomas y estancamiento ponderal en relación con la insuficiencia cardiaca congestiva.

How well did you know this?
1
Not at all
2
3
4
5
Perfectly
10
Q
  1. Paciente de 14 años, llevado por sus padres a su consulta para un chequeo. Le cuentan que el paciente desde la infancia ha tenido infecciones respiratorias recurrentes. Hace un mes recibió su último tratamiento antibiótico por el séptimo episodio de sinusitis en lo que va del año. Sus heces siempre son voluminosas, malolientes y “aceitosas”. Además, siempre ha sido muy delgado y no aumenta de peso, a pesar de que come muy bien. Al examen físico, se encuentra en el percentil 10 de altura y en el percentil 5 de peso. En la rinoscopia anterior, se observan múltiples pólipos nasales. ¿Qué otra alteración es muy probable encontrar en este paciente?
    A. Vaso deferente ausente
    B. Hipersensibilidad a la aspirina
    C. Deficiencia selectiva de IgA
    D. Situs inversus
A

Respuesta correcta: A.

Tema-Ítem: Enfermedades del aparato respiratorio-Fibrosis quística

Argumento: Este paciente tiene infecciones recurrentes de las vías respiratorias superiores y pólipos nasales, además de signos de insuficiencia pancreática (esteatorrea y malabsorción), con el consiguiente retraso en el crecimiento, todo lo cual sugiere fibrosis quística (FQ). Más del 98 % de los hombres con FQ tienen una ausencia bilateral congénita de los conductos deferentes, lo que provoca azoospermia e infertilidad.

La fibrosis quística (FQ) es un trastorno autosómico recesivo. Está causada por mutaciones en el gen CFTR, que codifica la proteína reguladora de la conductancia transmembrana de la FQ (CFTR). Estas mutaciones dan como resultado canales de cloruro defectuosos.

Las manifestaciones iniciales pueden incluir íleo meconial, retraso en el crecimiento y síntomas de malabsorción, y más adelante en la vida, los pacientes pueden experimentar síntomas como infecciones respiratorias recurrentes, pancreatitis recurrente e infertilidad. A medida que avanza la enfermedad, los canales de cloruro defectuosos en el tracto respiratorio dan como resultado una mucosidad bronquial espesa y una depuración mucociliar alterada, lo que conduce a infecciones respiratorias crónicas, colonización pulmonar con bacterias multirresistentes y una disminución progresiva de la función pulmonar. La disfunción de CFTR en el páncreas exocrino y el tracto biliar puede conducir en última instancia a enfermedad hepática y diabetes relacionadas con la FQ.
Si se sospecha FQ, el primer estudio de diagnóstico es una iontoforesis (prueba del sudor). Una concentración de cloruro en el sudor ≥ 60 mmol/L confirma el diagnóstico. Si el valor está entre 30 y 60 mmol/L, es necesario realizar un análisis del gen CFTR para confirmar el diagnóstico.
El manejo debe enfocarse en retrasar la progresión de la enfermedad pulmonar , erradicar y/o suprimir las infecciones crónicas, prevenir y/o tratar las exacerbaciones y abordar las complicaciones y comorbilidades.

La hipersensibilidad a la aspirina es un rasgo característico de la enfermedad respiratoria exacerbada por aspirina, que puede provocar poliposis nasal debido a la rinitis crónica. Se caracteriza por la tríada de Samter (asma, sinusitis, pólipos nasales). Esta enfermedad no provoca esteatorrea ni retraso en el crecimiento.

El situs inversus junto con infecciones sinusales y pulmonares recurrentes es característico del síndrome de Kartagener. La mitad de los pacientes con este síndrome tendrán situs inversus. También puede haber poliposis nasal y retraso en el crecimiento.

La deficiencia selectiva de IgA puede provocar infecciones sinusales y pulmonares recurrentes, diarrea (debido a infecciones gastrointestinales como la giardiasis) y retraso en el crecimiento. Sin embargo, las heces aceitosas y malolientes son más indicativas de esteatorrea que de diarrea por una infección. La esteatorrea no suele estar asociada con la deficiencia selectiva de IgA.

How well did you know this?
1
Not at all
2
3
4
5
Perfectly
11
Q
  1. Femenina de 9 años presenta congestión nasal persistente. Sus síntomas iniciaron hace 2 semanas con fiebre, rinorrea y congestión. Hace 1 semana desarrollo tos seca de predominio nocturno. El cuadro la ha obligado a faltar a sus clases de natación. Automedicada con acetaminofén y medicamentos para el resfriado de venta libre. Su padre es fumador, pero lo hace afuera de casa. Signos vitales: temp. de 37°C. Al examen físico eritema y edema de cornetes nasales + drenaje nasal purulento bilateral. El resto del examen físico es normal. Usted considera que el factor de riesgo más común para el desarrollo de la condición actual de esta paciente es:
    A. Infección viral de las vías respiratorias altas.
    B. Exposición crónica al cigarrillo.
    C. Deformidades del tabique nasal.
    D. Hipertrofia adenoidea.
A

Respuesta correcta: A

Tema e ítem: Rinosinusitis bacteriana aguda.

Argumento: Este paciente con cuadro clínico de congestión nasal prolongada (Mayor a 10 días) presenta una rinosinusitis bacteriana aguda, una infección de senos paranasales que se debe más comúnmente a H. influenzae, S. pneumoniae o M. catarrhalis. Los síntomas de presentación a menudo incluyen tos, secreción/congestión nasal, dolor facial y cefalea. La rinosinusitis bacteriana aguda se define por la presencia de cualquiera de los siguientes 3 criterios:

  • Síntomas que persisten por más de 10 días sin mejoría.
  • Síntomas de inicio severo (Fiebre ≥39°C + drenaje) durante más de 3 días.
  • Empeoramiento de los síntomas después de la mejoría inicial.

El factor de riesgo más común para la rinosinusitis bacteriana es la infección viral de las vías respiratorias superiores. Las secreciones espesas y la inflamación de la mucosa por la infección viral impiden el drenaje de los senos paranasales y alteran la eliminación mucociliar de bacterias contaminantes, lo que puede conducir a una infección bacteriana secundaria. El diagnóstico suele ser clínico y el tratamiento de primera línea es la amoxicilina con o sin clavulanato.

  • La hipertrofia adenoidea y la deformidad del tabique nasal pueden causar signos de obstrucción nasal, pero contribuyen de forma menor a la enfermedad de los senos paranasales.
  • Los irritantes ambientales, como el humo del cigarrillo y los productos químicos para piscinas, son factores de riesgo comunes para el asma, que se presenta con sibilancias. Los irritantes ambientales pueden inducir la inflamación de la mucosa y provocar sinusitis, pero su participación es menos frecuente en la rinosinusitis bacteriana.
How well did you know this?
1
Not at all
2
3
4
5
Perfectly
12
Q
  1. Femenina de 6 meses es llevada a urgencias por tos, congestión nasal y aumento del trabajo respiratorio en los últimos 2 días. Se encuentra irritable y con poco apetito. Sus familiares cercanos han tenido congestión nasal y rinorrea sin fiebre, al igual que sus compañeros de escuela. Signos vitales: temp. de 37°C y FR de 65 rpm. Al examen físico se encuentra irritable y taquipeneica con aleteo nasal y retracciones subcostales. A la auscultación pulmonar la fase espiratoria es prolongada con crepitantes difusos y sibilancias al final de la espiración. ¿Cuál de los siguientes es el manejo adecuado para esta paciente?
    A. Solo manejo de apoyo.
    B. Iniciar epinefrina racémica.
    C. Iniciar glucocorticoides.
    D. Iniciar manejo antibiótico.
A

Respuesta correcta: A

Tema e ítem: Bronquiolitis – Manejo

Argumento: Este paciente presenta tos, congestión nasal y aumento del trabajo respiratorio secundario a una bronquiolitis. La bronquiolitis es una infección de las vías respiratorias bajas que ocurre en niños menores de 2 años. El virus sincitial respiratorio es la causa más común. Otros patógenos virales comunes que causan bronquiolitis incluyen el rinovirus, el virus de la influenza y el virus de la parainfluenza. Los síntomas iniciales incluyen tos, congestión y rinorrea. La fiebre puede o no estar presente. Después de 2-3 días, la inflamación afecta a los bronquiolos, provocando sibilancias difusas y/o crepitantes. Los pacientes pueden desarrollar hipoxemia y signos de dificultad respiratoria, que normalmente alcanzan su punto máximo entre los días 3 y 5 de la enfermedad antes de desaparecer gradualmente. La bronquiolitis es un diagnóstico clínico. El tratamiento es de apoyo e incluye lavados nasales y succión de secreciones. La hospitalización está indicada para pacientes que requieren líquidos intravenosos u oxígeno suplementario.

  • Los antibióticos se usan para tratar la neumonía bacteriana, que se presenta con fiebre, tos y crepitantes focales. Este paciente está afebril con sibilancias difusas y crepitantes, lo que hace más probable la bronquiolitis viral.
  • Los glucocorticoides y la epinefrina racémica se usan en el tratamiento del crup, una infección de las vías respiratorias superiores. La presentación incluye fiebre, tos perruna y estridor inspiratorio, ninguno de los cuales se observa en este paciente. Los glucocorticoides también están indicados para las exacerbaciones del asma, que se presentan con sibilancias y dificultad respiratoria. Sin embargo, no son beneficiosos en niños menores de 2 años con un primer episodio de sibilancias inducidas por virus, como este paciente.
How well did you know this?
1
Not at all
2
3
4
5
Perfectly
13
Q
  1. Masculino de 5 años de edad es llevado a consulta por presentar dolor en cadera izquierda. Se levanto esta semana cojeando, con dolor sobre todo al moverse o caminar. Ayer tuvo un partido de futbol, pero no presento algún tipo de lesión. Tuvo una infección respiratoria alta hace 2 semanas actualmente resuelta. Signos vitales: temp. de 37°c, PA de 100/65 mmHg y FC de 92 lpm. Al examen físico el paciente es interactivo y activo, pero camina cojeando. La cadera izquierda izq. Esta ligeramente abducida y rotada externamente con disminución del rango de movimiento. Es capaz de pararse y sostener peso con la pierna izquierda. Rx de cadera normal. Paraclínicos: Hb de 13.5 g/dl, PLT de 287.000/mm3, LEU de 8.500/mm3. VSG de 15 mm/H. PCR 2 mg/L (Normal). ¿Cuál es el manejo adecuado?
    A. Realizar resonancia magnética de cadera izquierda.
    B. Iniciar manejo con ibuprofeno y seguimiento estrecho.
    C. Realizar aspiración de líquido sinovial de cadera izquierda.
    D. Iniciar manejo antibiótico.
A

Respuesta correcta: B

Tema e ítem: Tenosinovitis transitoria – Manejo.

Argumento: La sinovitis transitoria es una causa común de dolor de cadera en los niños, y generalmente ocurre en niños de 3 a 8 años. Se desconoce la etiología, pero los síntomas generalmente se desarrollan días o semanas después de una enfermedad viral leve, como en este paciente. La inflamación sinovial provoca dolor, disminución del rango de movimiento y cojera. En el examen físico, los pacientes tienen buen aspecto y están afebriles. La cadera afectada puede estar flexionada, ligeramente abducida y rotada externamente. Esta posición maximiza el espacio articular, lo que proporciona alivio del dolor. Las pruebas de laboratorio evidencian un recuento normal de leucocitos y marcadores inflamatorios normales debido a la ausencia de infección o inflamación significativa. Las radiografías a menudo revelan articulaciones normales de la cadera; la ecografía puede mostrar pequeños derrames unilaterales o bilaterales. El tratamiento consiste en reposo y medicamentos antiinflamatorios no esteroideos. Los niños generalmente se recuperan dentro de 1 a 4 semanas sin complicaciones.

  • Los hemocultivos, la aspiración articular y los antibióticos intravenosos son el tratamiento de la artritis séptica, que se debe sospechar en pacientes con dolor de cadera que se encuentran febriles, con mal aspecto o que tienen pruebas de laboratorio que indican infección o inflamación.
  • La resonancia magnética está indicada ante la sospecha de enfermedad de Legg-Calvé-Perthesporque las radiografías iniciales pueden parecer normales. Sin embargo, los pacientes suelen tener una marcha de Trendelenburg y una rotación interna y una abducción limitada, en contraste con este paciente que prefiere la abducción de la cadera.
How well did you know this?
1
Not at all
2
3
4
5
Perfectly
14
Q
  1. Femenina de 5 años es llevada a consulta por dolor articular y erupción cutánea. La semana pasada tuvo dolor de rodillas el cual resolvió, pero ahora se encuentra en tobillos y muñecas. Presenta además erupción rosada no pruriginosa en espalda. Tuvo faringitis hace unas semanas, pero resolvió por si solo. Temp. de 38.3°C, FC de 85 lpm y FR de 20 rpm. Al examen físico muñecas y tobillos rígidos y sensibles a la manipulación. Múltiples parches grandes, bien delimitados, eritematosos, no pruriginosos, con un contorno ligeramente elevado, presentes en tronco y extremidades proximales. Paraclínicos: Leu: 6.500/mm3, Hb de 12.4 g/dL, PLT de 380.000/mm3, PCR de 32 mg/dL, VSG de 62 mm/h. ¿Cuál es el diagnostico más probable?
    A. Artritis idiopática juvenil sistémica.
    B. Púrpura de Henoch-Schonlein.
    C. Fiebre reumática aguda.
    D. Enfermedad de Lyme.
A

Respuesta correcta: C

Tema e ítem: Fiebre reumática – Sospecha diagnostica.

Argumento: La fiebre reumática aguda es una complicación que generalmente ocurre de 2 a 4 semanas después de un episodio de faringitis estreptocócica del grupo A no tratada. El diagnóstico se basa en la evidencia clínica o de laboratorio de una infección previa por estreptococos del grupo A junto con 2 criterios mayores de Jones o 1 mayor más 2 menores. El dolor articular suele ser el hallazgo más temprano; otros criterios importantes incluyen carditis, nódulos subcutáneos, eritema marginado (erupción anular rosada con bordes afilados y elevados y limpieza central) y corea de Sydenham.

La fiebre, el sarpullido, la poliartritis migratoria (particularmente en las muñecas, los tobillos y las rodillas) y los reactivos de fase aguda elevados (proteína C reactiva y velocidad de sedimentación globula) de este paciente cumplen los criterios diagnósticos de artritis reumatoide. Los hallazgos de laboratorio de apoyo incluirían una prueba de antígeno estreptocócico positiva o un título elevado de antiestreptolisina O. Esta condición se puede prevenir con el tratamiento de la faringitis estreptocócica.

How well did you know this?
1
Not at all
2
3
4
5
Perfectly
15
Q
  1. Femenina de 3 años presenta letargo y fiebre. Tuvo diarrea desde hace varios días y sus padres dicen que “de repente empeoró”. Rechaza líquidos desde hace 12 horas y no ha orinado en 24 horas. No alergias y no toma medicamentos. Signos vitales: Temp. de 39°C, PA de 60/28 mmHg y FC de 145 lpm. Se encuentra letárgica y con poca turgencia en piel. Llenado capilar de 5 segundos. No se ha podido colocar una vía intravenosa periférica a pesar de varios intentos y la paciente continúa deteriorándose. ¿Cuál es el manejo adecuado?
    A. Traslado a UCI para colocación de catéter venoso central.
    B. Intentar colocar una vía intraósea.
    C. Colocar un catéter venoso central en la urgencia.
    D. Colocar una sonda nasogástrica.
A

Respuesta correcta: B

Tema e ítem: Manejo del shock en pediatría.

Argumento: El cuadro clínico de este paciente es preocupante para un shock hipovolémico o séptico y requiere reanimación con líquidos de emergencia. Cuando no se puede obtener un acceso intravenoso en casos de emergencia, debe intentarse inmediatamente un acceso intraóseo (IO). El acceso IO requiere menos habilidad y práctica que la colocación de una vía central, y los ensayos clínicos han demostrado que las vías IO son más seguras y rápidas que las vías centrales. Los catéteres IO proporcionan una cánula lo suficientemente grande para administrar fluidos y medicamentos rápidamente y para obtener muestras de sangre para análisis de laboratorio.

El sitio más común para el acceso IO es la tibia proximal debido a su superficie ancha y plana y a la distancia del esternón en caso de que la reanimación cardiopulmonar se realice simultáneamente. Sin embargo, se puede utilizar cualquier hueso grande. Los catéteres IO se pueden colocar manualmente o con un controlador. Las contraindicaciones para la colocación de IO incluyen infección que recubre el sitio de acceso, fractura o intentos previos de IO en la extremidad elegida, o fragilidad ósea.

How well did you know this?
1
Not at all
2
3
4
5
Perfectly
16
Q
  1. Hombre de 28 años es llevado a urgencias tras ser encontrado confuso y desorientado. Examen físico: PA 110/64 mmHg, FC 48/min, FR 22/min y SatO2 92% con aire ambiente. Está letárgico y diaforético. Ambas pupilas están mióticas y tiene ptialismo. La auscultación pulmonar revela sibilancias difusas y roncus dispersos. ¿Cuál es la conducta inicial más adecuada?
    A. Administrar Atropina 3 mg en bolo, considerar duplicar dosis en 5 min.
    B. Lograr atropinización con 0.02 mg/kg cada 5 min en bolo.
    C. Administrar Atropina 10 mg/h en infusión. Duplicar dosis o reducir 10% según evolución.
    D. Administrar Pralidoxima 1 g en bolo en 15 min, luego infusión de 250 mg/h por 24 h.
A

Respuesta correcta: A

Tema e item: Intoxicación por organofosforados (Bradicardia)-Tratamiento

Argumento: Este paciente con bradicardia, miosis y secreciones excesivas (diaforesis, roncus, ptialismo) tiene indicios de toxicidad colinérgica, probablemente debida a la exposición accidental o intencionada a un pesticida organofosforado. Los organofosforados inhiben la acetilcolinesterasa, lo que conduce a la hiperestimulación de los receptores colinérgicos muscarínicos (mnemotécnia: dumbbells) y nicotínicos (es decir, fasciculaciones musculares, debilidad, parálisis). El tratamiento de la toxicidad por organofosforados incluye la estabilización (vías respiratorias, respiración, circulación), la descontaminación para evitar la exposición continuada (es decir, retirada de la ropa contaminada, irrigación de la piel) y la reversión de la hiperestimulación colinérgica. La atropina, un inhibidor competitivo de la acetilcolina en el receptor muscarínico, debe administrarse inmediatamente y conduce a la resolución de los síntomas muscarínicos (broncoespasmo).

Dado que la atropina no actúa sobre los receptores nicotínicos, los pacientes con evidencia de disfunción neuromuscular deben recibir entonces pralidoxima, un agente reactivador de la colinesterasa que puede aliviar tanto los efectos muscarínicos como los nicotínicos; el uso de este último aún es controversial por cuestiones de eficacia, se ha sugerido en intoxicaciones por organofosforados tipo dietil, pero se sabe que no es útil para el manejo de intoxicaciones por carbamatos. La atropina se administra en bolo directo, sin diluir, de 1 a 3 mg; a los 5 min se debe administrar una nueva dosis (el doble de la primera). Esto se realizará hasta alcanzar las metas terapéuticas (FC >80/min, PAS >80 mmHg, ausencia de broncorrea). Ocasionalmente se puede requerir una infusión, la cual se calcula administrando el 10% de la dosis total aplicada en bolos por hora.

  • La dosis de 0.02 mg/kg es la indicada en niños, ningún adulto debería recibir menos de 0.5 mg.
  • La infusión se administra siempre posterior a las dosis de carga, inicialmente la atropina no se administra en forma lenta y diluida.
How well did you know this?
1
Not at all
2
3
4
5
Perfectly
17
Q
  1. Hombre, 32 años, presenta crisis asmática. T 37°C, FC 120/min, FR 28/min, SatO2 92% con 3 l/min. Tiene tiraje intercostal, no tolera el supino, ruidos respiratorios disminuidos. Rx: signos de hiperinsuflación, GB 11.000/mm3. Tiene nebulización de albuterol, ipratropio inhalado y metilprednisolona. Gases: pH 7,32, PaO2 65 mmHg, PaCO2 50 mmHg. ¿Cuál es la mejor conducta?
    A. Continuación del tratamiento actual.
    B. Realizar intubación endotraqueal.
    C. Iniciar antibióticos intravenosos.
    D. Administrar teofilina intravenosa.
A

Respuesta correcta: B

Tema e ítem: Asma-Manejo

Argumento: La presentación de este paciente es coherente con una exacerbación grave de asma con signos de insuficiencia respiratoria inminente. La exacerbación aguda del asma provoca un aumento del impulso respiratorio e hiperventilación, lo que lleva a una disminución de la presión parcial de dióxido de carbono (PaCO2). Una PaCO2 elevada o incluso normal, como se observa en este paciente, sugiere una incapacidad para satisfacer las mayores demandas respiratorias (probablemente debido a la fatiga) y una insuficiencia respiratoria inminente. Otros signos o síntomas clínicos de insuficiencia respiratoria inminente son:

  • Disminución marcada de los ruidos respiratorios.
  • Ausencia de sibilancias
  • Disminución del estado mental
  • Hipoxia marcada con cianosis

Los pacientes con parada respiratoria inminente o real requieren ingreso en la unidad de cuidados intensivos con intubación endotraqueal para mantener una oxigenación y ventilación adecuadas. La exacerbación asmática de leve a moderada suele responder al oxígeno y a los agonistas β2 de acción corta inhalados (SABA) para revertir eficazmente la obstrucción del flujo aéreo. Los pacientes que no responden requieren corticosteroides sistémicos. La exacerbación grave del asma requiere SABA, nebulizador de ipratropio y corticosteroides sistémicos. La adición de ipratropio a los SABA puede causar una mayor broncodilatación que cualquiera de los dos agentes por separado y se recomienda durante la atención en urgencias de las exacerbaciones graves. Los glucocorticoides sistémicos (orales o intravenosos) aumentan la tasa de mejoría sintomática, pero sus efectos beneficiosos no son clínicamente evidentes hasta varias horas después de su administración. También se recomienda una broncodilatación adicional con una infusión única de sulfato de magnesio intravenoso para las exacerbaciones asmáticas graves que no muestran mejoría tras una hora de tratamiento.

How well did you know this?
1
Not at all
2
3
4
5
Perfectly
18
Q
  1. Un hombre de 34 años consulta por dolor torácico durante 3 meses, es episódico, tipo opresivo retroesternal y se irradia al cuello. Suelen ocurrir en reposo y duran hasta 2 h. Sin disnea, palpitaciones, síncope ni edemas. Examen físico: PA 123/72 mmHg, FC 76/min, IMC 32 kg/m2. ECG y prueba de esfuerzo normales. ¿Cuál es la causa más probable del dolor torácico del paciente?
    A. Enfermedad coronaria
    B. Enfermedad esofágica
    C. Enfermedad musculoesquelética
    D. Enfermedad pleural
A

Respuesta correcta: B

Tema e ítem: ERGE-Diagnóstico

Argumento: Este paciente presenta episodios recurrentes de dolor torácico prolongado durante varios meses que no se asocian a actividad ni a otros síntomas acompañantes. La exploración física, el ECG y la prueba de esfuerzo son normales. Estos hallazgos sugieren un trastorno esofágico subyacente.

La enfermedad por reflujo gastroesofágico (ERGE) y los trastornos de la motilidad esofágica son causas frecuentes de dolor torácico no cardiaco. Las anomalías de la motilidad esofágica suelen manifestarse como pirosis, disfagia, regurgitación alimentaria y/o dolor torácico.

Las características que sugieren un origen esofágico del dolor torácico incluyen dolor prolongado de más de una hora de duración, síntomas posprandiales, pirosis o disfagia asociadas y alivio del dolor mediante tratamiento antirreflujo. El diagnóstico definitivo del dolor torácico esofágico suele ser difícil, y muchos pacientes requieren primero pruebas cardiacas para excluir la isquemia miocárdica. El dolor pleurítico, más que episódico, estaría provocado principalmente por las inspiraciones respiratorias. El dolor musculoesquelético sería también mucho más constante y no tendría un predominio durante el reposo.

How well did you know this?
1
Not at all
2
3
4
5
Perfectly
19
Q
  1. Hombre de 25 años consulta por úlcera indolora en el pene. Está afebril, sin secreción uretral. Hace unas semanas mantuvo relaciones sexuales sin protección. Estuvo en UCI por necrólisis epidérmica tóxica tras tomar penicilina. Tiene una úlcera poco profunda, no sensible y no exudativa, de base lisa, con una linfadenopatía inguinal bilateral leve. RPR (+), VIH (-). ¿Cuál es el manejo adecuado?
    A. Administrar Ciprofloxacina 30 mg/kg/día por 14 días.
    B. Administrar Clindamicina 600 mg c/6h por 14 días.
    C. Administrar Doxiciclina 100 mg c/12h por 14 días.
    D. Administra Trimetoprim-sulfametoxazol, 80 mg/ 400 mg c/24h por 14 días.
A

Respuesta correcta: C

Tema e ítem: Sífilis-Tratamiento

Argumento: Este paciente mantuvo relaciones sexuales sin protección y posteriormente desarrolló un chancro peneano (úlcera indolora, no exudativa), linfadenopatía inguinal y una prueba de reagina plasmática rápida (RPR) positiva, lo que indica sífilis primaria. El tratamiento de primera línea para la sífilis primaria es una dosis única de penicilina G benzatínica intramuscular, que proporciona niveles terapéuticos de penicilina durante aproximadamente 3 semanas. Sin embargo, los pacientes que no pueden tomar penicilina debido a una alergia grave a la penicilina pueden ser tratados con un régimen alternativo como 14 días de doxiciclina oral o 10-14 días de ceftriaxona intravenosa o intramuscular.

Por lo general, se recomiendan regímenes alternativos para los pacientes con alergia grave a la penicilina que presentan cualquier estadio de sífilis que no sea neurosífilis; los pacientes con neurosífilis que presentan alergia grave a la penicilina suelen requerir una desensibilización a la penicilina seguida de la administración de penicilina para garantizar la erradicación del organismo (la penicilina tiene una eficacia del 100% contra el organismo y una excelente penetración en el sistema nervioso central).

Todos los pacientes con sífilis requieren títulos no treponémicos (por ejemplo, RPR) en el momento del tratamiento y de nuevo a los 6-12 meses para asegurar una respuesta adecuada (una disminución de 4 veces en los títulos). Esto es especialmente importante para los pacientes que reciben regímenes alternativos porque el riesgo de fracaso del tratamiento es mayor que en los que reciben penicilina.

How well did you know this?
1
Not at all
2
3
4
5
Perfectly
20
Q
  1. Hombre de 63 años consulta por fatiga. Tiene HTA tratada con lisinopril. Ha tenido dolor en la rodilla derecha debido a artrosis y ha tomado frecuentemente naproxeno para el dolor. Paraclínicos: Hb 8,6 mg/dL, VCM 72 fl, GB 7.200/mm3, Plaq 165.000/mm3, Cr 1,2 mg/dL, VSG 15 mm/h. ¿Cuál de los siguientes cambios esperaría encontrar?
    A. Hierro sérico bajo, Ferritina sérica alta, Capacidad total de fijación del hierro baja, Saturación de transferrina baja.
    B. Hierro sérico bajo, Ferritina sérica baja, Capacidad total de fijación del hierro alta, Saturación de transferrina baja.
    C. Hierro sérico bajo, Ferritina sérica baja, Capacidad total de fijación del hierro baja, Saturación de transferrina alta.
    D. Hierro sérico alto, Ferritina sérica alta, Capacidad total de fijación del hierro baja, Saturación de transferrina alta.
A

Respuesta correcta: B

Tema e ítem: Anemia-Pruebas diagnósticas

Argumento: Este paciente tiene una anemia microcítica significativa, que es la causa probable de su fatiga. Las causas comunes de la anemia microcítica incluyen deficiencia de hierro, anemia de enfermedad crónica (ACD) y talasemia. Su consumo de naproxeno le pone en riesgo de úlceras pépticas/gastritis que provocan una pérdida crónica de sangre del tracto gastrointestinal y la consiguiente depleción de hierro.

La anemia no es una enfermedad sino un signo y como tal, debemos dirigir nuestros esfuerzos a encontrar su etiología. Según datos de la Organización Mundial de la Salud (OMS) este problema lo presenta el 24,8 % de la población mundial. Una vez se trate la causa, la anemia mejorará, por lo cual es indispensable una aproximación clínica y de laboratorio que le permita al clínico, en el menor tiempo posible y de manera costo efectiva, llegar al diagnóstico.

Los estudios de hierro pueden utilizarse para confirmar el diagnóstico de anemia ferropénica. Además del hierro sérico bajo, la ferritina sérica también suele ser baja porque refleja las reservas totales de hierro del organismo. El nivel de transferrina, medido por la capacidad total de fijación del hierro (TIBC), está aumentado en la ferropenia, ya que suele existir una relación inversa con el hierro sérico. La saturación de transferrina (hierro sérico/capacidad total de fijación del hierro) es baja dado el bajo nivel de hierro sérico y la elevada capacidad total de fijación del hierro.

How well did you know this?
1
Not at all
2
3
4
5
Perfectly
21
Q
  1. Femenina de 16 años, consulta por sangrado vaginal abundante desde hace 3 días. Sus menstruaciones son cada 4 a 5 meses, desde su menarquia a los 14 años; su última menstruación fue hace 4 meses. Es sexualmente activa y utiliza preservativos. Signos vitales dentro de rangos normales. En el examen pélvico, cantidad moderada de sangre en la cúpula vaginal y sangrado activo del orificio cervical. El examen bimanual muestra un útero pequeño, sin masas. La ecografía revela un útero pequeño con franja endometrial gruesa y ovarios normales. La prueba de embarazo en orina es negativa. Paraclínicos con: Hemoglobina 10 g/dL, VCM 76 fl, Plaquetas 320.000/mm3, TP 12 seg y TTP 26 seg. ¿Cuál es el manejo más adecuado de la paciente?
    A. Realizar dilatación y legrado de emergencia.
    B. Iniciar agonistas de la hormona liberadora de gonadotropina en dosis altas.
    C. Iniciar terapia anticonceptiva oral combinado a dosis altas.
    D. Realizar una evaluación de la deficiencia del factor de coagulación
A

Respuesta correcta: C

Tema e ítem: Sangrado Uterino Anormal-Tratamiento

Argumento: Esta paciente presenta un sangrado uterino anormal agudo (SUA) que probablemente se deba a una disfunción ovulatoria y requiere intervención clínica para prevenir más sangrado. En adolescentes, el SUA generalmente es causado por un eje hipotálamo-pituitario-ovárico inmaduro, lo que resulta en anovulación y sangrado abundante e irregular. Sin ovulación, existe una proliferación endometrial persistente que resulta en menstruaciones abundantes cuando ocurre la ovulación. La menstruación abundante es causada por una necrosis exagerada de la capa superficial del endometrio, que eventualmente se desnuda y continúa sangrando debido a la falta de estabilidad del tejido. La evaluación inicial de las pacientes que presentan sangrado vaginal agudo incluye un hemograma completo, una prueba de embarazo y estudios de coagulación. En pacientes hemodinámicamente estables, el sangrado uterino agudo puede tratarse médicamente. El tratamiento de primera línea incluye estrógeno intravenoso (estrógeno equino conjugado) o píldoras anticonceptivas orales de estrógeno/progestágeno en dosis altas. El estrógeno en dosis altas estabiliza rápidamente el endometrio denudado y detiene el sangrado menstrual.

  • En cuanto a realizar un legrado, no tiene indicación en esta paciente, debido a que está hemodinámicamente estable y, por ende, se puede manejar médicamente. Por otra parte, la indicación de administrar progestágenos en dosis altas, es para pacientes con contraindicaciones para los estrógenos (antecedentes de tromboembolismo). Además, recordemos que los progestágenos también pueden estabilizar el revestimiento del endometrio y prevenir una mayor proliferación, pero no son tan efectivos como el estrógeno.
  • Finalmente, no hay indicación de estudios para factores de coagulación, debido a que tanto el TP como el TTP se encuentran dentro de rangos normales.
How well did you know this?
1
Not at all
2
3
4
5
Perfectly
22
Q
  1. Femenina de 34 años, consulta al programa de anticoncepción. Durante los últimos 8 meses, la paciente ha experimentado un sangrado impredecible desde que le colocaron un implante de progestágeno subdérmico y solicita su extracción. Refiere que usó anticonceptivos orales combinados previamente, y le gustaría volver a ese método. Antecedente de G1P1, diagnóstico de hipertensión arterial primaria hace un año en adecuado manejo con hidroclorotiazida, sin otro antecedente médico. Signos vitales PA de 130/75 mm/Hg. El examen físico es normal. ¿En esta paciente, el uso de anticonceptivos orales combinados, aumentan el riesgo de cuál de las siguientes entidades?
    A. Fibroadenoma de mama.
    B. Cáncer de endometrio.
    C. Cáncer de ovario.
    D. Empeoramiento de la hipertensión arterial.
A

Respuesta correcta: D

Tema e ítem: Empeoramiento de la hipertensión por anticonceptivos orales combinados

Argumento: Los anticonceptivos orales combinados (AOC) que combinan estrógeno/progestina tienen riesgos y beneficios que se deben discutir con las pacientes antes de comenzar la terapia. Aunque la mayoría de las mujeres no tienen cambios en la presión arterial, un riesgo del uso de AOC es la hipertensión debido al aumento de la síntesis de angiotensinógeno por parte de los estrógenos durante el metabolismo hepático de primer paso; sin embargo, las mujeres que tienen hipertensión controlada pueden tomar AOC combinados con un control estricto de la presión arterial, para garantizar un control continuo. Por otra parte, las mujeres con hipertensión no controlada, daño de órganos diana o mayores de 35 años que consumen tabaco NO deben recibir AOC, debido a que la combinación de estos factores da como resultado un mayor riesgo de infarto de miocardio y accidente cerebrovascular. Por otro lado, recordemos que un efecto secundario adicional de los AOC es un mayor riesgo de tromboembolismo debido a las propiedades hipercoagulables de los estrógenos.

Finalmente, es importante recalcar que el uso de anticonceptivos orales combinados tiene un efecto protector con los fibroadenomas de mama, el cáncer de ovario y de endometrio, cuando existe un uso prolongado (decadadas) de los AOC. Lo anterior se ha evidenciado en grandes estudios epidemiológicos, pero no se tiene claridad absoluta del mecanismo involucrado en este efector protector, no obstante, en cuanto al cáncer de endometrio, se ha planteado que este beneficio protector se atribuye al componente de progestina, que suprime la proliferación endometrial.

How well did you know this?
1
Not at all
2
3
4
5
Perfectly
23
Q
  1. Femenina de 39 años lactante de su bebe de 3 meses, consultó por falta de apetito, malestar general, eritema y dolor en seno derecho desde hace 3 días, para el cual ha tomado doxiciclina desde entonces. No refiere náuseas, diarrea ni cefalea. Antecedente de G2P2 y madre con cáncer de mama a los 54 años. Signos vitales Tº de 37,7 Cº, PA de 124/68 mm/Hg y FC de 98 lpm. Al examen físico, ambos senos tienen congestión moderada y un agrietamiento lineal del pezón. La mama derecha tiene una nodularidad dolorosa y eritema. El resto del examen físico es normal. ¿Cuál sería el siguiente paso en el manejo de esta paciente?
    A. Realizar una biopsia de espesor total.
    B. Iniciar corticoide tópico.
    C. Cambiar antibiótico a cefalexina.
    D. Solicitar ecografía mamaria.
A

Respuesta correcta: D

Tema e ítem: Mastitis-Pruebas diagnósticas.

Argumento: Esta paciente que está lactando, presentó en el curso de su cuadro clínico una mastitis de la lactancia, caracterizada por síntomas similares a los de la gripe (fiebre o malestar general), eritema y dolor mamario. Si bien la paciente recibió el tratamiento empírico adecuado, el cual es doxiciclina o cefalexina (son efectivos contra el patógeno más común, el Staphylococcus aureus sensible a la meticilina (MSSA)), no ha presentado mejoría del cuadro clínico en las primeras 48 horas como se esperaría, por lo que se debe sospechar de un absceso mamario, generado por una retención de leche (por vaciamiento incompleto) infectada que conduce a una acumulación focal de líquido purulento. Debido a que el absceso forma una cavidad tapizada, es menos susceptible a la penetración de antibióticos, lo que explica porque no hay mejoría de los síntomas con la terapia antibiótica adecuada. Recordemos, que para realizar el diagnóstico de absceso mamario basta con la clínica (masa fluctuante discreta palpable), no obstante, en esta paciente sin el hallazgo clínico claro de un absceso y que presenta simultáneamente nódulos sensibles en el seno, se debe realizar una ecografía para diferenciar la mastitis grave de un absceso. Una vez diagnosticado el absceso, el tratamiento consiste en realizar una aspiración con aguja guiada por ecografía o incisión quirúrgica y drenaje.

En cuanto a la opción de iniciar un corticosteroide tópico, no hay evidencia que sustente el uso y beneficio de este tratamiento en un cuadro clínico de mastitis o de un absceso mamario. Por otra parte, la paciente ya está con uso de doxiciclina, un antibiótico que al igual que la cefalexina cubren el agente causal más común de mastitis (MSSA), pero no ha presentado mejoría, por lo que el manejo no es hacer un cambio a un antibiótico similar, sino buscar porque hay ausencia de mejoría, en este caso sospechar de un absceso mamario que forma una cavidad tapizada que dificulta la penetración del antibiótico. Finalmente, esta paciente no tiene indicación de realizar una biopsia de espesor total, debido a que la clínica de la paciente no hace referencia a cambios malignos, así como tampoco hay estudios imagenológicos que ameriten por el momento definir una conducta.

How well did you know this?
1
Not at all
2
3
4
5
Perfectly
24
Q
  1. Femenina de 42 años, G0P0, consultó por secreción sanguinolenta del pezón derecho. No ha tenido dolor, edema ni trauma en los senos. Nunca se ha hecho una mamografía. Sin antecedentes médicos relevantes. Los signos vitales son normales. Al examen físico, senos colgantes simétricos, sin ulceración, sin formación de costras o descamación de los complejos areola-pezón. La presión manual provoca secreción sanguinolenta de un solo conducto del pezón derecho. Se palpa una masa firme, mal definida, de 2 cm en el cuadrante inferior externo de la mama derecha, con retracción de la piel suprayacente. La mama izquierda es normal, y no hay adenopatías. ¿Cuál es el diagnóstico más probable en esta paciente?
    A. Absceso mamario
    B. Papiloma intraductal
    C. Carcinoma ductal invasivo
    D. Ectasia del conducto mamario
A

Respuesta correcta: C

Tema e ítem: Cáncer de mama-Diagnóstico

Argumento: Esta paciente tiene secreción sanguinolenta unilateral patológica por el pezón y una masa mamaria firme y mal definida. Esta presentación es más preocupante para el cáncer de mama (por ejemplo, un carcinoma ductal invasivo), especialmente dada la retracción de la piel que recubre la masa, que generalmente se debe a la invasión del tumor en los ligamentos suspensorios de la mama. El crecimiento tumoral y/o la necrosis dentro del sistema de conductos galactóforos pueden provocar una secreción sanguinolenta del pezón. Como próximos pasos para evaluar tanto la secreción patológica del pezón como la masa mamaria palpable, esta paciente debe someterse a estudios de imagen mamaria, mamografía, ecografía) seguidos de una biopsia. Recordemos que el enfoque de secreción del pezón, que no coincide con la lactancia, se clasifica como fisiológica o patológica: La secreción fisiológica del pezón suele ser bilateral, multiductal, lechosa o no sanguinolenta y se expresa solo con la manipulación de la mama, es causada por un proceso benigno fuera del seno (hiperprolactinemia). Por otra parte, la secreción patológica suele ser unilateral, uniductal, sanguinolenta o serosa y espontánea, y suele indicar la presencia de un proceso anormal (papiloma, carcinoma, absceso) dentro de la mama.

En cuanto a la sospecha de un absceso, es poco probable, debido a que esta paciente presenta hallazgos que sugieren más un proceso neoplásico, como la presencia de una masa firme, mal definida, con retracción de la piel suprayacente, además no hay eritema ni signos inflamatorios presentes. Por otra parte, la ectasia del conducto mamario y el papiloma intraductal, son entidades que se categorizan dentro del espectro de enfermedades benignas de la mama; y por ende, en el cuadro clínico de esta paciente son poco probables, no obstante para realizar la diferenciación se hace necesario de ayudas imagenológicas como la ecografía.

How well did you know this?
1
Not at all
2
3
4
5
Perfectly
25
Q
  1. Femenina de 16 años, consultó a hospital de 4 nivel para recibir anticoncepción de emergencia. Tuvo relaciones sexuales sin protección anoche y le preocupa que pueda quedar embarazada. Su último período menstrual fue hace 1 semana y ha tenido menstruaciones regulares desde la menarquia. El examen físico es normal y la prueba de embarazo en orina es negativa. La paciente solicita una opción de píldora y que sus padres no sean notificados. ¿Cuál de los siguientes es el tratamiento más adecuado para esta paciente?
    A. Obtener el consentimiento de los padres antes de proporcionar anticoncepción de emergencia.
    B. Recetar pastillas de misoprostol (prostaglandina E1).
    C. Recetar píldora de levonorgestrel.
    D. Suministrar a la paciente un régimen combinado o método Yuzpe.
A

Respuesta correcta: C

Tema e ítem: Anticonceptivos de Emergencia

Argumento: Las pacientes con relaciones sexuales recientes sin protección pueden ser candidatas para la anticoncepción poscoital o de emergencia (AE) para la prevención del embarazo. El estado de embarazo determina la elegibilidad para el uso de AE: En el primer escenario, las pacientes con una prueba de embarazo positiva no son candidatas para la AE porque ya se ha producido la implantación, por otra parte, en el segundo escenario, en las pacientes con una prueba de embarazo negativa aún corren el riesgo de un embarazo no deseado y son candidatas para la AE, así como es el caso de esta paciente. La píldora de levonorgestrel es la forma de AE más fácilmente disponible; la alta dosis de progestina en estas píldoras previene el embarazo al inhibir el aumento de LH, lo que retrasa la ovulación. Los mecanismos adicionales incluyen el engrosamiento del moco cervical y el adelgazamiento del revestimiento endometrial. Las pastillas de levonorgestrel son muy eficaces (es decir, hasta un 94 %); sin embargo, la eficacia disminuye en el transcurso de 72 horas (es decir, tan bajo como 59%). Por lo tanto, las píldoras de levonorgestrel deben administrarse lo antes posible después de una relación sexual sin protección.

En cuanto a las otras opciones, en Colombia se permite que los adolescentes mayores de 14 años reciban atención confidencial para la anticoncepción, el embarazo y las enfermedades de transmisión sexual sin el consentimiento de los padres. Por otro lado, el misoprostol tiene un uso como medicamento para interrupción de la gestación, debido a que aumenta la frecuencia de las contracciones uterinas; no obstante, no tiene indicación como anticonceptivo de emergencia, debido a que lo que se busca es que no se da la implantación del óvulo fecundado en el endometrio. Finalmente, el método Yuzpe puede ser una alternativa eficaz cuando se encuentran disponibles, y que son de bajo costo; sin embargo, no se recomiendan cuando hay entornos hospitalarios que cuenten con terapia con levonorgestrel, debido a que se ha evidenciado que el uso de terapia combinada es menos efectiva, produce más efectos secundarios y tiene interacción con antirretrovirales.

How well did you know this?
1
Not at all
2
3
4
5
Perfectly
26
Q
  1. Masculino de 84 años consulta por perdida gradual de su resistencia y poca energía. Ya no puede correr la misma distancia que hacia de forma rutinaria sin detenerse a descansar. Antecedente de HTA por lo que toma lisinopril. Signos vitales: PA de 130/80 mmHg y FC de 52 lpm. Frecuencia cardiaca regular, resto de examen físico normal. Paraclínicos dentro de limites normales. Se evidencia un ECG. Un holter de ECG de 48 horas evidencia una FC máxima de 72 lpm durante actividad física vigorosa. Un ecocardiograma y una prueba de esfuerzo realizada hace 1 año por dolor torácico es normal. ¿Cuál es el manejo adecuado para este paciente?
    A. Hospitalizar para realizar estudios adicionales.
    B. Realizar implantación de marcapasos.
    C. Cambiar lisinopril por amlodipino.
    D. Tranquilidad y seguimiento clínico.
A

Respuesta correcta: B

Tema e ítem: Bradiarritmias – Manejo.

Argumento: La colocación de un marcapasos es el manejo adecuado para el manejo de este paciente con síntomas secundarios a disfunción del nódulo sinusal. Las indicaciones para la implantación de un marcapasos permanente incluyen la bradicardia sintomática sin causa reversible, bloqueo de rama alterno y bloqueo cardíaco completo, Bloqueo AV de segundo grado Mobitz 2 y bloqueo AV de alto grado independientemente de los síntomas. Este paciente tiene bradicardia al inicio del estudio, no presenta respuesta taquicárdica a la actividad física en la monitorización ambulatoria y se encuentra sintomático. Un desafío común entre estos pacientes, muchos de los cuales son mayores, es diferenciar entre la disminución de la actividad física relacionada con la edad y la disfunción del nódulo sinusal patológica y sintomática. Es menos probable que el primero mejore con la estimulación cardíaca, y el segundo a menudo mejora de manera significativa. En este caso, el paciente es muy activo, con clara pérdida de energía y hallazgos electrocardiográficos de bradicardia sinusal. Los resultados recientes de la prueba de esfuerzo confirman una función ventricular izquierda normal y sin isquemia; por lo tanto, la estimulación cardíaca es el siguiente paso adecuado.

  • Los pacientes con disfunción del nódulo sinusal rara vez están inestables, pero si hay evidencia de inestabilidad hemodinámica, está indicada la hospitalización. Los signos y síntomas de advertencia de inestabilidad hemodinámica que justifican la hospitalización incluyen hipotensión, estado mental alterado, dolor torácico isquémico e insuficiencia cardíaca aguda.
  • Es poco probable que Lisinopril contribuya a la bradicardia de este paciente, aunque su antecedente de hipertensión arterial probablemente sea un factor de riesgo para la disfunción del nodo sinusal. El uso de amlodipina, un bloqueador de los canales de calcio dihidropiridínico, puede empeorar la bradicardia.
How well did you know this?
1
Not at all
2
3
4
5
Perfectly
27
Q
  1. Femenina de 26 años presenta palpitaciones desde hace 2 meses. Dicho cuadro comienza sin factores desencadenantes específicos, que duran de 5 a 15 minutos y se alivian con el reposo o la respiración profunda. Ocurren algunas veces por semana. Entre los episodios, desarrolla sus actividades diarias sin limitaciones. No tiene antecedentes pertinentes y no toma medicamentos. El examen físico y signos vitales son normales. El ECG en reposo muestra un ritmo sinusal y no presenta anomalía. ¿Cuál es la prueba diagnostica más adecuada en esta paciente?
    A. ECG de esfuerzo.
    B. Grabador de bucle implantable.
    C. Telemetría cardiaca móvil.
    D. Monitor de eventos de 30 días.
A

Respuesta correcta: D

Tema e ítem: Enfoque del paciente con palpitaciones.

Argumento: Un monitor de eventos de 30 días es la prueba diagnostica adecuada en esta paciente con palpitaciones. Los monitores de eventos son dispositivos activados por el paciente que son ideales para capturar arritmias sintomáticas. La elección del monitor de ritmo cardíaco ambulatorio depende de la frecuencia y duración de los síntomas, así como de cualquier otro síntoma, signo o hallazgo asociado. Para pacientes como este, con síntomas que ocurren con frecuencia en el transcurso de un mes, pero no a diario, lo más apropiado es un monitor de eventos de 30 días. El paciente puede activar el monitor cuando se presenten síntomas, siempre que los síntomas duren lo suficiente como para ser captados por la captura activa y retrospectiva del dispositivo. Los síntomas que duran más de 1 a 2 minutos son ideales para los monitores de eventos.

  • Un ECG de esfuerzo permite el diagnóstico de arritmias relacionadas con el ejercicio y también permite la evaluación del efecto del ejercicio sobre la presión arterial y los síntomas. La mayoría de las arritmias no son inducidas por el ejercicio, y esta paciente indicó claramente que no hubo eventos desencadenantes de sus palpaciones; por lo tanto, el ECG de esfuerzo no está indicado.
  • Un grabador de bucle implantable es más apropiado para arritmias poco frecuentes o muy sintomáticas en las que los síntomas, como el síncope, pueden impedir que el paciente active el dispositivo. Estos monitores se implantan debajo de la piel y se pueden usar durante meses o años. Los síntomas de este paciente ocurren semanalmente y no están asociados con síncope; por lo tanto, no es necesaria una grabadora de bucle implantable.
  • La telemetría cardíaca móvil proporciona un registro de ECG ambulatorio continuo para la cuantificación precisa o la captura de arritmias raras. Esta no está indicada para este paciente cuyos síntomas aún no se han correlacionado con ninguna arritmia específica; por lo tanto, la cuantificación de la carga de arritmia es innecesaria en este momento. Se usa a menudo para detectar arritmias que pueden no estar asociadas con síntomas.
How well did you know this?
1
Not at all
2
3
4
5
Perfectly
28
Q
  1. Masculino de 37 años consulta a urgencias por dolor en cuadrante superior derecho, fiebre y malestar desde hace 3 días. Signos vitales: temperatura de 38.9°C, PA de 100/40 mmHg y FC de 94 lpm. Al examen físico el paciente se encuentra ictérico y hay dolor abdominal con defensa en cuadrante superior derecho. Paraclínicos: leucocitosis, BT de 9 mg/dl y lipasa normal. La ecografía de abdomen revela colelitiasis, dilatación y calculo visible en colédoco. Se inicia manejo con piperacilina-tazobactam. ¿Cuál es el manejo adecuado para este paciente?
    A. Drenaje biliar laparoscópico.
    B. Colangioresonancia.
    C. Colecistectomía urgente.
    D. CPRE urgente.
A

Respuesta correcta: D

Tema e ítem: Patología de la vía biliar – Manejo.

Argumento: El tratamiento más adecuado es la colangiopancreatografía retrógrada endoscópica (CPRE) urgente. Este paciente tiene colangitis aguda, una infección del árbol biliar anunciada por la tríada de Charcot (fiebre, ictericia y dolor en el cuadrante superior derecho). La colangitis es potencialmente mortal; el tratamiento debe incluir una terapia antimicrobiana inmediata dirigida a Enterobacteriaceae en combinación con la descompresión urgente del árbol biliar con esfinterotomía endoscópica y extracción de cálculos. Además, la colecistectomía debe realizarse durante la hospitalización inicial o dentro de las 2 semanas para reducir el riesgo de complicaciones posteriores.

How well did you know this?
1
Not at all
2
3
4
5
Perfectly
29
Q
  1. Masculino de 32 años es llevado a urgencias inconsciente luego de ser encontrado cerca de frascos de metadona y lorazpam. En su momento con FR de 6 rpm, se le administró O2 suplementario y se administró naloxona intranasal, seguido de naloxona IV. En urgencias se repite naloxona IV. Signos vitales: Temp. de 37°C, PA de 120/68 mmHg, FC de 80 lpm, FR de 10 rpm y SatO2 de 90% con O2 a 5L/min por cánula nasal. Hay miosis pupilas, pero responde a la luz. Nivel de glucosa sérico de 88 mg/dL. Rx evidencia infiltrado en el lóbulo inferior derecho. ¿Cuál es el manejo adecuado para este paciente?
    A. Realizar entubación endotraqueal.
    B. Administrar flumazenil.
    C. Infusión IV de naloxona.
    D. Administrar oxigeno por mascara de no reinhalación.
A

Respuesta correcta: A

Tema e ítem: Manejo del paciente intoxicado.

Argumento: Este paciente tiene un síndrome tóxico que se manifiesta como depresión respiratoria y existe preocupación por una posible aspiración. Las circunstancias del síndrome tóxico en este paciente no están claras y pueden involucrar más de una sustancia. Sin embargo, una sospecha inicial es la toxicidad por opioides, con antecedentes de acceso a la metadona, depresión respiratoria, pupilas mióticas, nivel de conciencia deprimido y una respuesta transitoria de la frecuencia respiratoria a la naloxona. Además de identificar los agentes o las clases de fármacos consumidos, la atención de los pacientes que han tomado una sobredosis es principalmente de apoyo; el mantenimiento de la permeabilidad de las vías respiratorias y la ventilación es de vital importancia en pacientes con deterioro del estado mental. La incapacidad de este paciente para proteger las vías respiratorias y la preocupación por la aspiración requieren intubación de las vías respiratorias.

  • El flumazenil, un antagonista del receptor de benzodiacepinas, generalmente no está indicado en pacientes con depresión de la conciencia debido a intoxicación por varias sustancias porque puede precipitar convulsiones por abstinencia. Además, debido a que los efectos depresores respiratorios de las benzodiazepinas son proporcionalmente pequeños en relación con los de los opioides, la reversión con flumazenil no beneficiará a un paciente con insuficiencia respiratoria inminente.
  • La naloxona es un antídoto opioide que se puede administrar por vía intravenosa, intramuscular, intranasal y endotraqueal. Si la depresión respiratoria mejora considerablemente, se puede administrar naloxona en dosis repetidas o como una infusión intravenosa hasta que el opioide se haya metabolizado y eliminado lo suficiente. En muchos casos, la naloxona puede evitar la necesidad de ventilación mecánica. En este paciente, sin embargo, es necesaria la intubación debido a su aparente incapacidad para proteger las vías respiratorias.
  • Proporcionar niveles más altos de oxígeno suplementario podría aumentar modestamente los niveles de oxígeno en la sangre del paciente, pero no afectará los problemas primarios subyacentes de hipoventilación alveolar y la incapacidad para proteger las vías respiratorias de la aspiración.
How well did you know this?
1
Not at all
2
3
4
5
Perfectly
30
Q
  1. Paciente masculino de 42 se encuentra en control para su hipertensión arterial de reciente diagnóstico. Se inició manejo con clortalidona hace 1 mes sin embargo el control de presión en casa evidencia lecturas por encima del objetivo a pesar de la adherencia al tratamiento y las intervenciones en el estilo de vida. Signos vitales: PA de 150/90 mmHg y FC de 72 lpm. IMC de 22 kg/m2. Examen físico normal. Paraclínicos evidencian creatinina de 1.0 mg/dl y K+ sérico de 3.5 mEq/L. ¿Cuál es el tratamiento adicional adecuado para este paciente?
    A. Amlodipino.
    B. Doxazosina.
    C. Hidralazina.
    D. Metoprolol.
A

Respuesta correcta: A

Tema e ítem: Manejo de la hipertensión arterial

Argumento: En este paciente, un objetivo de presión arterial (PA) <130/80 mm Hg es razonable. Se recomiendan dos o más medicamentos antihipertensivos para lograr un objetivo de PA de <130/80 mm Hg en la mayoría de los adultos con hipertensión. De acuerdo con la guía del American College of Cardiology/American Heart Association, cuatro clases de medicamentos (diuréticos tiazídicos, BCC, inhibidores de la ECA o BRA) reducen la PA y reducen los resultados cardiovasculares o renales. Excepto por la combinación de inhibidores de la ECA y ARB, los regímenes que contienen una combinación de estas clases son razonables para lograr el objetivo de PA.
La doxazosina, la hidralazina y el metoprolol no son fármacos recomendados para la terapia inicial o como terapia adicional en pacientes con hipertensión sin la presencia de indicaciones convincentes. La doxazosina podría considerarse en pacientes masculinos con síntomas urinarios bajos molestos debido a hiperplasia prostática benigna; hidralazina y metoprolol en pacientes seleccionados con insuficiencia cardíaca; o metoprolol en pacientes con infarto de miocardio reciente. Ninguna de estas indicaciones está presente en este paciente.

How well did you know this?
1
Not at all
2
3
4
5
Perfectly
31
Q
  1. Masculino de 42 años consulta para extirpar un lunar en su antebrazo izquierdo, el cual ha tenido desde su juventud, refiriendo que lo adquirió por usar protector solar contaminado por toxinas. Refiere un colega le está “enviando pensamientos negativos que podrían hacer que se agrande”. Refiere que un compañero de trabajo lo elogió esta semana, y ahora le preocupa que este se encuentre celoso de él y desee hacerle daño. Ha trabajado en la misma empresa toda su vida y no tiene relaciones cercanas. Al examen físico se visualiza un nevus simétrico de 3 mm con bordes bien definidos en su brazo izquierdo. El paciente tiene mínimo contacto visual, afecto restringido y habla de manera formal. ¿Cuál es el diagnostico más probable?
    A. Trastorno paranoide de la personalidad.
    B. Trastorno esquizoide de la personalidad.
    C. Trastorno esquizotípico de la personalidad.
    D. Esquizofrenia.
A

Respuesta correcta: C

Tema e ítem: Trastornos de la personalidad - Trastorno esquizotípico de la personalidad.

Argumento: Las creencias extrañas/el pensamiento mágico de este paciente y el habla idiosincrásia de este paciente sugieren un trastrono esquizotípico de la personalidad (TEP). Las personas con TEP suelen parecer extrañas y socialmente incómodas, a menudo con un afecto restringido. El habla puede ser de naturaleza vaga, metafórica o idiosincrásica. Los pacientes con TEP tienden a evitar las relaciones sociales y tienen una capacidad limitada para las relaciones íntimas. Las creencias paranoicas (Que su colega está celoso y desea hacerle daño) son comunes y pueden provocar ansiedad social. Las creencias extrañas y las distorsiones perceptivas (ilusiones, sensaciones corporales inusuales) que se observan en pacientes con TEP no alcanzan proporciones psicóticas. El TEP se puede diferenciar de los trastornos psicóticos primarios por la ausencia de delirios persistentes bien formados o alucinaciones francas.

  • Aunque este paciente comparte algunas características con otros trastornos de personalidad, como la suspicacia que se observa en el trastorno de personalidad paranoide y el desapego emocional que se observa en el trastorno de personalidad esquizoide, ninguno de estos trastornos de personalidad explicaría las extrañas creencias, el pensamiento mágico de este paciente. El habla idiosincrásica y apariencia excéntrica, que son características del TEP.
  • La esquizofrenia se caracteriza por ≥6 meses de delirios, alucinaciones, habla desorganizada, comportamiento desorganizado y síntomas negativos (apatía, abulia), que a menudo resultan en un deterioro significativo en múltiples entornos. Este paciente, sin embargo, es capaz de mantener un trabajo y no está muy desorganizado. Aunque sus creencias son extrañas, no tienen una intensidad delirante; las alucinaciones están ausentes.
How well did you know this?
1
Not at all
2
3
4
5
Perfectly
32
Q
  1. Habitante de la calle de 25 años con antecedente de esquizofrenia es llevado a urgencias por la policía luego de destruir elementos de una tienda electrónica de su ciudad, luego se puso agitado y violento cuando los empleados intentaron intervenir. Se ingresa a urgencias y usted le coloca una dosis de haloperidol. Más tarde en la noche el paciente se pregunta “¿Qué me está pasando?”. Se encuentra molesto, se niega a sentarse y se señala el cuello. Al examen físico hay una contracción muscular sostenida del cuello hacia el lado derecho. ¿Cuál es el manejo adecuado para este paciente?
    A. Difenhidramina.
    B. Levodopa.
    C. Lorazepam.
    D. Quetiapina.
A

Respuesta correcta: A

Tema e ítem: Efectos adversos del uso de psicofarmacos.

Argumento: Este paciente está experimentando una reacción distónica aguda debido al haloperidol, un antipsicótico de primera generación de alta potencia. La distonía aguda es un tipo de síntoma extrapiramidal que se desarrolla a las pocas horas o días de iniciar o aumentar la dosis de un antipsicótico. Se caracteriza por espasmos musculares o rigidez en la cabeza y el cuello, incluyendo protuberancias o torsión de la lengua, muecas faciales, tortícolis, opistótonos (espalda) y crisis oculógira (elevación forzada y sostenida de los ojos en una posición hacia arriba). El riesgo de distonía aguda es mayor con antipsicóticos de alta potencia como el haloperidol en comparacion con aquellos de baja potencia y antipsicóticos de segunda generación. La distonía aguda debe distinguirse de la acatisia (inquietud interna) y parkinsonismo inducido por fármacos (temblor, rigidez, bradicinesia, facies enmascarada). Las reacciones distónicas agudas se tratan con medicamentos anticolinérgicos (benztropina) y antihistamínicos (difenhidramina) que tienen una actividad anticolinérgica significativa. Otras estrategias de manejo posteriores para los SEP incluyen la reducción de la dosis de antipsicóticos si es factible o el cambio a un SGA con un menor riesgo de efectos extrapiramidales.

How well did you know this?
1
Not at all
2
3
4
5
Perfectly
33
Q
  1. Femenina de 65 años con antecedente de hipertensión arterial, diabetes mellitus tipo 2, hiperlipidemia y artrosis. Sus cifras tensionales no están en metas y lleva un mal control glucemico con medicamentos antidiabeticos orales. IMC de 34 kg/m2. Al examen fisico su tercer y cuarto dedo de los pies parecen “en forma de garra”, con dorsiflexión de falanges proximales en articulaciones metatarsofalángicas y flexión plantar de aritculaciones interfalángicas proximales y distales. ¿Cuál es la causa más probable de la condición actual de esta paciente?
    A. Neuropatía diabetica.
    B. Artrosis del pie.
    C. Neuroma de Morton.
    D. Fascitis plantar.
A

Respuesta correcta: A

Tema e ítem: Neuropatía diabetica.

Argumento: Este paciente tiene una deformidad del dedo del pie en garra, caracterizada por dorsiflexión en las articulaciones metatarsofalángicas y flexión plantar en las articulaciones interfalángicas proximales y distales. Las deformidades del dedo del pie en garra y el dedo en martillo (similar al dedo en garra, pero con dorsiflexión en las articulaciones interfalángicas distales) son comunes en pacientes de edad avanzada y reflejan un desequilibrio en la fuerza y la flexibilidad entre los grupos de músculos flexores y extensores. Estas deformidades a menudo se deben a un problema mecánico simple, como zapatos que no calzan crónicamente, pero en un paciente con diabetes prolongada pueden sugerir una neuropatía diabética periférica subyacente.

La etiología de las deformidades del pie en pacientes con neuropatía diabética implica múltiples factores, incluida la atrofia de los músculos intrínsecos del pie debido a la degeneración de los axones motores, disminución de la sensación de dolor y propiocepción, y trastornos ortopédicos y vasculares concurrentes. Las complicaciones a largo plazo de la neuropatía diabética incluyen callos, ulceración, subluxación articular y artropatía de Charcot (destrucción y deformidad de huesos y tejidos blandos).

How well did you know this?
1
Not at all
2
3
4
5
Perfectly
34
Q
  1. Masculino de 4 años es evaluado por presentar ambos “ojos cruzados”. Su madre notó que el ojo izquierdo del paciente se ha desviado muy levemente hacia la nariz durante los ultimos meses. Presenta una agudeza visual 20/”0 en ojo derecho y 20/50 en ojo izquierdo. Cuando el ojo derecho está cubierto, el ojo izquierdo cambia la fijación de una dirección nasal a una temporal. Cuando el ojo izquierdo está cubierto, el ojo derecho permanece en la posición de la línea media. El reflejo rojo es más intenso en el ojo izquierdo. Además de la indicación de lentes correctivos. ¿Cuál es el manejo adecuado para este paciente?
    A. Colocar un parche en su ojo derecho.
    B. Colocar un parche en su ojo izquierdo.
    C. No es necesaria ninguna intervención adicional.
    D. Gotas de latanoprost en ojo derecho.
A

Respuesta correcta: A

Tema e ítem: Estrabismo – Manejo.

Argumento: Este paciente tiene estrabismo, que es una desalineación ocular caracterizada por desviación nasal (esotropía) o temporal (exotropía) del ojo. Se puede esperar estrabismo intermitente en bebés menores de 4 meses debido a la inmadurez de los músculos extraoculares (inestabilidad ocular de la infancia). Sin embargo, el estrabismo más allá de la primera infancia debe tratarse para prevenir la ambliopía, una reducción funcional de la agudeza visual por falta de uso del ojo desviado, como se observa en este paciente.

Los hallazgos del examen en el estrabismo incluyen reflejos rojos asimétricos y reflejos de luz corneales, así como desviación del ojo durante la prueba de cobertura. La prueba de cobertura se realiza cubriendo un ojo y observando el movimiento en el otro mientras el niño enfoca un objeto. Un ojo normal mantiene la misma posición sin moverse; un ojo desalineado cambia para volver a fijarse en el objeto cuando se cubre el ojo normal.

El tratamiento consiste en anteojos recetados para la corrección de errores de refracción significativos (si los hay) y promover el uso del ojo afectado. Por ejemplo, el ojo ambliópico puede fortalecerse con un parche en el ojo normal o haciendo borrosa la visión del ojo normal con gotas ciclopléjicas (atropina). Sin tratamiento, la agudeza visual de este paciente en el ojo izquierdo seguirá empeorando. La detección del estrabismo debe realizarse en cada visita de niño sano hasta los 5 años porque los resultados del tratamiento son mejores cuando se inicia cuando la corteza visual es más maleable.

How well did you know this?
1
Not at all
2
3
4
5
Perfectly
35
Q
  1. Femenina de 37 años consulta por dolor intenso en flanco izquierdo de 1 día de evolución. El dolor inició de forma aguda, es intermitente, de tipo cólico y se irradia a la ingle. Ha tenido náuseas, 2 episodios emeticos y hematuria. Hoy con fiebre y escalofrios. Signos vitales: Temperatura de 38.4°C, PA de 140/90 mmHg, FC de 105 lpm y FR de 18 rpm. Al examen físico puño percusión renal izquierda positiva. Un UROTAC revela un cálculo de 5 mm en uréter medio izquierdo con dilatación de uréter proximal. ¿Cuál es la indicación más fuerte para solicitar valoración por urología?
    A. Fiebre y escalofrios.
    B. Hematuria macroscopica.
    C. El tamaño del cálculo.
    D. La dilatación ureteral.
A

Respuesta correcta: A

Tema e ítem: Ureterolitiasis – Indicaciones de valoracion por urología.

Argumento: Este paciente tiene dolor agudo en el costado y náuseas debido a ureterolitiasis, que se confirma en la tomografía computarizada. El tratamiento inicial incluye hidratación, analgésicos (los fármacos antiinflamatorios no esteroideos suelen preferirse a los opiáceos) y antieméticos (metoclopramida). Se debe aconsejar a los pacientes que revisar la orina para confirmar el paso del cálculo y recuperar el cálculo examenes adicionales.

Aquellos pacientes con signos de urosepsis (pfiebre, taquicardia), como en este paciente; anuria; Lesión renal aguda; o el dolor refractario requieren una consulta urológica urgente para una posible nefrostomía percutánea o inserción de un stent ureteral. Además, aquellos con cálculos grandes que es poco probable que se eliminen espontáneamente (≥10 mm), o aquellos cuyos cálculos no se eliminan dentro de las 4 a 5 semanas, también justifican una evaluación urológica (en el ámbito ambulatorio) para considerar intervenciones más agresivas (litotricia, intervención ureteroscópica).

  • La hematuria (macroscópica o microscópica) está presente en la mayoría de los pacientes con cálculos sintomáticos. La dilatación ureteral también se observa en >80% de aquellos con cálculos sintomáticos y sería de esperar dado que la obstrucción ureteral es la fuente de gran parte del dolor en estos pacientes. Sin embargo, en ausencia de obstrucción completa (anuria) y dilatación ureteral marcada o hidronefrosis, no se requiere tratamiento intervencionista agudo.
How well did you know this?
1
Not at all
2
3
4
5
Perfectly
36
Q
  1. Masculino de 40 años, consultó por 5 días de rinorrea, tos que empeora en la noche, dolor de garganta, sin mejoría ni atenuantes. El día de hoy refiere que el dolor ha empeorado, y que presenta disfagia y odinofagia, incluso dificultad para tragar líquidos. Signos vitales 38,4 ºC, PA de 148/90 mm/Hg, FC de 110 lpm y FR de 22 rpm y SO2 de 99% al ambiente. Al examen físico, acumulación de secreciones orales. La orofaringe posterior aparece levemente eritematosa debido al goteo postnasal. El cuello anterior es blando pero sensible a la palpación. El examen pulmonar revela un estridor débil sin crepitantes ni ronquidos. ¿Cuál es el paso a seguir para establecer el diagnóstico en este paciente?
    A. Realizar radiografia de torax
    B. Prueba rapida de antigeno para estreptococo del grupo A
    C. Realizar radiografia lateral de cuello
    D. Realizar gram y cultivo de esputo
A

Respuesta correcta: C

Tema e ítem: Epiglotis-Diagnóstico

Argumento: Este paciente con dolor de garganta, fiebre, sensibilidad laringotraqueal a la palpación y empeoramiento del edema que genera dificultad para la deglución, junto al acúmulo de secreciones orales y compromiso respiratorio (taquipnea, estridor), nos deben hacer sospechar en una epiglotitis. Recordemos que en adultos, la mayoría de los casos son causados por Streptococcus pneumoniae o Haemophilus influenzae. El diagnóstico se confirma mediante visualización directa o radiografía lateral del cuello de los tejidos blandos, que por lo general muestra epiglotis agrandada, pérdida del espacio aéreo vallecular y/o distensión de la hipofaringe. Las radiografías también pueden ayudar a excluir otras condiciones. El compromiso respiratorio es menos común en adultos, pero puede progresar rápidamente; por lo tanto, los pacientes con compromiso significativo (hipoxia resistente a la intervención no invasiva) requieren el establecimiento de vías respiratorias antes de considerar la radiografía de cuello.

En cuanto a la radiografía de tórax y la tinción/cultivo de Gram de esputo, se utilizan para diagnosticar la neumonía. Aunque este paciente tiene tos, su oximetría de pulso es normal y sus pulmones no tienen evidencia de patología intrapulmonar (crepitos). La presencia de estridor y secreciones orales acumuladas hace que un problema de las vías respiratorias superiores, como la epiglotitis, sea mucho más probable que la neumonía. Por otra parte, la faringitis por estreptococo del grupo A generalmente se presenta con dolor de garganta de inicio agudo, exudados amigdalinos, linfadenopatía cervical y ausencia de tos (criterios de Centor). La aparición lenta de dolor de garganta, tos, acumulación de secreciones orales y estridor en este paciente hacen que la epiglotitis sea mucho más probable.

How well did you know this?
1
Not at all
2
3
4
5
Perfectly
37
Q
  1. Masculino de 43 años, es encontrado deambulando por la calle y llevado a urgencias. El paciente está confundido y no puede proporcionar ninguna historia. No tiene antecedentes hospitalarios previos. Signos vitales, Tº 35ºC, el resto normales. Al examen fisico, paciente letargico, pero sigue las ordenes. La mucosa oral está húmeda. El paciente no tiene linfadenopatía cervical, pero hay edema bilateral no doloroso de las mejillas compatibles con agrandamiento de las glándulas salivales. ¿Cuál de las siguientes es la causa más probable de este último hallazgo?
    A. Alcoholismo
    B. Sindrome de Sjögren
    C. Adenoma pleomórfico
    D. Parotiditis
A

Respuesta correcta: A

Tema e ítem: Agrandamiento bilateral de las glándulas parótidas (Alcoholismo)-Clínica

Argumento: Este paciente tiene agrandamiento bilateral no doloroso de las glándulas parótidas, que es compatible con sialoadenosis. La sialoadenosis es una inflamación benigna y no inflamatoria de las glándulas salivales. Puede resultar de la acumulación excesiva de gránulos secretores en las células acinares (posiblemente debido a una inervación autónoma anormal) en pacientes con consumo crónico de alcohol, bulimia o desnutrición. También puede resultar de la infiltración grasa de las glándulas en pacientes con diabetes mellitus o enfermedad hepática. Recordemos que la sialoadenosis se presenta con un agrandamiento gradual de las glándulas que no fluctúa y no está asociado con la alimentación. El reconocimiento de esta condición debe impulsar una investigación de la enfermedad subyacente

Por otra parte, la parotiditis debida a las paperas puede presentarse con edema de la parótida; sin embargo, suele ser doloroso y se presenta pocos días después del inicio de los síntomas sistémicos (fiebre, cefaleas, mialgias). En otro lado, en cuanto a un adenoma pleomórfico, recordemos es una neoplasia salival benigna que puede presentarse con un agrandamiento indoloro de la glándula parótida; sin embargo, el agrandamiento sería unilateral y típicamente se palpa una masa de características distintivas. Finalmente, el síndrome de Sjögren puede provocar inflamación bilateral de las glándulas parótidas debido a la infiltración linfocítica, sin embargo, a diferencia de este paciente, típicamente se presenta en mujeres y se asocia con boca seca en el examen (mientras que las membranas mucosas de este paciente están húmedas).

How well did you know this?
1
Not at all
2
3
4
5
Perfectly
38
Q
  1. Masculino de 55 años, consultó porque hace 6 meses le diagnosticaron cáncer papilar de tiroides y se sometió a tiroidectomía total, seguida de tratamiento con yodo radiactivo. Desde entonces, ha estado tomando levotiroxina y no ha tenido problemas. Sin antecedentes médicos relevantes. Los signos vitales están dentro de los límites normales. Al examen físico, incisión quirúrgica bien curada, sin masas palpables en el cuello ni ganglios linfáticos cervicales agrandados. Las pruebas de laboratorio muestran una concentración de tiroglobulina sérica significativamente elevada en comparación con el nivel anterior hace 6 meses. ¿Qué opción explica el aumento de tiroglobulina en este paciente?
    A. Efecto adverso del tratamiento con yodo radioactivo
    B. Recurrencia del cáncer de tiroides
    C. Exceso en la reposición de hormona tiroidea
    D. Retroalimentación negativa aumentada que genera aumento de TSH en la hipófisis
A

Respuesta correcta: B

Tema e ítem: Cáncer de tiroides- recurrencia

Argumento: Recordemos que la tiroglobulina (Tg) es el precursor de las hormonas tiroideas activas (T3 y T4) y es producida por tejido tiroideo normal o cáncer de tiroides diferenciado (es decir, papilar, folicular). La mayor parte de la Tg se almacena en la glándula tiroides, pero parte se libera a la circulación. Los pacientes como este que se han sometido a una tiroidectomía total y tratamiento con yodo radiactivo no deberían tener tejido tiroideo residual normal; por lo tanto, es probable que un nivel de Tg en aumento en estos pacientes se deba a un cáncer de tiroides diferenciado recurrente. Por otro lado, recordemos que los pacientes que se han sometido a una tiroidectomía por cáncer de tiroides diferenciado requieren suplementos de levotiroxina (T4) por 2 razones:
● La levotiroxina reemplaza la función de la glándula tiroides.
● La levotiroxina suprime la liberación pituitaria de TSH (retroalimentación negativa). Debido a que la TSH estimula el crecimiento del tejido tiroideo, la suplementación con levotiroxina para suprimir la TSH (es decir, provocando un estado levemente hipertiroideo) puede ayudar a prevenir la recurrencia del cáncer de tiroides.
En algunos pacientes, también se puede obtener un nivel de Tg “estimulado” para evaluar la recurrencia. Esta prueba mide el nivel de Tg después de retirar la suplementación con levotiroxina (para aumentar la liberación hipofisaria de TSH) o después de administrar TSH recombinante. Si hay cáncer recurrente, el aumento de TSH provocará una mayor liberación de Tg de las células cancerosas.

En cuanto a la opción “A”, debemos tener en cuenta que el yodo radiactivo provoca la destrucción de las células tiroideas. Debido a que el yodo puede ser absorbido por otras células, los efectos adversos incluyen sialadenitis (infección de las glándulas salivales), boca seca y fibrosis pulmonar, pero los niveles de Tg no se verían afectados. Por otra parte, en cuanto a las dos últimas opciones (C y D), se esperaría que el reemplazo excesivo de la hormona tiroidea (por levotiroxina), conduce a una mayor retroalimentación negativa a la pituitaria, generando una reducción del nivel de TSH, en lugar de aumentar. Incluso en pacientes con probable cáncer de tiroides recurrente que tengan aumentado el nivel de Tg.

How well did you know this?
1
Not at all
2
3
4
5
Perfectly
39
Q
  1. Masculino de 68 años, consultó por fiebre, malestar general, dolor en hipocondrio y flanco derecho. Refiere que hace 6 meses fue hospitalizado por síntomas similares y le diagnosticaron una infección del tracto urinario; por E.coli, la cual recibio manejo antibiotico. Durante el año pasado, el paciente experimentó micción nocturna, vacilación y goteo postmiccional, pero no hematuria macroscópica. Sin antecedentes relevantes. Signos vitales 38,7 ºC, PA de 130/80 mm/Hg. Al examen fisico, se nota gravemente enfermo, con sensibilidad el ángulo costovertebral derecho. Paraclínicos con Leucocitos 16,800/mm3, Creatinina 1.2 mg/dL, uroanalisis con hematuria, piuria y bacteriuria. ¿Cuál de las siguientes es la causa más probable de la condición actual del paciente?
    A. Hiperactividad del detrusor
    B. Alto volumen residual posmiccional
    C. Deficiencia selectiva de IgA
    D. Cálculos de Staghorn 
A

Respuesta correcta: B

Tema e ítem: Pielonefritis aguda - Diagnostico

Argumento: En este paciente, la presencia de fiebre, sensibilidad del ángulo costovertebral, leucocitosis y un urianálisis con bacteriuria, piuria y hematuria (debido a la inflamación del tracto urinario) son compatibles con una pielonefritis aguda. Adicionalmente, la presencia de sintomas de vaciado como vacilación (otros son chorro lento, chorro dividido, esfuerzo) o sintomas de almacenamiento vesical como la nicturia (otros son polaquiuria, urgencia) hacen pensar de una hiperplasia prostatica benigna (HPB), y a su vez, el agrandamiento prostático periuretralesta comprime la uretra e impide la evacuación completa de la vejiga; lo que permite que la flora fecal que ha contaminado las vías urinarias prolifere en lugar de ser evacuada con la micción, favoreciendo finalmente el ascenso bacteriano a los riñones lo que provoca pielonefritis unilateral (o menos comúnmente bilateral). Por lo anterior, este paciente tiene un alto volumen residual posmiccional debido su HPB, y esto a su vez, favorece una ITU que comprometio ya los riñones.

En cuanto a la hiperactividad del detrusor puede causar polaquiuria, urgencia, nocturia e incontinencia, siin embargo, los síntomas de vaciado (disminución de la fuerza del chorro, goteo) y las infecciones urinarias recurrentes son atípicos, porque no se produce obstrucción de la salida. Por otra parte, la deficiencia selectiva de IgA puede estar asociada con infecciones del tracto respiratorio (sinusitis, neumonía) y gastrointestinales (giardiasis), pero el riesgo de infección del tracto urinario generalmente no aumenta. Finalmente, los cálculos coralinos son causados por organismos productores de ureasa (p. Proteus, Klebsiella) y generalmente se presentan con dolor en el costado y hematuria. Escherichia coli no es un organismo productor de ureasa. Además, la presencia de síntomas miccionales hace que la HPB subyacente con residuos posmiccionales elevados sea el factor desencadenante más probable.

How well did you know this?
1
Not at all
2
3
4
5
Perfectly
40
Q
  1. Masculino de 38 años, consultó por 12 horas de una erección persistente y dolorosa sin estimulación sexual. No refiere traumatismo en la región perineal. En los últimos meses, el paciente también ha tenido fatiga. No tiene antecedentes médicos relevantes. Signos vitales con Tº de 36,7 ºC, PA de 120/78 mm/Hg y FC de 100 lpm. Al examen físico, no hay linfadenopatía, pero hay hepatoesplenomegalia. El pene está erecto, está sensible y tiene congestión venosa. El resto del examen sin anormalidades. El paciente recibe el tratamiento adecuado, con reducción de los síntomas. ¿Cuál es la prueba diagnóstica indicada para identificar la causa del cuadro clínico?
    A. Se debe realizar un test de monospot
    B. Realizar un hemoleucograma completo y un extendido de sangre periférica
    C. Realizar un electrocardiograma
    D. Descartar lesión de médula espinal
A

Respuesta correcta: B

Tema e ítem: Priapismo - Diagnóstico

Argumento: Este paciente que presentó una erección prolongada y dolorosa en ausencia de estimulación sexual continua, confirma el diagnóstico de un priapismo, el cual es causado por la activación parasimpática, que induce la dilatación de las arterias cavernosas, aumentando el flujo sanguíneo hacia los cuerpos cavernosos; lo que comprime las venas emisarias contra la túnica albugínea, bloqueando la salida de sangre. La estimulación simpática subsiguiente normalmente contrae el flujo arterial e induce la contracción del músculo liso cavernoso, lo que lleva a la detumescencia; sin embargo, la entrada persistente o la salida obstruida pueden provocar priapismo. La mayoría de los casos de priapismo son idiopáticos; sin embargo, el priapismo se puede observar en trastornos hematológicos que causan alteración de la viscosidad de la sangre o microtrombos locales, lo que conduce a una disminución del flujo de salida a través de las venas emisarias. Los trastornos comunes incluyen enfermedad de células falciformes, neoplasias malignas hematológicas (leucemia mielógena crónica), talasemia y mieloma múltiple. En este paciente con priapismo que adicionalmente se presenta con fatiga y hepatoesplenomegalia, se sugiere una posible malignidad hematológica; por ende, se debe solicitar un hemoleucograma completo y un extendido de sangre periférica.

En cuanto a la solicitud de un electrocardiograma, no es necesaria para la valoración de rutina para paciente con priapismo; no obstante, es importante mencionar algunos autores recomienda el electrocardiograma debido al uso de fenilefrina para el tratamiento de priapismo, debido a que puede causar hipertensión y bradicardia refleja. Por otra parte, la mononucleosis infecciosa puede causar hepatoesplenomegalia, pero no alteración de la viscosidad de la sangre ni priapismo. Además, este paciente no tiene otras características de mononucleosis (faringitis, linfadenopatía); por ende, no es necesario realizar un monospot test. Finalmente, los trastornos de la médula espinal (lesión traumática, metástasis espinal) pueden causar priapismo debido a la interrupción de la inervación autonómica; sin embargo, esto rara vez ocurre como un hallazgo aislado; la mayoría de los pacientes tienen anomalías neurológicas adicionales (retención de orina, shock espinal).

How well did you know this?
1
Not at all
2
3
4
5
Perfectly
41
Q
  1. Mujer de 31 años, G2P1, 30 sem de gestación. Consulta por una gran hemorragia vaginal desde hace 2 h. Su primer hijo nació en casa, vía vaginal, sin complicaciones. Monitorización materno-fetal normal. Eco TV: placenta previa completa. Tras estabilizar, el sangrado para. Usted comenta la opción de cesárea, pero ella se niega, quiere el parto en su casa. ¿Cuál es la respuesta más adecuada?
    A. “Sólo le recomendaré un parto por cesárea, porque el parto vaginal pone en riesgo su vida y la de su bebé”.
    B. “Un parto vaginal sería muy peligroso, pero si entiende y acepta estos riesgos, le permitiré dar a luz en casa”.
    C. “Puedes tener un parto vaginal, pero te recomiendo que vengas al hospital para que podamos ayudarte si algo va mal”.
    D. “Podemos volver a hablar de esto dentro de unos días, cuando haya tenido más tiempo para sopesar sus opciones con su pareja”.
A

Respuesta correcta: A

Tema e ítem: hemorragias durante la gestación - Manejo

Argumento: La mayoría de las decisiones médicas se toman mediante una toma de decisiones compartida, en la que se tienen en cuenta las preferencias y valores personales del paciente cuando se discuten ≥2 opciones de tratamiento médicamente razonables. Sin embargo, cuando sólo hay una opción de tratamiento razonable desde el punto de vista médico que tiene un apoyo claramente superior basado en la evidencia, es éticamente apropiado que el médico proporcione asesoramiento directivo, en el que se recomienda al paciente una única opción de tratamiento.

Esta paciente tiene una placenta previa completa, una afección en la que toda la placenta cubre el cuello uterino, que se asocia a un alto riesgo de morbilidad y mortalidad materna (hemorragia) y fetal (muerte intrauterina) con el parto vaginal. Debido a estos riesgos, la única opción segura para el parto es la cesárea.

Aunque esta paciente prefiere un parto vaginal, es inapropiado que el médico discuta esta elección como una opción segura y razonable. El médico tiene la responsabilidad ética de aconsejar explicando por qué el parto por cesárea es médicamente necesario.

Decidir con la ayuda de su pareja es un ejemplo de toma de decisiones asistida, en la que la familia o los cuidadores ayudan al paciente a tomar decisiones. La toma de decisiones asistida no es necesaria para esta paciente porque su preferencia o la de su pareja en cuanto al modo de parto no cambia la necesidad del parto por cesárea.

La placenta previa puede causar una hemorragia vaginal intensa inesperada, a menudo no provocada, como en la presentación inicial de esta paciente. El riesgo de hemorragia repetida es mayor tras un episodio inicial de hemorragia, y la toma de decisiones sobre el parto no debe retrasarse en caso de que se produzca otra hemorragia potencialmente mortal.

How well did you know this?
1
Not at all
2
3
4
5
Perfectly
42
Q
  1. Mujer de 19 años, G1P0, sin control prenatal, 24 sem de gestación. Tiene dolor abdominal en hemiabdomen derecho desde antier. Tiene náuseas y anorexia. Tiene contracciones no dolorosas, sin hemorragia ni flujo. SV: T 39°C, PA 116/70 mmHg y FC 120/min. Dolor en flanco derecho, sin irritación peritoneal. Cuello cerrado y membranas íntegras. ¿Cuál es el diagnóstico más probable?
    A. Torsión ovárica.
    B. Infección intraamniótica.
    C. Apendicitis aguda.
    D. Pielonefritis aguda.
A

Respuesta correcta: D

Tema e ítem: Pielonefritis aguda-Diagnóstico

Argumento: Las mujeres embarazadas tienen un mayor riesgo de padecer pielonefritis aguda debido al aumento de la estasis urinaria, la inmunosupresión leve y la dilatación ureteral relacionada con la progesterona. Como la mayoría de los pacientes, las embarazadas con pielonefritis suelen tener náuseas, fiebre alta y dolor en el costado. La fiebre y la taquicardia maternas también provocan taquicardia fetal. En la exploración, la sensibilidad del ángulo costovertebral es frecuente por inflamación retroperitoneal, que puede extenderse a la cavidad abdominopélvica y causar síntomas obstétricos, como irritabilidad uterina (contracciones no dolorosas) que pueden progresar a parto prematuro (contracciones dolorosas con dilatación cervical). La pielonefritis grave también puede causar sepsis materna, síndrome de distrés respiratorio agudo y compromiso fetal (parto prematuro, bajo peso al nacer). Debido al alto riesgo de complicaciones graves, el tratamiento de la pielonefritis aguda en el embarazo es con hospitalización y antibióticos empíricos (ceftriaxona).

La apendicitis aguda en el embarazo puede presentarse con fiebre, náuseas, anorexia y dolor en el lado derecho o superior del abdomen debido al desplazamiento uterino del apéndice en sentido cefálico; pero suele causar signos peritoneales como rebote o guardia, que no se observan en esta paciente.

La infección intraamniótica (IIA) es una causa frecuente de dolor abdominal en el embarazo. La IIA se define como fiebre materna (sin un foco claro) más taquicardia materna o fetal, líquido amniótico purulento o leucocitosis. Aunque esta paciente tiene fiebre y taquicardia materna, la IIA es un diagnóstico de exclusión y esta paciente tiene sensibilidad en el flanco que hace más probable una pielonefritis aguda. Además, el útero no sensible, el cuello uterino cerrado y las membranas intactas de esta paciente hacen que la IIA sea menos probable.

La torsión ovárica puede cursar con dolor abdominal unilateral, anorexia y fiebre baja (peritonitis); sin embargo, suele comenzar como un dolor intermitente seguido de un dolor intenso de aparición súbita compatible con una oclusión vascular completa. No suele asociarse a sensibilidad en el flanco.

How well did you know this?
1
Not at all
2
3
4
5
Perfectly
43
Q
  1. Mujer de 39 años, G1P0C0A0, 7 sem de gestación. Presenta manchado marrón oscuro y calambres uterinos leves desde ayer. Con fatiga y vómitos diarios por 1 sem. Examen físico: PA 112/68 mmHg y FC 86/min. Abdomen blando, sin irritación peritoneal, escasa sangre oscura en vagina y cuello cerrado. ECO pélvico: signo de lambda presente. ¿Cuál es el diagnóstico más probable?
    A. Mola hidatiforme completa.
    B. Gestación gemelar.
    C. Aborto espontáneo incompleto.
    D. Gestación anembrionada.
A

Respuesta correcta: B

Tema e ítem: Ultrasonografía en embarazo gemelar

Argumento: El manchado vaginal y los calambres abdominales leves son frecuentes durante el primer trimestre y suelen ser benignos en pacientes con el cuello uterino cerrado y una gestación intrauterina en la ecografía fetal. Esta paciente tiene un embarazo gemelar, en la ecografía. Entre los factores de riesgo de gestación gemelar se incluyen el aumento de la edad materna, las terapias para aumentar la fertilidad (inducción de la ovulación), el aumento de la paridad y los antecedentes familiares. El aumento de las náuseas y los vómitos en el primer trimestre pueden ser indicadores precoces de un embarazo gemelar y probablemente se deban a niveles elevados de hCG y progesterona. Los gemelos se clasifican en función de la corionicidad (número de placentas) y la amnionicidad (número de sacos amnióticos); esto se evalúa mejor durante el primer trimestre.

Los gemelos monocoriónicos-monoamnióticos (1 placenta y 1 saco amniótico) se diagnostican por la ausencia de una membrana divisoria entre los gemelos. Los gemelos monocoriónicos-diamnióticos (1 placenta y 2 sacos amnióticos) se diagnostican por un signo T, que se produce cuando ambas capas de amnios se unen en la placenta compartida en un ángulo de 90 grados. Las gestaciones dicoriónicas-diamnióticas (2 placentas y 2 sacos amnióticos) se presentan como 2 placentas visualmente separadas o la apariencia de una placenta fusionada identificada por una gruesa membrana entrelazada y una proyección placentaria donde la membrana entrelazada se encuentra con la placenta (signo lambda), son la gestación gemelar más frecuente y menos complicada.

Una gestación anembrionada, un embarazo sin desarrollo embrionario, puede presentarse con manchado vaginal. La ecografía suele revelar un saco gestacional vacío (saco gestacional ≥25 mm sin embrión ni saco vitelino).

Una mola hidatiforme completa se presenta clásicamente con un aspecto de queso suizo o tormenta de nieve en la ecografía en lugar de con 2 fetos.

Un aborto espontáneo incompleto se presenta con hemorragia vaginal y dilatación cervical con productos visibles de la concepción en el cuello uterino o la vagina. El cérvix de esta paciente está cerrado.

How well did you know this?
1
Not at all
2
3
4
5
Perfectly
44
Q
  1. Mujer de 33 años, G1P0, 27 sem de gestación. Sin controles prenatales. Hace unas semanas tuvo fiebre y una erupción maculopapular difusa, no pruriginosa por 2 días. ECO más actual: feto con ventriculomegalia bilateral y calcificaciones intracraneales en ganglios basales y en la unión sustancia gris-blanca, ascitis y hepatoesplenomegalia. PFE debajo del percentil 5. ¿Cuál es el agente causal?
    A. Toxoplasma gondii.
    B. Parvovirus B19.
    C. Listeria monocytogenes.
    D. Virus del herpes simple.
A

Respuesta correcta: A

Tema e ítem: Toxoplasmosis-Diagnóstico

Argumento: Los hallazgos ecográficos de esta paciente son sugestivos de toxoplasmosis congénita. La infección materna por Toxoplasma gondii puede producirse por el consumo de alimentos contaminados poco cocinados o por el contacto con las heces de un gato infectado, que es el huésped definitivo de los ooquistes del protozoo. Aunque suelen ser asintomáticos, los adultos pueden presentar fiebre, linfadenopatías, mialgias y/o una erupción maculopapular difusa, no pruriginosa, que se resuelve espontáneamente, como se observa en este paciente. Aunque la infección materna es leve, la transmisión transplacentaria puede provocar una infección fetal devastadora. T. gondii destruye preferentemente el tejido neural fetal, lo que se traduce en hallazgos ecográficos de ventriculomegalia bilateral (hidrocefalia) y calcificaciones parenquimatosas intracraneales difusas, especialmente en los ganglios basales. Otros hallazgos fetales asociados a la toxoplasmosis congénita son inespecíficos y pueden incluir ascitis, hepatoesplenomegalia y retraso del crecimiento fetal. Las pacientes deben someterse a una amniocentesis para realizar una prueba de PCR de T gondii en el líquido amniótico para confirmar el diagnóstico. El tratamiento antitoxoplasma prenatal (pirimetamina, sulfadiazina, ácido folínico) puede disminuir el riesgo de secuelas neurológicas como coriorretinitis, convulsiones y discapacidad intelectual.

En la infección por virus del herpes simple las madres suelen presentar úlceras genitales dolorosas en lugar de una erupción difusa. Además, los hallazgos ecográficos comunes incluyen calcificaciones de la placenta y el cordón umbilical, no del parénquima cerebral.

La infección materna por L. monocytogenes suele cursar con gastroenteritis febril. La transmisión vertical suele causar sepsis en el periodo neonatal, que suele causar abscesos diseminados, no calcificaciones intracraneales.

El parvovirus B19 puede causar una enfermedad leve con una erupción difusa en adultos, y puede producir ascitis debido a hidropesía fetal. Sin embargo, otras acumulaciones anormales de líquido (derrame pericárdico/pleural) son típicas, y las calcificaciones intracraneales no se asocian a esta enfermedad.

How well did you know this?
1
Not at all
2
3
4
5
Perfectly
45
Q
  1. Mujer, 32 años, G1P0C0A0, 21 sem de gestación. Lleva 1 sem con flujo vaginal, era claro, ahora sanguinolento. No hay dolor abdominal. Examen físico: T 36,8°C, PA 96/62 mmHg y FC 92/min. El útero no es sensible, FC fetal de 155/min. Una bolsa abultada con partes fetales sobresale a través del cuello uterino en 4 cm de dilatación. Prueba de nitrazina (-). ¿Cuál es el diagnóstico más probable?
    A. Parto prematuro.
    B. Aborto incompleto.
    C. Insuficiencia cervical.
    D. Ruptura prematura de membranas.
A

Respuesta correcta: C

Tema a item: Insuficiencia cervical-Diagnóstico

Argumento: Esta paciente presenta insuficiencia cervical, una debilidad estructural del cuello uterino que provoca una dilatación cervical espontánea e indolora y una posible pérdida del embarazo en el segundo trimestre. Los factores de riesgo incluyen anomalías del colágeno (síndrome de Ehlers-Danlos), anomalías uterinas (útero tabicado o bicorne), traumatismo obstétrico previo y conización cervical.

Las pacientes con insuficiencia cervical, incluso las que presentan una dilatación cervical avanzada, suelen tener sólo síntomas leves como aumento del flujo vaginal (debido a la pérdida del tapón mucoso), hemorragia vaginal leve (debido al estiramiento cervical) y presión pélvica (debido a que las partes fetales distensionan la parte superior de la vagina).

Las pacientes pueden ser diagnosticadas en la exploración, que revela una dilatación cervical visible y ausencia de contracciones uterinas. Estas pacientes suelen ser tratadas con un cerclaje. Sin embargo, en pacientes con membranas amnióticas abultadas o prolapsadas, no suele recomendarse un cerclaje de rescate debido al alto riesgo de rotura de membranas, complicaciones (infección intraamniótica, traumatismo cervical) y probable parto inminente.

Un aborto incompleto puede presentarse con dilatación cervical y partes fetales en el orificio cervical; sin embargo, se define como pérdida del embarazo en <20 semanas de gestación, mientras que esta paciente está en la semana 21. Además, suele ser doloroso debido a los calambres uterinos. En futuros embarazos, el tratamiento de las pacientes con un aborto espontáneo previo suele ser expectante; en cambio, las que cumplen los criterios de insuficiencia cervical pueden beneficiarse de un cerclaje profiláctico.

En contraste con la presentación indolora de esta paciente, las pacientes en trabajo de parto prematuro suelen tener contracciones uterinas regulares y dolorosas que conducen a la dilatación cervical.

La rotura prematura de membranas antes del parto puede causar un aumento del flujo vaginal claro o ligeramente sanguinolento, pero la prueba de nitrazina suele ser positiva.

How well did you know this?
1
Not at all
2
3
4
5
Perfectly
46
Q
  1. Femenina de 45 años, con historia de diabetes mellitus tipo 2 e hipertensión arterial de larga data. Fue hospitalizada hace 2 días por síndrome hiperosomolar hiperglicemico que requirió intubación orotraqueal. Medicamentos: metformina, amlodipino. En su tercer día de hospitalización presenta mejoría del cuadro clínico y es extubado pero su estado de conciencia fluctúa, además de no seguir órdenes y no tolerar la vía oral. Le realizaron varios estudios de laboratorio y el ionograma demostró hipernatremia (Na+: 153 mEq/L). ¿Cuál de los siguientes es el tratamiento correcto para esta paciente?
    A. DAD 5% IV
    B. SSN 0.45 % IV
    C. Hidratación por sonda nasogástrica
    D. SSN 0.9% IV
A

Respuesta correcta: C

Tema e ítem: Manejo de trastornos hidroelectrolíticos.

Argumento: Recuerda que el síndrome hiperglicémico hiperosmolar (SHH) es una complicación aguda de la diabetes mellitus tipo 2, el cual se manifiesta como una hiperglicemia grave y debe tratarse de forma temprana y agresiva para evitar las complicaciones potencialmente mortales de la deshidratación y el desequilibrio electrolítico. La glucosuria grave provoca una diuresis osmótica y una grave depleción de volumen, que puede verse exacerbada por las pérdidas gastrointestinales. La condición puede progresar hasta el letargo y la muerte. El tratamiento incluye una fuerte hidratación e insulinoterapia endovenosa.

Este paciente desarrolló hipernatremia debido al déficit de agua secundario a la diuresis osmótica inducida por la glucosuria. Además, el tratamiento de su hiperglicemia con insulina probablemente causó una disminución de la osmolalidad extracelular, haciendo que el agua se mueva hacia las células, exacerbando más la deshidratación. Por cada disminución de 100 mg/dL en la glicemia, el sodio aumentará aproximadamente 2,0 mEq/L. El déficit hídrico de este paciente debe ser corregido con reposición de volumen a través de una sonda gástrica por su estado confuso, de lo contrario, podría hacerse por vía oral si la tolera. Aunque las disminuciones de sodio sérico de hasta 1,0 mEq/L/h no parecen tener efectos adversos, la mayoría de las directrices recomiendan NO disminuir el Na+ no más de 10-12 mEq/L en 24 horas con el objetivo de disminuir el riesgo de edema cerebral. Debido a las continuas pérdidas de agua, la tasa de corrección calculada será algo menor que la tasa real.

How well did you know this?
1
Not at all
2
3
4
5
Perfectly
47
Q
  1. Mujer de 20 años, procedente del municipio de Magangué-Bolivar, es evaluada por un historial de pérdida de la consciencia de 40-45 segundos de duración y posterior temblor generalizado desde hace 10 meses. Refiere que la mayoría de los episodios ocurren en bipedestación, y experimenta pródromos que van desde ‘’visión de tunel’’, hipoacusia, mareos, náuseas y palidez. La madre refiere que durante cada episodio observa ‘’sacudidas’’ intermitentes en todas las extremidades, cada una de ellas de forma independiente. Pasado 45 segundos, la paciente despierta alerta, orientada y con capacidad de continuar su día con normalidad. El examen físico es anodino. ¿Cuál de los siguientes es el diagnóstico más probable?
    A. Crisis tónico-clónica.
    B. Crisis mioclónica.
    C. Crisis atónica.
    D. Síncope convulsivo.
A

Respuesta correcta: D

Tema e ítem: Síncope – diagnósticos diferenciales.

Comentario: El síncope se define como una pérdida completa y transitoria de la conciencia y del tono postural debida a una hipoperfusión cerebral global y PUEDE seguir patrones estereotipados (al igual que las convulsiones o las migrañas). Su aparición es brusca, con una recuperación espontánea, rápida y completa. La causa más común de síncope es el síncope mediado neuralmente, que generalmente ocurre con la bipedestación prolongada o la deshidratación y tiene un pródromo de náuseas, aturdimiento, calor, visión de túnel o palpitaciones.

Mientras que la actividad motora durante un episodio de perdida de la conciencia a menudo sugiere un diagnóstico de crisis convulsiva, la actividad motora breve, incluida la extensión tónica del tronco y las extremidades o varias sacudidas clónicas, puede ocurrir en un síncope no complicado. La combinación de actividad motora similar a una convulsión en el contexto de un síncope a veces se denomina ‘’síncope convulsivo’’. Es por lo anterior que los episodios estereotipados con signos de advertencia pueden confundirse con las crisis convulsivas, pero la duración de la pérdida de conciencia suele ser más breve en el síncope (<1 minuto) que, en las convulsiones, y la recuperación neurológica inmediata y completa es típica. Las mioclonías no epilépticas (como las de este paciente) y otros tipos de movimientos y temblores son típicos del síncope convulsivo; de hecho, el síncope sin movimientos es la excepción más que la regla.

How well did you know this?
1
Not at all
2
3
4
5
Perfectly
48
Q
  1. ¿Cuál de las siguientes opciones es una contraindicación ABSOLUTA de lactancia materna?
    A. Uso activo de cannabis.
    B. Infección crónica por VHB.
    C. Omeprazol.
    D. Uso activo de cocaína
A

Respuesta correcta: D

Tema e ítem: Contraindicaciones de la lactancia materna.

Comentario: Algunas drogas recreativas como la cocaína-fenciclidina se excretan por la leche materna y pueden producir toxicidad aguda y efectos negativos en el desarrollo neuroconductual del lactante a largo plazo, por lo que representan una contraindicación absoluta. Otras contraindicaciones absolutas de la lactancia materna incluyen infección activada por tuberculosis no tratada, infección por VIH (excepto en países con recursos limitados, en los que puede que no se disponga de fórmula), lesiones herpéticas en mama, infección activa por varicela, quimioterapia o radioterapia, galactosemia y fenilcetonuria.

Aunque el cannabis también es detectable en la leche materna, La AAP refiere que efectos adversos en los bebés lactantes son limitados. Sin embargo, aunque su consumo no está contraindicado, se desaconseja encarecidamente durante la lactancia. También se debe recomendar de forma práctica a las madres que se abstengan de fumar cualquier tipo de tabaco, debido al mayor riesgo de síndrome de muerte súbita del lactante.

How well did you know this?
1
Not at all
2
3
4
5
Perfectly
49
Q
  1. Mujer de 25 años. Consulta por lesiones en la vulva de 2 semanas de evolución, no dolorosa. Examen físico: se objetiva 3 ulceras de márgenes elevados, base eritematosa y fondo limpio en introito vaginal, de 6-8 mm de diámetro. No hay linfadenopatia inguinal asociada. Laboratorios: ELISA para VIH (-), VDRL no reactivo. ¿Cuál de los siguientes es el diagnóstico y tratamiento más probable?
    A. Sífilis primaria, penicilina benzatínica 2.400.000 U IM DU.
    B. Herpes genital, aciclovir 400 mg c/8h/7 días VO.
    C. Linfogranuloma venéreo, doxiciclina 100 mg VO cada 12 horas por 21 días.
    D. Granuloma inguinal, azitromicina 1 g VO c/7 días/3 semanas
A

Respuesta correcta: D

Tema e ítem: Ulceras genitales – enfoque.
Argumento: La presencia de úlceras genitales múltiples, induradas e indoloras, con una base eritematosa y márgenes elevados, junto con la ausencia de linfadenopatía inguinal, sugieren un granuloma inguinal (donovanosis), causado por Klebsiella granulomatis. Las lesiones comienzan en forma de nódulos (granulomas subcutáneos) en el lugar de inoculación tras un periodo de incubación muy variable (mediana de 50 días) antes de ulcerarse. También pueden desarrollarse lesiones ulcerosas en el periné.

El organismo causante del granuloma inguinal es difícil de cultivar. El diagnóstico requiere la visualización de cuerpos de Donovan en el campo oscuro de una biopsia. El tratamiento de elección se realiza con azitromicina 1 g VO semanal, por 3 semanas o hasta que la úlcera sane. Se debe dar tratamiento a las parejas sexuales de los 60 días previos al inicio de los síntomas.

How well did you know this?
1
Not at all
2
3
4
5
Perfectly
50
Q
  1. Niña de 5 años es llevada a consulta por desatención y múltiples quejas de sus maestros de escuela. La madre refiere que la niña no completa sus tareas y no sigue instrucciones de sus profesores. Se niega a hablar con padres y compañeros. Prefiere jugar sola con 5 carros de color rojo, colocándolos en línea recta. La niña evita el contacto visual con su madre y el médico en toda la visita. El examen físico y neurológico es normal. ¿Cuál es el diagnóstico más probable?
    A. Desorden hiperactivo y déficit de atención.
    B. Desorden del espectro autista.
    C. Discapacidad auditiva.
    D. Comportamiento adecuado para la edad.
A

Respuesta correcta: 5

Tema e ítem: Trastorno del espectro autista.

Argumento: Aunque la presentación clínica varía ampliamente, el trastorno del espectro autista se caracteriza por deterioro en la interacción social y patrones restrictivos de comportamiento o intereses. Los problemas de comunicación, la falta de atención, la negativa al hablar y hacer contacto visual, la fijación y el apego intenso a ciertos objetos y acciones repetidas son comportamientos que se observan típicamente en pacientes con trastorno del espectro autista. El manejo debe iniciar temprano, implica manejo educativo y conductual (Ej. Entrenamiento de competencia de habilidades sociales y habilidades de comunicación), asesoramiento familiar y terapia médica.

  • Los pacientes con desorden hiperactivo y déficit de atención suelen presentar dificultad para prestar atención y tienen un bajo rendimiento académico. Sin embargo, el desorden hiperactivo y de déficit de atención se acompaña de impulsividad e hiperactividad, que no están presentes en este paciente.
  • Los niños con discapacidad auditiva puede parecer que no prestan atención o no responden cuando se les habla. Sin embargo, este diagnóstico no explicaría las otras características de este paciente: negativa para hablar o hacer contacto visual, apego intenso por ciertos objetos y repetición de acciones específicas. Este paciente no tiene antecedentes de factores de riesgo para hipoacusia infantil: otitis infecciosa, malformaciones, obstrucciones o ingesta de fármacos ototóxicos.
  • Los niños con desarrollo normal del lenguaje hablan en oraciones de tres palabras a los 3 años y cuentan historias complejas a los 4 años. El comportamiento de este paciente no es normal para un niño de 5 años. Ser distraído, ignorar lo que dicen los demás y preferir jugar solo puede ser apropiado para la edad hasta cierto punto. Sin embargo, la evitación del contacto visual, la negativa a hablar y la frecuente repetición de acciones específicas de este paciente apuntan a un diagnóstico diferente.
How well did you know this?
1
Not at all
2
3
4
5
Perfectly
51
Q
  1. Paciente de 3 años. Llevado por su abuela a urgencias por presentar desde hace aproximadamente 12 horas, 3 deposiciones con sangre. Sin vómito. No ha presentado fiebre. Al examen físico, con temperatura de 37°C y con resto de signos vitales sin alteraciones, de buen aspecto, sin signos de deshidratación, leve dolor en hipogastrio a la palpación. Tolera vía oral. ¿Cuál consideraría el manejo más apropiado para este paciente?
    A. Es una disentería, se debe iniciar antimicrobiano empírico.
    B. Iniciar sales de rehidratación vía oral y dejar en observación.
    C. Iniciar hidratación intravenosa por la contraindicación a la vía oral.
    D. Dar signos de alarma y cita de revisión en 48 horas.
A

Respuesta correcta: D

Tema-Ítem: Enfermedad diarréica aguda – Evaluación del grado de hidratación.

Este paciente no tiene signos de deshidratación, ni está en riesgo de deshidratación. No tiene fiebre, ni signos clínicos de sepsis, por lo que no se requiere iniciar antimicrobiano. No tiene ninguna contraindicación para recibir líquidos vía oral.

La diarrea es la segunda causa de mortalidad en menores de cinco años en el mundo. Por este motivo es fundamental conocer el tratamiento adecuado.
Cuando nos enfrentamos a la población pediátrica, lo más importante es el triángulo de valoración: Evaluar apariencia, respiración y circulación. Esto nos puede definir si un niño se encuentra en sepsis o shock, en cuyo caso debemos iniciar bolos de líquidos y cubrimiento antimicrobiano inmediato.

Si el paciente no está en sepsis o shock, debemos evaluar posteriormente los signos de deshidratación: Que se encuentre alerta, ojos normales (no hundidos), con lágrimas, boca y lengua húmedas y que beba líquidos de forma normal o sin sed (que no beba ávidamente). Si presenta dos o más de estos signos, tiene algún grado de deshidratación y se debe iniciar el plan B.

Si el paciente no tiene signos de deshidratación, el paso siguiente es preguntarnos si tiene riesgo de deshidratación: diarrea de alto gasto (>4 deposiciones en 4 horas o >10 en 24 horas), vómito persistente (>3 vómitos en una hora o >5 vómitos en 4 horas) o rechazo a la vía oral. Si presenta uno de esto signos, el paciente debe iniciar el plan A supervisado, que consta de:
⇛ Reponer las pérdidas por vía oral con suero de rehidratación oral e iniciar alimentación ⇛ Observar respuesta y revalorar
⇛ Si tolera vía oral y es posible mantener hidratación en casa, dar salida con recomendaciones de PLAN A
⇛ Si persiste alto gasto, emesis o rechaza la vía oral por completo, realizar tratamiento en la institución.

Si no se encuentra en alto riesgo de deshidratación, podemos iniciar el plan A, con sales de rehidratación oral: en <2 años 50-100ml y en >2 años 100-200 ml.

Se consideran líquidos adecuados:
-Sales de Rehidratación Oral
-Líquidos preparados mediante cocción de cereales en grano o molidos (arroz, maíz o cebada) y cocinados en agua, o aquellos que se preparan con papa o plátano, o arroz*
Agua, siempre y cuando, se combine con la dieta habitual, que incluya alimentos con almidón (papa, yuca, ñame o maíz), preparados de preferencia con sal.

How well did you know this?
1
Not at all
2
3
4
5
Perfectly
52
Q
  1. El paciente del cuadro clínico anterior, es llevado de nuevo a urgencias por su abuela, debido a que ha iniciado con fiebre de 39.3°C, persiste con deposiciones con sangre (lleva 3 deposiciones hoy) y ahora con vómito (2 episodios hoy). Al examen físico, febril con 39.5°C, irritable, sin signos de deshidratación y sin aspecto tóxico. ¿Cuál es el paso a seguir?
    A. Iniciar líquidos intravenosos en bolo por shock séptico
    B. Iniciar cubrimiento antimicrobiano empírico con ceftriaxona
    C. Iniciar cubrimiento antimicrobiano empírico con azitromicina
    D. Iniciar plan B de hidratación
A

Respuesta correcta: B.

Tema-Ítem: Enfermedad diarréica aguda – Disentería grave

Argumentación: Este paciente >3 meses presenta fiebre >39°C + sangre en heces (disentería) lo que lo clasifica en disentería grave y por esto es necesario iniciar antimicrobiano.

La disentería es la sangre en las heces. Esto puede ser ocasionado más comúnmente por bacterias como Shigella o Salmonella. Algunos virus también podrían ocasionar disentería.
Cuando nos enfrentamos a un paciente con disentería, posterior al triángulo de valoración pediátrica y durante el examen físico, es fundamental evaluar presencia de fisuras anales que puedan explicar la presencia de heces con sangre y así evitar el uso indiscriminado de antimicrobianos que aumenta la resistencia bacteriana y altera la microbiota, además de predisponer a enfermedad diarréica por Clostridium difficile.

En un paciente con disentería, sin otros signos de alarma, no se debe administrar antimicrobiano. Sin embargo, si nos encontramos frente a una disentería grave, es necesario iniciar cubrimiento antibiótico empírico con cefotaxima o ceftriaxona, además de realizar coproscópico y coprocultivo. Los criterios para inicio de antimicrobiano son:
Sangre en heces y uno de los siguientes:
● Fiebre >38°C en menor de 3 meses o >39°C si es > de 3 meses
● Signos generales de alarma
● Compromiso del estado general
● Dolor abdominal intenso
● Distensión abdominal
● Factores de riesgo para bacteriemia (inmunosupresión, enfermedades crónicas de tracto gastrointestinal, hemoglobinopatías, desnutrición moderada o severa).

Posterior al inicio de antimicrobiano empírico, es importante dirigirlo de acuerdo a la bacteria encontrada.

How well did you know this?
1
Not at all
2
3
4
5
Perfectly
53
Q
  1. Matías, tiene 1 año y es llevado a su consulta por la madre, quien refiere que está preocupada porque presenta infecciones respiratorias cada mes y en lo que va del año, lleva seis episodios. Algunas se han presentado con fiebre que mejora con acetaminofén. En el momento, se encuentra sano. El paciente no presenta antecedentes, tiene PAI completo. Al examen físico, con signos vitales adecuados, índices antropométricos en rangos de normalidad. ¿Qué le realizaría a este paciente?
    A. No enviaría ningún estudio, explicar a la madre
    B. Sospecharía una inmunodeficiencia por encontrarnos frente a una fiebre recurrente
    C. Haría un hemoleucograma para evaluar neutropenia cíclica
    D. Es importante descartar una enfermedad reumatológica, enviaría perfil completo
A

Respuesta correcta: A.

Tema – Ítem: Fiebre recurrente en pediatría

Argumentación: Este paciente presenta cada mes un episodio de infección, probablemente viral, que no ha requerido hospitalización ni manejo antimicrobiano, lo cual se encuentra dentro de los parámetros normales, por lo que no requiere ningún estudio. Es importante explicar muy bien a los padres y dar signos y síntomas de alarma.

Fiebre recurrente se define como 3 o más episodios de fiebre objetiva, separados por intervalos libres de síntomas de al menos dos semanas, en un período mayor o igual a seis meses. La causa más frecuente son infecciones virales autolimitadas a repetición, que en la primera infancia (lactante mayor y preescolar) pueden alcanzar hasta 12-15 por año. Un porcentaje pequeño de estos pacientes tendrá una enfermedad autoinmune o inflamatoria, una neoplasia o una inmunodeficiencia.

Los signos de alarma para sospechar una inmunodeficiencia primaria, son:
1. ≥6-8 otitis medias agudas en un año.
2. ≥2 sinusitis graves en un año.
3. ≥2 meses con antibiótico con escasos resultados.
4. ≥2 neumonías en un año.
5. Retraso ponderoestatural.
6. Abscesos cutáneos profundos o viscerales recurrentes.
7. Aftas orales recurrentes en boca o infecciones micóticas en >1 año de edad.
8. Necesidad de antibiótico intravenoso para la curación de infecciones.

Dentro de las inmunodeficiencias, se encuentra el síndrome de neutropenia cíclica, un síndrome neutropénico que suele heredarse de forma autosómica dominante asociándose a una mutación del gen ELANE. Esta mutación ocasiona la interrupción periódica de la producción celular mieloide en la médula ósea.

El cuadro clínico típico se caracteriza por episodios de fiebre recurrente y periódica cada 3 semanas, con una duración de 3-7 días, asociada a úlceras orales e inflamación orofaríngea. Se inicia desde el primer año de vida y característicamente pueden asociar celulitis perianal. En el hemograma detectamos una neutropenia <200/µl durante 3-5 días, con nadir antes del inicio de la fiebre. Entre los episodios los individuos se encuentran asintomáticos, las cifras de neutrófilos se normalizan y el desarrollo es normal.
Para el diagnóstico se requieren hemogramas seriados, al menos 3 veces por semana (lunes, miércoles y viernes), durante 4-6 semanas, para confirmar el patrón cíclico.

How well did you know this?
1
Not at all
2
3
4
5
Perfectly
54
Q
  1. Una madre lleva a su hijo a la consulta porque le preocupa que no ha gateado. El paciente tiene 8 meses, nació a término con una edad gestacional de 39 semanas por parto vaginal, con adaptación neonatal temprana y sin complicaciones. Al examen físico, sin retraso pondoestatural, se sienta por sí solo, hace pinza gruesa y se desliza sobre sus glúteos y usa sus brazos para avanzar. ¿Qué le diría a la madre?
    A. Se deben hacer estudios adicionales para descartar retraso en el neurodesarrollo
    B. Tranquiliza a la madre y le da signos de alarma
    C. Se debe hacer seguimiento periódico más frecuente para vigilar evolución
    D. Realizar exoma para descartar enfermedades neurogenéticas
A

Respuesta correcta: B.

Tema – Ítem: Crecimiento y desarrollo – Hitos del neurodesarrollo

Argumentación: Este paciente de 8 meses, tiene una variante de gateo que se considera normal (sentado). Además, tiene los otros hitos del neurodesarrollo que deben cumplirse entre los 8-9 meses (pinza gruesa, sentarse por sí solo sin ayuda). Por lo tanto, se debe continuar con los controles habituales, sin realizar estudios adicionales.

Las variaciones en el neurodesarrollo que no se consideran patológicas, son:
1. Pinza manual entre el dedo pulgar y el medio
2. Variantes del gateo: Mano-pie, rodilla-pie, arrastrándose en decúbito ventral (como un soldado), sentado, utilizando las 3 extremidades
3. Marcha de pie sin pasar por la etapa de gateo (18% de los niños)
4. Retraso simple de la marcha con signo de “sentarse en el aire”
5. Marcha de puntillas, en las primeras semanas o meses después del inicio de la deambulación
6. Tartamudeo fisiológico entre 2-4 años
7. Dislalia fisiológica hasta los 4-5 años

How well did you know this?
1
Not at all
2
3
4
5
Perfectly
55
Q
  1. Paciente de 18 meses llevado al servicio de urgencias por fatiga desde ayer. Al examen físico, afebril, hipotenso, taquicárdico y taquipnéico. Palidez mucocutánea, con ictericia escleral bilateral. Se palpan varios ganglios linfáticos cervicales anteriores de 1 cm. A la auscultación, se evidencia un soplo de eyección sistólico de 2/6. Dolor a la palpación en hipocondrio izquierdo, con esplenomegalia a nivel del ombligo. Sin hepatomegalia. Los pulsos periféricos bilaterales débiles. ¿Cuál de las siguientes ayudas diagnósticas podría establecer el diagnóstico de este paciente?
    A. Electroforesis de proteína
    B. Biopsia de ganglios linfáticos
    C. Ecocardiografía
    D. Actividad enzimática de glucosa 6 fosfato deshidrogenasa
A

Respuesta correcta: A

Tema-Ítem: Enfoque de anemia – Anemia de células falciformes

Argumentación: Este paciente se encuentra inestable hemodinámicamente por sus signos vitales alterados y por su alteración en el triángulo de valoración pediátrica. Presenta esplenomegalia con dolor en hipocondrio derecho y signos de anemia aguda (palidez, fatiga, soplo cardíaco). Lo más probable es que esta presentación represente una crisis de secuestro esplénico aguda asociada con la enfermedad de células falciformes.

La anemia de células falciformes (ACF) es una enfermedad que se establece a partir de una mutación en el gen de la beta globina secundario a un cambio durante la transcripción genética, que genera la codificación de valina en lugar de ácido glutámico.

El secuestro esplénico es una complicación potencialmente mortal de la enfermedad de células falciformes que puede ser la característica de presentación en algunos niños. Debido a que los glóbulos rojos falciformes (GR) quedan atrapados en los senos esplénicos, se acumula un gran volumen de sangre en el bazo. El bazo aumenta de tamaño con rapidez (es decir, se produce esplenomegalia dolorosa) y la hemoglobina cae de forma aguda desde el valor inicial, lo que provoca fatiga, palidez, taquicardia e incluso shock (p. ej., hipotensión, pulso débil). Casi siempre se encuentra trombocitopenia debido al secuestro esplénico y la producción de reticulocitos en la médula ósea aumenta en respuesta a la disminución de los glóbulos rojos circulantes. La ictericia escleral refleja hemólisis subyacente. La electroforesis de hemoglobina que muestra la presencia de hemoglobina S (HbS) (mutación del gen de la globina beta) confirma el diagnóstico de enfermedad de células falciformes. La polimerización de HbS en su estado desoxigenado hace que los glóbulos rojos adopten una forma de hoz.

How well did you know this?
1
Not at all
2
3
4
5
Perfectly
56
Q
  1. Femenina de 41 años presenta cuadro clínico caracterizado por disnea grave, tos y hemoptisis. Refiere haber visitado a su padre recientemente a la india (El cual tiene los mismos síntomas). Una radiografía de tórax evidencia fibrosis en lóbulo superior derecho y una muestra de esputo es positiva para bacilos acidorresistentes. Se realiza el Dx de tuberculosis pulmonar activa y se inicia isoniazida, rifampicina, etambutol y pirazinamida. Un mes después en una cita de control refiere sentirse mejor. Una nueva prueba de esputo es negativa para bacilos acidorresistentes. Reporta los siguientes paraclínicos: Albúmina 4.8g/dL, BT: 0.9, BD: 0.2, FA: 42 U/L, ATS 75 U/L, ALT 97 U/L. ¿Cuál es el manejo adecuado para esta paciente?
    A. Continuar con el tratamiento actual y programar una nueva cita de control.
    B. Reemplazar el tratamiento actual por un régimen antituberculoso de segunda línea.
    C. Suspender el tratamiento antituberculoso.
    D. Realizar una biopsia percutánea de hígado.
A

Respuesta correcta: A

Tema e ítem: Manejo a largo plazo de la tuberculosis.

Argumento: La isoniazida es un antibiotico bactericida particularmente efectivo contra Mycobacterium tubercolosis al usarse con otros agentes como la rifampicina, el etambutol y la pirazinamica. Sin embargo, este antibiotico tiene efectos adversos asociandose con hepatotoxicidad, que puede manifestarse de forma leve a grave. Los casos de hepatitis grave se presentan con manifestaciones clínicas muy similares a los casos de hepatitis viral: fatiga, náuseas, síntomas similares a la gripe, ictericia, aminotrasnferasas >10 veces el limite superior normal. Se estima que el riesgo de desarrollar esta forma de hepatitis es del 2.6% para aquellos pacientes con consumo diario de alcohol, con enfermedad hepatica previa o que tienen más de 50 años de edad. Si se observan estos sintomas graves, el medicamento debe suspenderse de inmediato. En contraste, un 10 a 20% de pacientes que toman isoniazida experimentarán una lesión hepática subclínica leve demostrada con elevaciones pequeñas de aminotransferasas. El pronostico de estos pacientes es bueno y la condición es autolimitada, lo que permite continuar con el manejo antibitotico bajo vigilancia estrecha. Esta paciente es relativamente joven y se encuentra asintomática, por lo tanto podemos continuar con la isoniazida realizando pruebas de función hepática de forma regular.

  • En caso de hepatotoxicidad significativa con el régimen antibiotico inicial se deben reemplazar los medicamentos antituberculosos de primera línea por medicamentos de segunda línea. Este no es el caso.
  • Aunque el esputo de la paciente es negativo para bacilos acidorresistentes, no es probable que haya eliminado por completo los bacilos latentes después de un mes de tratamiento.
  • Una biopsia de hígado está indicada en pacientes con elevación asintomática de las enzimas hepáticas de etiología desconocida, ya que el examen de tejido permitirá una mejor comprensión del proceso patológico involucrado. Sin embargo, las elevaciones de aminotransferasa en este paciente probablemente se deban al tratamiento de la tuberculosis. Por lo tanto, no se indica ninguna biopsia en este momento.
How well did you know this?
1
Not at all
2
3
4
5
Perfectly
57
Q
  1. Masculino de 31 años presenta cefalea y confusión progresiva desde hace 2 semanas. Tiene antecedente de infección por VIH y no tiene buena adherencia a tratamiento antirretroviral. Tampoco ha recibido manejo profiláctico. Signos vitales: temp. de 38°C, PA de 118/80 mmHg y FC de 102 lpm. Al examen físico el paciente se ve descuidado y desorientado. Hay ataxia y hemiparesia derecha. Una RM de cerebro evidencia lesiones con realce anular. ¿Cuál de los siguientes medicamentos hubiera prevenido la condición actual de este paciente?
    A. Albendazol.
    B. Fluconazol.
    C. Ganciclovir.
    D. Trimetropin-sulfametoxazol.
A

Respuesta correcta: D

Tema e ítem: Toxoplasmosis en pacientes con VIH.

Argumento: Este paciente con VIH tiene síntomas neurológicos, fiebre y evidencia de lesiones con realce anular en la neuroimagen realizada, lo que debe generar una alta sospecha de toxoplasmosis. En la mayoría de ocasiones se debe a la reactivación de T. gondii debido a deficiencias graves en la inmunidad celular como lo es el estado del SIDA con recuentos de CD4 menores a 100/mm3. La profilaxis primaria contra la toxoplasmosis es eficaz para prevenir la afección en pacientes con SIDA avanzado. Por lo tanto, todo paciente con VIH debe someterse a una evaluación de laboratorio para detectar la exposición (IgG de toxaplasma) y aquellos con serología positiva y recuento de CD4 menor a 100/mm3 requerirán profilaxis primaria con trimetropin-sulfametoxazol. Los pacientes en manejo antirretroviral generalmente pueden interrumpir la profilaxis cuando el recuente de CD4 sea mayor a 200/mm3 durante 3 meses y existe una supresión viral adecuada. Además, el trimetropin-sulfametoxazol se usa como profilaxis contra neumonía por Pneumocystis en pacientes con CD4 menores a 200/mm3.

How well did you know this?
1
Not at all
2
3
4
5
Perfectly
58
Q
  1. Femenina de 85 años de edad presenta disnea de esfuerzos y nicturia desde hace 6 meses los cuales han empeorado progresivamente. Tiene antecedente de HTA por lo que toma lisinopril. Signos vitales: PA de 150/78 mmHg, FC de 82 lpm y FR de 28 rpm. Presión venosa yugular de 10 cm H2O. Al examen físico destaca leve distensión abdominal sin dolor, edema 2+ con fóvea en miembros inferiores. Paraclínicos: Péptido natriurético cerebral de 776 (Normal <100 pg/mL). Uroanálisis con proteína 1+. Rx de tórax normal. ¿Cuál es el diagnostico más probable?
    A. Desacondicionamiento.
    B. Insuficiencia cardiaca.
    C. Enfermedad pulmonar intersticial.
    D. Síndrome nefrótico.
A

Respuesta correcta: B

Tema e ítem: Diagnostico de la insuficiencia cardiaca.

Argumento: Este paciente presenta nicturia y disnea de esfuerzo progresiva, además con evidencia de sobrecarga de volumen, lo que sugiere una insuficiencia cardíaca descompensada. La elevación del péptido natriurético cerebral fortalece este diagnóstico clínico. La causa de la disnea en la insuficiencia cardiaca descompensada es multifactorial e implica más que solo el edema pulmonar: aumento de la demanda ventilatoria, disfunción muscular periférica y ventilatoria. Por lo tanto, la ausencia de edema pulmonar en este paciente no excluye el diagnóstico de insuficiencia cardiaca porque los pacientes con descompensación progresiva lenta pueden tener edema pulmonar mínimo o nulo. La cardiomegalia puede estar ausente si la insuficiencia cardiaca se debe a hipertrofia ventricular izquierda (en el contexto de hipertensión crónica) que produce disfunción diastólica (llenado alterado).

  • Aunque el desacondicionamiento puede causar disnea de esfuerzo, no explicaría el aumento de la presión venosa, el edema periférico o el BNP elevado.
  • La enfermedad pulmonar intersticial generalmente se presenta con tos seca y disnea de esfuerzo. También puede provocar aumento de presión yugular y edema periférico (por presión venosa central elevada causada por hipertensión pulmonar). Sin embargo, la auscultación pulmonar generalmente revela crepitantes finos y se espera que la radiografía de tórax muestre un aumento de las marcas pulmonares intersticiales.
  • El síndrome nefrótico generalmente se presenta con proteinuria y edema periférico prominente como resultado de la pérdida de albúmina y otras proteínas en la orina. Puede ocurrir disnea con el esfuerzo, pero no se esperan aumento de la presión yugular ni BNP elevado porque la hipoalbuminemia causa depleción del volumen intravascular.
How well did you know this?
1
Not at all
2
3
4
5
Perfectly
59
Q
  1. Masculino de 72 años consulta por dolor intenso en su pierna izquierda que inició con sensación de hormigueo hace varias horas, pero que desde entonces se ha vuelto doloroso. Tiene antecedente de HTA, DM2, FA paroxística y osteoartritis. Fue fumador de 45 paquetes-año. Signos vitales: PA de 142/80 mmHg y FC de 112 lpm e irregular. Al examen físico, la piel de la extremidad inferior izquierda aparece moteada y fría al tacto. Los pulsos distales de la pierna izquierda no son palpables. ¿Qué medicamento prevendría la condición actual de este paciente?
    A. Apixaban.
    B. Aspirina.
    C. Clopidogrel.
    D. Metoprolol.
A

Respuesta correcta: A

Tema e ítem: Anticoagulación en pacientes con Fibrilación Auricular.

Argumento: Este paciente con dolor agudo en las extremidades inferiores, parestesia y falta de pulso tiene isquemia aguda en las extremidades. Dados los antecedentes de fibrilación auricular paroxística del paciente y pulso irregular en el examen, la causa más probable es la embolia de un trombo en la aurícula izquierda, que podría haberse evitado con un tratamiento anticoagulante adecuado. Las guías actuales recomiendan el uso de la puntuación CHA2DS2-VASc para la evaluación del riesgo tromboembólico en pacientes con FA. Los pacientes masculinos con una puntuación CHA2DS2-VASc ≥2, como este paciente (puntuación = 3: hipertensión, diabetes mellitus, edad 65-74), tienen un alto riesgo de eventos tromboembólicos y deben tratarse con terapia anticoagulante. En estos pacientes, el beneficio de la reducción del riesgo de embolización supera el riesgo de hemorragia (hemorragia intracraneal). El apixabán es un anticoagulante oral no antagonista de la vitamina K (NOAC) que inhibe directamente el factor Xa, evitando que escinda la protrombina en trombina. Los NOAC (apixabán, dabigatrán, rivaroxabán, edoxabán) reducen significativamente el riesgo de embolización sistémica con una eficacia similar o superior a la de la warfarina, pero con menor riesgo de hemorragia grave (accidente cerebrovascular hemorrágico) y sin necesidad de pruebas de INR de rutina. Por el contrario, la terapia antiplaquetaria con aspirina o una combinación de aspirina y clopidogrel es significativamente menos efectiva para reducir el riesgo tromboembólico en comparación con la terapia anticoagulante con NOAC o Warfarina.

How well did you know this?
1
Not at all
2
3
4
5
Perfectly
60
Q
  1. Masculino de 55 años se presenta para un control de rutina. Refiere sentirse bien. Tiene antecedente de HTA de larga data por lo que toma enalapril e hidroclorotiazida. Signos vitales: PA de 140/90 mmHg y FC de 80 lpm. Al examen físico hay sobrepeso moderado (IMC de 27) con una circunferencia abdominal de 104 cm. Paraclínicos: Glucosa en ayunas de 112 mg/dL, colesterol total de 220 mg/dl, LDL de 140 mg/dL, triglicéridos 240 mg/dL. ¿Cuál es el factor patogénico más importante para esta condición?
    A. Secreción inadecuada de insulina.
    B. Resistencia a la insulina.
    C. Hiperactividad simpática.
    D. Vasodilatación mediada por insulina.
A

Respuesta correcta: B

Tema e ítem: Síndrome X – Síndrome metabólico.

Argumento: Este caso clínico describe a un paciente con la presentación clínica típica del síndrome metabólico, que incluye hipertensión, alteración de la glucosa en ayunas y dislipidemia. Los pacientes también tienen un sobrepeso característico (como se ve en este caso), con una distribución de grasa predominantemente central (abdominal) que se refleja en un aumento de la relación cintura-cadera. La resistencia a la insulina juega un papel central en la patogenia del síndrome metabólico. El síndrome metabólico se diagnostica cuando se cumplen al menos 3 de los 5 criterios siguientes:
- Obesidad abdominal (Hombres: Circunferencia de la cintura >40 pulgadas; Mujeres: Circunferencia de la cintura >35 pulgadas)
- Glucosa en ayunas >100 - 110 mg/dL
- Presión arterial > 130/80 mm Hg
- Triglicéridos >150 mg/dL
- Colesterol HDL (Hombres: <40 mg/dL; Mujeres: <50 mg/dL)

How well did you know this?
1
Not at all
2
3
4
5
Perfectly
61
Q
  1. Masculino de 55 años, consulta por dolor rectal agudo de 2 horas de evolución, de inicio súbito mientras estaba teniendo una evacuación intestinal. El paciente refiere el dolor como insoportable e implacable. Niega fiebre, dolor abdominal, hematoquezia o pérdida de peso inesperada. Antecedente de estreñimiento crónico. Signos vitales Tº 37,5 ºC, la PA de 125/75 mm/Hg, FC de 75 lpm. Al examen físico el abdomen no es doloroso, y el tacto rectal revela un bulto violáceo muy doloroso justo debajo de la línea dentada. ¿Cuál es el siguiente paso en el manejo de este paciente?
    A. Realizar hemorroidectomía bajo anestesia local.
    B. Realizar ligadura con banda elástica.
    C. Ordenar ablandadores de heces y antiespasmódicos.
    D. Ordenar ablandadores de heces, antiespasmódicos y antibióticos.
A

Respuesta correcta: A

Tema e ítem: Hemorroide anorrectal externa trombosada - Manejo

Argumento: Este paciente con dolor anorrectal severo y un bulto anal violáceo debajo de la línea dentada, generan la alta sospecha de una hemorroide externa trombosada (En ocasiones, la trombosis puede ser visible como un cordón palpable dentro de la propia hemorroide). Aunque el manejo conservador se recomienda en casos leves, la hemorroidectomía externa está indicada para aquellos con casos graves de dolor. Esta cirugía, que se puede realizar bajo anestesia local, se realiza haciendo una escisión elíptica en la piel que rodea la hemorroide, permitiendo su completa extracción. Si se retrasa la cirugía, se puede lograr un alivio temporal mediante una incisión hemorroidal y eliminación de trombos; sin embargo, la recurrencia es común, por lo que se debe realizar una hemorroidectomía eventualmente. Recordemos que las hemorroides se forman como resultado de la hipertensión venosa dentro del plexo venoso hemorroidal. La inflamación intravascular puede provocar inflamación de la mucosa ulceración y finalmente trombosis. A su vez, pueden ser hemorroides externas o internas.

En cuanto al uso de ligadura con banda elástica y escleroterapia, se reserva el manejo de hemorroides internas sintomáticas no trombosadas que han fracasado al tratamiento conservador, y en cuanto Estas técnicas se evitan con las hemorroides externas, porque la ubicación debajo de la línea dentada, está altamente inervada y estos procedimientos causarían grave dolor. Por otra parte, en pacientes con dolor leve y no debilitante, las trombosis externas de las hemorroides pueden resolverse con un tratamiento conservador que consiste en ablandadores de heces, baños de asiento, antiinflamatorios y antiespasmódicos tópicos (supositorios de glucocorticoides, crema de nitroglicerina). Este paciente tiene un dolor insoportable, lo que requiere intervención quirúrgica inmediata. Finalmente, los antibióticos están indicados para los abscesos perirrectales, pero no indican para el tratamiento de hemorroides trombosadas.

How well did you know this?
1
Not at all
2
3
4
5
Perfectly
62
Q
  1. Masculino de 48 años, ingresa a urgencias, posterior a herida por arma cortopunzante en región abdominal. No sufrió otras lesiones. Signos vitales PA de 114/68 mm/Hg y FC de 118 lpm. Al examen físico, murmullo vesicular conservado de forma bilateral, sonidos cardiacos normales sin agregados. El examen abdominal muestra una lesión que no sangra en el cuadrante superior izquierdo que mide aproximadamente 2,5 × 1 cm. El abdomen está rígido y presenta sensibilidad difusa. El tacto rectal no revela sangre. Ya se inició reanimación con líquidos. ¿Cuál es el siguiente paso en el manejo de este paciente?
    A. Realizar una tomografía computarizada de abdomen con contraste intravenoso.
    B. Realizar lavado peritoneal diagnóstico.
    C. Realizar laparotomía exploratoria.
    D. Realizar una exploración local de la herida.
A

Respuesta correcta: C

Tema e ítem: Trauma abdominal penetrante anterior - Manejo

Argumento: Este paciente tiene un traumatismo abdominal penetrante (TAP) (es decir, una herida por arma blanca) en el cuadrante superior izquierdo, con posible lesión de los órganos subyacentes (estómago, intestino delgado, bazo). Además de la taquicardia (es decir, posible shock hemorrágico temprano), el paciente tiene peritonitis. (rigidez, sensibilidad difusa). Lo que debe indicar una posible lesión intraabdominal con hemorragia en curso; por ende, se justifica realizar una laparotomía exploratoria inmediata. Recordemos en pacientes con TAP las indicaciones de exploración por laparotomía exploratoria inmediata:

  • Evidencia de hemorragia en curso: inestabilidad hemodinámica (presión arterial sistólica <90 mm Hg) o sangre franca en la sonda nasogástrica o en el recto.
  • Peritonitis (rigidez abdominal, dolor de rebote).
  • Evisceración (es decir, intestinos expuestos externamente).
  • Empalamiento (es decir, objeto penetrante todavía in situ).

Por el contrario, los pacientes sin indicación de laparotomía inmediata deben someterse a exámenes para determinar si se produjo penetración peritoneal (tomografía computarizada o exploración local de la herida) y/o si las lesiones intraabdominales están presentes (evaluación enfocada con ecografía para traumatismos, tomografía computarizada). En caso de encontrarse en estos exámenes lesión, se justifica ya la exploración quirúrgica. En cuanto a realizar un lavado peritoneal diagnóstico, este no se realiza comúnmente, pero puede ayudar a descartar lesiones intraabdominales en pacientes con TAP que no tienen indicación (peritonitis) de laparotomía inmediata; sin embargo, recordemos este paciente tiene signos de peritonitis. Finalmente, en pacientes con heridas punzantes anteriores aisladas, se puede realizar una exploración local de la herida con anestesia local para buscar violaciones del peritoneo (visualización del contenido intraabdominal), lo que indicaría una exploración quirúrgica; sin embargo, la peritonitis de este paciente implica penetración peritoneal con lesión intraabdominal y justifica una intervención quirúrgica.

How well did you know this?
1
Not at all
2
3
4
5
Perfectly
63
Q
  1. Masculino de 35 años, previamente sano, consulta por dolor abdominal grave, durante las últimas 4 horas, el cual inició en el área periumbilical y se ha desplazado al cuadrante inferior derecho. Se acompaña de náuseas y emesis. Signos vitales Tº de 38,3 ºC, PA de 126/80 mm/Hg, FC de 100 lpm. Al examen físico, dolor a la palpación en el cuadrante inferior derecho que no empeora con la inspiración, Signo de Rovsing presente. Uronanálisis normal. Hemoglobina 14,2 g/dL, Plaquetas 220.000, leucocitos 16.000, y neutrófilos 86%. ¿Qué estudio se debe realizar inicialmente en este paciente?
    A. Realizar una tomografía computarizada de abdomen y pelvis.
    B. No es necesario solicitar imágenes adicionales.
    C. Realizar radiografías abdominales en decúbito supino y lateral.
    D. Realizar un ultrasonido del abdomen.
A

Respuesta correcta: A

Tema e ítem: Apendicitis-Diagnóstico

Argumento: Este paciente con dolor migratorio en el cuadrante inferior derecho, náuseas, emesis, fiebre, signo de Rovsing positivo y leucocitosis son preocupantes para una apendicitis aguda (Este paciente tiene 9 puntos en la escala de Alvarado, lo que se interpreta como certeza de apendicitis).

Recordemos que históricamente, la sospecha de apendicitis se consideraba suficiente para realizar la apendicectomía; sin embargo, al proceder de esta manera se daba una tasa de apendicectomía negativa (el apéndice extirpado no estaba inflamado en la patología); por ende, se plantea que para disminuir este tipo de situaciones se haga uso de ayudas diagnósticas por imagen. En el caso de este paciente, la indicada sería la tomografía computarizada (TC) del abdomen y la pelvis (con contraste), debido a que se trata de un adulto no embarazado. Además, la TC tiene una alta precisión diagnóstica y es menos probable que produzca un resultado indeterminado, junto a que depende menos del operador que la ecografía, a su vez es mejor tolerada, más barata y disponible que la resonancia magnética. Por último, un elemento importante de la tomografía, es que permite dirigir, según el reporte, enfocar el tratamiento: Las posibilidades del reporte por TC son Apéndice normal, Apendicitis no perforada, Apendicitis perforada.

En cuanto a realizar radiografías abdominales, pueden ser útiles para diagnosticar algunas afecciones intraabdominales (niveles hidroaéreos en la vista en decúbito lateral en el intestino en casos de obstrucción). Sin embargo, las radiografías simples no visualizan el apéndice y no pueden detectar inflamación apendicular.

Finalmente, para evitar la exposición a radiaciones ionizantes, la ecografía abdominal es la modalidad de imagen inicial. Se utiliza en niños y mujeres embarazadas, sin embargo, la tomografía computarizada del abdomen y la pelvis (con contraste) es la opción preferida.

How well did you know this?
1
Not at all
2
3
4
5
Perfectly
64
Q
  1. Femenina de 59 años, consulta por dos días de dolor abdominal sordo y persistente en el cuadrante inferior izquierdo, acompañado de náuseas y emesis. Además, refiere ha tenido urgencia urinaria leve, sin disuria y hematuria. Comentó no haber defecado los últimos 3 días, pero si refiere paso de flatos. Antecedente de un quiste ovárico. Signos vitales Tº de 38ºC, PA de 134/82 mm/Hg y FC de 94/min. Al examen físico, IMC de 33, dolor a la palpación en el cuadrante inferior izquierdo del abdomen sin rebote. No hay masa palpable ni organomegalia. Peristaltismo presente. Una tirilla reactiva en orina es positiva para esterasa leucocitaria y negativa para nitritos. ¿Cuál es el diagnóstico más probable?
    A. Cistitis bacteriana aguda.
    B. Diverticulitis aguda.
    C. Obstrucción intestinal.
    D. Ruptura de quiste ovárico.
A

Respuesta correcta: B

Tema e ítem: Diverticulitis- Diagnóstico

Argumento: Esta paciente que se presenta con dolor abdominal en cuadrante inferior izquierdo, náuseas, emesis, fiebre, síntomas vesicales (urgencia), piuria estéril (esterasa leucocitaria positiva con nitrito/bacterias negativas) y alteración de los hábitos intestinales (estreñimiento), nos deben hacer sospechar de un cuadro de diverticulitis aguda, una complicación de la diverticulosis. Finalmente, el diagnóstico suele realizarse mediante imágenes (tomografía computarizada del abdomen). Recordemos que la diverticulosis, se caracteriza por herniación de la mucosa/submucosa del colon a través de las vías circular y la capa muscular longitudinal, debido a la presión intraluminal elevada. El riesgo aumenta con la edad y es mayor en personas con obesidad, mala alimentación. Aunque la diverticulosis suele ser asintomática, aproximadamente el 10% de los pacientes van a desarrollar hemorragia diverticular y/o diverticulitis. En la diverticulitis, la presión luminal elevada o las partículas de comida espesas causan erosión de la pared diverticular, que conduce a micro o macroperforación y posterior inflamación colónica. La mayoría de los casos surgen en el colon sigmoide (el sitio de mayor concentración intraluminal presión).

En cuanto a la sospecha de una cistitis bacteriana aguda, se debe descartar porque no genera usualmente síntomas sistémicos (fiebre, emesis), tampoco genera dolor en un cuadrante específico y además, los nitritos urinarios suelen estar positivos. Es probable que los síntomas urinarios de esta paciente y su piuria estéril, se deban a la irritación de la vejiga secundaria a la inflamación adyacente del colón sigmoide. Por otra parte, los quistes ováricos rotos se presentan con mayor frecuencia con dolor abdominal severo y repentino, o dolor después de las relaciones sexuales. La gran mayoría de los casos ocurren en mujeres de edad reproductiva. Finalmente, la obstrucción del intestino delgado generalmente ocurre en personas que han tenido una cirugía abdominal o pélvica y han desarrollado adherencias o hernias postoperatorias. Además, los pacientes suelen tener paroxismos de dolor abdominal tipo cólico periumbilical (no sordo, persistente, dolor en el cuadrante inferior izquierdo) y a menudo no pueden expulsar flatos.

How well did you know this?
1
Not at all
2
3
4
5
Perfectly
65
Q
  1. Masculino de 67 años, consultó por náuseas, vómitos y dolor abdominal creciente en las últimas 3 horas en región periumbilical. Hace cuatro semanas fue dado de alta del hospital después de un ingreso hospitalario por infarto agudo de miocardio. Antecedente de DM2 e hipercolesterolemia. Signos vitales Tº de 37,8 Cº, la PA es de 150/90 mm/Hg, FC de 110 lpm. Al examen físico, ruidos intestinales disminuidos y presencia de dolor abdominal difuso a la palpación, pero es más pronunciado sobre la región periumbilical. Paraclínicos con pH con acidosis láctica, bicarbonato 14 mEq/L, BUN 25 mg/dL, Hemoglobina de 19 gr/dl, leucocitos en 12.000, Creatinina 1,1 mg/dL, Glucosa 185 mg/dL, Amilasa 275 U/L, Lipasa 80 U/L y uroanálisis normal. ¿Cuál es el diagnóstico más probable?
    A. Apendicitis aguda.
    B. Pancreatitis aguda.
    C. Isquemia intestinal.
    D. Cetoacidosis diabética.
A

Respuesta correcta: C

Tema e ítem: Isquemia intestinal (isquemia mesentérica aguda) - Diagnóstico

Argumento: Este paciente que presenta dolor periumbilical repentino, que aparentemente es generalizado, pero que se focaliza, que se acompaña de leucocitosis, hemoglobina elevada (hemoconcentración), elevación amilasa y acidosis metabólica debido al aumento de los niveles de lactato sérico (junto a un bicarbonato disminuido) más el antecedente de infarto reciente, nos hacen sospechar de una isquemia mesentérica aguda. Recordemos que esta entidad es más comúnmente ocasionada por una oclusión arterial abrupta, secundaria a eventos embólicos cardíacos en el contexto de fibrilación auricular, enfermedad valvular, enfermedad cardiovascular o aneurismas. Este paciente probablemente desarrolló un trombo ventricular izquierdo después de su reciente infarto; este trombo luego se embolizó hacia la arteria mesentérica y generó isquemia. Los pacientes con evidencia de infarto intestinal deben someterse a una evaluación quirúrgica inmediata; no obstante, es importante recordar que esta entidad puede confirmarse radiológicamente mediante angiografía por tomografía computarizada.

En cuanto a la apendicitis aguda, es poco común en pacientes de edad avanzada; si ocurre, la presentación es menos aguda. Además, en ausencia de perforación e hipotensión significativa, los pacientes con apendicitis rara vez desarrollan acidosis metabólica.

Por otra parte, la pancreatitis aguda (PA) puede presentarse con náuseas y dolor abdominal que se irradia hacia la espalda; no obstante, un resultado normal de lipasa hace que la PA sea mucho menos probable, ya que la lipasa es más sensible y específica que la amilasa.

Finalmente, la cetoacidosis diabética puede presentarse con manifestaciones gastrointestinales y acidosis, pero generalmente se desarrolla en pacientes con diabetes mellitus tipo 1, no tiende a causar dolor abdominal localizado, y sería poco probable con un nivel de glucosa <250 mg/dL.

How well did you know this?
1
Not at all
2
3
4
5
Perfectly
66
Q
  1. Femenina de 24 años, consulta por un historial de náuseas y vómitos de 1 semana de evolución. No refiere otros síntomas asociados. Antecedentes: enfermedad pélvica inflamatoria hace 2 años tratada con antibioterapia. Medicamentos: Anticonceptivos orales combinados. Examen físico: Tº 37.1, PA 116/77, FC 111 lpm, FR 16 lpm. El resto del examen físico es normal. La prueba de embarazo en orina es positiva. La β-hCG sérica es de 910 mIU/mL. Además de repetir la concentración sérica de B-hcG, ¿cuál de los siguientes es el siguiente paso más apropiado en el diagnóstico?
    A. Solicitar ecografía abdominal.
    B. Programar para dilatación + curetaje.
    C. Programar para laparotomía exploratoria emergente.
    D. Solicitar ecografía transvaginal en 3 días
A

Respuesta correcta: D

Tema e ítem: Determinar la ubicación del embarazo.

Argumento: En pacientes con sospecha clínica de embarazo ectópico, después de una prueba de embarazo positiva, la ubicación debe confirmarse mediante una ecografía transvaginal. Sin embargo, es posible que no detecte un embarazo intrauterino antes de las 5 semanas de gestación o antes de que la concentración de β-hCG sea >1500–2000 mIU/mL. Por lo tanto, en pacientes con β-hCG < 1000 mIU/mL (como en este caso) la ecografía transvaginal debe programarse para dentro de 2-3 días después de la prueba inicial.

También está indicado solicitar niveles séricos de β-hCG cada 2,5 días dentro de las primeras 4 semanas de embarazo con el objetivo de identificar aumentos en la concentración de β-hCG. De esta manera, tenemos que:

  • Aumento lento puede indicar un embarazo ectópico o un aborto.
  • Aumento rápido puede indicar un tumor secretor de β-hCG (Mola hidatiforme, coriocarcinoma) o embarazo gemelar
How well did you know this?
1
Not at all
2
3
4
5
Perfectly
67
Q
  1. Femenina de 24 años consulta porque tuvo relaciones sexuales hace 2 días con su pareja sin métodos anticonceptivos. Refiere que no desea quedar embarazada en este momento ya que se encuentra a 6 meses de terminar sus estudios universitarios, pero que le gustaría concebir una vez se gradúe. No hay alteraciones en su menstruación. Refiere alergia a metales de joyería. La prueba de embarazo es negativa. ¿Cuál es el manejo inmediato adecuado para esta paciente?
    A. Colocar un dispositivo intrauterino de cobre.
    B. Colocar un dispositivo intrauterino de progestágenos.
    C. Administrar acetato de ulipristal.
    D. Administrar acetato de medroxiprogesterona.
A

Respuesta correcta: C

Tema e ítem: Métodos anticonceptivos de emergencia.

Argumento: En caso de que las pacientes soliciten métodos anticonceptivos de emergencia pero que a su vez consideren un embarazo en un futuro cercano, el acetato de ulipristal oral o el levonorgestrel oral son la opción anticonceptiva de emergencia adecuada. El acetato de ulipristal tiene una eficacia de hasta el 99% para prevenir el embarazo si se administra dentro de 5 días posteriores a la relación sexual sin protección, y el lenonorgestrel tiene una eficacia de hasta el 98% si se administra dentro de los 3 días posterior a la relación sexual. Estos anticonceptivos orales de emergencia son más afectivos durante la primera mitad del ciclo menstrual y en pacientes con peso saludable.

  • Los dispositivos intrauterinos de cobre y levonorgestrel son más efectivos que los métodos anticonceptivos orales de emergencia, sin embargo, estos métodos no son adecuados en esta paciente que se encuentra considerando un embarazo en un futuro cercano. El DIU de cobre debería evitarse en pacientes con antecedentes sugestivos de alergia de cobre.
  • El acetato de medroxiprogesterona se usa ampliamente como método anticonceptivo a largo plazo, particularmente en pacientes que no desean tomar un anticonceptivo vía oral diario, ya que este se administra de forma IM cada 3 meses. No se usa como método anticonceptivo de emergencia.
How well did you know this?
1
Not at all
2
3
4
5
Perfectly
68
Q
  1. Femenina de 32 años G2P1 con embarazo de 32 semanas consulta para control prenatal. Refiere cefalea y mareos frecuentes. No tiene antecedentes de importancia y toma sus oligoelementos como han sido indicados. Signos vitales: temperatura de 37°C, FC de 90 lpm y PA de 170/100 mmHg. Al examen físico es normal para la edad gestacional. La FCF es de 155 lpm. Dentro de los paraclínicos destaca un recuento de plaquetas en 60.000/mm3. El resto de paraclínicos son normales. ¿Cuál es el manejo adecuado para esta paciente?
    A. Iniciar manejo con sulfato de magnesio y labetalol.
    B. Transfundir plaquetas.
    C. Iniciar manejo con lisinopril.
    D. Realizar parto por cesárea.
A

Respuesta correcta: A

Tema e ítem: Trastornos hipertensivos del embarazo.

Argumento: Esta paciente tiene hipertensión grave (PAS >160 mmHg) y trombocitopenia lo que indica una preeclampsia con características de severidad. El manejo de la preeclampsia con características de severidad está determinado por la edad gestacional, la estabilidad hemodinámica de la madre y/o el feto y las complicaciones en las funciones pulmonar, hepática o renal de la madre. Si la edad gestacional es <34 semanas y tanto la madre como el feto se encuentran estables, como en este caso, el manejo apropiado es el manejo expectante con tratamiento conservador. Además, un intento de retrasar la gestación hasta más allá de las 34 semanas aumentará la viabilidad del feto y, por lo tanto, es apropiado para esta paciente. El manejo conservador incluye la administración de sulfato de magnesio como profilaxis para la eclampsia, así como antihipertensivos como el labetalol. También debería administrarse dexametasona para inducir maduración pulmonar fetal, lo que reduciría el riesgo de síndrome de dificultad respiratoria neonatal, hemorragia intraventricular y muerte neonatal.

  • La transfusión de plaquetas se encuentra indicada en pacientes con preeclampsia si el recuento de plaquetas es menor a 10.000 o 20.000/mm3 en pacientes sin evidencia de sangrado. Si la paciente presenta sangrado activo, se encuentra indicado con un conteo de plaquetas menor a 50.000/mm3.
  • El lisinopril y otros IECA se encuJentran contraindicados durante el embarazo ya que aumenta el riesgo de complicaciones fetales.
  • El parto vaginal es preferible a la cesárea si la edad gestacional, el estado cervical y la condición materno-fetal han progresado lo suficiente, dado que se asocia a menor riesgo de complicaciones materna y fetales. Además, no se debe inducir al parto antes de las 34 semanas, excepto en caso de complicaciones. La preeclampsia con características graves no es una indicación de parto por cesárea.
How well did you know this?
1
Not at all
2
3
4
5
Perfectly
69
Q
  1. Femenina de 30 años G2P1 con embarazo de 31 semanas ingresa por trabajo de parto espontaneo. Durante el embarazo desarrolló anemia ferropénica e hipotiroidismo controlado. No tiene antecedentes de importancia. Signos vitales: FC de 90 lpm, FR de 17 rpm y PA de 130/80 mmHg. La paciente ha presentado 8 contracciones en la última hora. El cérvix tiene 3 cm de dilatación. El monitoreo fetal es normal. Se administra dexametasona e indometacina. ¿Cuál de los siguientes se debe incluir en el manejo de esta paciente?
    A. Cesárea de emergencia.
    B. Administrar inmunoglobulina anti-D.
    C. Administrar sulfato de magnesio.
    D. Administrar azitromicina profiláctica.
A

Respuesta correcta: C

Tema e ítem: Manejo del parto pretérmino.

Argumento: La neuroprotección fetal con sulfato de magnesio se encuentra indicada si se prevé un parto pretérmino < 32 semanas de gestación. Además, el sulfato de magnesio tiene actividad tocolítica y se puede administrar como agente único o en combinación con otro agente tocolítico, como la indometacina indicada hasta la semana 32, con el fin de retrasar el trabajo de parto hasta 48 horas. Este retraso permite la acción de los corticoides prenatales para inducir la madurez pulmonar fetal. La administración concomitante de sulfato de magnesio y un bloqueador de canales de calcio debe evitarse por su alto riesgo de efectos secundarios.

  • Esta paciente tiene un embarazo de 31 semanas y no existe un riesgo inmediato para muerte materna o fetal, la principal prioridad es retrasar el trabajo de parto para permitir la administración de corticoides y brindar profilaxis contra la parálisis cerebral.
  • Por lo general, la inmunoglobulina anti-D se administra a las 28 semanas de gestación y 72 horas posteriores al nacimiento. No nos dan datos sobre la hemoclasificación materna como para tomar esta conducta.
  • Los antibióticos se encuentran indicados en pacientes con trabajo de parto pretérmino y un estado de infección para S. del grupo B desconocido como forma de profilaxis contra la infección neonatal. Sin embargo, la penicilina G y la ampicilina son agentes de primera línea más apropiados en este caso que la azitromicina.
How well did you know this?
1
Not at all
2
3
4
5
Perfectly
70
Q
  1. Femenina de 40 años consulta porque desea usar un método anticonceptivo. Actualmente es sexualmente activa con una pareja y no han utilizado anticonceptivos. Refiere ser fumadora desde los 15 años. Es alérgica al látex y al cobre. Una prueba de embarazo es negativa. ¿Cuál de los siguientes anticonceptivos no consideraría utilizar en esta paciente?
    A. Acetato de medroxiprogesterona.
    B. Dispositivo intrauterino.
    C. Condones.
    D. Píldora anticonceptiva oral combinada.
A

Respuesta correcta: D

Tema e ítem: Anticonceptivos – Contraindicaciones.

Argumento: La edad y el antecedente de tabaquismo de esta paciente aumentan su riesgo de tromboembolismo venoso, particularmente si se usa este método de anticoncepción. Las píldoras anticonceptivas orales combinadas no se recomiendan en pacientes de ≥35 años que fuman debido al riesgo de coagulopatía. El estrógeno se encuentra asociado con la coagulopatía ya que aumenta las concentraciones plasmáticas de varios factores de la coagulación y de fibrinógeno. La edad avanzada y el tabaquismo también alteran la coagulación y la circulación. Tampoco se aconseja el uso de anticonceptivos combinados en pacientes con enfermedades cardiovasculares (Enfermedad de arterias coronarias, trombosis venosa profunda, ACV e hipertensión arterial), trastornos metabólicos (Ej: diabetes insulinodependiente) tumores dependientes de estrógenos, lupus eritematoso sistémico y/o vasculitis.

  • El acetato de medroxiprogesterona es una progestina inyectable que se administra cada 3 meses. Debido a que esta carece de estrógenos, puede usarse en pacientes mayores de 35 años que fuman.
  • Aunque esta paciente debe evitar los condones de látex dada su alergia, podría usar condones que no sean de látex. Con un uso constante y correcto, los condones tienen una tasa mínima de fallas. Además, reducen el riesgo de infecciones de transmisión sexual.
  • Aunque esta paciente informa alergia al cobre y no sería candidata para un dispositivo intrauterino (DIU) de cobre, aún podría recibir un DIU de progestina. Los DIU de progestina brindan anticoncepción reversible de acción prolongada durante 3 a 5 años, según el tipo, y también son apropiados para fumadoras mayores de 35 años.
How well did you know this?
1
Not at all
2
3
4
5
Perfectly
71
Q
  1. Femenina de 51 años consulta para realizarse una prueba para detección de cáncer de colon. Niega alguna sintomatología en específico. Tiene antecedente de hipertensión arterial y su madre desarrolló cáncer de colon a los 85 años. Sus signos vitales y examen físico se encuentran dentro de lo normal. La paciente se realiza una colonoscopia en el que se encuentran y extirpan 2 pólipos hiperplásicos de aprox. 8mm. ¿Cuál es el manejo adecuado para esta paciente?
    A. Realizar una nueva colonoscopia en 1 año.
    B. Realizar una nueva colonoscopia en 10 años.
    C. Realizar una tomografía computarizada de abdomen.
    D. Realizar una sangra oculta en heces en 6 meses.
A

Respuesta correcta: B

Tema e ítem: Cáncer colorrectal – Tamizaje.

Argumento: Esta paciente tiene pólipos hiperplásicos, una anomalía no neoplásica que a menudo se encuentra en las colonoscopias. Los pólipos hiperplásicos pequeños (<1 cm) no aumentan el riesgo de cáncer de colon y, por lo tanto, los pacientes pueden continuar con las pruebas de detección colorrectales en los intervalos habituales:
- Realizar nueva colonoscopia en 10 años para personas con un riesgo promedio.

  • Repetir nueva colonoscopia en 5 años para personas con antecedentes familiares de alto riesgo (Ejemplo. pariente de primer grado con cáncer colorrectal a <60 años).

La madre de este paciente tenía más de 60 años en el momento del diagnóstico, lo que no aumentaría apreciablemente el riesgo del paciente; es apropiado un intervalo de detección de 10 años.

  • La detección más frecuente (<10 años) está indicada para pacientes con pólipos adenomatosos, y el intervalo específico depende del tamaño y el número. Los pólipos hiperplásicos no cambian la frecuencia normal de detección.
  • La tomografía computarizada abdominal es una medida de estadificación adecuada para pacientes con un nuevo diagnóstico de cáncer colorrectal. No es necesario en la evaluación de pólipos neoplásicos o no neoplásicos.
  • La prueba periódica de sangre oculta en heces es una modalidad de detección adecuada para personas con riesgo promedio de cáncer colorrectal. Por lo general, se realiza en intervalos de 1 año (no de 6 meses). A este paciente se le acaba de realizar una colonoscopia con hallazgos de bajo riesgo; No serán necesarias pruebas de detección adicionales hasta dentro de varios años.
How well did you know this?
1
Not at all
2
3
4
5
Perfectly
72
Q
  1. Masculino de 72 años consulta por seguimiento. Tiene un diagnostico reciente de adenocarcinoma de ciego de 4.2 cm y la TAC no evidencia metastasis. Fue sometido a hemicolectomía derecha, y la histopatología indicó que el tumor solo afectaba mucosa y submucosa. Se logró resección tumoral completa con márgenes limpios sin afección ganglionar. Sus signos vitales son normales. Al examen fisico hay una cicatriz quirurgica de buen aspecto en abdomen. ¿Cuál es la recomendación que se debe dar a este paciente?
    A. Realizar tomografía computarizada en 1 años.
    B. Realizar sangre oculta en heces cada año.
    C. Realizar colonoscopia en 1 año.
    D. Realizar colonoscopia en 5 años.
A

Respuesta correcta: C

Tema e ítem: Cáncer colorrectal – Seguimiento.

Argumento: Este paciente se sometió a una resección quirúrgica de un adenocarcinoma localizado estadio I. Estos tumores se limitan a las capas superficiales del colon (mucosa y submucosa) y no son metastásicos. Aunque la mayoría de los pacientes con adenocarcinoma de colon localizado se curan después de la resección quirúrgica, algunos desarrollan lesiones neoplásicas recurrentes del colon. La detección temprana es clave porque el tratamiento con cirugía y/o quimioterapia es más eficaz cuando se inicia rápidamente. La colonoscopia es la prueba diagnóstica más precisa y debe realizarse poco después de la resección quirúrgica (1 año) porque la neoplasia recurrente puede desarrollarse rápidamente. La vigilancia se continúa de por vida (o hasta que las comorbilidades ya no lo permitan) con colonoscopia periódica cada 3-5 años. Puede haber algunas diferencias en las pautas exactas de vigilancia de diferentes sociedades profesionales.

  • A diferencia del adenocarcinoma de colon localizado, los tumores que invaden las capas muscular y serosa (estadio II) o afectan los ganglios linfáticos (estadio III) tienen un riesgo significativo de metástasis a distancia a pesar del tratamiento adecuado. La vigilancia en estos individuos requiere tomografías computarizadas abdominales frecuentes y niveles seriados de antígeno carcinoembrionario (CEA) en suero, además de la realización de la colonoscopia.
  • Aunque la prueba anual de sangre oculta en heces se puede utilizar para la detección del cáncer de colon en personas de bajo riesgo, no se recomienda en personas de alto riesgo (antecedentes personales o familiares de cáncer de colon o pólipos) porque es menos precisa. que la colonoscopia para diagnosticar lesiones neoplásicas.
How well did you know this?
1
Not at all
2
3
4
5
Perfectly
73
Q
  1. Femenina de 76 años presenta dolor en hemiabdomen inferior desde hace 2 días no asociado a otros sintomas. Tiene antecedente de estreñimiento. Signos vitales: 37.9°C, PA de 144/90 mmHg y FC de 90 lpm. Al examen fiscio hay dolor en cuadrante inferior izquierdo, una TAC sugiere diverticulitis aguda por lo que se inicia manejo con ciprofloxacina y metronidazol VO. Tres días despues la paciente regresa con dolor abdominal persistente, nauseas, fiebre y ultima evacuación hace 12 horas. Una nueva TAC evidencia colección líquida perisigmoidea de 5 cm que realza el borde. Paraclínicos: Hb de 13.5 g/dL, PLT de 445.000 mm3 y LEU de 14.500/mm3. ¿Cuál es el manejo adecuado en este caso?
    A. Iniciar antibiotico de amplio espectro IV y dejar en observación.
    B. Realizar laparotomía para resección de colon.
    C. Realizar laparotomía para derenaje y debridamiento.
    D. Realizar drenaje percutaneo.
A

Respuesta correcta: D

Tema e ítem:

Argumento: Este paciente tiene diverticulitis aguda. La diverticulitis se caracteriza por una inflamación diverticular del colon que provoca dolor en el cuadrante inferior izquierdo, fiebre y leucocitosis. La tomografía computarizada puede mostrar cambios inflamatorios como tejido blando trenzado y engrosamiento de la pared del colon. La diverticulitis se puede clasificar como no complicada (75%) o complicada (25%).

La diverticulitis no complicada en pacientes estables se puede tratar de forma ambulatoria con reposo intestinal, antibióticos orales y observación. Sin embargo, se recomienda la hospitalización y los antibióticos intravenosos para pacientes con diverticulitis no complicada que son ancianos, inmunodeprimidos, tienen fiebre alta o leucocitosis significativa, o tienen comorbilidades importantes.

La diverticulitis complicada se refiere a la diverticulitis asociada con un absceso (es decir, acumulación de líquido, como se observa en este paciente), perforación, obstrucción o formación de fístula. Para el tratamiento de la diverticulitis complicada por absceso se debe tener en cuenta:

  • Una colección de líquido <3 cm (algunas referencias mencionan <4 cm) se puede tratar con antibióticos intravenosos y observación, y la cirugía se reserva para pacientes sin respuesta al tratamiento.
  • Una colección de líquido ≥3 cm (algunas referencias enumeran ≥4 cm) debe recibir antibióticos y drenaje percutáneo guiado por TC. Si los síntomas no se controlan (En unos pocos días), se recomienda drenaje quirúrgico y desbridamiento.

La resección sigmoidea generalmente se reserva para pacientes con fístulas, perforación con peritonitis, obstrucción o ataques recurrentes de diverticulitis.

How well did you know this?
1
Not at all
2
3
4
5
Perfectly
74
Q
  1. Femenina de 43 años presenta distensión abdominal leve y difusa 4 días despues de realizarse una colecistectomía electiva. Niega expulsar gases desde la cirugia. En el perioperatorio recibió antibióticos, morfina para el dolor y metoclopramida para las náuseas. Signos vitales: PA de 130/90 mmHg y FC de 76 lpm. El IMC es de 33 kg/m2. Al examen fisico el abdomen está distendido y timpánico con disminución de los ruidos intestintales. Hay dolor leve y difuso, pero no hay rebote ni defensa. ¿Cuál es la causa de la condición actual de esta paciente?
    A. Impacto de un calculo biliar en el íleo.
    B. Adhesiones posoperatorias.
    C. Morfina.
    D. Metoclopramida.
A

Respuesta correcta: C

Tema e ítem: Íleo paralitico.

Argumento: El íleo es un defecto funcional en la motilidad intestinal sin una obstrucción física asociada. Las manifestaciones incluyen náuseas, vómitos, distensión abdominal, dificultad para eliminar flatos o heces (estreñimiento) y ruidos intestinales hipoactivos. Se produce cierto grado de íleo después de la mayoría de los procedimientos abdominales; sin embargo, la persistencia de los signos y síntomas (>3 a 5 días después de la operación) se denomina íleo posoperatorio prolongado (o “patológico”). Los desencadenantes incluyen un aumento del tono simpático del nervio esplácnico después de la instrumentación peritoneal, la liberación local de mediadores inflamatorios y el uso posoperatorio de analgésicos opiáceos (que provocan una disminución de la motilidad gastrointestinal y trastornos del peristaltismo). Las técnicas para prevenir el íleo posoperatorio incluyen anestesia epidural, cirugía mínimamente invasiva y uso perioperatorio prudente de líquidos intravenosos (para minimizar el edema gastrointestinal). El diagnóstico es clínico, aunque las radiografías abdominales que revelan asas intestinales dilatadas sin punto de transición respaldan el diagnóstico.

Las adherencias, que comienzan a formarse inmediatamente después de las cirugías abdominales, son la causa más común de obstrucción intestinal mecánica produciendo manifestaciones similares. Sin embargo, la obstrucción mecánica suele provocar ruidos intestinales hiperactivos (no hipoactivos). Además, los pacientes con obstrucción mecánica posoperatoria suelen tener un retorno temporal de la función intestinal antes de la aparición de los síntomas (a diferencia de este paciente).

How well did you know this?
1
Not at all
2
3
4
5
Perfectly
75
Q
  1. Masculino de 29 años consulta por vómitos persistentes y dolor abdominal desde hace 24 horas. Su ultima evacuación fue hace 3 días, niega diarrea. La emesis aparece verde sin sangre y el paciente no tolera la vía oral. Signos vitales: Temp. de 36.8°C, FC de 91 lpm y PA de 110/70 mmHg mientras está sentado y de 90/60 mmHg al estar de pie. El abdomen está distendido con ruidos intestinales hiperactivos, timpanico a la percusión y con sensibilidad difusa, sin signos de irritación peritoneal. Laboratorios: Hematocrito 45%, Leucocitos 9.600 células, Sodio 147 mEq/L, Potasio 3,1 mEq/L, Creatinina 1,0 mg/dL, AST 20 U/L, ALT 12 U/L, Bilirrubina 0,8 mg/dL ¿Cuál de los siguientes seria la causa más probable de la condición de este paciente?
    A. Apendiceptomia hace 1 años.
    B. Alto consumo de alcohol.
    C. Perdida reciente de peso.
    D. Episodio reciente de fiebre y diarrea no sanguinolenta.
A

Respuesta correcta: A

Tema e ítem: Obstrucción intestinal – Causas.

Argumento: Este paciente tiene una obstrucción mecánica del intestino delgado. Los vómitos causaron hipopotasemia y, junto con la disminución de la ingesta oral, provocaron deshidratación y ortostasis en este paciente. La obstrucción mecánica del intestino delgado se clasifica por ubicación anatómica (es decir, proximal versus media/distal). Las obstrucciones proximales completas se caracterizan por vómitos tempranos, malestar abdominal y llenado anormal de contraste en las radiografías. Las obstrucciones medias o distales suelen presentarse como dolor abdominal tipo cólico, vómitos retardados, distensión abdominal prominente, estreñimiento, ruidos intestinales hiperactivos y asas intestinales dilatadas en la radiografía abdominal. Lo más probable es que este paciente tenga una obstrucción de intestino delgado media o distal.

Las adherencias son, con diferencia, la causa más común de obstrucción de intestino delgado. Pueden ser congénitos en niños, pero generalmente son el resultado de operaciones abdominales o procesos inflamatorios. Es probable que este paciente adulto con obstrucción de intestino delgado haya tenido una cirugía abdominal, como una apendicectomía.

How well did you know this?
1
Not at all
2
3
4
5
Perfectly
76
Q
  1. Mujer, 18 años, consulta por disuria. Tomó nitrofurantoína por 7 días sin mejoría. Presenta flujo vaginal. Signos vitales: T 36,7°C, PA 118/74 mmHg y FC 68/min. Especuloscopia: secreción cervical espesa, amarilla, el cuello es friable. No hay dolor en la exploración. Microscopía: ↑ leucocitos, sin células clave ni organismos móviles. Uroanálisis: esterasa (+), nitritos y sangre (-). ¿Cuál es el manejo?
    A. Iniciar Ceftriaxona 500 mg IM DU + Doxiciclina 100 mg c/12h por 7 días.
    B. Iniciar Azitromicina 1 g VO DU + Doxiciclina 100 mg c/12h por 7 días.
    C. Iniciar Clindamicina 300 mg VO dos veces/día por 7 días + Gentamicina 3-5 mg/kg/día IV.
    D. No se indica tratamiento hasta disponer de los resultados de las pruebas.
A

Respuesta correcta: A

Tema e ítem: Tema: Cervicitis por ETS - Tratamiento

Argumento: Lo más probable es que esta paciente tenga una cervicitis aguda causada por Chlamydia trachomatis o Neisseria gonorrhoeae, prevalentes en mujeres de <25 años. El diagnóstico es clínico y se basa en la presencia de secreción cervical mucopurulenta y cuello uterino friable; algunas también presentan uretritis concomitante debido a la proximidad de la uretra a la vagina. Por el contrario, la sensibilidad al movimiento cervical, un signo de infección ascendente (enfermedad inflamatoria pélvica), puede estar ausente con un recuento bacteriano bajo o en una infección temprana.

  • Es necesario un tratamiento empírico antes de los resultados microbiológicos para prevenir la transmisión a las parejas sexuales y disminuir el riesgo de secuelas a largo plazo (embarazo ectópico, infertilidad). El tratamiento empírico de primera línea es la terapia dual con ceftriaxona y doxiciclina. La ceftriaxona cubre N gonorrhoeae, un diplococo gramnegativo, y la doxiciclina cubre C trachomatis, un organismo atípico.
  • Tanto la azitromicina como la doxiciclina cubren el C trachomatis, pero juntas son insuficientes como tratamiento empírico de la cervicitis aguda porque ninguna de ellas cubre adecuadamente los gramnegativos N gonorrhoeae.
  • La clindamicina más gentamicina se utiliza para tratar la endometritis posparto, una infección uterina polimicrobiana causada por el ascenso de la flora vaginal normal durante el parto. La clindamicina cubre las bacterias grampositivas (estreptococos del grupo B) y anaerobias (Bacteroides fragilis); la gentamicina cubre las bacterias gramnegativas, incluida N gonorrhoeae. Sin embargo, ambas carecen de cobertura frente a C trachomatis y, por lo tanto, no se utilizan para el tratamiento empírico de la cervicitis aguda.
How well did you know this?
1
Not at all
2
3
4
5
Perfectly
77
Q
  1. Mujer, 29 años, consulta por masa dolorosa en mama izquierda. Está preocupada porque su prima tiene ca de mama. Fuma cigarrillo y consume alcohol ocasionalmente. Ecografía: masa bien circunscrita (4 x 5 x 6 cm) con realce acústico posterior, sin restos ecogénicos ni componentes sólidos. Aspiración con aguja: líquido claro con resolución de la masa. ¿Cuál es la conducta?
    A. Realizar seguimiento en 12 meses.
    B. Realizar cultivo del líquido aspirado.
    C. Realizar seguimiento en 2 meses.
    D. Realizar mamografía.
A

Respuesta correcta: C
Tema e ítem: Masa movible - Ultrasonido
Argumento: Una masa sensible y móvil en una paciente joven es probablemente benigna. En una paciente de edad <30 años con una masa clínicamente benigna, la ecografía mamaria es el estudio de imagen de primera línea. La ecografía de esta paciente muestra realce acústico posterior (indicativo de líquido) y ausencia de restos ecogénicos o componentes sólidos; estas características son compatibles con un quiste mamario simple.

La presentación de un quiste mamario simple es variable, desde la ausencia de síntomas hasta el dolor intenso y localizado. Las pacientes sintomáticas pueden beneficiarse de la aspiración, que debería producir un líquido claro y provocar la desaparición de la masa, confirmando así el diagnóstico. Como el líquido quístico puede volver a acumularse, la paciente debe volver en 2-4 meses para un examen clínico de seguimiento de la mama.

  • Solo si en el seguimiento a mediano plazo la paciente no vuelve a presentar síntomas ni signos de recidiva, puede reanudarse el cribado anual.
  • El tabaquismo se ha asociado a procesos de metaplasia escamosa de ductos distales, los cuales se vuelven ectásicos, lo cual lleva a sobreinfección con Gram negativos y anaerobios, produciendo así abscesos periareolares que pueden terminar en fístulas periareolares si no se tratan a tiempo, creando un proceso crónico. La manifestación inicial es dolor y eritema con fiebre, malestar general, clínica que no solo no está presente en la paciente, sino que también el líquido aspirado no posee características de sobreinfección, por lo que no es necesario su cultivo.
  • Se considera que en menores de 35 años la ecografía tiene una mayor utilidad que la mamografía en el estudio de la patología mamaria, pues la densidad mamaria a edades más tempranas hace que la sensibilidad sea muy limitada.
How well did you know this?
1
Not at all
2
3
4
5
Perfectly
78
Q
  1. Mujer de 39 años consulta por episodios intermitentes de pérdida involuntaria de orina. Inició hace varios meses con el esfuerzo físico. Ahora, toser y reír le provocan grandes pérdidas. Ha empezado a usar pañal. Sus 2 hijos nacieron por vía vaginal y ambos tuvieron macrosomía fetal. Uroanálisis: nitritos y esterasa leucocitaria (-). Tiene constipación y hemorroides. ¿Cuál es el manejo apropiado?
    A. Terapia con Fesoterodina 4 mg al día por 2 semanas.
    B. Micción programada por 2 semanas + Darifenacina 7.5 mg al día por 2 semanas.
    C. Iniciar Estrógenos conjugados equinos 0.3 mg/día VO.
    D. Realizar ejercicios del piso pélvico.
A

Respuesta correcta: D

Tema e item: SIU (Incontinencia Urinaria) - Manejo

Argumento: La pérdida de orina de esta paciente con el aumento de la presión intraabdominal (al levantar peso, toser, reír) se debe a incontinencia urinaria de esfuerzo (IUE). La vejiga y la uretra se mantienen normalmente en la posición anatómica adecuada gracias a los músculos del suelo pélvico (elevadores del ano). Las pacientes desarrollan IUE por debilidad de los músculos del suelo pélvico debida a una presión crónica o a una lesión de estos músculos; esto ocurre a menudo en mujeres con múltiples partos vaginales (especialmente los complicados por macrosomía fetal). La debilidad crónica del suelo pélvico puede crear hipermovilidad uretral, en la que la uretra se mueve de forma inadecuada y es incapaz de comprimirse completamente contra la pared vaginal anterior en momentos de aumento de la presión intraabdominal (maniobra de Valsalva). Por lo tanto, el tratamiento de primera línea de la IUE son los ejercicios para los músculos del suelo pélvico (Kegel), que fortalecen y estabilizan la musculatura pélvica. La musculatura pélvica fortalecida proporciona una base para la compresión uretral y suprime las contracciones de la vejiga, mejorando la incontinencia. Puede utilizarse un pesario de continencia junto con los ejercicios de Kegel, y puede realizarse un procedimiento de cabestrillo mediouretral para quienes deseen un tratamiento quirúrgico o no respondan a la terapia conservadora. Los ejercicios de los músculos pélvicos fortalecen la musculatura del suelo pélvico para que la uretra se comprime sobre ella e inhibe de forma refleja las contracciones del detrusor. El régimen básico consiste en tres series de 8 a 12 contracciones sostenidas durante 8 a 10 segundos cada una, realizadas tres veces al día. Las pacientes deben intentar hacerlo todos los días y continuar durante al menos 15 a 20 semanas.
* La terapia farmacológica es uno de los pilares terapéuticos, pero no es el de primera línea. Se debe tener precauciones e intervenir concomitantemente a aquellas pacientes que reciben antimuscarínicos y tienen constipación, pues se puede empeorar el cuadro.
* La micción programada suele durar en promedio 6 semanas para que pueda demostrar resultados. En el mismo sentido, la terapia antimuscarínica debe administrarse por al menos 4-6 semanas para poder evaluar una respuesta.
* En las pacientes con incontinencia secundaria al síndrome urogenital de la menopausia pueden ser tratadas con terapias estrogénicas. Sin embargo, este no corresponde al caso expuesto y si así lo fuera, dicha terapia siempre es intravaginal, no se recomienda la terapia oral.

How well did you know this?
1
Not at all
2
3
4
5
Perfectly
79
Q
  1. Mujer, 33 años, G3P0A2, 28 sem de gestación, asiste a control prenatal. Tiene DM1. La mayoría de las glucemias postprandiales son > 200 mg/dL. Eco de detalle: normal; eco a las 24 sem: percentil 30; eco reciente: percentil 4 e ILA de 3 cm (normal: >5 cm). Examen físico: PA 124/82 mmHg, FC 88/min. Altura uterina: 24 cm. NST reactivo y tranquilizador. ¿Cuál es la conducta más adecuada?
    A. Realizar Eco doppler de arteria umbilical.
    B. Descartar aneuploidía fetal.
    C. Solicitar fibronectina fetal.
    D. Realizar una nueva ecografía en 2 semanas.
A

Respuesta correcta: A

Tema e ítem: Restricción del crecimiento fetal-Manejo

Argumento: Esta paciente tiene restricción del crecimiento fetal (RCF), un peso fetal estimado en <10 percentil para la edad gestacional. La RGF puede ser una complicación de una enfermedad vascular materna (diabetes mellitus tipo 1, hipertensión crónica), que causa vasoconstricción placentaria crónica e isquemia con la consiguiente insuficiencia uteroplacentaria. A medida que empeora la insuficiencia uteroplacentaria (es decir, disminuye la perfusión placentaria), también disminuyen la oxigenación y la nutrición fetal. En respuesta, el crecimiento fetal se ralentiza y la sangre se desvía preferentemente de los riñones fetales al cerebro, provocando un RGF y un oligohidramnios asociado (es decir, un índice de líquido amniótico ≤5 cm), como se observa en esta paciente.

Las pacientes con RGF tienen un alto riesgo de muerte fetal intrauterina y requieren una ecografía Doppler de la arteria umbilical inmediata para evaluar la perfusión placentaria. Esta prueba mide el flujo intravascular y la resistencia en la arteria umbilical; el aumento de la resistencia indica una disminución de la perfusión placentaria y un empeoramiento de la hipoxia fetal. Estas mediciones se utilizan para identificar a las pacientes que requieren un parto urgente para minimizar el riesgo de muerte fetal.
* La fibronectina fetal es una glucoproteína que se cree que actúa como adherente en la superficie de contacto materno-fetal, está presente habitualmente en las secreciones cervicovaginales al final del segundo y el principio del tercer trimestre. Se ha considerado positivo un valor >50 ng/dL. Las mujeres asintomáticas con una fibronectina fetal positiva presentan un mayor riesgo de parto prematuro antes de la semana 35 de gestación, especialmente en las 2 semanas siguientes a un resultado positivo. Sin asociación con RCF.
* La clínica de esta gestación es mucho más sugestiva de RCF que de alguna aneuploidía, las cuales se comienzan a sospechar desde edades gestacionales mucho más tempranas, con el desarrollo de la primera ecografía.
* Retrasar un procedimiento, que adicionalmente, carece de utilidad, seguramente no es la estrategia terapéutica para pacientes que deben recibir una atención pronta.

How well did you know this?
1
Not at all
2
3
4
5
Perfectly
80
Q
  1. Mujer, 24 años, víctima de violencia sexual hace 2 h. FUM hace dos semanas. Está llorando y evita el contacto visual. Hematomas en: abdomen y muñecas. Con abrasiones sensibles en los labios vulvares menores. Se activó el sector justicia, se reúne material probatorio en cadena de custodia, prueba de embarazo (-) y se administra anticoncepción de emergencia. ¿Cuál es el mejor paso a seguir?
    A. Sólo observación y tranquilización.
    B. Tratamiento empírico de las infecciones de transmisión sexual.
    C. Tiene indicación psiquiátrica de manejo intrahospitalario.
    D. Revisión de los factores de riesgo de agresión sexual.
A

Respuesta correcta: B

Tema e ítem: Abuso sexual (riesgo de transmision de ETS) -Manejo

Argumento: Las agresiones sexuales son frecuentes, con una prevalencia a lo largo de la vida en mujeres en Estados Unidos de aproximadamente el 20%, y los agresores suelen ser conocidos de la paciente. La evaluación inicial de las pacientes que buscan atención médica tras una agresión sexual incluye un examen físico y forense (sangre, semen) con documentación detallada de las lesiones (hematomas, abrasiones).
El tratamiento de la agresión sexual incluye el tratamiento empírico de las infecciones de transmisión sexual porque el tratamiento precoz ayuda a prevenir la morbilidad (enfermedad inflamatoria pélvica, infertilidad) asociada a la infección. Por lo tanto, todos los pacientes reciben profilaxis postexposición (PPE) contra la Clamidia (azitromicina), la N. gonorrhoeae (ceftriaxona) y la tricomoniasis (metronidazol).
Las pacientes que acuden a la consulta en las 72 horas siguientes a una agresión también pueden beneficiarse de la profilaxis contra el VIH tras un asesoramiento individualizado sobre los riesgos y beneficios de la terapia. Además de la profilaxis postexposición, a las mujeres con una prueba de embarazo negativa se les ofrece anticoncepción de emergencia.

  • A las víctimas de agresiones sexuales se les debe ofrecer apoyo psicosocial integral, incluidos servicios de salud mental, porque pueden experimentar emociones complicadas (ira, miedo, vergüenza) o desarrollar ansiedad, depresión o trastorno de estrés postraumático. La evaluación psiquiátrica hospitalaria y la terapia antidepresiva están indicadas para pacientes con riesgo inmediato de suicidio u homicidio. Sin embargo, el llanto y evitar el contacto visual (como en esta paciente) inmediatamente después de una agresión son normales y no significan necesariamente que la paciente esté en riesgo de hacerse daño a sí misma o a otros.
  • Revisar los factores de riesgo de agresión sexual inmediatamente después de la agresión de una paciente es insensible y culpa parcialmente a la paciente.
How well did you know this?
1
Not at all
2
3
4
5
Perfectly
81
Q
  1. Femenina de 54 años, consulta por astenia, adinamia y pérdida de peso en los últimos 3 meses, 10 kg de forma inintencionda. Antecedente de obesidad grado II por IMC de 34 kg/m2. Signos vitales: T° 36,9 C°, la PA es de 120/80 mm/Hg, FC de 80 lpm. Al examen físico, abdomen blando, no doloroso, con matidez desplazable a la percusión, hay hepatomegalia y un nódulo hepático palpable. Tiene edema grado II con fóvea de forma bilateral en miembros inferiores. Paraclínicos con: Hb 11,6, Plaquetas 120.000, Leucocitos 9.000, Albúmina 2,5 g/dL, Bilirrubina total 2,3 mg/dL, FA 370, U/L, AST 154 U/L, ALT134 U/L y Alfafetoproteína sérica 12 ng/mL. ¿Cuál es el diagnóstico más probable?
    A. Una hiperplasia nodular focal
    B. Un adenoma hepático
    C. Un carcinoma Hepatocelular
    D. Un quiste hidatídico
A

Respuesta correcta: C

Tema e ítem: Cirrosis hepática y carcinoma hepatocelular, Enfoque diagnóstico.

Argumento: Esta paciente que se presenta con matidez desplazable (signo de ascitis), hipoalbuminemia, función hepática alterada (AST-ALT elevadas), trombocitopenia e hiperbilirrubinemia, configuran la sospecha de una cirrosis subyacente. La mayoría de las cirrosis son causadas por consumo de alcohol, hepatitis viral crónica o enfermedad del hígado graso no alcohólico (paciente obesa sin otro antecedente). Los pacientes con cirrosis a menudo desconocen el diagnóstico, hasta que desarrollan complicaciones como sangrado por várices o carcinoma hepatocelular (CHC). Recordemos que el riesgo de CHC en pacientes con cirrosis es del 1% al 8% por año. En estos pacientes, el CHC suele presentarse con insuficiencia hepática descompensada (ascitis, ictericia, hipoalbuminemia), pérdida de peso, posible hepatomegalia y nódulo hepático palpable (Como esta paciente). La alfafetoproteína (AFP), una glicoproteína producida por el hígado fetal y el saco vitelino, está elevada en aproximadamente el 50% de los casos; por lo tanto, la AFP puede servir como una pista diagnóstica importante (cuando está elevada); no obstante, no debe utilizarse para descartar el diagnóstico. La tomografía computarizada con contraste arterial de triple fase del abdomen es diagnóstica en la mayoría de los casos.

En cuanto a la hiperplasia nodular focal, recordemos se trata de una lesión hepática benigna que generalmente surge en mujeres jóvenes. Aunque una minoría desarrolla un nódulo hepático palpable, la mayoría es asintomática. Este tipo de lesión generalmente no se rompen, no sufren de transformaciones malignas, no causan insuficiencia hepática, ni generan cambios en el peso del paciente. Por otra parte, el adenoma hepático es una lesión hepática benigna común en mujeres jóvenes que reciben tratamiento oral anticonceptivo; si bien, algunas lesiones sufren una transformación maligna (a CHC), la insuficiencia hepática descompensada es poco común (porque el resto del hígado ha conservado su función). Finalmente, el quiste hidatídico es una infección por tenia Echinococcus. La mayoría de los casos son asintomáticos, pero los quistes hidatídicos grandes, pueden causar hepatomegalia, dolor en el cuadrante superior derecho o náuseas/emesis; sin embargo, sería poco común la insuficiencia hepática descompensada y la pérdida de peso.

82
Q
  1. Femenina de 21 años, consultó por cefalea que no atenúa con naproxeno. La cefalea inicio posterior a traumatismo craneoencefálico ocurrido mientras jugaba fútbol hace 24 horas. La paciente presentó un episodio de emesis, sin pérdida del estado de conciencia. Fue llevada a urgencias, donde estaba confundida y con incapacidad para recordar los eventos de su lesión. Una tomografía computarizada de la cabeza no tuvo hallazgos anormales; el examen neurológico de la paciente fue normal; y fue dada de alta. Al momento de reconsultar, examen neurológico sin hallazgos anormales. A la paciente le gustaría saber cuándo puede volver a practicar fútbol, ¿Cuál es la mejor respuesta ante la duda de la paciente?
    A. Puede aumentar gradualmente su actividad durante 1 semana si no tiene síntomas.
    B. Puede volver a su actividad después de que se resuelvan sus síntomas actuales.
    C. Debe evitar los deportes de contacto de forma indefinida.
    D. Debe iniciar fenitoína profiláctica para prevenir convulsiones.
A

Respuesta correcta: A

Tema e ítem: Traumatismo craneoencefálico leve-Pronóstico

Argumento: Esta paciente con un traumatismo craneoencefálico reciente, complicado por emesis y amnesia transitoria, son elementos que configuran la presencia de una conmoción cerebral (se caracteriza por una alteración neurológica transitoria) que resulta de una lesión cerebral traumática leve (LCTL). Es importante tener en cuenta que el regreso prematuro a la práctica deportiva aumenta el riesgo de conmociones cerebrales futuras y secuelas (encefalopatía traumática crónica, síndrome del segundo impacto); por lo tanto, se ha recomendado un protocolo gradual de regreso al juego para reducir este riesgo. Los adultos con riesgo promedio, deben descansar durante ≥24 horas antes de aumentar gradualmente el nivel de actividad (cada 24 horas asintomáticas del paciente permiten ir aumentando gradualmente su actividad física). Este aumento gradual debe darse durante 6 días; por ende, los pacientes no deben retomar sus actividades de contacto antes de una semana.

En cuanto a evitar los deportes de contacto de forma indefinida, la Declaración de consenso de 2012 sobre la conmoción cerebral en el deporte, recomienda es un regreso al deporte de contacto de forma gradual en un tiempo de 6 días; por ende, en esta paciente que presentó una conmoción cerebral, no se debe indicar la suspensión indefinida de su actividad. Por otra parte, un regreso más rápido al juego en paciente asintomáticos puede considerarse; no obstante, esta paciente presentó cefalea sin mejoría 24 horas posterior al evento, por ende se debe plantear un tiempo de espera de 1 semana con aumento gradual de la actividad física.

Finalmente, los pacientes seleccionados con lesión cerebral traumática moderada o grave pueden ser tratados con medicamentos anticonvulsivos profilácticos a corto plazo. Los pacientes con lesión cerebral traumática leve, generalmente no requieren tratamiento profiláctico anticonvulsivo.

83
Q
  1. Masculino de 47 años, consultó por inicio súbito de dolor torácico paraesternal y diaforesis. Durante la consulta el paciente refiere sensación de mareos y súbitamente deja de responder, y pierde el conocimiento, por lo que se inicia reanimación cardiopulmonar. El paciente recupera el conocimiento después de 60 segundos. Antecedente de DM2 controlada con dieta, hipertensión e hiperlipidemia. En el departamento de urgencias, PA de 142/88 mm Hg y FC de 92 lpm. Realizaron un electrocardiograma que reportó ritmo sinusal normal, complejos ventriculares prematuros y elevación del segmento ST de 3 mm en las derivaciones V1-V3. ¿Cuál es el mecanismo fisiopatológico primario más probable responsable del episodio sincopal de este paciente?
    A. Una fibrilación auricular.
    B. Un bloqueo de la conducción auriculoventricular.
    C. Una taquicardia supraventricular paroxística.
    D. Una arritmia ventricular.
A

Respuesta correcta: D

Tema e ítem: Síncope- Enfoque diagnóstico

Argumento: Las arritmias ventriculares, incluidos los latidos ventriculares prematuros, la taquicardia ventricular sostenida o no sostenida y la fibrilación ventricular, son bastante comunes en el período inmediatamente posterior al infarto de miocardio (IM). La fibrilación ventricular es la arritmia subyacente más frecuente responsable del paro cardíaco repentino en el contexto de un IM agudo; más del 50% ocurren dentro de la primera hora después de la aparición de los síntomas. La reentrada es el mecanismo predominante responsable de las arritmias ventriculares en el período inmediatamente posterior al infarto. El mecanismo subyacente responsable de las arritmias ventriculares periinfarto varía según el tiempo transcurrido desde el inicio del IM. Las arritmias que ocurren dentro de los 10 minutos posteriores a la oclusión coronaria se conocen como arritmias ventriculares “inmediatas” o de fase 1a. La isquemia aguda causa heterogeneidad de la conducción con áreas de conducción CON marcado enlentecimiento y activación retardada, lo que a su vez predispone a arritmias reentrantes. Por el contrario, las arritmias “retardadas” o de fase 1b ocurren entre 10 y 60 minutos después del infarto agudo y se cree que son el resultado de un automatismo anormal.

Por otra parte, el bloqueo de la conducción auriculoventricular puede ocurrir en pacientes con IM agudo, especialmente aquellos con IM de la pared inferior (Este paciente tiene elevación en el EKG que sugiere más bien isquemia en región anteroseptal del corazón). Además, suele ser transitorio y se resuelve después de una reperfusión exitosa con trombolíticos o una intervención coronaria percutánea primaria. Finalmente, las taquicardias supraventriculares distintas de la fibrilación o el aleteo auricular son poco comúnes en el período periinfarto. Incluso la fibrilación auricular ocurre raramente (alrededor del 3%) durante las primeras 3 horas de IM agudo; si ocurre, rara vez se presenta con síncope.

84
Q
  1. Ema tiene 3 años. Es llevada a urgencias por su madre debido a edema en rodilla derecha desde hace 2 meses, pero los atribuyó a los juegos bruscos con su hermano. Tenía además cojera que aparecía por las mañanas y desaparecía al mediodía. Desde ayer, aumentó notablemente el edema. La paciente niega dolor articular u otros síntomas. Se encuentra afebril, con signos inflamatorios locales. ¿Cuál de las siguientes patologías es más probable en esta paciente?
    A. Hemorragia muscular
    B. Uveítis
    C. Enfermedad intestinal inflamatoria
    D. Bloqueo de la conducción auriculoventricular
A

Respuesta correcta: B

Tema-Ítem: Artritis idiopática juvenil-Complicaciones

Argumentación: Es probable que este paciente tenga artritis idiopática juvenil (AIJ) oligoarticular, debido a la rigidez matinal, compromiso de grandes articulaciones y ausencia de síntomas sistémicos.

La AIJ oligoarticular, la forma más común, se caracteriza por la afectación de ≤4 articulaciones (es decir, <5 articulaciones) dentro de los primeros 6 meses de la aparición de la enfermedad. Usualmente, se presenta edema en articulaciones, pero con dolor mínimo o ausente. Las articulaciones grandes (p. ej., rodillas, tobillos) son las más comúnmente afectadas; sin embargo, la afectación de la cadera es rara. No se presentan síntomas sistémicos (p. ej., fiebre, sarpullido).

La uveítis es una complicación grave que se desarrolla en una minoría de pacientes con AIJ oligoarticular. Estos pacientes requieren evaluaciones oftalmológicas periódicas porque la uveítis no tratada suele ser asintomática y puede provocar una pérdida irreversible de la visión. Aunque no están presentes en todos los pacientes con AIJ oligoarticular, los anticuerpos antinucleares son un factor de riesgo de uveítis. Los datos de laboratorio adicionales (p. ej., recuento de leucocitos, hemoglobina, velocidad de sedimentación globular) suelen ser normales en la AIJ oligoarticular.

El bloqueo de la conducción auriculoventricular es una complicación de la enfermedad de Lyme, que típicamente se presenta como monoartritis, más comúnmente de la rodilla. El eritema migratorio y los signos sistémicos (p. ej., fiebre, cefalea) suelen preceder a las anomalías articulares y la inflamación articular no desaparece a lo largo del día.

La enfermedad inflamatoria intestinal se asocia con espondiloartritis, que típicamente se presenta en hombres adultos con dolor lumbar crónico debido a la afectación de la articulación axial (p. ej., sacroilíaca).
La hemorragia intramuscular ocurre con la hemofilia, un trastorno hemorrágico recesivo ligado al cromosoma X que puede causar dolor agudo en las articulaciones y edema debido a una hemartrosis. La artritis crónica en una niña hace que la hemofilia y la hemartrosis sean poco probables.

La fibrosis pulmonar se observa en asociación con la artritis reumatoide, que se presenta más comúnmente con artritis de las articulaciones pequeñas (p. ej., manos, pies) en mujeres adultas. Las manifestaciones pulmonares no ocurren en la AIJ oligoarticular.

85
Q
  1. Paciente femenina de 2 semanas de edad, actualmente hospitalizada en la unidad de cuidados intensivos neonatales por prematuridad (29 semanas de gestación). Desde hace 3 horas con episodios de apnea y bradicardia. Nació por parto vaginal, pesó 1000 kg. Ha estado recibiendo alimentación enteral por sonda nasogástrica con leche de fórmula. Al examen físico, estable hemodinámicamente, con ligera distensión abdominal. Se toman un ionograma y un hemoleucograma, sin alteraciones. ¿Cuál considera que es el mejor paso a seguir?
    A. Rx de abdomen
    B. Continuar observación y vigilar otros signos/síntomas
    C. Intubar de inmediato
    D. Realizar ecocardiografía
A

Respuesta correcta: A

Tema-Ítem: Neonatología-Enterocolitis necrotizante

Argumentación: Paciente prematuro y de bajo peso al nacer, con apnea, bradicardia y distensión abdominal, por lo que debemos sospechar enterocolitis necrotizante (ECN).

La ECN es una afección potencialmente mortal caracterizada por inflamación intestinal y necrosis debido a bacterias productoras de gas. Los recién nacidos prematuros (especialmente con edad gestacional <32 semanas) y aquellos con muy bajo peso al nacer que reciben alimentación enteral, en particular con fórmula, tienen mayor riesgo de ECN. Se requiere un alto índice de sospecha en estos pacientes porque los hallazgos iniciales pueden ser sutiles pero pueden conducir a un rápido deterioro clínico.

La presentación puede ocurrir semanas después del nacimiento y a menudo es inespecífica e incluye apnea, letargo, intolerancia alimentaria o inestabilidad de los signos vitales (p. ej., hipotermia). El abdomen suele estar distendido y puede volverse tenso y eritematoso. Los paraclínicos también son inespecíficos y el recuento de leucocitos puede ser normal, bajo o elevado. En particular, la trombocitopenia y la acidosis metabólica se asocian con enfermedad avanzada.

El siguiente paso en la evaluación de una sospecha de ECN es una radiografía abdominal y si encontramos neumatosis intestinal (es decir, aire en la pared intestinal) es diagnóstica. La ECN complicada por perforación intestinal revelaría neumoperitoneo (es decir, aire libre debajo del diafragma) en la radiografía y es una indicación de cirugía.

El aumento de la apnea en un recién nacido prematuro puede ser un signo de insuficiencia cardíaca debido a un ductus arterioso persistente. La ecocardiografía puede estar indicada si hay signos cardiopulmonares, como soplos, crépitos y/o signos de dificultad respiratoria. Sin embargo, la distensión abdominal de este paciente y el examen cardiopulmonar normal hacen que la ECN sea más probable que una etiología cardíaca.
Aunque puede ser necesaria la intubación endotraqueal en un paciente con apnea sostenida, actualmente no está indicada en este recién nacido con oxigenación normal y sin signos de dificultad respiratoria (p. ej., sin estridor ni retracciones).

La apnea del prematuro es debida a la respiración inmadura. Sin embargo, el inicio suele ser en los primeros días de vida. Los episodios de apnea cada vez más recientes y en aumento, especialmente en combinación con distensión abdominal, requieren una evaluación adicional.

86
Q
  1. Carlos, 15 meses. Llevado a su consulta para cita de crecimiento y desarrollo. Actualmente, cumpliendo todos los hitos del desarrollo. Su guardería tuvo un brote de tos ferina hace 6 semanas que obligó a cerrar por unos días, pero el paciente se encuentra asintomático. A los 6 meses de edad, tuvo una convulsión tónico clónica generalizada, de 2 minutos de duración, a las pocas horas de recibir la vacuna contra la difteria, el tétanos y la tos ferina acelular (DTaP). Cuando sea vacunado de nuevo, ¿Cuál recomendación le daría a la madre?
    A. Administrar la vacuna contra difteria, tétanos y tos ferina acelular según lo programado
    B. Administrar los toxoides diftérico y tetánico; evitar el componente de tos ferina
    C. Evitar la aplicación de todas las vacunas que contengan estos componentes de la vacuna
    D. Prescribir únicamente antibióticos de profilaxis postexposición
A

Respuesta correcta: A

Tema e ítem: Esquemas de vacunación en pediatría

Argumentación: La vacuna contra la difteria, el tétanos y la tos ferina acelular (DTaP) es una vacuna combinada que contiene antígeno de tos ferina acelular con toxoides diftérico y tetánico. Las vacunas combinadas son seguras, efectivas para prevenir infecciones y beneficiosas para reducir la cantidad de “chuzones” necesarios (lo que resulta en una disminución del dolor y una mejor adherencia). De acuerdo al PAI, los niños deben recibir 5 dosis de la vacuna DTaP inactivada a los 2, 4 y 6 meses de edad; 18 meses y 5 años.

El riesgo de una reacción adversa a la vacuna DTaP es bajo y generalmente incluye eritema/edema leve en el lugar de la inyección y/o fiebre. Las convulsiones, desencadenadas por fiebre o por el componente de la vacuna contra la tos ferina, son raras y suelen ser breves (<5 minutos) y autolimitadas.

Los pacientes con antecedentes familiares de convulsiones febriles o epilepsia pueden tener un mayor riesgo. Sin embargo, ni los antecedentes personales ni familiares de convulsiones constituyen una contraindicación para la vacunación. Específicamente, las convulsiones no complicadas después de la administración de la vacuna no son una contraindicación para una futura vacunación. El beneficio de la vacuna DTaP, especialmente en el contexto de un brote de tos ferina, supera el improbable riesgo de efectos secundarios importantes.

La vacuna DTaP está contraindicada en algunas circunstancias como cuando se desarrolla anafilaxia después de la administración de DTaP. Además, los trastornos neurológicos inestables (p. ej., espasmos infantiles, epilepsia no controlada) y la encefalopatía (es decir, coma, disminución del nivel de conciencia, convulsiones prolongadas) dentro de una semana después de la administración de la vacuna DTaP son contraindicaciones para la vacuna combinada; por esto, se deben solamente aplicar los toxoides diftérico y tetánico, sin tos ferina.

La profilaxis antibiótica postexposición está indicada para contactos cercanos (p. ej., hogar, guardería) de cualquier persona con tos ferina en los últimos 21 días. Debido a que este niño estuvo expuesto a la tos ferina hace 6 semanas, ya no se requieren antibióticos profilácticos.

87
Q
  1. José tiene 7 meses. La abuela lo lleva al consultorio por sangre en heces en dos ocasiones desde hace una semana. Recibió lactancia materna exclusiva hasta hace un mes, cuando se introdujeron en su dieta cereales de arroz y fórmula a base de leche de vaca. Anteriormente, las heces eran blandas y amarillas, con frecuencia de al menos una vez al día. Durante las últimas 3 semanas, son Bristol 1, con frecuencia cada 2 o 3 días. Al examen físico, abdomen ligeramente distendido, con matidez en cuadrante inferior izquierdo y una pequeña fisura anal a las seis del reloj. ¿Cuál de los siguientes es el manejo más adecuado para este paciente?
    A. Iniciar cambios en la dieta
    B. Cambiar a una fórmula a base de soja
    C. Cambie a una fórmula ampliamente hidrolizada
    D. Obtener una radiografía abdominal
A

Respuesta correcta: A

Tema-Ítem: Constipación-Constipación funcional

Argumentación: Este paciente con aparición reciente de heces infrecuentes, Bristol 1, después de la introducción de alimentos sólidos probablemente tenga constipación funcional.

El estreñimiento funcional es la causa más común de estreñimiento en los niños y no tiene una causa orgánica grave. Por el contrario, se debe sospechar estreñimiento patológico si hay signos de alarma, como retraso en el paso del meconio en el período neonatal, crecimiento deficiente o distensión abdominal intensa.

El estreñimiento funcional suele presentarse alrededor de los 6 meses de edad porque el principal factor de riesgo en este grupo de edad es el cambio en la dieta (p. ej., alimentos sólidos con relativamente poca fibra y menor contenido de líquido). El crecimiento es normal y la exploración física suele ser normal. Sin embargo, puede haber una leve distensión abdominal y matidez en el cuadrante inferior izquierdo o heces palpables debido a la retención de heces. Además, las heces de gran calibre pueden causar una fisura anal y sangre de color rojo brillante visible en las heces.

El diagnóstico en lactantes suele ser clínico y el tratamiento inicial es con cambios en la dieta, con carbohidratos no digeribles y osmóticamente activos que atraen agua hacia el tracto gastrointestinal, ablandando así las heces. Al igual que el tratamiento del estreñimiento funcional en niños mayores, se puede considerar la adición de laxantes (p. ej., polietilenglicol) en lactantes que tienen estreñimiento refractario a los cambios en la dieta.
Se utiliza un enema de bario para el diagnóstico de la enfermedad de Hirschsprung, que generalmente se presenta en el período neonatal con retraso en el paso del meconio, pero que puede causar estreñimiento crónico refractario en la infancia. Una radiografía abdominal también puede mostrar ausencia de aire rectal y asas intestinales proximales dilatadas. A diferencia de este paciente, se esperaría retraso del crecimiento y distensión abdominal grave. Además, las imágenes no están indicadas en la evaluación del estreñimiento funcional, pero pueden considerarse en casos que son refractarios al tratamiento inicial.

Las fórmulas hidrolizadas, que contienen proteínas preprocesadas, menos alergénicas, se utilizan para alergias alimentarias como la alergia a la proteína de la leche de vaca. También se puede considerar la fórmula de soja. Este trastorno se presenta típicamente en bebés <6 meses con heces blandas que contienen moco y sangre mezcladas, sin hallazgos de fisuras anales al examen físico.

88
Q
  1. Masculino de 4 años, llevado a urgencias por su padre por fatiga persistente y que ha empeorado progresivamente. Hace tres semanas, el paciente presentó fiebre y congestión, esta última ya desapareció, pero la fiebre persistió. Además inicio con hiporexia y pérdida de peso. Se encuentra febril, taquicárdico, con altura y peso en +1 desviaciones estándar. Conjuntivas pálidas, adenopatías no dolorosas en las regiones cervical, axilar e inguinal, con hepatomegalia y petequias. La palpación profunda a lo largo del tercio medio de ambos fémures provoca dolor. Se observan múltiples petequias. En paraclínicos: Plaquetas 18.000, GB 2800.
    ¿Qué ayudas diagnósticas enviaría para confirmar el diagnóstico?
    A. Aspirado de médula ósea
    B. Electroforesis de hemoglobina
    C. Epstein Barr
    D. Anticuerpos antinucleares
A

Respuesta correcta: A

Tema-Ítem: Trastornos hematológicos-Leucemia linfoblástica aguda

Argumentación: Este paciente tiene pancitopenia, que generalmente es causada por una mayor destrucción celular o una producción celular deficiente. En este caso, el dolor óseo (que se traduce en fatiga) y las adenopatías difusas del paciente hacen sospechar leucemia linfoblástica aguda (LLA).

En la LLA, el crecimiento descontrolado de células malignas en la médula ósea da como resultado una producción deficiente de líneas celulares normales.
Aunque los síntomas iniciales de la LLA suelen ser inespecíficos (p. ej., fiebre, falta de apetito, pérdida de peso), el examen físico con frecuencia revela palidez y hematomas/petequias debido a anemia y trombocitopenia, respectivamente. Puede ocurrir leucopenia con infecciones recurrentes; sin embargo, a menudo se observa leucocitosis debido a la diseminación de linfoblastos desde la médula ósea hacia la periferia. A medida que las células leucémicas proliferan y expanden la médula ósea, muchos niños desarrollan dolor óseo (p. ej., dolor a la palpación profunda, negativa a caminar) que afecta los huesos largos (p. ej., fémur, tibia). Además, la diseminación leucémica extramedular puede provocar linfadenopatía y hepatoesplenomegalia.

El aspirado de médula ósea es diagnóstica; en la médula normal, los blastos (células inmaduras) constituyen un pequeño porcentaje de células, mientras que el diagnóstico de LLA se confirma por un aumento anormal de linfoblastos (p. ej., >20%) en el aspirado de médula ósea.
Los anticuerpos antinucleares detectan lupus eritematoso sistémico (LES), un diagnóstico poco común en niños pequeños que puede causar pancitopenia además de fiebre, linfadenopatía y esplenomegalia. Sin embargo, encontraríamos edema/dolor en articulaciones, y el dolor a la palpación del tercio medio de un hueso largo en este paciente hace que este diagnóstico sea menos probable.

La mononucleosis infecciosa causada por el virus de Epstein-Barr (VEB) puede presentarse con fiebre y fatiga prolongadas, así como linfadenopatía difusa y esplenomegalia; puede ocurrir pancitopenia. En contraste con este caso, encontraríamos adenopatías dolorosas y no se observaría dolor óseo.

La electroforesis de hemoglobina es diagnóstica de la enfermedad de células falciformes, que a menudo cursa con esplenomegalia en niños pequeños y puede presentarse con fiebre y dolor óseo (es decir, crisis de dolor vasooclusivo) en el contexto de una infección (p. ej., parvovirus). Aunque el parvovirus puede causar crisis aplásica en estos pacientes, esta complicación se caracteriza por anemia grave, no por linfopenia ni trombocitopenia. Además, no se observa linfadenopatía difusa.

89
Q

Paciente masculino de 58 años de edad, obrero, consulta por dolor lumbar desde hace 2 días. Refiere que el dolor se irradia a miembro inferior derecho con incapacidad para caminar en puntas con esa pierna. Signos vitales normales. Al examen físico hay debilidad a la extensión de hallux, parestesia plante y disminución del reflejo aquiliano derecho. ¿Cuál es la etiología más probable de la condición actual de este paciente?
A. Radiculopatía S1.
B. Radiculopatía L5.
C. Compresión del nervio tibial posterior.
D. Afección del nervio femoral

A

Respuesta correcta: A

Tema e ítem: Dolor lumbar y radiculopatía lumbar

Argumento: El dolor radicular lumbar constituye una causa frecuente de consulta en urgencias. Este síntoma puede ser originado por razones de tipo no compresivo, tales como la radiculitis inflamatoria, el herpes zoster, las mono o polineuropatías, la infección por VIH, entre otras causas comunes. Hablando de la radiculopatía de tipo compresivo, tenemos un importante grupo de enfermedades entre las que se cuentan: procesos infecciosos como la espondilodiscitis, patología neoplásica tanto primaria como metastásica, trauma, enfermedad degenerativa de la columna y por supuesto, la hernia discal lumbar. Una radiculopatía asociada a hernia discal lumbar (HDL) se define como el desplazamiento del disco intervertebral que comprime una o varias raíces espinales originando dolor y/o paresia que se distribuye en un dermatoma o miotoma. Con frecuencia la HDL produce solo dolor, pero en los casos más graves, los síntomas pueden ser motores y multirradiculares. El examen físico de un paciente con sospecha de radiculopatía lumbar asociada a hernia discal debe estar precedido por un adecuado interrogatorio. En el caso específico de la radiculopatía por HDL siempre debe documentarse el signo de Lasègue: La forma correcta de realizar la maniobra es con el paciente en decúbito supino y la rodilla extendida, se eleva la extremidad hasta 45°; si el paciente reproduce o exacerba los síntomas siguiendo un dermatoma puede entenderse que la prueba es positiva. El Lasègue invertido y el Slump (Lasègue sentado) son maniobras complementarias que pueden elevar la sensibilidad del examen. En el caso del Lasègue invertido, al hacer la maniobra en la extremidad no afectada el paciente refiere dolor en el miembro inferior sintomático y en el caso del Slump el paciente se examina sentado con flexión del tronco y extensión del miembro inferior sintomático buscando reproducir el síntoma. La raíz afectada puede ser diagnosticada teniendo en cuenta el miotoma, dermatoma y reflejo osteotendinoso especifico. En este caso la afección de S1 produce dolor distribuido en la cara posterior de la pierna y posterolateral del pie, con cambios sensitivos en la pierna posterolateral, cara lateral del pie y la panta. Se asocia además a debilidad a la plantiflexión, flexión de la rodilla, extensión de la cadera y hay anomalías en el reflejo aquileano.

90
Q

Andrea de 27 años, sin antecedentes relevantes, acude a su consulta por presentar un “bulto”palpable en mama izquierda detectado por autoexamen hace 3 días. Refiere FUM hace 1 semana, ciclos regulares; 25-27/4-5. Al examen físico se evidencia un nódulo de 1.7 cm en la coordenada 2-5, móvil, márgenes definidos, sin cambios inflamatorios en la piel circundante y sin adenopatías regionales o axilares.
¿Cuál sería la conducta adecuada a a seguir?
A. Tranquilizar y educar a la paciente sobre dicha lesión haciendo énfasis a una posible etiologia benigna y cita de control en 1 mes
B. Realizar ecografía
C. Programar punción con aguja fina (ACAF) de la lesión
D. Solicitar mamografía + ecografía

A

Respuesta correcta: B

Tema e ítem: Enfoque del paciente con nódulo mamario - ¿Qué hacer?

Argumento: Pregunta sencilla. Aunque usualmente los nódulos mamarios (NM) son benignos, el examen físico per se nunca será suficiente para descartar malignidad. Lo anterior, implica aplicar la triada diagnóstica que consiste en realizar un examen físico completo, evaluación por imágenes y estudio histológico. Esto debido a que los tres estudios combinados realizados adecuadamente y con resultados concordantes, se aproximan a una precisión diagnóstica del 100%.

La conducta a seguir en esta paciente sería solicitar estudios imagenológicos que permitan caracterizar el nódulo mamario. La recomendación general es solicitar de entrada una mamografía en mujeres > 35 años acompañada de ecografía. En mujeres < 35 años se solicita únicamente ecografía.

La ecografía tiene una sensibilidad del 89 % y especificidad del 78 % para detección de cáncer, y es más eficaz para detectar lesiones en mamas con tejido denso. Además, tiene la gran ventaja respecto a la mamografía de diferenciar si las lesiones densas son quistes o masas sólidas y es la razón por la que se recomienda en complemento en pacientes > 35 años.

91
Q

Femenina de 30 años se encuentra en su tercer día de manejo conservador ante un diagnostico de obstrucción intestinal. Refiere sentir mejora de dolor abdominal con la administración de morfina. Tiene antecedente de apendicetomía hace 3 años por una apendicitis perforada. Signos vitales hoy: FC de 100 lpm y PA de 100/70 mmHg. La sonda nasogástrica presenta 600 mL de líquido intestinal en las ultimas 24 horas. Al examen físico el abdomen se encuentra distendido, peristalsis disminuida y timpánico. No hay signos de irritación peritoneal. La rx seriada de abdomen con medio hidrosoluble evidencia medio de contraste hasta intestino delgado distal sin llegada al ciego a las 72h. ¿Cuál es el manejo más adecuado?
A. Esperar 24 horas para verificar la llegada de medio de contraste al ciego.
B. Se encuentra indicado el manejo quirúrgico.
C. Se debe continuar con manejo médico y observación clínica por 24 horas más.
D. Realizar una TAC de abdomen para descartar complicaciones.

A

Respuesta correcta: B

Tema e ítem: Manejo médico y quirúrgico de la obstrucción intestinal.

Argumento: El manejo de la obstrucción intestinal tiene como objetivo resolver alteraciones del flujo intestinal, aliviar los síntomas, corregir trastorno electrolíticos y ácido base, corregir la deshidratación y definir de forma oportuna los pacientes que requieren manejo quirúrgico. Aprox. el 85% de los pacientes con bridas (Causa principal de obstrucción intestinal) resolverán su cuadro con manejo médico. El manejo médico o conservador se basa en medidas de soporte básicos: Nada vía oral, sonda nasogástrica a libre drenaje, analgésicos, líquidos endovenosos, reposición de electrolitos y restauración del equilibrio ácido-base.

El seguimiento conservador debe hacerse idealmente por parte del servicio de cirugía general, donde debe evaluarse la mejoría de la distensión abdominal y el dolor, presencia de flatos y deposiciones, así como la vigilancia del débito de la sonda nasogástrica que debe ser meno de 500 cc en las ultimas 24 de 72 horas de haber iniciado el tratamiento. El seguimiento de paraclínicos se hará en casos puntuales: deterioro clínico o si tenía alteraciones previas que requieran de seguimiento.

La valoración urgente por cirugía general para definir si se requiere o no de manejo quirúrgico se encuentra indicada en pacientes en quienes se sospecha de compromiso intestinal por isquemia, necrosis o perforación, o aquellos que cursar con etiologías que requieren corrección quirúrgica de obstrucción en asa cerrada, hernia encarcelada, vólvulos, intususcepción, íleo biliar, cuerpos extraño o tumores. El fracaso del manejo médico se presenta en aprox. 25% de los pacientes. Se define ante síntomas obstructivos persistentes a los 3 días de seguimiento, débito de la sonda nasogástrica mayor a 500 cc/día en este mismo periodo, y si hay exacerbación de síntomas y/o sospecha clínica de complicaciones como isquemia o peritonitis.

Este paciente presenta un debito elevado por la sonda nasogástrica, persiste con abdomen distendido, timpánico y con peristalsis disminuida a las 72 horas de tratamiento, por lo tanto, el manejo médico ha fracasado y este paciente requiere de manejo quirúrgico.

92
Q

1) Samuel tiene 10 años y llega a la consulta por otalgia derecha desde hace 3 días y desde hoy, parálisis facial. Fue automedicado con gotas de glicerina para realizar lavado del oído, sin mejoría del cuadro. El paciente no presenta antecedentes relevantes. Se encuentra afebril, con dificultad para cerrar el ojo derecho y parálisis en hemicara derecha. El resto del examen neurológico es normal. La otoscopia derecha muestra erupciones costrosas en el meato auditivo externo. ¿Cuál de las siguientes es la causa más probable de la condición actual de este paciente?
A. Infección neumocócica de la mastoides
B. Infección invasiva por P. aeruginosa
C. Síndrome de Ramsay Hunt
D. Otitis externa fúngica

A
  1. Respuesta correcta: C

Tema-Ítem: Síndrome de Ramsay Hunt – Manifestaciones clínicas

Argumento: El herpes zoster ótico es una manifestación de la reactivación latente del virus varicela-zoster (VZV). La infección primaria por VZV se presenta con una erupción vesicular difusa asociada con fiebre, malestar general y faringitis. Luego, el virus migra a través de los nervios sensoriales hasta los nervios craneales y los ganglios espinales dorsales, donde permanece inactivo durante años. La reactivación del virus ocurre típicamente en un solo ganglio y causa una erupción vesicular dolorosa en una distribución dermatomal. La reactivación ocurre comúnmente en adultos y rara vez ocurre en niños, ya sea después de una infección primaria, vacunación o infección materna durante el embarazo.

El herpes zoster ótico es causado por la reactivación del VZV latente en el ganglio geniculado, lo que altera la función del nervio facial. Un exantema vesicular en el conducto auditivo externo (CAE) es clásico. La diseminación posterior al nervio vestibulococlear (VIII par) puede provocar alteraciones auditivas (p. ej., pérdida de audición, hiperacusia, tinnitus) y vestibulares (p. ej., vértigo, náuseas/vómitos).

El tratamiento es con corticosteroides y medicamentos antivirales. El inicio temprano (<3 días) se asocia con una mejor recuperación de la función del nervio facial; sin embargo, incluso con tratamiento muchos pacientes tienen debilidad facial residual. Como en todos los pacientes con parálisis del nervio facial, la protección del ojo en el lado paralizado de la cara (p. ej., lágrimas artificiales, ungüentos lubricantes) es de suma importancia.

93
Q

2) Salomé tiene 6 meses y llega al consultorio para una consulta de crecimiento y desarrollo. La paciente recibe lactancia materna exclusiva y recientemente ha comenzado a comer alimentos sólidos en puré. Balbucea, pero no dice “mamá” ni “papá”. Su hermano de 2 años tiene un retraso en el habla y se encuentra en terapia de lenguaje. Sus medidas antropométricas se encuentran adecuadas. La paciente pasa un juguete de su mano izquierda a la derecha y se sienta sin apoyo, pero después de unos segundos cae hacia adelante. Cuando su padre la llama por su nombre, ella se vuelve hacia él y sonríe. No puede ponerse de pie y no gatea. Los reflejos de Moro y prensión están ausentes. Tiene Babinski presente bilateralmente. ¿Cuál de las siguientes es la clasificación más adecuada de acuerdo con los hallazgos?
A. Retraso motor grueso, motor fino y cognitivo
B. Retraso motor grueso, sin alteración en motor fino y cognitivo
C. Retraso motor fino, sin alteración en motor grueso y cognitivo
D. Retraso motor fino, motor grueso y cognitivo sin alteraciones

A
  1. Respuesta correcta: D

Tema-Ítem: Neurodesarrollo – Hitos

Argumento: Cada evaluación de rutina del niño sano debe incluir una evaluación de los hitos del desarrollo, ya que el reconocimiento rápido permite una intervención temprana con terapias para optimizar tanto la adquisición de habilidades como la salud general a largo plazo.

Esta paciente demuestra habilidades de desarrollo motoras y cognitivas apropiadas para un paciente de 6 meses. A los 6 meses, el niño debería poder sentarse momentáneamente con las manos apoyadas y comenzar a sentarse sin apoyo, como se observa en esta paciente. Pasar objetos de una mano a otra también es un hito motor de los 6 meses; sin embargo, no se espera que gatee y se ponga de pie hasta dentro de unos meses. Esta niña reconoce a sus padres y responde apropiadamente a su nombre, pero llora cuando el médico la examina. La ansiedad ante los extraños es una fase normal del desarrollo cognitivo y social que puede comenzar a los 6 meses y durar hasta los 18-24 meses.

A los 6 meses de edad, los reflejos primitivos (p. ej., Moro, prensión) ya han desaparecido cuando el bebé inicia movimientos intencionados de las extremidades. El reflejo de protrusión de la lengua también desaparece entre los 4 y 6 meses de edad, lo que permite la coordinación para ingerir alimentos sólidos a esta edad. El reflejo de Babinski es el único reflejo primitivo que puede persistir en niños sanos hasta los 2 años de edad, pero que puede desaparecer tan pronto como los 12 meses.

94
Q

3) Paciente de sexo masculino de 5 días, hospitalizado en UCIN. Nació a las 32 semanas de gestación por parto vaginal. Se encuentra con alimentación enteral con leche materna y fórmula para prematuros. Inició con vómito súbito, posterior a las últimas 3 tomas; los dos primeros eran verdes y el último tenía aspecto de cuncho de café. Se encuentra afebril, taquicárdico, somnoliento, pero alertable. El abdomen está moderadamente distendido y parece doloroso a la palpación, con disminución de movimientos intestinales. La radiografía de abdomen revela neumatosis intestinal. ¿Cuál es el mejor siguiente paso en el tratamiento de este paciente?
A. Hemocultivos e inicio de antibiótico
B. Continuar solo con leche materna
C. Laparotomía inmediata
D. Inicio de esteroides IV

A
  1. Respuesta correcta: A

Tema-Ítem: Enterocolitis necrotizante – Diagnóstico oportuno y manejo

Argumento: La neumatosis intestinal se refiere al aire en la pared intestinal y es el hallazgo radiológico característico de la enterocolitis necrotizante (ECN).

La ECN es una afección potencialmente mortal que se presenta principalmente en bebés prematuros y de muy bajo peso al nacer debido a la invasión de bacterias productoras de gas en la pared de la mucosa intestinal. La inflamación y la necrosis intestinal causan intolerancia alimentaria (p. ej., residuos gástricos, vómitos biliosos), abdomen doloroso y distendido y hemorragia gastrointestinal (p. ej., hematoquezia, hematemesis), como se observa en este paciente. Sin un reconocimiento y tratamiento oportunos, la infección progresiva puede provocar shock séptico, perforación intestinal y la muerte.

El tratamiento inmediato de la ECN incluye reposo intestinal (es decir, interrupción de la alimentación enteral, descompresión gástrica, nutrición parenteral), líquidos intravenosos y antibióticos empíricos de amplio espectro. Debido a que la bacteriemia es común en la ECN, lo ideal es obtener hemocultivos antes de administrar antibióticos para ayudar a guiar el tratamiento. La respuesta al tratamiento se controla de cerca mediante análisis de sangre seriados, exámenes abdominales e imágenes.

95
Q

4) Carlos tiene 15 años, no tiene ningún antecedente patológico. Acude a su consulta por un cuadro de 3 meses de dolor y edema en hombro izquierdo, que han empeorado de forma progresiva. El paciente levanta pesas y juega béisbol 5 días a la semana. Ha tomado acetaminofén e ibuprofeno con poca mejoría. Al examen físico, se palpa una masa dolorosa en el húmero proximal izquierdo, con un rango de movimiento normal. Trae paraclínicos: FA, VSG y LDH elevadas, con rx de hombro derecho donde se evidencia lesión en húmero proximal, con reacción perióstica en sol naciente y triángulo de Codman. ¿Cuál es el diagnóstico más probable?
A. Sarcoma de Ewing
B. Osteoma osteoide
C. Osteosarcoma
D. Osteomielitis

A
  1. Respuesta correcta: C

Tema-Ítem: Osteosarcoma – Características comunes de los tumores óseos

Argumento: Lo más probable es que el paciente tenga un osteosarcoma, el tumor óseo primario más común que afecta a niños y adultos jóvenes, principalmente entre los 13-16 años. En los niños, el tumor se presenta con mayor frecuencia en las metáfisis de los huesos largos, como el fémur distal, la tibia proximal y el húmero proximal. Los síntomas constitucionales como fiebre, pérdida de peso y malestar general suelen estar ausentes. En el examen físico, el hallazgo más importante es una masa dolorosa de tejido blando.

Los hallazgos radiológicos característicos incluyen un patrón espiculado en forma de “rayo de sol” o “sol naciente” y una elevación del periostio conocida como triángulo de Codman. La fosfatasa alcalina y la lactato deshidrogenasa aumentan debido al recambio de osteocitos dañados; niveles altos pueden correlacionarse con un pronóstico adverso. El aumento de la velocidad de sedimentación globular es un marcador inespecífico de inflamación. El tratamiento incluye extirpación del tumor y quimioterapia.

96
Q

5) Paciente de sexo femenino de 2 días de vida, con 39 semanas de edad gestacional por parto vaginal espontáneo, sin complicaciones. La madre recibió anestesia epidural. Desde su nacimiento, la niña se encuentra hiporéxica, con regurgitación constante y distensión abdominal. Aún no ha eliminado meconio. Al examen físico, se evidencia un hoyuelo sacro de 5 mm de diámetro. Se realiza un tacto rectal con evidencia de aumento del tono y posterior liberación de heces y gases. Se toma una radiografía de abdomen con evidencia de asas intestinales dilatadas y ausencia de aire en recto, sin aire libre infradiafragmático. De acuerdo al diagnóstico sospechado, ¿Cuál sería la fisiopatología?
A. Obstrucción intestinal por heces espesas
B. Falla en la migración de las células de la cresta neural
C. Defectos en cierre del tubo neural
D. íleo intestinal por exposición a analgesia epidural en la madre

A
  1. Respuesta correcta: B

Tema-Ítem: Enfermedad de Hirschprung – Implicaciones y fisiopatología

Argumento: Se debe sospechar enfermedad de Hirschsprung (EH) o megacolon agangliónico congénito, en recién nacidos con retraso en la eliminación de meconio porque prácticamente todos los recién nacidos a término defecan dentro de las 48 horas posteriores al nacimiento.

Esta patología resulta de una migración fallida de las células de la cresta neural durante el desarrollo fetal del sistema nervioso entérico. El segmento intestinal afectado, más comúnmente el colon rectosigmoide, carece de inervación y de capacidad para relajarse y, por lo tanto, permanece crónicamente contraído. Aunque suele ser un defecto aislado, la EH también está fuertemente asociada con el síndrome de Down.

Los recién nacidos presentan mala alimentación, distensión abdominal, dificultad para eliminar el meconio y vómito bilioso. El tacto rectal muestra aumento del tono y puede producir una expulsión explosiva de gases y heces (“squirt sign” o signo de la explosión”) debido al alivio temporal de la obstrucción.
El enema de contraste muestra una zona de transición entre el segmento agangliónico estrechado y el colon dilatado (megacolon), normalmente inervado. La ausencia de células ganglionares en la biopsia por succión rectal es diagnóstica y el tratamiento es la resección quirúrgica del segmento agangliónico.

97
Q
  1. Masculino de 31 años, consulta al programa control de diabetes. Tiene antecedente de un episodio reciente (5 semanas) de cetoacidosis diabética sin etiología identificada, siendo dado de alta con esquema basal. Refiere cambios en su plan alimentario y perdida de 5 kg de peso desde el alta con disminución gradual de la dosis de insulina. También menciona que su madre y hermana han sido diagnosticados de DM2. Registra glucometrías con glicemias en ayunas entre 95-98 mg/dL y postprandial de 110 mg/dL. Al examen fisico presenta signos vitales estables, IMC 30, moderado panículo adiposo de distribución central y acantosis nigricans. Trae reporte de analítica: Hb1Ac 5.8%, péptido C 3,2 ng/mL (VR: 0,8-3,1 ng/mL), glicemia en ayunas 105 mg/dL, anticuerpos Anti GAD negativos. Se decide suspender el esquema basal.
    ¿Cuál de los siguientes tratamientos debe ser iniciado en este paciente?
  2. Dapaglifozina
  3. Continuar con el plan alimentario actual y cita de control en 1 mes
  4. Glimepiride
  5. Metformina
A

(C)

Respuesta correcta: C

Tema e ítem: Diabetes mellitus con predisposición a la cetoacidosis - ¿Qué tan común es?

Argumento: El término “diabetes con predisposición a la cetosis” incluye varios síndromes glicémicos y también se conoce como diabetes mellitus tipo 2 propensa a la cetosis, “diabetes de Flatbush”, diabetes idiopática tipo 1, diabetes tipo 1B y diabetes atípica. Estos síndromes cursan con cetoacidosis diabética (CAD) episódica como consecuencia del déficit de insulina, y también puede tener presentación de periodos variables de dependencia e independencia de esta. En los individuos con diabetes propensos a la cetosis, es necesario un tratamiento con insulina para la CAD hasta que ésta se haya resuelto disminuyendo la toxicidad glicémica en las células β y puedan producir cantidades suficientes de insulina para inhibir la lipólisis. La evaluación de la reserva de células β con un nivel de péptido C en ayunas debe realizarse semanas o meses después del episodio de CAD. Los pacientes también deben estudiarse como una posible diabetes tipo 1 con la medición del péptido C en ayunas o anticuerpos Anti-GAD. Este paciente presenta las características clínicas de la diabetes de tipo 2 (resistencia a la insulina, obesidad, antecedentes familiares importantes), una función intacta de las células β, anticuerpos negativos y recibe dosis relativamente bajas de insulina para su peso corporal. En este caso, la conducta adecuada a seguir en esta paciente es suspender el esquema basal e iniciar manejo oral con metformina.

  • Dado el episodio previo de CAD, los cambios en el estilo de vida por sí solos sin tratamiento farmacológicos son ineficientes para mantener las metas logradas en este paciente.
  • No hay evidencia actual de que los iSGLT2 sean un tratamiento eficaz en la diabetes propensa a la cetosis. Además, esta clase de medicamentos conlleva un mayor riesgo de CAD euglucémica, por lo que no sería una opción adecuada.
  • La glimepirida es una sulfonilurea asociada al aumento de peso que suele ser entre 2.5-5 kg y está relacionado con el incremento de la insulina plasmática. No es la mejor opción para este paciente con obesidad, que se beneficiaría más de un sensibilizador a la insulina como la metformina.
98
Q
  1. Varón de 45 años, quien consulta al Servicio de Urgencias por un cuadro de 1 día evolución caracterizado por dificultad respiratoria asociada a hipodinamia y astenia. Tiene antecedente de diabetes tipo 2 y obesidad grado (P: 90 kg). Medicamentos: metformina, empaglifozina, orlistat. Al ingreso se encuentra alerta, orientado en las tres esferas, PA 110/75 mh, FC 115 lpm, taquipnea con SatO2 96% a aire ambiente, crépitos en base pulmonar derecha, glucometría 175 mg/dL. Analítica: pH 7.25, HCO3 16, glucosuria, cetonemia, Cr: 1.3 mg/dL, Na+: 138 mEq/L, K+: 4.7 mEq/L. Rx de tórax: opacidad alveolar en base derecha. Además de iniciar terapia antibiótica, ¿Cuál de los siguientes manejos también deben ser iniciado en este paciente?
    A. Iniciar SSN 0.9% con DAD 10% en infusión a 125 ml/hora + 9U/h insulina cristalina
    B. Iniciar SSN 0.9% 1500 mL en la primera hora + 9U/h insulina cristalina
    C. Iniciar SSN 0.9% con DAD 5% 1000 mL en la primera hora + 5U/h insulina cristalina
    D. Iniciar SSN 0.9% 1500 mL en la primera hora, luego continuar a razón de 250 ml/h + 14U/h de insulina cristalina
A

(A)

Respuesta correcta: A

Tema e ítem: Cetoacidosis euglicémica (CAE) – Situaciones especiales en el servicio de Urgencias

Argumento: Tal como su nombre lo indica, la CAE es una cetoacidosis diabética (CAD) en pacientes con glucemias normales, la cual ha venido en aumento después de que los iSGLT2 se hicieran disponibles como estrategia terapéutica.
El diagnóstico de esta complicación suele retrasarse por la ausencia de hiperglicemia (de ahí el refrán: no diagnosticamos lo que no leemos), sin embargo, la CAE debe sospecharse en cualquier paciente en tratamiento con iSGLT2 (empaglifozina, dapaglifozina) que se presente con alteración del estado mental, poliuria, glucosuria, taquipnea, taquicardia o cetosis, sobre todo si se presenta con un factor de riesgo asociado. Los principales factores de riesgo que predisponen a un paciente en terapia con iSGLT2 a sufrir CAE son la falta de adherencia al tratamiento médico, infección (nuestro paciente cursa con una neumonía adquirida en la comunidad), cirugías mayores y una diabetes autoinmune inadvertida en un paciente previamente diagnosticado como DM2.

El cotransportador de sodio y glucosa (SGLT2) presente en el túbulo contorneado proximal, facilita la reabsorción de glucosa. Los iSGLT2 bloquean su funcionamiento, por lo que disminuyen la glicemia al aumentar la eliminación de glucosa por la orina; se ha evidenciado que, de manera concomitante, reducen desenlaces cardiovasculares y renales. Los mecanismos que podrían explicar el aumento de las cetonas con los iSGLT2 son múltiples, entre ellos un aumento de su reabsorción, y un incremento de su producción a partir de la estimulación de la lipólisis.

El tratamiento de la CAE se hace de manera exactamente igual al de la CAD con unas excepciones. Se recomienda iniciar SSN con glucosa al 10 % de inmediato a un flujo de 125 ml/hora porque la glucosa está <250 mg/dl. La tasa de infusión de la insulina cristalina será a 0,1 unidades/Kg/hora. Si la glucemia desciende a pesar de la suplencia con dextrosa, se debe reducir el goteo de insulina a 0,05 unidades/kg/hora para evitar la hipoglicemia (la cual aumenta la mortalidad). Recuerda que el objetivo principal es detener lipólisis y cetogénesis, NO es disminuir la glicemia.

99
Q
  1. Masculino de 61 años, sin antecedentes de interés, es evaluado por un historial de 3 meses de evolución consistente en fatiga creciente e hiporexia. El examen físico es anodino. Trae reporte de estudios realizados recientemente: Hto 28%, Cr 1.6 mg/dL, glucosa plasmática 101 mg/dL, Na+ 130 mEq/L, K+ 3.8 mEq/L, Ca+ 10.5 mg/dL, albumina 3.0 g/dL, osmolalidad plasmática 290 mOsm/kg H2O, proteínas totales 9.7 g/dL, osmolaridad urinaria 240 mOsm/kg H2O, Na+ urinario: 45 mEq/L.
    ¿Cuál es la causa más probable de la hiponatremia del paciente?
    ¿Cuál de las siguientes es la causa más probable de la hiponatremia de este paciente?
    A. síndrome de secreción inadecuada de ADH
    B. Ingesta ineficiente de sodio en la dieta
    C. Mieloma múltiple
    D. Polidipsia psicógena
A

(C)

Respuesta correcta: C

Tema e ítem: Enfoque etiológico de la hiponatremia

Argumento: Cuando se evalúa a un paciente con hiponatremia, el primer paso siempre será medir la osmolalidad plasmática. En este paciente, la osmolalidad sérica es normal (VR: 275-295 mOsm/kg H2O), por lo tanto, tiene una hiponatremia isotónica. La hiponatremia isotónica es un artefacto de laboratorio causado por el método con el que se mide el sodio plásmatico. El plasma se divide en una fase acuosa (93%), en la que se disuelve el sodio, y una fase sólida (7%), compuesta por lípidos y proteínas. Es decir, la fase solida puede aumentar por una hipertrigliceridemia grave o por incremento de las proteínas, como las que se encuentran en el mieloma múltiple, disminuyendo así el porcentaje de la fase acuosa. La etiología de la hiponatremia de este paciente se enfoca más al mieloma múltiple por la evolución crónica del cuadro clínico referido y por los paraclínicos que muestran Ca++ sérico elevado, anemia, proteínas totales aumentadas e hipoalbuminemia.

La mayoría de los laboratorios utilizan un electrodo de dilución en el que la muestra se diluye diez veces al medir los electrolitos. Al disminuir la fase acuosa, la dilución ya no es 1:10, sino incluso mayor, lo que da lugar a un valor reducido del Na+.

100
Q
  1. Varón de 65 años, consulta al servicio de Urgencias por cuadro clínico de 1 día de evolución dado por dolor en miembro inferior derecho, progresivo, 8/10 en la EAD. Antecedentes: diabetes tipo 2, HTA, enfermedad arterial periférica. Medicamentos: metformina, clortalidona, atorvastatina, cilostazol, ASA. Examen físico: FC 110 lpm e irregular, PA 110/75 lpm, FR 20 rpm, ruidos cardiacos arrítmicos. El miembro inferior derecho presenta temperatura local disminuida, coloración cianótica, sensibilidad conservada, amplitud de movimiento pasivo conservado, pulso poplíteo y tibial ausentes. EKG: ausencia de ondas P y R-R irregular.
    ¿Cuál de las siguientes es la conducta inicial más adecuada?
    A. Administrar alteplase IV
    B. Realizar angiografía
    C. Administrar heparina no fraccionada IV
    D. Iniciar apixaban VO
A

(C)

Respuesta correcta: C

Tema e ítem: Manejo de la isquemia aguda de extremidades en el servicio de Urgencias

Argumento: Clásicamente, los pacientes con isquemia aguda de las extremidades (IAE) presentan al menos una de las ‘’seis P’’:
Parestesia, dolor, palidez, ausencia de pulso, fría a la palpación y parálisis.

La causa más frecuente de la isquemia aguda de las extremidades es la trombosis arterial aguda en miembros inferiores, trombosis del stent o un injerto de bypass. Otras causas son la tromboembolia, disección vascular (que suele producirse periprocedimiento) o un traumatismo. Estamos indudablemente frente a un caso compatible con IAE debida a un tromboembolismo secundario a la fibrilación auricular de este paciente, y la conducta adecuada a seguir es:
(1) Iniciar anticoagulación intravenosa con heparina no fraccionada como elección
(2) Realizar angiografía para confirmar diagnóstico
(3) Establecer un plan de reperfusión para lograr la preservación del miembro inferior

Es necesario resaltar que la angiografía invasiva de miembro inferior debe realizarse en el menor tiempo posible, ya que la revascularización endovascular o quirúrgica está justificada para preservar la función de la extremidad y evitar una amputación mayor. Sin embargo, la anticoagulación, normalmente con heparina no fraccionada, se debe iniciar tan pronto como se sospeche el diagnóstico, antes de realizar los estudios pertinentes.
* Este paciente con fibrilación auricular necesitará anticoagulación indudablemente. Sin embargo, dado que es necesario intervenir para restablecer el flujo sanguíneo, debe iniciarse ahora la heparina no fraccionada, y no un anticoagulante oral como el apixabán. La heparina no fraccionada tiene un inicio de acción rápido, lo que permite una mayor flexibilidad en el ajuste de la dosis o su interrupción cuando es necesaria para procedimientos invasivos.

  • Sé que pensaste en la trombólisis como manejo inicial, pero infortunadamente y a diferencia del SCACEST/TEP grave/ACVi en tiempo de ventana, la trombólisis IV en este contexto no ha demostrado ser eficaz en la mayoría de los pacientes con isquemia aguda de las extremidades. Sin embargo, la trombólisis con catéter es eficaz en pacientes con isquemia aguda de las extremidades y una extremidad salvable.
101
Q
  1. Don Pablo de 67 años, consulta al programa de control. Tiene antecedentes de diabetes tipo 2 y enfermedad renal diabética estadio G3b. Está actualmente en tratamiento con valsartan/amlodipino y metformina. Al examen físico presenta PA 130/85 mh, FC 91 lpm y regular. La exploración física es anodina. Analítica: Hb1Ac 8.4%, K+: 4.2, TFG 35 ml/min, relación proteína/creatinina en orina 2550 mg/g.
    ¿Cuál de los siguientes es el mejor medicamento para intensificar la terapia antidiabética en esta paciente?
    A. Dapaglifozina
    B. Pioglitazona
    C. Lixisenatida
    D. Linagliptina
A

(A)

Respuesta correcta: A

Tema e ítem: Tratamiento de la diabetes mellitus (DM) y enfermedad renal crónica (ERC)

Argumento: Los individuos con DM y ERC están en riesgo de complicaciones relacionadas con la diabetes tanto agudas como crónicas incluidas hipoglucemia, crisis hiperglucémicas, retinopatía, neuropatía, compromiso cardiovascular e insuficiencia renal avanzada con necesidad de terapia de reemplazo o trasplante renal. Existe un número limitado de intervenciones que han demostrado ralentizar el deterioro de la función renal en pacientes con enfermedad renal crónica (ERC).

En las intervenciones relacionadas con el control glucémico, la guía KDIGO 2022 recomienda que en pacientes con DM2, ERC y TFG ≥30 ml/min por 1,73 m2 la terapia recomendada de primera línea es metformina y un inhibidor del cotransportador de sodio-glucosa 2 (iSGLT2) como empaglifozina/dapaglifozina porque ralentizan la progresión de la ERC y reduce el riesgo de muerte por complicaciones renales o cardiovasculares. Incluso aunque la meta de HbA1c con metformina en monoterapia se haya logrado, se debe adicionar un iSGLT2 por el beneficio renal y cardiovascular.

En pacientes con reciente diagnóstico de DM2 y ERC se podría iniciar monoterapia con metformina con la introducción temprana de iSGLT2 o incluso inicio de terapia combinada con metformina e iSGLT2 con monitoreo continuo de eventos adversos.

De los agonistas del receptor GLP1 que han demostrado beneficio cardiovascular en ERC son liraglutida, semaglutida inyectable y dulaglutida; no se recomienda exenatida ni lixisenatida debido a que no han demostrado beneficio cardiovascular a pesar de su seguridad.

102
Q
  1. Femenina de 35 años, G3POA3, consulta por su pérdida recurrente de embarazo. Ha tenido 3 abortos espontáneos, todos en el primer trimestre y se encuentra preocupada ya que no cree poder llevar un embarazo al término. Sus menstruaciones son regulares con duración de 4 a 5 días con sangrado moderado y cólicos. Signos vitales normales. IMC de 24 kg/m2. Al examen físico el útero es pequeño, antevertido y no doloroso. Una ECO-transvaginal revela un fibroma submucoso de 2 cm en fondo uterino. El estudio para trombofilias es negativo. ¿Cuál es el manejo adecuado para prevenir nuevas perdidas?
    A. Administrar heparina de bajo peso molecular.
    B. Progesterona vaginal.
    C. Miomectomía histeroscópica.
    D. Inducción de ovulación con clomifeno.
A

Respuesta correcta: C

Tema e ítem: Miomatosis uterina – Manejo

Argumento: Esta paciente ha tenido 3 perdidas consecutivas de embarazo, es decir, abortos espontáneos, lo que la diagnostica con pérdida gestacional recurrente. En este grupo de pacientes, el manejo inicial incluye la realización de ecografía pélvica, cariotipo y pruebas de trombofilia porque los trastornos anatómicos, genéticos y trombóticos son las causas más comunes. La pérdida recurrente del embarazo de esta paciente probablemente se deba a su fibroma submucoso, un tumor benigno del músculo liso (miometrial) que puede causar una anatomía intrauterina anormal al extenderse hacia la cavidad o incidir (es decir, alterar) el endometrio. Esto compromete la implantación o el crecimiento del embrión, probablemente debido a la disminución del grosor del endometrio y la vascularización reducida del fibroma (en relación con el miometrio). Por lo tanto, el tratamiento más apropiado para la pérdida recurrente del embarazo debido a fibromas submucosos es la miomectomía histeroscópica (incluso si los fibromas no causan menstruaciones abundantes). Este procedimiento restaura la anatomía intrauterina normal, reduciendo así el riesgo de pérdida de embarazos en el futuro.

103
Q
  1. Femenina de 52 años consulta por prurito vulvar de una semana de evolución. Refiere micción frecuente, disuria y nicturia. Ha tenido síntomas similares 4 veces en los últimos 3 meses en los que se automedica con óvulos de venta libre que mejoran los síntomas, sin embargo, los síntomas reaparecen en las siguientes semanas. La paciente tenía períodos abundantes e irregulares que mejoraron después de la inserción de un dispositivo intrauterino que contenía progestina. Los exámenes de rutina que incluyen colonoscopia, mamografía y prueba de Papanicolaou eran normales hace 2 años. Es sexualmente activa y mantiene una relación monógama. Signos vitales: PA de 130/80 mm Hg y FC de 84/min. El IMC es de 25 kg/m2. El examen pélvico revela eritema vulvar y múltiples excoriaciones. Hay una secreción espesa y blanca por toda la cúpula vaginal. No se observan lesiones cervicales o vaginales. La microscopía revela pseudohifas y ausencia de organismos móviles. ¿Cuál de las siguientes pruebas adicionales se debe realizar en este paciente?
    A. Hemoglobina glicosilada.
    B. Prueba de amplificación de ácidos nucleicos.
    C. Biopsia vulvar.
    D. Medir niveles de FSH.
A

Respuesta correcta: A

Tema e ítem: Factores de riesgo para candidiasis vaginal recurrente.

Argumento: Esta paciente tiene candidiasis vulvovaginal que se caracteriza por flujo vaginal blanco y prurito vulvovaginal, secundario a Candida albicans. Otras características clínicas incluyen disuria e irritación vulvar, que provocan eritema vulvar y vaginal y excoriaciones que se observan en el examen físico. Una evaluación más detallada de la secreción generalmente revela un pH normal; En el microscopio se observan levaduras en ciernes y pseudohifas.

Los factores de riesgo de la candidiasis vulvovaginal incluyen niveles elevados de estrógeno (Ejemplo: embarazo), inmunosupresión y diabetes mellitus. La diabetes mellitus aumenta el riesgo de candidiasis ya que la glucosa facilita la adhesión de Candida a la superficie epitelial. Por lo tanto, las mujeres con infecciones recurrentes por candidiasis vulvovaginal (≥4 episodios en un año) y otros signos de diabetes mellitus (Ejemplo: nicturia, polaquiuria) deben evaluarse con una prueba de hemoglobina glicosilada. El fluconazol es el tratamiento para la candidiasis vulvovaginal y un mejor control de la glucemia puede reducir el riesgo de recurrencia.

104
Q
  1. Femenina de 16 años presenta periodos menstruales dolorosos desde la menarquia, sin embargo, se han hecho cada vez más dolorosas por lo que ha presentado ausentismo escolar. Su menstruación ocurre cada 25 – 27 días con 5 días de sangrando moderado con dolor tipo cólico en los primeros 4 días siempre en cuadrante inferior derecho que se irradia a flanco ipsilateral. El dolor mejora con AINEs y medidas térmicas. Presenta además fatiga y náuseas que inician el día anterior a la menstruación y desaparece cuando cesa el dolor. No tiene antecedentes y no es sexualmente activa. Su FUM fue hace 3 semanas. Signos vitales normales. No se realiza examen pélvico por no haber iniciado vida sexual pero el resto de examen físico es normal. ¿Cuál de los hallazgos sugiere una causa secundaria de dismenorrea?
    A. Síntomas asociados.
    B. Localización del dolor.
    C. Edad de inicio.
    D. Patrón de sangrado menstrual.
A

Respuesta correcta: B

Tema e ítem: Dismenorrea – Características clínicas.

Argumento: La dismenorrea, o menstruación dolorosa, es común en mujeres en edad reproductiva. Se divide en causas primarias (fisiológicas) y secundarias (patológicas). La dismenorrea primaria es la causa más común de menstruaciones dolorosas y típicamente se presenta en adolescentes después de que se establecen ciclos menstruales regulares y ovulatorios. Los pacientes suelen tener dolor pélvico en la línea media que puede irradiarse a las piernas bilaterales o la espalda, así como síntomas sistémicos asociados, como fatiga, náuseas, vómitos y diarrea. La dismenorrea primaria se puede diagnosticar basándose únicamente en la historia clínica y no requiere examen físico.

Por el contrario, las pacientes con cualquiera de las siguientes características clínicas deben ser evaluadas para detectar causas secundarias de dismenorrea:

  • Inicio de los síntomas a la edad >25 años.
  • Dolor pélvico unilateral (fuera de la línea media) (como se observa en el dolor en el cuadrante inferior derecho de este paciente que se irradia hacia el flanco derecho).
  • Sin síntomas sistémicos (ejemplo: fatiga, náuseas) durante la menstruación.
  • Sangrado uterino anormal (sangrado intermenstrual, manchado poscoital).

Estas características sugieren una variedad de causas patológicas de dismenorrea: enfermedad inflamatoria pélvica, endometriosis, masas anexiáleses y una anomalía uterina (cuerno uterino rudimentario). Se realiza un examen físico para evaluar estas posibles causas; los pacientes con hallazgos anormales en el examen físico (masa anexial, sensibilidad al movimiento cervical) pueden requerir imágenes adicionales (ecografía pélvica) y posible cirugía (laparoscopia).

105
Q
  1. Femenina primigravida de 32 años con 20 semanas de gestación consulta a su primera cita de control prenatal. Niega sangrado, perdida de líquido o actividad uterina. Sus exámenes resultan dentro de la normalidad. La paciente tiene un grupo sanguíneo O+ y su marido es AB+. El examen físico se encuentra normal para la edad. La paciente está preocupada por los diferentes grupos sanguíneos y el riesgo de complicaciones del embarazo. ¿Cuál de las siguientes es la respuesta más adecuada para este paciente?
    A. “La mayoría de los recién nacidos afectados por la incompatibilidad ABO sólo tendrán una enfermedad hemolítica leve”.
    B. “Usted es tolerante a los antígenos ABO de su hijo, por lo que su recién nacido no tendrá enfermedad hemolítica”.
    C. “Los antígenos ABO provocan sólo una respuesta IgM, por lo que no corres ningún riesgo con este embarazo”
    D. “Los anticuerpos contra los antígenos ABO no son hemolíticos”
A

Respuesta correcta: A

Tema e ítem: Incompatibilidad ABO durante el embarazo.

Argumento: Esta paciente con grupo sanguíneo O, cuyo marido es del grupo sanguíneo AB, tendrá un hijo con el grupo sanguíneo A o B. En estas situaciones de incompatibilidad ABO, los signos de enfermedad hemolítica suelen ser leves y aparentes sólo en aproximadamente un tercio de los bebés. Los bebés afectados suelen ser asintomáticos o levemente anémicos al nacer; dentro de las primeras 24 horas de vida pueden desarrollar ictericia (hiperbilirrubinemia), que normalmente responde a la fototerapia.

El sistema inmunológico de una mujer embarazada no está deprimido en general; La interacción entre los sistemas inmunológicos innato (neutrófilos, macrófagos) y adaptativo (anticuerpos) permite una respuesta adecuada en la interfaz feto-materna. La exposición a los antígenos A y B del feto provoca una respuesta IgM durante el embarazo, pero estos anticuerpos son pentámeros grandes que no pueden cruzar la placenta. Sin embargo, los pacientes con el grupo sanguíneo O están expuestos a los antígenos A y B a una edad temprana (por exposición a antígenos similares a A y B presentes en alimentos, bacterias y virus) y producen anticuerpos contra estos antígenos. Cuando una mujer con grupo sanguíneo O queda embarazada, ya tiene anticuerpos IgG anti-A y anti-B que atraviesan la placenta; por lo tanto, la incompatibilidad ABO puede afectar un primer embarazo.

106
Q
  1. Femenina de 24 años consulta por disuria y hematuria desde hace 3 días. Niega fiebre o tenesmo vesical. Niega dolor entre las micciones, refiere episodios similares desde hace 6 meses que inician días antes de la menstruación con posterior resolución espontanea, permanece asintomática y luego reaparece antes de la menstruación. Signos vitales normales. Al examen físico hemiabdomen inferior ligeramente doloroso a la palpación sin signos de irritación peritoneal. En el examen pélvico, la vejiga está sensible a la palpación de la pared vaginal anterior. Uroanálisis: Sangre moderada, Esterasa leucocitaria negativa, Nitritos negativos, Glóbulos blancos 5-10/hpf, Glóbulos rojos muchos/hpf El urocultivo es negativo. ¿Cuál de los siguientes es el diagnóstico más probable en este paciente?
    A. Endometriosis.
    B. Cistitis intersticial.
    C. Divertículo uretral.
    D. Nefrolitiasis.
A

Respuesta correcta: A

Tema e ítem: Endometriosis

Argumento: Esta paciente con disuria cíclica y hematuria asociada con la menstruación probablemente tenga endometriosis, la implantación ectópica de glándulas endometriales y estroma en la cavidad abdominopélvica. Aunque algunos pacientes son asintomáticos y se les diagnostica de manera incidental, aquellos que son sintomáticos a menudo tienen diferentes presentaciones y hallazgos en el examen que dependen de la ubicación de la endometriosis.

Es probable que este paciente tenga focos que se infiltran profundamente en la vejiga; Estos implantes endometriales se desprenden con la menstruación mensual, provocando dolor de vejiga y sensibilidad suprapúbica. Además, los pacientes pueden tener disuria con hematuria asociada y leucocitos en el análisis de orina debido a la inflamación de la vejiga. Los implantes endometriales ectópicos y la fibrosis posterior pueden causar la sensibilidad y/o nodularidad característica de la pared vaginal anterior, como se observa en esta paciente. Cuando termina el ciclo menstrual, los síntomas clásicamente desaparecen hasta el siguiente ciclo.

El diagnóstico definitivo de endometriosis se realiza mediante visualización directa y biopsia durante la exploración quirúrgica. El tratamiento conservador incluye anticonceptivos orales y antiinflamatorios no esteroides. La cirugía puede eliminar las lesiones visibles, pero la recurrencia es común.

107
Q
  1. Masculino de 20 años, consulta por miedo intenso que le genera una presentación académica en la Universidad, refiere “tengo miedo de que me vean ansioso””. El paciente informa que tuvo que irse en medio de su última presentación por diaforesis, taquicardia, rubefacción facial y quebrantamiento de la voz. El paciente refiere que es algo que le ocurre la mayoría de las veces que tiene un evento social o académico. Tiene antecedente de consumo de marihuana y alcohol; madre y padre con diagnóstico de ansiedad y depresión. Signos vitales: PA de 134/82 mm/Hg, FC de 86 lpm, FR de 12rpm. Al examen físico no hay hallazgos anormales. Al examen mental, paciente inicialmente tenso que se relaja a medida que avanza la entrevista. ¿Cuál es el manejo adecuado para este paciente?
    A. Ordenar Lorazepam
    B. Ordenar propranolol
    C. Ordenar paroxetina
    D. Ordenar buspirona
A

Respuesta correcta: B

Tema e ítem: Ansiedad social – Diagnóstico

Argumento: Este paciente que tiene ansiedad en eventos que requieran un desempeño académico, y que presenta síntomas sistémicos que le dificulta ejecutar su actividad en público, hace sospechar de un trastorno de ansiedad social exclusivo del rendimiento (Cumple con criterios del DSM-V). Los individuos con trastorno de ansiedad social exclusivo del desempeño no temen las situaciones sociales sin desempeño y, en general, no tienen ansiedad de carácter social. El tratamiento farmacológico del trastorno de ansiedad social exclusivo del rendimiento, incluye betabloqueantes o benzodiazepinas, según sea necesario, en lugar de medicación de mantenimiento. Los betabloqueantes (propanolol) ayudan a controlar la respuesta autónoma asociada (temblores, taquicardia, diaforesis).

Por otra parte, debido a su potencial adictivo, las benzodiazepinas (diazepam, lorazepam) generalmente se evitan en pacientes con antecedentes personales o familiares de trastorno por uso de sustancias. Además, no se recomiendan cuando el rendimiento podría verse afectado por la sedación y los efectos secundarios cognitivos (realizar una presentación, como en este caso). En cuanto a los inhibidores selectivos de la recaptación de serotonina (paroxetina) y los inhibidores de la recaptación de serotonina y norepinefrina (venlafaxina) se usan en el tratamiento del trastorno de ansiedad social que no se limita a situaciones de desempeño (es decir, generalizado) o si hay es depresión comórbida. Sería inapropiado prescribir una medicación diaria a esta paciente porque su ansiedad se limita a situaciones de desempeño y no tiene depresión. Finalmente, la buspirona se usa para tratar el trastorno de ansiedad generalizada, no el trastorno de ansiedad social.

108
Q
  1. Masculino de 19 años, consulta por lesión ocular izquierda de 1 hora de evolución, secundaria a trauma por objeto extraño mientras jugaba paintball. El paciente refiere que inmediatamente sintió dolor en el ojo izquierdo, se colocó una bolsa de hielo encima. Al examen ocular se aprecia hifema que ocupa más de la mitad de la cámara anterior del ojo izquierdo, junto a inyección conjuntival y disminución de la agudeza visual ipsilateral. ¿Cuál de las siguientes complicaciones es más probable se genere por el cuadro clínico de este paciente?
    A. Formación de cataratas.
    B. Queratitis infecciosa.
    C. Hipertensión intraocular.
    D. Hemorragia subconjuntival.
A

Respuesta correcta: C

Tema e ítem: Hipema- Complicaciones

Argumento: Este paciente que presentó un trauma ocular, que se presenta con hipema (acumulación de sangre en la cámara anterior del ojo), junto con dolor ocular y una disminución de la agudeza visual que se correlaciona con el tamaño del hipema, configuran el diagnóstico de un hipema traumático, entidad en la cual se debe prevenir complicaciones como el resangrado y la hipertensión intraocular, que pueden provocar lesión del nervio óptico (glaucoma) y pérdida permanente de la visión. Recordemos que los hipemas son más comunes después de un traumatismo ocular cerrado (paintball), pero también pueden ocurrir con una lesión penetrante (Cuando se aplica fuerza al ojo, la presión intraocular aumenta bruscamente, provocando la rotura de los vasos sanguíneos del cuerpo ciliar y del iris). Los pequeños hipemas generalmente se resuelven solos en unos días; sin embargo, se requiere la evacuación quirúrgica del coágulo en caso de un hipema grande y persistente o en caso de hipertensión intraocular que no responde a las terapias tópicas y orales.

En cuanto a la opción de respuesta de formación de cataratas, recordemos, es una entidad indolora que causa visión borrosa, la cual generalmente se desarrolla como una complicación de una afección inflamatoria (uveítis, escleritis). Pueden ocurrir cataratas traumáticas, pero más comúnmente se desarrollan después de una lesión de globo abierto (traumatismo penetrante), que típicamente se presenta con una pupila excéntrica o en forma de lágrima. Por otra parte, los factores de riesgo comunes de queratitis infecciosa incluyen el uso prolongado de lentes de contacto y la inmunosupresión, no un traumatismo cerrado ni un hipema. Finalmente, la hemorragia subconjuntival es una acumulación indolora de sangre debajo de la superficie del ojo que no afecta la agudeza visual. El sangrado puede ser espontáneo o secundario a un aumento de la presión venosa (maniobra de Valsalva), pero no es una complicación del hipema.

109
Q
  1. Femenina de 47 años, consulta por vértigo constante que en las últimas horas se ha asociado a náuseas y emesis, junto a inestabilidad al caminar. Antecedente de hospitalización por infección viral de tracto respiratorio superior con desarrollo de hipoacusia en el oído derecho. Signos vitales dentro de rangos normales. Al examen físico, la maniobra de Dix-Hallpike evidencio un nistagmo unidireccional del lado derecho. La prueba de impulso cefálico evidencia movimientos sacádicos. El resto del examen neurológico es normal. ¿Cuál es el diagnóstico más probable?
    A. Vértigo postural paroxístico benigno.
    B. Laberintitis
    C. Enfermedad de Meniere
    D. Síndrome de Ramsay Hunt.
A

Respuesta correcta: B

Tema e ítem: Laberintitis – Diagnóstico

Argumento: Esta paciente que presenta nistagmo unidireccional con la maniobra de Dix-Hallpike y movimientos sacádicos con la prueba del impulso cefálico sugieren un vértigo periférico. A su vez, la aparición de un cuadro de vértigo agudo sostenido posterior a una infección viral, nos debe hacer sospechar de una laberintitis (neuronitis vestibular o estibulopatía periférica aguda). Recordemos que la laberintitis es causada por la inflamación post viral de ambas ramas del nervio vestibulococlear (nervio craneal VIII), lo que resulta en vértigo persistente, severo y de inicio repentino, acompañado de pérdida de la audición.

En cuanto al vértigo posicional paroxístico benigno, es una causa frecuente de vértigo periférico; sin embargo, por lo general causa breves episodios de vértigo provocados por el cambio de posición, en lugar del vértigo continuo descrito por este paciente. Además, no suele asociarse a pérdida auditiva. Por otra parte, los síntomas de la enfermedad de Meniere típicamente incluyen vértigo episódico con aparición insidiosa de pérdida auditiva neurosensorial y tinnitus. La enfermedad de Meniere causaría hallazgos en el examen consistentes con vértigo periférico, pero sería poco común que la enfermedad de Meniere se presenta con vértigo constante que empeora rápidamente.

Finalmente, el síndrome de Ramsay Hunt (infección por herpes zoster que afecta al VII nervio craneal) causa vértigo periférico, pero tiene características clínicas diferentes a las de este paciente; suele presentarse con dolor de oído y vesículas en el conducto auditivo. También puede causar parálisis facial unilateral (por afectación del nervio craneal VII), pérdida del gusto o disgeusia (por afectación de la cuerda del tímpano de la rama del nervio craneal VII) y alteración de la audición (si está afectado el nervio craneal VIII).

110
Q
  1. Femenina de 25 años, consulta a urgencias por trauma craneoencefálico secundario a accidente en calidad de motociclista, sin uso de casco. Signos vitales PA 170/95 mmhg, FC 48 lpm, FR 10 rpm. Al examen físico, paciente con Glasgow 12/15, laceración de 2 cm en región temporal derecha con gran hematoma y dolor en miembro inferior derecho con hemiparesia. ¿Cuáles hallazgos son más probables de encontrar al examen físico de la paciente?
    A. Ptosis palpebral, limitación de la aducción, supra e infraducción ocular y midriasis ipsilateral.
    B. Anisocoria, limitación de la aducción ocular y ptosis palpebral
    C. Neuralgia en territorio del trigémino, parálisis de los músculos de la masticación, peribucales y orbiculares
    D. Ptosis palpebral, limitación de la aducción, supra e infraducción ocular y miosis pupilar.
A

Respuesta correcta: A
Tema e ítem: Hematoma epidural - Diagnóstico
Argumento: Esta paciente que presenta un trauma craneoencefálico (TEC) y compromiso en región temporal (laceración, hematoma) debemos pensar indudablemente como principal diagnóstico diferencial en un hematoma epidural (HE) por lesión de la arteria meníngea media. Es probable que dicho HE se haya expandido de forma rápida aumentando la presión intracraneal, generando un efecto de compresión en el lóbulo temporal y culminando en una herniación del uncus (localizado en la región posteromedial de dicho lóbulo). Cuando se produce una herniación de dicha estructura, los pacientes pueden desarrollar los siguientes déficits neurológicos:

● Hemiparesia contralateral (por tanto, el caso clínico debió referirse al miembro inferior izquierdo y no derecho), sin embargo, también puede presentarse un signo conocido como fenómeno de Kernohan (hemiparesia ipsilateral) que se produce por la compresión del pedúnculo cerebral contralateral contra el tentorio a medida que avanza la herniación.
● Comprensión del nervio oculomotor (III P.ar craneal) ipsilateral, lo cual se evidencia por midriasis pupilar ipsilateral en un inicio, y de forma más tardía por ptosis palpebral ipsilateral, debido al compromiso de los músculos oculomotores. A su vez, desviación del ojo hacia abajo y hacia afuera (Por limitación de los movimientos propios del III par craneal), lo que genera dificultad para la aducción, supra e infraducción ocular.
● Hemianopsia homónima contralateral con preservación macular.

Por lo anterior, la opción B que menciona los posibles hallazgos clínicos, pero que no menciona la lateralidad de los mismos (debe ser ipsilateal a la lesión), se debe descartar. Por otra parte, la opción D que menciona miosis, se descarta debido a que la herniación uncal produce midriasis. Finalmente, la herniación uncal se relaciona con compromiso del III par craneal y no del V par craneal o nervio trigémino.

111
Q
  1. Masculino de 79 años, consulta a urgencias por cuadro clínico de 2 días de evolución consistentes con cefalea, confusión, episodios eméticos persistentes, astenia y adinamia. Antecedente de fractura abierta de fémur hace 3 meses con intervención quirúrgica. Signos vitales PA 140/92 mmhg, FC 70 lpm con pulso irregular, FR 13 rpm. Imágenes: TAC simple de cráneo reporta foco hemorrágico entre el córtex cerebral y la duramadre en forma de ‘’semiluna’’. ¿Cuál es el diagnóstico más probable?
    A. Hematoma subdural agudo.
    B. Hematoma epidural
    C. Hemorragia subaracnoidea
    D. Daño axonal difuso
A

Respuesta correcta: A

Tema e ítem: Hemorragia subdural aguda-Diagnóstico

Argumento: Este paciente que presenta ‘’foco hemorrágico en el espacio subdural en forma de semi/medialuna’’ configura el diagnóstico de hematoma subdural (HS). Además, dado el tiempo de evolución ubicado entre 34 a 48 horas nos permite hablar de un HS agudo. Recordemos que el HS corresponde a un sangrado extraxial entre el córtex cerebral y la duramadre en forma de semiluna evidenciando en la TAC simple de cráneo. Este se produce por rotura de vasos corticodurales, y en pacientes de edad avanzada, este riesgo aumenta considerablemente debido a la atrofia cerebral propia del envejecimiento (espacio más grande entre el córtex cerebral y la duramadre, lo que aumenta la tensión en las venas puente). Además de la atrofia cerebral, los traumas craneoencefálicos previos son un antecedente muy común, y este caso no es la excepción, ya que comentan un antecedente quirúrgico que fue la fractura abierta de fémur, por lo que deberíamos sospechar o inferir que este paciente tuvo una caída reciente, lo que provocó un desgarro de las venas puentes desarrollando un HS. Para agregar, es importante tener en cuenta que la TAC simple de cráneo suele mostrar una hiperdensidad (foco hemorrágico) en forma de medialuna que cruza las líneas de sutura. Los hematomas pequeños se tratan de forma conservadora, mientras que los más grandes que causan déficits neurológicos requieren una evacuación quirúrgica.

En cuanto al hematoma epidural, recordemos que el sangrado epidural generalmente no cruza las líneas de sutura y produce la apariencia de colección hiperdensa biconvexa bien definida. Por otra parte, la hemorragia subaracnoidea, ocurre como resultado de la ruptura de pequeños vasos piales a lo largo de la superficie del cerebro. En la TAC, se observa delimitando la geometría giral con una morfología lineal o curvilínea cuando se localiza en surcos cerebrales, fisuras cerebelosas, o espacios cisternales. Finalmente, la lesión axonal difusa, puede deberse como resultado de fuerzas de cizallamiento secundarias a una aceleración rotacional que deforma y corta la interfaz entre la sustancia blanca y gris, al cual se le suman alteraciones metabólicas que ensombrecen la lesión secundaria de los axones. En la TAC se pueden observar lesiones hemorrágicas múltiples, sin embargo, menos del 20% tienen un componente hemorrágico, por lo que la mayoría de pacientes no tendrán ningún hallazgo en este estudio y pueden requerir imágenes avanzadas.

112
Q
  1. Femenina de 35 años, sin antecedentes de interés, consulta al servicio de Urgencias refiriendo la presencia de un ‘’bulto’’ de aproximadamente 8x10 cm en región cervicolateral derecha desde hace 2 meses. Niega sintomatología asociada. Al examen físico presenta signos vitales estables y no se evidencia lesión ocupante de espacio en la región referida. Sin embargo, la paciente persistente convencida de que el bulto está presente. El afecto es normal y no hay indicios de pensamiento desorganizado.
    ¿Cuál de los siguientes es el diagnóstico más probable?
    A. Trastorno límite de la personalidad
    B. Trastorno delirante
    C. Esquizofrenia
    D. Trastorno de síntomas somáticos
A

Respuesta correcta: B

Tema e ítem: Trastorno delirante - ¿Cómo identificarlo?

Argumento: La presentación clínica de esta paciente es compatible con el trastorno delirante, un trastorno psicótico caracterizado por falsas creencias persistentes que el paciente sigue convencido de que son ciertas a pesar de que un correcto examen físico demuestra lo contrario. Los síntomas deben estar presentes durante al menos 1 mes, suelen afectar mínimamente a la funcionalidad y pueden o no ser angustiantes para el paciente. Dado que los pacientes a menudo no perciben el delirio como un problema, es posible que no busquen atención médica voluntaria. Los temas delirantes comunes incluyen:
* Paranoia (persecución o infidelidad)
* Erotomanía, definida como la creencia de que una persona con un estatus elevado está enamorada del paciente
* Grandiosidad (creencia en un valor, conocimiento o poder)
* Delirios somáticos de defecto físico o anormalidad (el caso de la paciente).

OJO: No deben estar presentes
síntomas compatibles otros trastornos psicóticos, como pensamiento desorganizado, disfunción motora y síntomas negativos.

  • Los trastornos de la personalidad implican patrones consistentes de comportamiento interpersonal y percepciones que son inflexibles. El trastorno límite de la personalidad se caracteriza por lo siguiente: relaciones caóticas (tanto idealizadas y devaluadas) y una imagen frágil de sí mismo, miedo al abandono, emociones lábiles e intensas, sensación de vacío, participación en conductas impulsivas y de riesgo y posiblemente autolesiones y suicidio.
  • La esquizofrenia es un trastorno psicótico que puede incluir delirios o alucinaciones. Los pacientes con esquizofrenia experimentan pensamientos y comportamientos desorganizados y síntomas negativos, como retraimiento social, afecto aplanado y pobreza de palabra y pensamiento. El funcionamiento social y laboral suele estar muy deteriorado. Este paciente con un delirio aislado sobre un defecto físico no presenta síntomas que orienten a pensar en una esquizofrenia.
  • El trastorno de síntomas somáticos se caracteriza por síntomas médicamente inexplicables que causan malestar emocional y deterioro psicosocial. Los pacientes con estos trastornos tienen tasas de utilización de la atención sanitaria muy elevadas y, sin embargo, a menudo están insatisfechos con la atención que reciben.
113
Q
  1. Femenina de 41 años, acude a consulta refiriendo historial de 3 meses de evolución dado por rinorrea persistente, cefalea frontal y anosmia. Refiere automanejo con antihistamínicos sin mejoría alguna. Al examen físico presenta dolor a la digitopresión en región maxilar y frontal. La rinoscopia anterior se evidencian cornetes hipertróficos y edematizados con presencia de rinorrea de aspecto amarillento. El resto del examen físico es anodino.
    ¿Qué prueba diagnostica solicitaría en esta paciente?
    A. Rx simple de senos paranasales
    B. RNM de senos paranasales
    C. Muestra nasofaríngea para cultivo e iniciar antibioticos empíricos
    D. TAC simple de senos paranasales
A

Respuesta correcta: D

Tema e ítem: Sinusitis crónica – Enfoque diagnóstico basado en la evidencia

Argumento: La sinusitis crónica se manifiesta clínicamente con al menos 12 semanas de congestión nasal y drenaje purulento, anosmia o dolor a la digitopresión en la región de senos paranasales. Puede asociarse a poliposis nasal (fuerte asociación con el asma). Para el diagnóstico es necesario demostrar el compromiso de la mucosa mediante endoscopia nasal o pruebas de imagen (TAC simple de elección). Los hallazgos más frecuentes en la TAC incluyen engrosamiento de la mucosa, obstrucción ostial de los senos, pólipos y opacificación de los senos. El tratamiento incluye glucocorticoides y antibióticos.

Aunque el compromiso de la mucosa
puede demostrarse con RNM, la TAC tiene mayor resolución y mejor precisión diagnóstica en este contexto. Además, la RNM es más costosa que la TAC y no proporciona un diagnóstico más preciso. Debe considerarse cuando hay sospecha de infección o inflamación que se extiende más allá de los senos paranasales.

La radiografía simple de senos paranasales puede mostrar cambios sugestivos de sinusitis crónica, incluida la opacificación de los senos y el engrosamiento de la mucosa. Sin embargo, estos hallazgos son inespecíficos. Además, la radiografía simple carece de la sensibilidad suficiente para determinar la presencia o el alcance de una posible erosión ósea por infección.

114
Q
  1. Don Javier de 55 años, acude al programa de control. Tiene antecedente de falla cardiaca crónica con fracción de eyección reducida (FEVI 35%) diagnosticada hace 2 años. Refiere disnea a mínimos esfuerzos (clase funcional III de la NYHA) y ha permanecido estable durante los últimos 6 meses. Se encuentra actualmente en manejo con salcubitril-valsartan, empaglifozina, carveridol. Al examen físico presenta PA 130/70 mh, FC 81 lpm regular, sin edemas o signos de congestión. Analítica: Cr 1.1 mg/dL, Na + 137 mEq/L, K+ 3.8 mEq/L.
    ¿Cuál de los siguientes medicamentos debe ser iniciado en don Javier?
    A. Ivabradina
    B. Furosemida
    C. Vericiguat
    D. Espironolactona
A

Respuesta correcta: D

Tema e ítem: Los cuatros fantásticos en el manejo de la falla cardiaca crónica

Argumento: Sin duda alguna, la espironolactona debe ser añadida al manejo farmacologico de este paciente. Está clínicamente estable con síntomas de insuficiencia cardíaca de clase funcional III de la NYHA, y el objetivo del tratamiento en este momento es optimizar la terapia médica para mejorar su calidad de vida y reducir la mortalidad y hospitalizaciones por causa cardiovascular. El tratamiento con antagonistas de la aldosterona (espironolactona o eplerenona) está indicado en todos los pacientes con insuficiencia cardíaca sintomática con fracción de eyección reducida y función renal normal (TFG ≥30 mL). La eplerenona es un antagonista de la aldosterona más selectivo que la espironolactona; se asocia a menos efectos secundarios endocrinos y a una menor incidencia de ginecomastia (1% vs 10%).

115
Q
  1. Don Juan de 81 años, sin antecedentes de interés, acude a consulta por historial de fatiga de 2 años de duración. Niega otros síntomas asociados. El examen físico es normal. Trae reporte de perfil tiroideo con TSH 7 mU/L (TSH de 6.9 mU/L hace 6 meses), T4L 1,2 ng/dL, anti-TPO negativos.
    ¿Cuál sería una conducta adecuada a seguir?
    A. Iniciar levotiroxina
    B. Realizar nuevo perfil tiroideo en 6 meses con cita de control
    C. Solicitar anticuerpos anti-receptor de tirotropina (Ac anti- TSH-r)
    D. Solicitar T4 total
A

Respuesta correcta: B

Tema e ítem: Hipotiroidismo subclínico - ¿Cómo y cuando realizar los controles?

Argumento: El diagnostico de este paciente es un hipotiroidismo subclínico que se caracteriza por un nivel elevado de TSH asociado a un nivel normal de hormona tiroidea. Este grupo de pacientes suelen ser asintomáticos o presentar síntomas leves de hipotiroidismo. Los pacientes con hipotiroidismo subclínico pueden presentar elevaciones leves de los niveles séricos de colesterol total, colesterol LDL e incluso proteína C reactiva, e incluso algunos metaanálisis han demostrado un aumento del riesgo de aterosclerosis y eventos cardíacos. Entre el 2% y el 4% de los pacientes con hipotiroidismo subclínico evolucionan anualmente a hipotiroidismo manifiesto, mientras que en un tercio de los pacientes la función tiroidea vuelve espontáneamente a la normalidad. El intervalo normal de TSH aumenta con la edad, y un nivel de TSH de hasta 7,9 µU/mL está dentro del intervalo normal para personas de 80 años o más. Por lo tanto, es razonable repetir los estudios de la función tiroidea en 6 meses.

116
Q
  1. Doña Aurora de 58 años, acude a consulta por cuadro clínico de 3 meses de evolución consistente en astenia progresiva, fatiga, intolerancia al frio, voz ronca, amnesia y lentitud del habla. Refiere aumento de peso de 5 kg. Examen físico: PA 110/90 mh, FC 51 lpm, FR 15 rpm, P: 69 kg, cara abotargada, piel seca y pálida, cabello quebradizo, reflejos tendinosos disminuidos. Perfil tiroideo: TSH 11 mU/L (VR: 0,4-5,0 mUI/L), T4L 0.20 ng/dL. Basados en el consenso colombiano para el abordaje y tratamiento del hipotiroidismo, ¿Cuál es la dosis correcta de levotiroxina que debe recibir doña Aurora?
    A. Levotiroxina 125 mcg/día
    B. Levotiroxina 75 mcg/día
    C. Levotiroxina 150 mcg/día
    D. Levotiroxina 50 mcg/día
A

Respuesta correcta: B

Tema e ítem: Hipotiroidismo subclínico – Recomendaciones del
Consenso Colombiano

Argumento: Estamos indudablemente frente a un caso clínico típico de hipotiroidismo primario; sugerido por el espectro clínico del paciente y confirmado por el perfil tiroideo.

El consenso colombiano para el diagnóstico, tratamiento y seguimiento del hipotiroidismo en la población adulta recomienda usar los valores de referencia de TSH adaptados a Colombia. De esta manera, tenemos que dichos valores de referencia normales se determinan:
* 18-60 años: 0,4-5,0 mUI/L
* >60 años: 0,4-6,0 mUI/L
* En pacientes de ≥ 80 años el valor normal puede ser hasta de 7,9 mUI/L

De la misma manera, dicho consenso adapta las dosis de levotiroxina dependiendo de los valores de TSH del paciente:
* >5 - <10 mUI/L: 0,6-0,9 mcg/kg/día
* >10 - <20 mUI/L: 1-1,3 mcg/kg/día
* >20 mUI/L: 1,4-1,6 mcg/kg/día

En esta paciente de 58 años con una TSH de 11 de mU/L, la dosis diaria de levotiroxina deben iniciarse entre 1,-1,3 mcg/kg/día: 69-84 mcg/día de levotiroxina, aproximando a las opciones de respuesta la opción correcta sería la B.

117
Q
  1. Paciente de 3 años, sin antecedentes patológicos. Ingresa a reanimación del servicio de urgencias donde usted trabaja, según la madre refiriendo que no respira y que ella cree que se “atragantó” con un juguete. Usted observa que el paciente está tosiendo, se observa pálido, sin cianosis y con sibilancias audibles. ¿Cuál manejo instauraría de inmediato?

A. Animar al paciente a seguir tosiendo para que expulse el cuerpo extraño
B. Iniciar exploración de vía aérea para retirar el cuerpo extraño
C. Realizar maniobra de Heimlich
D. Llevar a fibrobroncoscopia urgente

A

Respuesta correcta: A.

Tema-Ítem: Urgencias pediátricas-Ingesta/aspiración de cuerpos extraños

Argumentación: Este paciente, presentó aspiración de cuerpo extraño evidenciada por la madre, posterior a lo cual presenta dificultad respiratoria, con tos activa. En este caso, debemos animar al paciente a seguir tosiendo para que expulse el cuerpo extraño. Intentar desobstruir la vía aérea podría empeorar el cuadro y paradójicamente, obstruirla aún más.

La aspiración de CE (cuerpo extraño) se define como la presencia de un elemento no habitual a cualquier nivel del árbol respiratorio.

Existe un riesgo incrementado de aspiración en los tres primeros años de vida de forma general. Hay dos picos de incidencia, el primero y más importante ocurre durante el segundo año de vida y el segundo, en la edad escolar.

El MEJOR factor PREDICTIVO de aspiración de cuerpo extraño es el antecedente de atragantamiento/crisis asfíctica, definido como inicio repentino de tos o disnea o cianosis en un niño previamente san, autolimitado, entre segundos o minutos, que puede seguirse de un periodo libre de síntomas.

La tríada clásica consistente en antecedente de asfixia/tos aguda, sibilancias e hipoventilación universal se presenta en el 15-25% de los casos.

Están contraindicadas las maniobras exploratorias invasivas en un paciente con sospecha de aspiración de CE, salvo en pacientes inconscientes con obstrucción total de la vía aérea en que sean necesarias para intentar extracción.

El tratamiento inicial de un niño con sospecha de aspiración de un cuerpo extraño debe ir dirigido a mantener la oxigenación y la ventilación, y a prevenir o tratar la obstrucción total de las vías aéreas. Si hay sospecha de localización en vía aérea superior por obstrucción completa (posición en trípode, distrés universal, cianosis e incapacidad para el habla) o porque el paciente está inconsciente, se considera una urgencia vital y se debe iniciar de forma inmediata maniobras de desobstrucción de la vía aérea. En caso de una obstrucción incompleta (niño consciente y con tos efectiva, como en el caso de nuestro paciente), administrar oxígeno y poner en posición cómoda, animarlo a que siga tosiendo, continuando una observación estrecha del niño y vigilando si expulsa el cuerpo extraño o, por el contrario, la tos se hace inefectiva, deja de respirar o se deteriora el estado de consciencia.

118
Q

2.Paciente de 7 semanas, nació a las 29 semanas por desprendimiento de placenta, pesó 1150 g. Desde entonces, en UCIN. Seis horas después del parto, la paciente inició con taquipnea y cianosis, con rx de tórax con evidencia de opacidades en vidrio esmerilado. Se administró surfactante, requirió ventilación mecánica durante las primeras 3 semanas de vida pero ha sido difícil el desmonte de oxígeno, aun requiriendo O2 por cánula nasal. Ha permanecido afebril y sin taquipnea, sin síntomas respiratorios; se ausculta un soplo sistólico de 1/6 en el borde esternal inferior izquierdo, roncus y estertores dispersos en ambos campos pulmonares. Rx de tórax con opacidades difusas y disminución de volúmenes pulmonares. ¿Cuál cree que es el diagnóstico más probable?
A. Displasia broncopulmonar
B. Hipertensión pulmonar persistente
C. Bronquiectasias
D. Cardiopatía congénita con cortocircuito de derecha a izquierda

A

Respuesta correcta: A.

Tema-Ítem: Displasia broncopulmonar-Neonatología

Argumentación: Este paciente prematuro y de bajo peso al nacer desarrolló inicialmente un síndrome de dificultad respiratoria (taquipnea, cianosis, opacidades en vidrio esmerilado en la rx de tórax) que requirió terapia con surfactante y ventilación mecánica. Ahora tiene una necesidad persistente de oxígeno con taquipnea, roncus y hallazgos radiológicos de disminución del volumen pulmonar. Esta presentación es compatible con una displasia broncopulmonar (DBP), resultado de agresiones repetidas al pulmón neonatal por factores como la ventilación mecánica, la exposición prolongada al oxígeno y la inflamación.
Los factores de riesgo de DBP incluyen prematuridad, bajo peso al nacer (<2500 g), síndrome de dificultad respiratoria y ventilación mecánica. La terapia con surfactante no previene su desarrollo, pero puede reducir la mortalidad por esta causa. La mayoría de los pacientes con DBP mejoran en 2 a 4 meses; algunos desarrollan hipertensión arterial pulmonar.

La bronquiectasia es el resultado de
la remodelación y el engrosamiento de las vías respiratorias debido a inflamación crónica. En los niños, generalmente se asocia con fibrosis quística (debido a secreciones viscosas que provocan infección recurrente). No está relacionado con la prematuridad ni con el síndrome de dificultad respiratoria.

La cardiopatía congénita con derivación de derecha a izquierda puede presentarse con dificultad respiratoria, estertores e hipoxemia. Sin embargo, el soplo leve de 1/6 de este paciente, probablemente sea un soplo de flujo y no es consistente con un defecto del tabique interventricular (más fuerte y más áspero) o un conducto arterioso persistente (similar a una máquina). Además, estas condiciones habrían empeorado progresivamente antes de las 7 semanas de vida.

La hipertensión pulmonar persistente se presenta con taquipnea e hipoxia. Se produce cuando la resistencia vascular pulmonar neonatal no disminuye durante la transición desde la circulación fetal. Sin embargo, no se asocia con síndrome de dificultad respiratoria y es poco común en bebés prematuros.

119
Q
  1. Emilio tiene 4 años. Llega a su consultorio por fiebre de 1 día de duración de 38.4°C y edema del cuello en el lado izquierdo. Niega traumas o caídas. La madre niega sintomatología adicional, sin nexo de contagio. PAI al día. Se observa una masa roja, dolorosa y fluctuante de aproximadamente 2 cm de diámetro en la cadena ganglionar cervical anterior izquierda. No hay otros hallazgos al examen físico. ¿Cuál de las siguientes es la etiología infecciosa más probable?
    A. S. aureus
    B. B. henselae
    C. Adenovirus
    D. Virus de Epstein-Barr
A

Respuesta correcta: A.

Tema-Ítem: Adenitis bacteriana-Enfermedades infecciosas en Pediatría

Argumentación: Este paciente tiene una linfadenitis cervical unilateral aguda, la cual es causada con mayor frecuencia por Staphylococcus aureus o Streptococcus pyogenes.
La linfadenitis cervical aguda ocurre predominantemente en niños <5 años y con frecuencia afecta los ganglios submandibulares, aunque cualquier ganglio cervical puede verse afectado. En la anamnesis, podemos encontrar una infección reciente de las vías respiratorias superiores, pero a menudo no hay síntomas previos. Los pacientes suelen tener buen aspecto y pueden tener fiebre, como en este niño. Se encuentran adenopatías, dolorosas y con signos inflamatorios. La linfadenitis por S. aureus o S. pyogenes a menudo progresa hacia supuración y absceso, que se identifican por fluctuación a la palpación, como se observa en este paciente.

El diagnóstico suele ser clínico. El tratamiento antibiótico empírico para la linfadenitis cervical unilateral aguda consiste en clindamicina, que tiene actividad contra S. aureus resistente a meticilina, así como contra S. pyogenes.

Los adenovirus y otras infecciones virales de las vías respiratorias superiores son la causa más común de linfadenitis cervical bilateral aguda. Además, el adenovirus suele causar fiebre marcada, faringitis y conjuntivitis, hallazgos que no se observan en este paciente.

Bartonella henselae causa la enfermedad por arañazo de gato, que puede presentarse con una lesión cutánea papular o nodular en el sitio de una mordedura o arañazo de gato y se acompaña de linfadenitis ipsilateral. A diferencia de la infección por S. aureus, la linfadenitis por arañazo de gato tiene un inicio indolente (es decir, es más subagudo).m

El citomegalovirus y el virus de Epstein-Barr causan linfadenitis bilateral subaguda o crónica en el contexto de síntomas de mononucleosis infecciosa (p. ej., fiebre, fatiga, faringitis). Este paciente tiene un inicio agudo de linfadenitis cervical unilateral, lo que hace poco probable una causa viral.

120
Q
  1. Paciente de 15 meses, sin antecedentes patológicos, es llevada a urgencias por sus padres debido a un episodio de desconexión con el medio. La paciente estaba jugando con sus juguetes y, cuando el padre se los quitó para prepararse para ir a dormir, comenzó a llorar sin cesar. Hubo una larga pausa en el llanto, durante la cual la paciente no hizo ningún ruido y dejó de respirar hasta que se puso cianótica, quedó flácida y no respondió durante unos segundos. Después de este episodio, alerta y sin alteraciones. No tiene antecedentes descritos, PAI al día. Al examen físico, sin ninguna alteración. De acuerdo a la patología que sospecha, ¿Cuál de las siguientes cree que encontraría en esta paciente?
    A. Arritmias
    B. Enfermedad cardíaca estructural
    C. Epilepsia
    D. Neurodesarrollo normal
A

Respuesta correcta: D

Tema-Ítem: Espasmos del sollozo-Neurología pediátrica

Argumentación: Los espasmos del sollozo, son episodios de apnea y pérdida del conocimiento precipitados por frustración, ira, miedo o dolor. Suele ser benignos y aparecen en niños de 6 meses a 2 años. Cuando está acompañado de cianosis o palidez, puede representar una variante del síncope vasovagal debido a disfunción autonómica.

Un espasmo del sollozo cianótico, como se observa en este paciente, es el subtipo más común y clásicamente comienza con llanto y retención de la respiración durante la espiración forzada; a esto le siguen apnea, cianosis y pérdida del conocimiento. El evento es breve con un rápido retorno al estado basal. Por el contrario, un espasmo del sollozo pálido no se asocia con llanto, el niño inicialmente se observa pálido y sudoroso (en lugar de cianótico) y puede estar confundido y somnoliento durante unos minutos después.

El diagnóstico es clínico y la evaluación incluye la detección de anemia por deficiencia de hierro (p. ej., hemograma completo). En ausencia de deficiencia de hierro, no se requiere tratamiento y se debe asegurar a los padres que, aunque los episodios pueden repetirse, generalmente se resuelven a los 5 años. Los niños generalmente tendrán un neurodesarrollo normal, sin alteraciones.
Si se presentan frecuentemente, son prolongados (>1 minuto) o hay antecedentes familiares de enfermedad cardíaca, síncope o muerte súbita deben someterse a un ECG para evaluar arritmias (p. ej., síndrome QT).

Se puede diferenciar de una epilepsia por un historial de provocación (p. ej., llanto) anterior a la pérdida de la conciencia. Además, por anamnesis se debe descartar presencia de movimientos anormales o período posictal.

La tetralogía de Fallot puede presentarse con cianosis posterior al llanto. Sin embargo, durante el episodio se produce respiración rápida y profunda, y no contención de la respiración, y se esperaría un soplo de eyección sistólico intenso debido a la obstrucción del tracto de salida del ventrículo derecho.

121
Q

5.Un paciente de 15 años, previamente sano, es llevado a su consulta por la madre debido a mal rendimiento académico, irritabilidad y mal humor en los últimos 3 meses. La madre dice que el paciente solía ser un buen estudiante. El paciente le refiere durante el interrogatorio que ha escuchado voces. Al examen físico, irritable, con contacto visual mínimo, con evidencia de sialorrea, aumento del tono en los brazos y un leve temblor. ¿Cuál de los siguientes procesos fisiológicos esperaría encontrar alterado en este paciente?
A. Metabolismo de ácidos grasos
B. Transporte hepatocelular
C. Absorción intestinal
D. Regulación de hormona tiroidea

A

Respuesta correcta: B

Tema-Ítem: Enfermedades genéticas en pediatría

Argumentación: Este adolescente tiene una psicosis de nueva aparición y al examen físico, manifiesta alucinaciones auditivas. Tiene un examen neurológico anormal (p. ej., temblor, hipertonía, rigidez facial). Estos hallazgos deberían hacernos sospechar un parkinsonismo por enfermedad de Wilson.

La enfermedad de Wilson es un trastorno autosómico recesivo caracterizado por un transporte de cobre hepático defectuoso, lo que resulta en una acumulación de cobre a nivel sistémico. La acumulación inicial ocurre dentro del hígado, lo que lleva a la apoptosis hepatocelular y la liberación de cobre libre al torrente sanguíneo. Luego, el cobre extrahepático se deposita en varios tejidos, incluidos el SNC, el riñón y la córnea. Los ganglios basales son el principal sitio de depósito en el SNC y explican las anomalías del movimiento que se observan en la enfermedad de Wilson. Debido a que los ganglios basales, la corteza frontal y el sistema límbico se comunican, la acumulación de cobre también afecta el estado de ánimo y la cognición.

Los síntomas neuropsiquiátricos a menudo preceden a los síntomas hepáticos (como hepatomegalia o ictericia) en los adolescentes y pueden atribuirse erróneamente a un cambio normal de la adolescencia o a una enfermedad psiquiátrica primaria. Los síntomas pueden ser sutiles (p. ej., cambio de personalidad, deterioro académico, irritabilidad) o más graves, como en este paciente. Los hallazgos neurológicos también pueden incluir disartria, distonía y temblor. La disminución de la expresividad y la sialorrea de este paciente son consistentes con el parkinsonismo observado en la enfermedad de Wilson.

122
Q
  1. Masculino de 16 años presenta molestias intermitentes y sensación de pesades escrotal desde hace algunos meses. No es sexualmente activo y niega antecedentes de importancia. Sus signos vitales son normales. Al examen físico hay una masa de apariencia irregular y viscosa en ambos lados de la parte superior de su escroto. La masa aumenta de tamaño con las maniobras de Valsalva y se reduce en posición supina. El resto del examen físico es normal. Usted indica que el manejo es quirúrgico con el fin de prevenir:
    A. Epididimitis crónica.
    B. Infertilidad.
    C. Estrangulación intestinal.
    D. Torsión testicular.
A

Respuesta correcta: B

Tema e ítem: Varicocele – Complicaciones.

Argumento: Un varicocele es una dilatación tortuosa del plexo pampiniforme de venas que rodean el cordón espermático y los testículos. Ocurre en casi el 20% de los hombres (generalmente entre 15 y 25 años) y se presenta como una masa blanda e irregular (“bolsa de gusanos”) que aumenta de tamaño al ponerse de pie y al realizar maniobres de Valsalva. También se puede observar una reducción en la posición supina (debido a la descompresión del plexo). Los varicoceles son más comunes en el lado izquierdo porque la vena espermática izquierda drena hacia la vena renal izquierda, que es vulnerable a la compresión cuando pasa entre la arteria mesentérica superior y la aorta. Los varicoceles bilaterales también son comunes; sin embargo, los varicoceles unilaterales derechos son relativamente raros y deben hacer sospechar de trombosis o compresión maligna.

El diagnóstico de varicocele se puede confirmar mediante ecografía, que muestra dilatación del plexo pampiniforme y flujo venoso retrógrado. Las intervenciones iniciales para el malestar escrotal debido a un varicocele incluyen soporte escrotal y analgésicos simples (fármacos antiinflamatorios no esteroides).

Los varicoceles se asocian con un mayor riesgo de infertilidad y atrofia testicular, que puede deberse a un ligero aumento de la temperatura escrotal. Para los hombres mayores que no tienen preocupaciones sobre la fertilidad, es posible que no se necesite ninguna intervención. Sin embargo, se debe realizar un seguimiento de los hombres más jóvenes para detectar signos de atrofia testicular o cambios en el análisis de semen; los hallazgos anormales generalmente justifican una intervención quirúrgica. Cuando está indicada la intervención, la ligadura venosa quirúrgica puede mejorar la fertilidad.

123
Q
  1. Masculino de 43 años consulta por 24 horas de dolor creciente en hemiabdomen inferior, escroto y perineo asociado a nauseas, vómitos y malestar general. Tiene antecedente de obesidad mórbida, HTA, DM-2 e ICC con FEV1 conservada. Signos vitales: Temp. de 39.8°C, FC de 112 lpm, PA de 80/60 mmHg y FR de 18 rpm. Al examen físico la piel de la parte inferior del abdomen, el escroto y el perineo está tensa, sensible, eritematosa e hinchada con evidencia de crepitación. Hay edema leve, bilateral, en las extremidades inferiores. Paraclínicos: Leucocitos de 27.800/mm3, Bandas 20%, HCO3- 18 mEq/L, Creatinina de 1.7 mg/dL, Glucosa de 280 mg/dl. Se toman hemocultivos, se inician líquidos IV y antibióticos. ¿Cuál es el manejo adecuado a seguir?
    A. Realizar TAC de abdomen y pelvis.
    B. Realizar cirugía de emergencia.
    C. Iniciar terapia con oxigeno hiperbárico.
    D. Indicar traslado a unidad de cuidados intensivos.
A

Respuesta correcta: B

Tema e ítem: Gangrena de Fournier – Manejo.

Argumento: La infección cutánea de aparición rápida de la parte inferior del abdomen, el escroto y el perineo de este paciente con crepitación y manifestaciones sistémicas significativas (hipotensión, fiebre alta, leucocitosis) probablemente indica gangrena de Fournier, una fascitis necrotizante potencialmente mortal que progresa rápidamente a sepsis y muerte sin intervención. La mayoría de los casos surgen en el contexto de lesiones cutáneas en la región perianal o genital, que permiten una puerta de entrada para organismos polimicrobianos del colon o urogenitales. Luego, la infección se propaga a lo largo de la grasa subcutánea a través de planos fasciales y produce microtrombos en los vasos cutáneos, lo que lleva a la posterior gangrena de la piel. El riesgo es mayor en personas con diabetes mellitus mal controlada y obesidad.

Las manifestaciones generalmente surgen rápidamente e incluyen:
- Edema, sensibilidad y crepitación en el perineo, el escroto y/o la parte inferior del abdomen.
- Síntomas sistémicos graves como fiebre alta e hipotensión.
- Anomalías de laboratorio significativas pero inespecíficas, como leucocitosis, acidemia, insuficiencia renal y coagulopatía.

Se deben administrar antibióticos de amplio espectro y líquidos intravenosos con urgencia, pero el diagnóstico y tratamiento definitivos requieren exploración quirúrgica y desbridamiento tempranos. Aunque la intervención quirúrgica temprana mejora los resultados y las tasas de supervivencia, >20% de los pacientes con gangrena de Fournier mueren durante la hospitalización.

124
Q
  1. Masculino de 48 años acude a urgencias por asimetría facial. Refiere que tuvo una leve molestia en el oído izquierdo por la mañana y que mientras conducía al trabajo tuvo dificultades para cerrar el ojo izquierdo a contraluz. Cuatro horas más tarde, no podía beber con pitillo y un amigo le comentó que su rostro parecía caído. Niega otros síntomas. Signos vitales: Temp. de 37°C, PA de 140/80 mmHg y FC de 84 lpm. Al examen físico se observa asimetría facial izquierda con caída de la comisura izquierda de la boca y aplanamiento del surco nasolabial. No puede cerrar completamente el ojo izquierdo. El resto del examen físico es normal. ¿Cuál es el manejo adecuado para este paciente?
    A. Aspirina y estatinas.
    B. Carbamazepina.
    C. Ganciclovir.
    D. Prednisona.
A

Respuesta correcta: D

Tema e ítem: Parálisis facial – Manejo.

Argumento: Este paciente con caída facial unilateral de inicio agudo que afecta la parte superior e inferior de la cara probablemente tenga parálisis de Bell, una neuropatía periférica que afecta al par craneal VII (nervio facial). El inicio de los síntomas suele ser agudo (en el transcurso de unas horas) y progresivo, y la debilidad máxima se produce en un plazo de tres semanas. Puede estar precedido por un pródromo de dolor auricular o disacusia (distorsión del sonido). La parálisis facial unilateral se caracteriza por caída de la boca, desaparición del pliegue nasolabial y, lo más importante, afectación de la parte superior de la cara (incapacidad para cerrar el ojo, debilidad para levantar la ceja), lo que ayuda a distinguir la parálisis de Bell del accidente cerebrovascular agudo (típicamente respeta la región superior de la cara). Los pacientes con parálisis de Bell también pueden tener disminución del lagrimeo, hiperacusia y disminución del sentido del gusto en los dos tercios anteriores de la lengua. La sensación suele estar intacta.
La base del tratamiento son los glucocorticoides orales (prednisona). Debido a que se cree que muchos casos son causados por la reactivación del virus del herpes simple, algunos estudios también sugieren que la adición de valaciclovir o aciclovir puede acelerar el tiempo hasta la recuperación completa de la función facial. Los pacientes con cierre ocular incompleto también deben recibir protección ocular. La mayoría de los pacientes recuperan el movimiento facial en 3 semanas; las parálisis que duran más de 3 o 4 meses sugieren otro diagnóstico.

125
Q
  1. Masculino de 55 años asiste a una cita de control para su hipertensión arterial. Fue diagnosticado hace 5 años y desde entonces toma enalapril y amlodipino. Su PA se ha encontrado constantemente alta en los últimos 6 meses a pesar de sus modificaciones en estilo de vida. Signos vitales: PA de 156/94 mmHg y FC de 80 lpm. IMC de 30 kg/m2. Se considera agregar clortalidona a la terapia actual del paciente para mejorar cifras tensionales. ¿Cuál de los siguientes efectos metabólicos debería esperarse con esta terapia?
    A. Hiperglucemia.
    B. Hiperpotasemia.
    C. Hipermagnesemia.
    D. Hipocalcemia.
A

Respuesta correcta: A

Tema e ítem: Efectos secundarios de manejo antihipertensivo.

Argumento: Los diuréticos tiazídicos (clortalidona, hidroclorotiazida) son medicamentos antihipertensivos ampliamente utilizados que son eficaces como monoterapia o en combinación con otros agentes. En dosis más bajas, la clortalidona es más eficaz que otras tiazidas y se asocia con una tasa más baja de insuficiencia cardíaca en comparación con otros agentes antihipertensivos (posiblemente debido a su capacidad para lograr una presión arterial más baja como agente único).
Sin embargo, las tiazidas pueden elevar el nivel de glucosa en sangre del paciente debido al deterioro tanto de la liberación de insulina del páncreas como de la utilización de la glucosa en los tejidos periféricos. La intolerancia a la glucosa inducida por tiazidas se observa con mayor frecuencia en pacientes con diabetes mellitus y síndrome metabólico (es decir, hipertensión, dislipidemia y obesidad abdominal). Los efectos metabólicos adversos de los diuréticos tiazídicos dependen de la dosis, por lo que se puede lograr una reducción significativa de la presión arterial con efectos secundarios mínimos cuando se usan dosis bajas.

126
Q
  1. Masculino de 46 años consulta por debilidad y opresión torácica intermitente desde hace 24 horas. Niega antecedentes de importancia. Su padre tuvo un infarto cardiaco a los 68 años y su madre tiene miastenia gravis. Se realiza un ECG que muestra ausencia de onda P y R-R irregular, FC de 120 lpm con QRS estrecho. Usted debe realizar paraclínicos adicionales para diagnosticar:
    A. Disección aórtica.
    B. Síndrome de Cushing.
    C. Feocromocitoma.
    D. Hipertiroidismo.
A

Respuesta correcta: D

Tema e ítem: Hipertiroidismo y fibrilación auricular.

Argumento: El ECG de esta paciente muestra un ritmo irregular con intervalos R-R variables, sin ondas P claramente discernibles (También debería o podría evidenciarse taquicardia de complejo estrecho). Estos hallazgos son compatibles con fibrilación auricular con respuesta ventricular rápida (es decir, frecuencia >100/min). La FA puede ser asintomática o presentarse con síntomas como palpitaciones, debilidad, mareos, presíncope, disnea y/o dolor torácico.
La FA es la arritmia supraventricular más común y puede ocurrir debido a una variedad de trastornos cardíacos y/o sistémicos. Entre las opciones de respuesta, el hipertiroidismo es la causa más común y probable de FA de aparición repentina. A todos estos pacientes con FA de reciente aparición se les deben medir los niveles de TSH y T4 libre para detectar hipertiroidismo oculto como causa subyacente.

127
Q
  1. Don Javier de 65 años, antecedente de HTA, es ingresado al servicio de Urgencias tras desarrollar un evento cerebrovascular isquémico transitorio. El electrocardiograma, ecocardiograma, ecografía carotidea y La TAC simple de cráneo son normales. Analítica: CT 199 mg/dL, LDL 102 mg/dL, HDL 37 mg/dL, TG 350 mg/dL. Se inicia tratamiento con aspirina y asesoramiento sobre cambios en el estilo de vida.
    ¿Cuál de las siguientes opciones debe ser añadida al manejo de Don Javier?
    A. Atorvastatina 20 mg/día + ezetimiba 10 mg/día
    B. Rosuvastatina 40 mg/día
    C. Atorvastatina 20 mg/día
    D. Icosapent etílico
A

Respuesta correcta: B

Tema e ítem: Estatinas y reducción de recurrencias por enfermedad cardiovascular ateroesclerótica

Argumento: La antiagregación, asesoramiento sobre cambios en el estilo de vida y las estatinas de alta intensidad son componentes claves del manejo de los pacientes que han cursado con un evento cerebrovascular isquémico transitorio. La prevención secundaria de la enfermedad cardiovascular aterosclerótica (ECVA) con estatinas de alta intensidad está indicada en todos los pacientes con manifestaciones clínicas de la misma.

La ECVA clínica comprende el síndrome coronario agudo, la revascularización coronaria, el ictus o el accidente isquémico transitorio, y la enfermedad arterial periférica. El tratamiento con estatinas de alta intensidad incluye atorvastatina (40-80 mg/día) o rosuvastatina (20-40 mg/día). Se debe realizar un perfil lipídico de control entre las 4 y 12 semanas después de iniciar el tratamiento con estatinas para garantizar una reducción de LDL del 50% o más. Recuerda que los pacientes con un riesgo muy elevado de enfermedad cardiovascular recurrente deben tener un objetivo de colesterol LDL inferior a 70 mg/dl.

La ezetimiba es un
hipocolesterolemiante de segunda línea que puede añadirse al tratamiento con estatinas en los pacientes con ECVA que no han logrado una reducción del 50% o más de sus valores de LDL o un nivel de LDL inferior a 70 mg/dL . También puede utilizarse en pacientes con ECVA que no toleran las estatinas a pesar de múltiples intentos. La ezetimiba más una estatina puede ser eventualmente una terapia apropiada si este paciente no alcanza su objetivo de colesterol LDL de menos de 70 mg/dL, pero no una recomendación de tratamiento inicial.

128
Q
  1. Masculino de 47 años, sin antecedentes de interés, acude a consulta tras la detección incidental y reciente de un nódulo pulmonar sólido mientras se evaluaba un dolor abdominal en el servicio de Urgencias con TAC simple de abdominal. La TAC reportó un nódulo solido de 7.5 mm en la periferia del lóbulo inferior izquierdo, sin alteraciones adyacentes del parénquima pulmonar o asociado a adenopatías.
    ¿Cuál sería la conducta adecuada a seguir?
    A. Realizar tomografía por emisión de positrones con 18F-fluorodeoxiglucosa (PET-FDG)
    B. Realizar biopsia transtorácica guiada por TC
    C. Repetir TAC en 12 meses
    D. Educar al paciente sobre el bajo riesgo de malignidad del nódulo e informar que no es necesario realizar seguimiento con imágenes
A

Respuesta correcta: C

Tema e ítem: Nódulo pulmonar – Enfoque práctico para exámenes de Residencia

Argumento: Los nódulos pulmonares son pequeñas opacidades radiográficas asintomáticas <3 cm rodeadas de parénquima pulmonar normal. Suelen encontrarse incidentalmente en TAC realizadas por otras indicaciones. El paso inicial en la evaluación de un nódulo pulmonar es evaluar la probabilidad de malignidad. Los factores de riesgo de malignidad incluyen antecedentes de tabaquismo, edad avanzada, sexo femenino, antecedentes familiares de cáncer de pulmón, enfisema, exposición al amianto, mayor tamaño (>8 mm), localización en el lóbulo superior y tipo de nódulo. El riesgo puede estimarse cuantitativamente con calculadoras de riesgo como la de la Universidad de Brock o la calculadora del modelo de la Clínica Mayo. También deben realizarse exploraciones previas, si se dispone de ellas, para evaluar la estabilidad del nódulo a lo largo del tiempo.
Aparte del tamaño del nódulo, este paciente tiene un riesgo bajo de malignidad. De acuerdo con las recomendaciones de la Sociedad Fleischner, los pacientes de bajo riesgo con un nódulo sólido de 6-8 mm de tamaño deben ser sometidos a seguimiento con TAC en un plazo de 6 a 12 meses, y debe considerarse la posibilidad de realizar una segunda exploración de seguimiento a los 18 o 24 meses.

  • Una PET-FDG no sería útil para delimitar mejor el riesgo de malignidad en este paciente con un nódulo de 7.5 mm. La sensibilidad de una PET para detectar cáncer de pulmón se ve afectada por la masa del nódulo y su tasa metabólica. Son más útiles para evaluar una posible malignidad con lesiones de al menos 10 mm de tamaño, con un mayor potencial de resultados falsos negativos para cánceres menores de este tamaño.
  • Por el riesgo bajo de malignidad, la biopsia transtorácica no está indicada en este paciente.
  • este paciente presenta un riesgo bajo de cáncer de pulmón, el tamaño del nódulo (6-8 mm) obliga a realizar una evaluación y un seguimiento adicionales. En los pacientes de bajo riesgo que tienen un nódulo pulmonar sólido menor de 6 mm de tamaño, podríamos considerar no realizar seguimiento por imágenes.
129
Q
  1. Varón de 25 años, consulta al servicio de Urgencias por cuadro clínico de 3 días de evolución dado por tumefacción y dolor en la rodilla, tobillo y muñeca izquierda, con impotencia funcional y febrícula. Refiere que dos semanas antes había un episodio de diarrea autolimitada. Al examen físico se evidencia derrame articular en las regiones referidas y múltiples placas psoriasiforme en palmas, dedos y plantas de los pies.
    ¿Cuál es el diagnóstico más probable?
    A. Artritis reumatoide
    B. Artritis psoriásica
    C. Sífilis secundaria
    D. Artritis reactiva
A

Respuesta correcta: D

Tema e ítem: Artritis reactiva - ¿Cuándo pensar en ella?

Argumento: Las placas psoriasiformes de este paciente son compatible con queratodermia blenorrágica (erupción psoriasiforme en palmas, dedos y plantas de los pies), asociada a artritis reactiva. La artritis reactiva es una causa poco frecuente de artritis inflamatoria que puede aparecer tras infecciones genitourinarias y gastrointestinales específicas, como Chlamydia trachomatis y Ureaplasma urealyticum en la uretra y Campylobacter, Escherichia coli, Salmonella, Shigella y Yersinia en el intestino. Los síntomas de la artritis inflamatoria suelen aparecer entre 2 y 3 semanas después de la infección. La artritis es asimétrica y puede ser monoarticular u oligoarticular; las articulaciones comúnmente afectadas son la muñeca, rodilla y tobillo. Además del queratoderma blenorrágico, otras manifestaciones extraarticulares pueden ser la conjuntivitis (más común que la uveítis anterior) y la balanitis circinada (erupción psoriasiforme en el pene).

130
Q
  1. Aurora de 45 años, acude a consulta con un cuadro de inicio insidioso de 6 meses de evolución consistente en dolor, eritema y aumento de la temperatura ambas manos. Refiere rigidez matutina de 1 hora de duración. Se automedica con acetaminofén + ibuprofeno con mejoría parcial del dolor. Al examen físico se evidencia tumefacción bilateral y dolor a la palpación en 2ª y 3ª articulación metacarpofalángicas y 3ª articulación interfalángica proximal. Analítica: Hb 11 g/dL, factor reumatoide (+), anticuerpos anti-CCP (+), ANA (-).
    ¿Cuál de los siguientes estudios sería el más adecuado a realizar en Aurora?
    A. Realizar RNM de manos
    B. Medir anticuerpos anticitoplasma de neutrófilo (ANCA)
    C. Realizar Rx simple de manos AP y lateral
    D. Anticuerpos anti-ADN de doble cadena (anti-dsDNA)
A

Respuesta correcta: C

Tema e item: Artritis reumatoides y estudios diagnósticos ideales en su enfoque diagnostico-terapéutico

Argumento: Estamos indudablemente frente a un caso clínico compatible con artritis reumatoide (AR). Es cierto, ninguna característica radiográfica por sí sola es diagnóstica de AR. Sin embargo, rasgos como la osteopenia periarticular, estrechamiento del espacio articular y las erosiones óseas son característicos de la enfermedad establecida y pueden ayudar a consolidar el diagnóstico de artritis reumatoide cuando los hallazgos de la exploración física son equívocos. Si están presentes, también pueden subrayar la necesidad de un tratamiento precoz con agentes modificadores de la enfermedad, que pueden incluir fármacos biológicos, ya que la presencia de lesiones radiográficas en el momento del diagnóstico de la artritis reumatoide predice una evolución más grave de la patología. Así pues, la radiografía simple puede ayudar tanto en la toma de decisiones diagnósticas como terapéuticas, así como en el establecimiento de una línea de base que ayude a monitorizar la actividad de la enfermedad.

  • Los ANCA se dirigen contra antígenos que se encuentran en los gránulos citoplasmáticos de neutrófilos y monocitos. Estas suelen realizarse para ayudar a diagnosticar o descartar la granulomatosis con poliangeítis y la poliangeítis microscópica. Sin embargo, la artritis inflamatoria simétrica poliarticular de las articulaciones pequeñas de las manos, como se observa en este paciente, no es una característica de la vasculitis por ANCA, por lo que no sería una prueba que aporte al enfoque diagnostico o terapéutico del paciente.
  • Los anticuerpos anti-ADN de doble cadena (anti-dsDNA) deben reservarse para los pacientes con un resultado positivo en la prueba de ANA y un síndrome clínico que sugiera una enfermedad reumatológica subyacente.
  • La RNM puede ser una herramienta valiosa en la evaluación de patologías con compromiso musculoesqueléticas, pero carece de especificidad para el diagnóstico de la AR. El primer paso en la evaluación radiológica de un paciente con síntomas articulares suele ser la radiografía simple, ya que es relativamente barata, está fácilmente disponible y permite evaluar los hallazgos característicos de la artritis reumatoide.
131
Q
  1. Varón de 35 años, sin antecedentes de interés, consulta al servicio de Urgencias por cuadro clínico de 2 días de evolución dado por secreción de aspecto purulento e inflamación en su ojo izquierdo. Niega dolor o cambios en la agudeza visual. Al examen físico se evidencia ojo izquierdo: hiperemia conjuntival, epifora y secreción mucopurulenta. La agudeza visual es 20/20 en ambos campos oculares. El resto del EF es anodino.
    ¿Cuál de los siguientes es el tratamiento más adecuado para este paciente?
    A. Ceftriaxona 1gr IM DU + irrigación ocular con suero salino cada 60/minutos
    B. Tobramicina colirio 0,3% cada 4-6 horas
    C. Olopatadina colirio 0.2% cada 7 horas
    D. Prednisolona colirio 1% cada 12 horas
A

Respuesta correcta: B

Tema e ítem: Conjuntivitis bacteriana aguda – ¿Cuál es el tratamiento ideal?

Argumento: La solución oftálmica de tobramicina colirio 0,3% cada 4-6 horas es el tratamiento tópico más adecuado para este paciente con conjuntivitis bacteriana (hiperemia conjuntival, epifora y secreción mucopurulenta). Según la guía 2019 de la Academia Americana de Oftalmología, la conjuntivitis bacteriana leve suele ser autolimitada, y suele resolverse espontáneamente sin tratamiento específico en adultos inmunocompetentes. Sin embargo, el uso de terapia antibacteriana tópica se asocia con una remisión clínica y microbiológica más temprana en comparación con el placebo en los días 2 a 5 del tratamiento. El tratamiento puede reducir la transmisibilidad. La elección del antibiótico suele ser empírica. Dado que un ciclo de un antibiótico tópico de amplio espectro suele ser eficaz, puede seleccionarse la opción más conveniente o menos costosa; no hay pruebas clínicas que sugieran la superioridad de ningún antibiótico en particular. De todos los antibioticos tópicos existentes, los medicamentos más comercializados en Colombia según evidencia son:
(a) Tobramicina colirio 0,3% cada 4-6 horas
(b) Ciprofloxacina colirio 0,3% cada 4-6 horas
(c) Azitromicina colirio 1,5% cada 12 horas los primeros 2 días y luego 1 vez al día por 3 días más.
(d) Gentamicina colirio 0,3% cada 4-6 horas (Mayor toxicidad a la superficie ocular)

  • Los antihistamínicos tópicos, como la olopatadina, no tienen ningún papel en el tratamiento de la conjuntivitis bacteriana aguda. La olopatadina se utiliza habitualmente en pacientes con conjuntivitis alérgica, que suelen presentar hiperemia conjuntival bilateral, prurito, secreción acuosa e hinchazón de los párpados.
  • Los glucocorticoides oftalmológicos tópicos, como la prednisolona sola o en combinación con un antibiótico, no tienen un papel en el tratamiento de la conjuntivitis bacteriana aguda. Cuando se administran a pacientes con queratitis bacteriana o vírica, pueden provocar daños en la córnea, incluida la perforación y la cicatrización.
132
Q
  1. Femenina de 25 años de edad, G1P0A0, con 38 semanas de gestación, consulta por rotura espontánea de membranas y contracciones dolorosas. El cultivo para estreptococo del grupo B es negativo. Al examen físico presenta 8 cm de dilatación, borramiento del 90%, estación 0. Se administra analgesia epidural. Refiere un aumento de la presión rectal; se repite la exploración cervical: sin cambios. Tiene 4 contracciones cada 10 min. 4 h más tarde, cuello uterino sin cambios. El monitoreo fetal es categoría 1. ¿Cuál es el mejor paso a seguir?
    A. Administrar oxitocina intravenosa.
    B. Realizar parto por cesárea.
    C. Repetición del examen cervical en 2 horas.
    D. Parto vaginal instrumentado + amniotranfusión.
A

Respuesta correcta: B

Tema e ítem: Etapas y fases del parto – Manejo

Argumento: La primera fase del parto comienza con el inicio de contracciones regulares y finaliza cuando el cuello uterino está dilatado 10 cm. Consta de una fase latente con dilatación cervical gradual y una fase activa con dilatación rápida. La transición entre la fase latente y la activa se produce a los 6 cm de dilatación.

Esta paciente presenta detención de la fase activa, definida como ausencia de cambios cervicales durante ≥4 horas a pesar de contracciones adecuadas o ausencia de cambios cervicales durante ≥6 horas con contracciones inadecuadas (y en infusión de oxitocina). Las causas incluyen etiologías uterinas (contracciones inadecuadas), fetales (malposición, macrosomía) o pélvicas (deformidad o fractura).

  • El mejor paso siguiente en el tratamiento de la parada de la fase activa es el parto por cesárea porque no se espera más dilatación cervical y retrasar el parto aumenta la morbilidad materno-fetal (infección intraamniótica, hemorragia posparto, entre otras).
  • La oxitocina aumenta el trabajo de parto al incrementar la frecuencia y la fuerza de las contracciones. La oxitocina se utiliza en pacientes que cumplen los criterios de protracción del parto, pero no en las que cumplen los criterios de parada del parto.
  • Se realiza un parto vaginal instrumentado durante la segunda fase del parto (10 cm de dilatación hasta el parto fetal) para acelerar el parto en caso de trazados de categoría III o agotamiento materno. Esta paciente tiene 8 cm de dilatación. La amnioinfusión se realiza mediante un catéter de presión intrauterina y está indicada para anomalías del trazado de la frecuencia cardiaca fetal (desaceleraciones variables). No se utiliza de rutina, ni para aumentar el trabajo de parto porque no afecta a la dilatación cervical.
133
Q
  1. Femenina de 32 años de edad, G4P4, asiste a consulta rutinaria a las 5 semanas postparto. Quiere recibir asesoría anticonceptiva. Cree que algunas opciones pueden dificultar la pérdida de peso. Previo a la gestación, sus menstruaciones eran abundantes y presentaba diagnóstico de anemia leve. A menudo olvidaba tomar su suplementación de hierro oral. Su hijo recibe lactancia exclusiva. ¿Cuál es el método anticonceptivo más adecuado para esta paciente?
    A. Dispositivo intrauterino que contiene levonorgestrel.
    B. Dispositivo intrauterino de cobre.
    C. No se necesita anticoncepción durante la lactancia.
    D. Inyección de medroxiprogesterona.
A

Respuesta correcta: A

Tema e ítem: Anticoncepción – definir método en el postparto.

Argumento: El dispositivo intrauterino (DIU) con levonorgestrel es un anticonceptivo reversible de acción prolongada que evita el embarazo mediante la liberación de levonorgestrel (una progestina), que crea una barrera física al espesar el moco cervical y dificultar la implantación mediante la decidualización del endometrio. Tiene una eficacia superior al 99% y su uso está autorizado en Estados Unidos durante un máximo de 7 años. Un efecto secundario frecuente es la amenorrea, que puede utilizarse para mejorar la anemia y las hemorragias uterinas anormales. Un pequeño porcentaje de mujeres experimenta efectos secundarios sistémicos (cambios de humor, sensibilidad mamaria, dolores de cabeza). El aumento de peso no es un efecto secundario.

  • El DIU de cobre, otra opción anticonceptiva reversible de acción prolongada, está aprobado en Estados Unidos por un periodo de hasta 10 años. El DIU de cobre puede provocar hemorragias menstruales abundantes y no debe colocarse en mujeres con hipermenorrea o anemia, ya que puede agravar estas afecciones.
  • La medroxiprogesterona es una opción para la anticoncepción a largo plazo, en la que las pacientes reciben inyecciones cada 3 meses. Dado que el aumento de peso es un efecto secundario frecuente, la medroxiprogesterona no es óptima para esta paciente, que está preocupada por las dificultades con la pérdida de peso posparto.
  • La lactancia puede causar anovulación y, por lo tanto, cierto grado de anticoncepción, ya que los niveles elevados de prolactina inhiben la liberación de la hormona liberadora de gonadotropina del hipotálamo. Sin embargo, la lactancia no se considera un método anticonceptivo fiable porque la ovulación puede reanudarse mientras la madre sigue amamantando.
134
Q
  1. Femenina de 29 años de edad con 12 semanas de gestación asiste a control prenatal. Refiere sensación de presión pélvica, sin hemorragia ni flujo vaginal. Completó un ciclo de antibióticos a las 10 sem por un urocultivo que reportó ≥100.000 UFC de E. coli. Ha tenido polaquiuria, pero niega disuria, tenesmo y/o hematuria. Signos vitales: Temp. de 37°C, PA 96/68 mmHg, FC 90/min. El examen físico es normal para su edad gestacional ¿Cuál es el mejor paso a seguir?
    A. Prueba de volumen residual posmiccional.
    B. Realizar cistouretrograma miccional.
    C. Supresión/administración diaria de antibióticos.
    D. Realizar paraclínicos para confirmar tratamiento exitoso.
A

Respuesta correcta: D

Tema e ítem: Bacteriuria en el embarazo - conducta postmanejo.

Argumento: Los síntomas urinarios (polaquiuria) son frecuentes en el embarazo debido a la compresión de la vejiga por el aumento de tamaño del útero. Dado que a menudo es difícil distinguir entre la cistitis y los síntomas urinarios normales relacionados con el embarazo, todas las mujeres se someten a un cultivo de orina de cribado en la visita prenatal inicial para evaluar la infección del tracto urinario. Las gestantes tienen un mayor riesgo de pielonefritis aguda (y complicaciones maternofetales subsiguientes) debido a la relajación del músculo liso relacionada con la progesterona que causa dilatación ureteral, lo que permite que las bacterias asciendan fácilmente al tracto urinario superior. Por lo tanto, las pacientes con ≥100.000 unidades formadoras de colonias/mL de bacterias requieren tratamiento incluso en ausencia de síntomas (bacteriuria asintomática). El tratamiento consiste en antibióticos orales (nitrofurantoína). Un tercio de los pacientes no erradica completamente la bacteriuria con el tratamiento inicial. Por lo tanto, se realiza un urocultivo-uroanálisis de repetición unas semanas después de finalizar el tratamiento antibiótico. Las pacientes con un urocultivo repetido negativo de orina negativo pueden reanudar los cuidados prenatales habituales. Por el contrario, aquellas con otro urocultivo positivo requieren un retratamiento y otra prueba de curación. Las declaraciones de grupos de expertos también resaltan el valor incierto de repetir la detección de bacteriuria asintomática. Las pautas de la IDSA de 2019 informaron que no hay evidencia directa que evalúe el beneficio de la detección repetida después de un episodio inicial de bacteriuria asintomática y que no se sabe si el retratamiento de la bacteriuria recurrente o persistente mejora los resultados. Las directrices de la ACOG establecen que en la literatura se recomienda un urocultivo de seguimiento después del tratamiento de la bacteriuria asintomática, pero indican que se necesitan más datos para determinar la eficacia de esta estrategia

  • La supresión antibiótica diaria puede estar indicada en pacientes con bacteriuria persistente en el urocultivo repetido o pielonefritis aguda durante el embarazo.
  • El volumen residual posmiccional se mide en pacientes con síntomas de retención urinaria (urgencia, retención), pero no se utiliza para evaluar la bacteriuria asintomática.
  • El cistouretrograma miccional se realiza para evaluar el reflujo vesicoureteral (RVU) en niños, que pueden beneficiarse de un manejo quirúrgico para prevenir ITU recurrentes. El RVU leve puede ser un hallazgo normal en el embarazo que se resuelve después del parto; por lo tanto, aunque se diagnostique RVU leve en el embarazo, no suele requerir tratamiento.
135
Q
  1. Femenina de 38 años, G5P4, cursando con 10 semanas de gestación asiste a control prenatal. Presenta antecedente de diabetes mellitus tipo 2 en manejo actual con insulina. No presenta urgencia urinaria, hematuria ni disuria. Signos vitales: Temp. de 36,7°C, Presión Arterial de 130/80 mmHg. Su IMC 29 kg/m2 y la FCF 155/min. Un urocultivo reciente reporta >100.000 UFC/mL de E. coli. ¿Cuál es la conducta más apropiada?
    A. Amoxicilina-clavulanato, 500 mg por vía oral cada 8h por 7 días.
    B. Trimetoprim-sulfametoxazol, 800/160 mg, 1 comprimido cada 12h por 5 días.
    C. Ciprofloxacina, 30 mg/kg/día repartido en 2 dosis por 5 días.
    D. No se indica ningún tratamiento o estudio adicional.
A

Respuesta correcta: A

Tema e ítem: Bacteriuria en el embarazo - Manejo

Argumento: La bacteriuria asintomática (BA) es el crecimiento de ≥100.000 unidades formadoras de colonias/mL de una sola bacteria a partir de una muestra de orina limpia en ausencia de síntomas de infección urinaria. La diabetes mellitus previa al embarazo, como en esta paciente, es un factor de riesgo. El cribado y tratamiento de la BA no suele realizarse en la población general porque no disminuye las tasas de infección sintomática ni los resultados adversos. En cambio, las gestantes tienen un mayor riesgo de pielonefritis y complicaciones (parto prematuro, bajo peso al nacer).

Esto está probablemente relacionado con la relajación del músculo liso inducida por la progesterona (es decir, dilatación ureteral, laxitud de la válvula) que permite a las bacterias ascender al tracto urinario superior.

Por lo tanto, todas las pacientes embarazadas se someten a un cribado de BA en la visita prenatal inicial. Las pacientes cuyos urocultivos de cribado son positivos requieren tratamiento antibiótico. El patógeno más frecuente es Escherichia coli, como en esta paciente. El tratamiento antibiótico de primera línea incluye cefalexina, amoxicilina-clavulánico o nitrofurantoína. Recuerde, la bacteriuria asintomática siempre se trata de cardo al reporte de sensibilidad antibiótica del urocultivo, la intención de la pregunta fue analizar las contraindicaciones antibióticas durante el embarazo. Se repite el urocultivo (prueba de curación) una vez finalizado el tratamiento antibiótico para garantizar la desaparición de la bacteriuria.

  • Las fluoroquinolonas (ciprofloxacino) están contraindicadas en el embarazo debido a su posible asociación con deformidades óseas fetales y artropatía.
  • La doxiciclina y otros antibióticos de tetraciclina están contraindicados en el embarazo porque interfieren en el desarrollo óseo y dental del feto. Los niños expuestos a la tetraciclina en el útero también pueden desarrollar decoloración gris de los dientes.
  • La trimetoprima-sulfametoxazol es segura en el segundo trimestre pero está contraindicada en el primero (10 semanas) porque interfiere con el metabolismo del ácido fólico, que es fundamental para el desarrollo temprano del feto. También está contraindicado en el tercer trimestre debido al riesgo de kernicterus neonatal.
136
Q
  1. Femenina de 45 años G3P3 consulta por sangrado menstrual abundante y prolongado. Las menstruaciones ocurren a intervalos regulares de 28 días y dura 7 días con flujo abundante. Sus signos vitales son normales. Al examen físico hay un útero firme, irregular, de tamaño consistente a un embarazo de 15 semanas. Un hemograma evidencia anemia microcitica. Una ecografía pélvica evidencia múltiples masas intramurales en un útero irregular. La paciente refiere haber culminado su maternidad por lo que le gustaría un tratamiento definitivo para sus síntomas por lo que se programa para histerectomía total en 2 meses. ¿Cuál es el adecuado previo a la intervención quirúrgica?
    A. Anticonceptivos orales de solo progestágeno.
    B. Manejo con leuprolida.
    C. Terapia con ácido tranexámico.
    D. Anticonceptivos orales combinados de estrógenos y progestágenos.
A

Respuesta correcta: B

Tema e ítem: Miomatosis uterina – Enfoque terapéutico

Argumento: Es probable que las alteraciones en la menstruación de esta paciente sean secundarias a la presencia de leiomiomas, los cuales se pueden visualizar en la ecografía ya realizada como múltiples masas intramurales. Además, esta paciente refiere que tiene una paridad satisfecha, por lo que se programa para la realización de una histerectomía, sin embargo, durante el abordaje prequirúrgico se requiere manejo para disminuir el tamaño del útero miomatoso y para corregir la anemia disminuyendo la pérdida de sangre. Se encuentra indicada la terapia con leuprolida (Un agonista de GnRH) para pacientes con leiomiomas uterinos grandes y/o múltiples, 2 a 3 meses antes de la cirugía. La leuprolida puede reducir el tamaño de los miomas al inducir hipoestrogenismo e hipoprogesteronismo. Una reducción en el tamaño de un útero leiomiomatoso disminuye el tiempo quirúrgico, acelera la recuperación posoperatoria e incluso puede permitir la realización de una histerectomía vaginal que generalmente causa menos pérdida de sangre que una histerectomía abdominal. Además, la leuprolida también ayuda a tratar la menorragia al inducir amenorrea, lo que mejora los niveles de hemoglobina antes de la cirugía.

  • Las píldoras anticonceptivas de solo progestágenos pueden indicarse en ciertas ocasiones ante la presencia de un sangrado uterino anormal. Sin embargo, existe una alta evidencia de que este tipo de anticonceptivos podría aumentar el tamaño de los miomas, por lo tanto, no se recomienda su uso en esta paciente.
  • La terapia a corto plazo con ácido tranexámico se puede usar para tratar la menorragia grave y se puede administrar poco antes de una histerectomía para reducir el sangrado durante la cirugía. Sin embargo, no tiene efecto sobre el tamaño de los miomas.
  • La terapia anticonceptiva oral combinada es eficaz para reducir el flujo sanguíneo menstrual y se ha demostrado que puede disminuir la indicidencia de leiomiomas. Sin embargo, no es eficaz para disminuir el tamaño de estos.
137
Q
  1. Joshua es un niño de 10 años, llevado a urgencias por un brote que apareció ayer al despertar, localizado en el tronco, más eritematoso en las axilas e ingle. Hace dos días inicia con cefalea, fiebre y odinofagia. El papá dice que dos niños de su colegio tenían los mismos síntomas y ahora tienen descamación en manos y pies. Al examen físico febril, taquicárdico y taquipnéico, de buen aspecto. Se evidencian pápulas eritematosas y rugosas en el tronco que palidecen con la digitopresión, la orofaringe posterior se observa eritematosa, con exudados amigdalinos. Hay púrpura húmeda. ¿Cuál es el diagnóstico más probable?
    A. Kawasaki
    B. Enfermedad de manos, pies y boca
    C. Escarlatina
    D. Mononucleosis
A

Respuesta correcta: C.

Tema-Ítem: Escarlatina-Exantemáticas

Argumentación: La escarlatina es una enfermedad causada por estreptococos del grupo A que se desarrolla con mayor frecuencia en niños que tienen faringitis estreptocócica. Ciertas cepas de Streptococcus pyogenes producen exotoxinas eritrogénicas, que son responsables de la erupción característica de la escarlatina.
La fiebre, odinofagia y cefalea suelen preceder a la erupción, que suele extenderse por el tronco, la ingle y las axilas. El exantema tiene una textura característica de “papel de lija”, finamente papular y es más pronunciado en los pliegues de la piel. A medida que la enfermedad se resuelve, la descamación de la erupción provoca la descamación de las manos y los pies.
Los hallazgos clásicos en la exploración son una faringe eritematosa con exudados amigdalinos, petequias palatinas y lengua en fresa. El área alrededor de la boca puede parecer pálida en comparación con las mejillas rojas, dando la apariencia de palidez peribucal. La linfadenopatía cervical anterior, dolorosa y unilateral, como se observa en este paciente, es otro hallazgo inespecífico.

El diagnóstico se confirma mediante una prueba rápida de antígeno estreptocócico o un cultivo de garganta y el tratamiento es con penicilina.

-El eritema infeccioso es causado por el parvovirus B19 y causa una erupción clásica en forma de “mejilla abofeteada”, no de papel de lija.

-La enfermedad de manos, pies y boca es una infección común por el virus Coxsackie que se presenta en niños pequeños con lesiones en las manos/pies y ulceraciones orales, hallazgos que no están presentes en este paciente.

-El diagnóstico de la enfermedad de Kawasaki (EK) requiere 5 días de fiebre y ≥4 criterios clínicos: ganglio cervical >1,5 cm, erupción polimorfa, edema de manos/pies, conjuntivitis y cambios en las mucosas (p. ej., lengua en fresa, labios agrietados). La EK es poco probable en este paciente con sólo 2 días de fiebre y 2 de los criterios enumerados.
-La mononucleosis puede presentarse con faringitis exudativa y erupción cutánea; sin embargo, la linfadenopatía cervical posterior es típica.

138
Q
  1. Paciente de 8 años de edad, con antecedentes de síndrome de fiebre periódica familiar. Llevada por su madre a urgencias debido a un cuadro clínico de aproximadamente 13 días de evolución de fiebre objetiva con pico máximo de 40°C, que inicia con deposiciones diarreicas que se autolimitan. Hace 6 días inicia con congestión nasal, odinofagia y tos seca. Al examen físico, taquicárdica y febril, sin signos de dificultad respiratoria, orofaringe eritematosa, con un brote papular en manos y pies, con sibilancias a la espiración forzada. En la rx de tórax, con infiltrados multilobares bilaterales. ¿Qué microorganismo podría ocasionar la enfermedad que sospechamos?
    A. Enterovirus
    B. No hay microorganismo, es por su enfermedad de base
    C. M. pneumoniae
    D. Adenovirus
A

Respuesta correcta: C.

Tema-Ítem: Infección por M. pneumoniae en niños-Enfermedades infecciosas

Argumentación: Esta paciente presenta un cuadro clínico larvado y subagudo, con deposiciones diarréicas y posteriormente síntomas respiratorios leves. Al examen físico, con brote papular y con sibilancias a la auscultación. Estas son algunas de las manifestaciones pulmonares y extrapulmonares del M. pneumoniae.

M. pneumoniae causa un amplio espectro de enfermedades. Muchas infecciones por M. pneumoniae son asintomáticas, Las manifestaciones clínicas de la infección sintomática por M. pneumoniae generalmente se dividen en manifestaciones del tracto respiratorio (las más comunes) y extrapulmonares (menos comunes). Ninguna de las manifestaciones es exclusiva de M. pneumoniae. Aunque las infecciones por M. pneumoniae generalmente son leves y autolimitadas, los pacientes de todas las edades pueden desarrollar neumonía adquirida en la comunidad (NAC) grave o manifestaciones extrapulmonares.
Los síntomas y signos comúnmente incluyen:
*Fiebre (86 a 96 por ciento)
*Tos, que suele ser no productiva y puede persistir durante semanas o meses
*Fatiga
*Disnea
*Cefalea
*Odinofagia
*Auscultación anormal

*Manifestaciones extrapulmonares
Otros hallazgos clínicos relacionados con el tracto respiratorio incluyen sensibilidad en los senos nasales, rinorrea, eritema leve de la orofaringe posterior, otalgia, eritema o ampollas en membrana timpánica y adenopatía cervical no prominente.
M. pneumoniae puede causar una enfermedad respiratoria inespecífica similar a una infección del tracto respiratorio superior (p. ej., dolor de garganta no exudativo, coriza, dolor de cabeza, dolor de oído, tos prolongada).
La infección por M. pneumoniae puede empeorar los síntomas del asma y puede estar asociada con sibilancias en niños que no tienen asma (como en la paciente del caso).

Las erupciones mucosas o cutáneas son una manifestación extrapulmonar común de la infección por M. pneumoniae y ocurren hasta en el 25 por ciento de los pacientes.
Las manifestaciones dermatológicas varían desde una erupción eritematosa maculopapular o vesicular leve (que se observa con mayor frecuencia junto con infecciones del tracto respiratorio), hasta el síndrome de guantes y calcetines purpúrico papular ampolloso y una erupción mucocutánea infecciosa reactiva (un síndrome similar a Stevens-Johnson).
Para los niños con neumonía documentada por M. pneumoniae, se sugieren: macrólidos, tetraciclinas o fluoroquinolonas. El régimen más comúnmente utilizado es: Azitromicina 10 mg/kg en una dosis (dosis máxima 500 mg) por vía oral o intravenosa (IV) el primer día y 5 mg/kg en una dosis (dosis máxima 250 mg) durante los siguientes cuatro días.

139
Q
  1. Juan Pablo, 1 año de edad, con sospecha de asma del lactante, usa salbutamol en caso de crisis. Es llevado por sus padres a urgencias debido a cuadro clínico de dos días de evolución de rinorrea hialina, tos húmeda y pico febril de 38.5°C. Al examen físico, afebril, saturando 89% al aire ambiente, taquicárdico y taquipnéico, de buen aspecto, irritable, con retracciones subcostales, intercostales y supraesternales, con sibilancias difusas a la auscultación. ¿Cuál es el tratamiento más apropiado?
    A. Salbutamol 4 puff en esquema de crisis + bromuro de ipratropio en esquema de crisis + dosis única de esteroide oral
    B. Salbutamol 4 puff cada 20 minutos por una hora + esteroide en dosis única y revalorar
    C. Salbutamol en esquema de crisis + bromuro de ipratropio cada 20 minutos por una hora + esteroide por 3-5 días
    D. Salbutamol en esquema de crisis + bromuro de ipratropio en esquema de crisis + esteroide por 3-5 días
A

Respuesta correcta: C

Tema-Ítem: Crisis asmática-Urgencias pediátricas

Argumentación: Este paciente tiene una sospecha de asma del lactante, por lo que estamos enfrentando una exacerbación de su asma o una crisis asmática. Satura 89% y está con sibilancias, lo que nos habla de un broncoespasmo, por lo que debemos iniciar manejo con salbutamol en esquema de crisis, bromuro de ipratropio solo por la primera hora y luego por horario y completar el esteroide sistémico por 3-5 días.

La crisis asmática se define como un episodio agudo de incremento progresivo de síntomas respiratorios con deterioro progresivo de la función pulmonar por obstrucción grave al flujo aéreo, que lleva a modificación del tratamiento para mejorar los síntomas. Puede ocurrir en un paciente con diagnóstico previo de asma por agudización o como debut de la enfermedad en un paciente sin antecedentes de la enfermedad.

Es importante que en todos los pacientes midamos la severidad de la crisis, con alguna de las escalas que se encuentran disponibles, como Pulmonary Index Score (PIS), Pediatric Respiratory Assessment Measure (PRAM) o Pediatric Asma Score (PAS) y dejarlo consginado en la historia clínica y en cada evolución, para hacer seguimiento de la mejoría/empeoramiento de los signos y síntomas y poder realizar cambios en el manejo de forma inmediata.

En una crisis asmática, los pilares del tratamiento son: Oxígeno, B2 agonistas, bromuro de ipratropio y esteroide sistémico.
El salbutamol, se podría usar tanto inhalado como nebulizado, pues tiene la misma evidencia de éxito en el manejo en crisis leves-moderadas. Se usa en esquema de crisis tal como lo indican las guías actuales:
2-4 puff cada 20 minutos (en 1h)
2-4 puff cada 30 minutos (en 2h)
2-4 puff cada 1 hora (en 3h)
2-4 puff cada 2 horas (en 6h)
2-4 puff cada 4 horas (en 2 días)
Se deben usar máximo 10-12 puff en casos graves. También podríamos hacerlo de acuerdo al peso:
5-10 kg: 4 puff
10-20 kg: 6 puff
>20 kg: 8-10 puff

El bromuro de ipratropio es útil para usarlo EN COMBINACIÓN con el salbutamol en crisis moderadas/severas (podría considerarse su uso en crisis leves también) porque reduce días de estadía, sin embargo se debe usar solo por la primera hora y no en esquema de crisis como el salbutamol. Después podemos dejarlo por horario (cada 6-8 horas), por períodos corto de tiempo (solo por 48-72 horas). NUNCA debe formularse como monoterapia. En los niños menores de un año se puede hacer solo una nebulización.

Los esteroides sistémicos mejoran el efecto inmunosupresor de linfocitos T mediados por hipersensibilidad, generando una acción antiinflamatoria, disminuyendo secreciones y previniendo recurrencias. NO son de acción inmediata, necesitan hasta 4-6 horas para actuar. Se deben usar de forma temprana y por un total de 3-5 días.

140
Q
  1. La jefe de enfermería le comenta que el tratamiento que usted le ordenó a Juan Pablo ya fue culminado con éxito y se ve mucho mejor, listo para una revaloración. Usted va al cubículo donde está ubicado el paciente, lo revisa y la mamá le dice que lo observa mucho mejor, más tranquilo. Encuentra paciente dormido, con frecuencia respiratoria de 38, afebril, sin taquicardia, con retracciones subcostales y sin otros signos de dificultad respiratoria, con murmullo vesicular disminuido sin agregados. Usted decide continuar con igual tratamiento. A los cinco minutos, lo llaman por paro cardiorrespiratorio. ¿Cuál considera que fue la causa del paro?
    A. Crisis asmática grave
    B. Shock séptico
    C. Broncoaspiración
    D. Neumonía
A

Respuesta correcta: A.

Tema-Ítem: Tema-Ítem: Crisis asmática-Urgencias pediátricas

Argumentación: Este paciente presentó una crisis asmática grave con una evolución desfavorable y refractaria al tratamiento instaurado. SIEMPRE debemos despertar a los pacientes pediátricos para revisarlos y revalorarlos, pues este paciente progresó a una fatiga muscular (ya no estaba taquipnéico, tenía disminución de los signos de dificultad respiratoria) y tenía un tórax silente (sin sibilancias), lo que significa un empeoramiento marcado de su condición clínica. Estaba dormido, debido a la somnolencia que genera la hipercapnia. Pudimos habernos percatado de su mal estado clínico de forma precoz y temprana, antes de llegar a un paro cardiorrespiratorio.

Para realizar el examen físico de un niño, debemos iniciar siempre por el triángulo de valoración pediátrica, el cual se enfoca en 3 aspectos esenciales: Aspecto, circulación y respiración. Es importante recordar los valores normales de frecuencia respiratoria, de acuerdo a la edad:

< 2 meses: > 60 rpm
2–12 meses: > 50 rpm
1–5 años: > 40 rpm
6–8 años: > 30 rpm
> 8 años: > 20 rpm

En los casos de crisis asmática grave, se deben iniciar medidas salvadoras importantes, tales como determinar si el paciente requiere intubación orotraqueal e iniciar sulfato de magnesio.
El sulfato de magnesio, inhibe la contracción del musculo liso y la estimulación colinérgica, por lo que asociado a SABA y esteroide ayuda a la broncodilatación, además tiene efecto antiinflamatorio. Está indicado en paciente con falla al manejo inicial con SABA + SAMA + esteroide IV, asma grave, hipoxemia persistente. Entre más rápido se inicie mejores resultados.
Solo debería usarse en >2 años, en <5 años no hay una clara evidencia si podría usarse en casos graves (como en este paciente). La dosis es de 30-50 mg/kg/bolo, máximo 2 g (idealmente en dosis única) en 20-30 minutos mínimo, diluidos en 100 cc de salino para pasar en 1 hora. En estados refractarios incluso se puede poner en infusión a 25-50 mg/kg/h.

141
Q
  1. Paciente femenina de un día de vida, nace con 39 de edad gestacional. Al examen físico, estable hemodinámicamente. Se evidencia una masa firme y no fluctuante al lado derecho del cráneo. Además de control de paraclínicos, ¿Qué otras intervenciones considera que necesita?
    A. Manejo quirúrgico inmediato
    B. Neuroimagen
    C. Casco moldeador de cráneo
    D. Seguimiento y observación
A

Respuesta correcta: D

Tema-Ítem: Parto traumático-Neonatología

Argumentación: El cefalohematoma es una acumulación de sangre entre el cráneo y el periostio (es decir, el espacio subperióstico) causada por la rotura de los vasos sanguíneos subperiósticos debido a un traumatismo del parto. Los pacientes que nacen con ayuda de fórceps corren un mayor riesgo.
Debido a la naturaleza lenta del sangrado, los cefalohematomas se presentan horas después del nacimiento, como una masa en el cuero cabelludo, firme, no dolorosa y no fluctuante. La inflamación está bien delimitada y confinada a la superficie de un solo hueso porque el hematoma no cruza las líneas de sutura.

El diagnóstico es clínico y el tratamiento es con observación. A medida que la sangre del cefalohematoma se degrada, el paciente tiene un mayor riesgo de hiperbilirrubinemia y puede requerir fototerapia. Se debe asegurar a los cuidadores que los cefalohematomas generalmente se reabsorben espontáneamente en un mes, tiempo durante el cual el área afectada puede sentirse más firme debido a la coagulación o la calcificación.

142
Q
  1. Femenina de 56 años, consultó por 1 semana de fiebre, disnea y tos húmeda expectorante con flema maloliente y verdosa. Refiere que recibió hace 5 días manejo con azitromicina para un cuadro clínico similar; sin embargo, no ha presentado mejoría a pesar de tomar el medicamento. Antecedente de una endoscopia digestiva alta hace 10 días por pirosis. Signos vitales: Tº 38.8 °C, PA de 130//80 mm Hg, FC de 108 lpm y FR 22 rpm. Al examen físico, crepitos en ápex pulmonar derecho, sin otro hallazgo anormal. El hemoleucograma con leucocitos en 14.500/mm3, y una radiografía de tórax evidenció un infiltrado en el lóbulo superior derecho. ¿Cuál de los siguientes antibióticos es el indicado para esta paciente?
    A. Amoxicilina/clavulánico
    B. Ciprofloxacina
    C. Dicloxacilina
    D. Metronidazol
A

Respuesta correcta: A
Tema e ítem: Neumonía bacteriana aspirativa (endoscopia) tramiento
Argumento:Esta paciente que presenta fiebre, síntomas respiratorios, esputo maloliente, leucocitosis, crepitos en ápex pulmonar derecho, que junto a una radiografía que reporta infiltrado en el lóbulo superior derecho, nos debe hacer sospechar de un cuadro de neumonía adquirida en la comunidad. Además, esta paciente se sometió a una endoscopia gastrointestinal superior y posteriormente desarrolló neumonía, probablemente debido a aspiración bacteriana durante el procedimiento. Recordemos, la endoscopia superior aumenta el riesgo de aspiración, porque la sonda del endoscopio deteriora el cierre de la glotis y del esfínter esofágico inferior, lo que puede provocar neumonitis química a las pocas horas debido a aspiración de ácido gástrico o neumonía varios días después por la aspiración de microbios gástricos. Aunque durante mucho tiempo se consideró que los anaerobios (Peptostreptococcus, Fusobacterium) eran los organismos dominantes involucrados en la neumonía por aspiración bacteriana, según indica la evidencia más reciente, los patógenos aeróbicos comunes (bacilos gramnegativos, Streptococcus pneumoniae, Staphylococcus aureus) están presentes en la mayoría de las infecciones. Por lo tanto, la terapia antibiótica empírica ahora está diseñado para cubrir patógenos aeróbicos y anaeróbicos, de la siguiente manera:

● Ámbito ambulatorio: amoxicilina-clavulánico
● Hospitalizado, pero no grave: ampicilina-sulbactam
● Hospitalizado y en estado crítico: carbapenem o piperacilina-tazobactam

En cuanto a la ciprofloxacina, proporciona cobertura contra gramnegativos, pero no es efectiva para anaerobios o la mayoría de los organismos grampositivos. Por otra parte, la doxiciclina cubre organismos atípicos en pacientes con neumonía adquirida en la comunidad, pero no tiene buena actividad contra los anaerobios. Finalmente, el metronidazol proporciona cobertura contra anaerobios gramnegativos, pero tiene eficacia limitada contra muchos anaerobios grampositivos y no cubre adecuadamente los aerobios.

143
Q
  1. Masculino de 43 años, consultó por dolor torácico opresivo, no irradiado. Refiere que es la primera vez que le ocurre esto, pero comentó que ha presentado disnea y tos en el pasado. Antecedente de rinitis alérgica y dermatitis atópica. Se realiza EKG que evidenció depresión del segmento ST en las derivaciones laterales; no obstante, marcadores cardíacos no evidencian IAM. El paciente es tratado con aspirina, clopidogrel, heparina de bajo peso molecular, metoprolol y lisinopril. A la mañana siguiente, presenta disnea y tos seca, sin dolor torácico. Signos vitales Tº 37,1 Cº, PA de 122/70 mm Hg, FC de 63 lpm y FR de 22 rpm. Al examen físico, espiración prolongada con sibilancias bilaterales, sin otro hallazgo anormal. ¿Cuál es la causa de los síntomas respiratorios del paciente?
    A. Efecto adverso de la medicación
    B. Infección bronquial
    C. Pericarditis
    D. Embolia pulmonar
A

Respuesta correcta: A
Tema e ítem: Exacerbación de asma por medicamentos - diagnóstico
Argumento: Este paciente que ingresó inicialmente por dolor torácico (posiblemente angina inestable debido a las depresiones del segmento ST en el ECG) y que fue tratado adecuadamente (terapia antiplaquetaria, betabloqueantes, IECA y anticoagulante), que desarrolló posteriormente de forma aguda disnea y sibilancias espiratorias, indican una broncoconstricción. Dado los antecedentes de rinitis alérgica, dermatitis atópica, y su historial previo episodios intermitentes de disnea y tos, es probable que el paciente tenga síntomas no diagnosticados de asma, que ahora se ve exacerbada por la aspirina o por la terapia con beta bloqueadores. Recordemos que la aspirina es un desencadenante común de broncoconstricción en pacientes con asma. En cuanto a los beta-bloqueadores no selectivos (propranolol) actúan sobre los receptores β1 y β2 y a menudo desencadenan broncoconstricción en pacientes con asma subyacente. Por otro lado, los beta-bloqueadores cardioselectivos (metoprolol) actúan predominantemente sobre los receptores β1 y se consideran seguros de usar en pacientes con asma leve a moderada; sin embargo, todos los beta-bloqueadores pueden desencadenar broncoconstricción, especialmente cuando se administran en grandes dosis. Además, los IECA (lisinopril) pueden causar tos seca a través del aumento de los niveles circulantes de bradicinina y la consiguiente irritación bronquial.

En cuanto a la bronquitis aguda, generalmente es causada por una infección viral de las vías respiratorias superiores; y se caracteriza por tos persistente, a veces con producción de esputo. El momento de aparición de síntomas (1-2 días después de tomar nuevos medicamentos) y ausencia de otros síntomas, hacen poco probable este diagnóstico. Por otra parte, la pericarditis aguda se caracteriza por un dolor torácico pleurítico agudo que a menudo se alivia al inclinarse hacia adelante, junto a la presencia de un roce pericárdico. Finalmente, los pacientes con embolia pulmonar pueden tener dolor torácico, disnea y/o taquipnea. Las sibilancias y la espiración prolongada son posibles, pero no típicas.

144
Q
  1. Masculino de 44 años, consultó por astenia y adinamia de varios meses de evolución. Antecedente de consumo de alcohol diario hasta alcanzar la embriaguez y bajo consumo de carne. Sin antecedentes de importancia. Al examen físico, palidez conjuntival interna y pulsos carotídeos hiperdinámicos. Se realiza un hemoleucograma que reporta hemoglobina de 7,7 g/dL, VCM de 104 fl, HCM de 35 pg y HbA1c de 5.2%. Se inicia terapia con ácido fólico. Cinco semanas después, el nivel de hemoglobina del paciente es de 9,1 mg/dL, pero refiere parestesias en los dedos de los pies, sin otro hallazgo anormal. ¿Cuál es la causa más probable de los síntomas actuales de este paciente?
    A. Sindrome de abstinencia
    B. Deficiencia de hierro
    C. Deficiencia de cobalamina
    D. Neuropatía diabética
A

Respuesta correcta: C
Tema e ítem: Anemia por deficiencia de cobalamina - Diagnóstico
Argumento: Este paciente tiene una anemia macrocítica con HCM elevado, para la cual fue tratada empíricamente como una anemia por deficiencia de folato, en el contexto de consumo excesivo de alcohol. Ahora tiene síntomas neurológicos que empeoran, consistentes con la coexistencia de una deficiencia de cobalamina (vitamina B12) no tratada. Recordemos que la suplementación con cobalamina no se administra de forma rutinaria porque, en estos pacientes, la deficiencia de cobalamina es rara en ausencia de pancreatitis crónica; sin embargo, puede desarrollarse una deficiencia de cobalamina al tener una ingesta limitada de proteínas animales (carne) o con una edad avanzada, debido a una disminución del factor intrínseco (anemia perniciosa) y posterior malabsorción de vitamina B12. En etapas avanzadas, la deficiencia de cobalamina puede provocar neuropatía periférica (parestesias) o defectos de la columna posterior, debido a una síntesis defectuosa de mielina (el folato y la cobalamina son cofactores en la conversión de homocisteína a metionina). Además, debido a que la cobalamina también se usa en otros procesos, los déficits neurológicos pueden precipitarse o empeorar cuando se administra ácido fólico sin suplementos de cobalamina. Por lo tanto, es importante considerar la deficiencia de cobalamina antes de iniciar el reemplazo de ácido fólico.

En cuanto a sospechar una deficiencia de hierro, los síntomas de esta entidad son cefalea, disnea de esfuerzo, síndrome de piernas inquietas y pica. Además, la anemia inicial de este paciente es una anemia macrocítica, que no es el patrón de una anemia ferropénica (es normocítica o microcítica). Por otro lado, este paciente no presenta síntomas de un cuadro de abstinencia como la anhedonia, irritabilidad, alteración de la concentración, del patrón del sueño o algún nivel de agitación psicomotora. Finalmente, para considerar una neuropatía diabética, se debe plantear primero si el paciente tiene el diagnóstico previo de diabetes (este paciente no tiene diagnóstico previo). Además, si se sospechara de una diabetes no diagnosticada que cursara ya con una neuropatía diabética, se esperaría tener valores elevados de una hemoglobina glicada junto a síntomas propios de la diabetes.

145
Q
  1. Femenina de 35 años, consultó por 4 semanas de fatiga progresiva, junto a parestesias en miembros inferiores (MI). No refiere dolor de espalda, alteración de la marcha, ni problemas intestinales o síntomas vesicales. Antecedente de enfermedad de Crohn. No consume alcohol. Signos vitales con T° de 36,9 °C. Al examen físico, fuerza muscular de 5/5 en MI, los reflejos están intactos, la vibración y la propiocepción están preservadas. Paraclínicos con hemoglobina 8 g/dL, VCM de 102 fl, HCM 35 pg, leucocitos 3.000/mm3, creatinina sérica de 1 mg/dL, calcio 8,6 mg/dL. ¿Qué examen se debe solicitar a este paciente?
    A. Realizar resonancia magnética de la columna
    B. Solicitar una electroforesis de proteínas séricas
    C. Solicitar prueba de nivel de vitamina B en suero
    D. Pedir plumbemia en sangre
A

Respuesta correcta: C
Tema e ítem: Anemia- Diagnóstico
Argumento: Este paciente que presenta anemia, junto a un VCM y HCM elevado, la presencia de parestesias bilaterales de MI y el antecedente de un trastorno inflamatorio del intestino delgado, nos debe hacer sospechar de una deficiencia de vitamina B12, debido a una reducción de la absorción del complejo factor intrínseco-vitamina B12 en el íleon, por lo que se debe solicitar medición de vitamina B12 en suero. Recordemos que la vitamina B12 es esencial para la síntesis de ADN (a través de la reducción de 5-metiltetrahidrofolato a tetrahidrofolato) y formación de mielina (mediante metilación de lípidos y proteínas neuronales). La deficiencia afecta con mayor frecuencia a células hematológicas que se dividen rápidamente (anemia megaloblastica-leucopenia), posteriormente los pacientes clásicamente desarrollan desmielinización de la columna dorsal de la médula espinal (reducción sensación vibratoria, ataxia sensorial de la marcha) y los tractos corticoespinales laterales (paresia espástica, reflejos mejorados); sin embargo, los nervios mielinizados del sistema nervioso periférico suelen verse afectados primero, lo que lleva a parestesias en las extremidades inferiores sin otros hallazgos neurológicos. Se deben solicitar niveles de ácido metilmalónico y homocisteína para aquellos con resultados dudosos/no concluyentes.

En cuanto a solicitar una resonancia magnética de la columna, no se realizan de forma rutinaria, está reservada para una evaluación adicional después de la evaluación inicial con tomografía computarizada. Además, la clínica de este paciente no presenta hallazgos que sugieran compromiso de la médula espinal (síntomas de compresión). Por otra parte, la realización de una electroforesis de proteinas (EP) está indicada en todos los pacientes en los que se sospecha mieloma múltiple, macroglobulinemia de Waldenström, amiloidosis primaria. La EP también debe considerarse en cualquier paciente con una proteína sérica total elevada o signos y síntomas inexplicables que sugieran la presencia de un trastorno de las células plasmáticas (Anemia con dolor de espalda inexplicable, osteopenia o lesiones osteolíticas, insuficiencia renal con sedimento de orina blando, hipercalcemia, entre otras). Finalmente, este paciente no presenta factores de riesgo para sospechar de una intoxicación por plomo (consumo, balas de plomo, riesgo laboral, ambiente industrial). Además, la intoxicación por plomo cursa la mayoría de veces con una anemia microcítica normocrómica, y este paciente tiene una anemia macrocítica.

146
Q
  1. Femenina de 42 años, en seguimiento por cuadro de ascitis hace 3 semanas. Ha estado tomando lactulosa, espironolactona y furosemida con mejoría en sus síntomas. Antecedente de consumo de 12 cervezas diarias durante 15 años. Además, refiere haber recibido todas las vacunas y refuerzos de la infancia; y un refuerzo, de toxoides tetánico y diftérico tras una lesión traumática a los 30 años. A la exploración física, leve ictericia, eritema palmar, abdomen ligeramente distendido. Tiene negativo resultado de Hepatitis A IgM, Hepatitis A IgG, Antigeno de superficie de Hepatitis B, anticuerpo de superficie de Hepatitis B y Hepatitis C IgM. ¿Cuál de las siguientes vacunas se recomienda para esta paciente?
    A. Hepatitis A, hepatitis B e influenza
    B. Hepatitis A, hepatitis B , Tdap y PPSV23
    C. Hepatitis A, hepatitis B, influenza y Tdap
    D. Hepatitis A, hepatitis B, influenza, PPSV23 y Tdap
A

Respuesta correcta: D
Tema e ítem: Vacunación en el adulto con cirrosis

Argumento: Argumento: Esta paciente tiene características clínicas compatibles con cirrosis alcohólica, recordemos que los pacientes con hepatopatía crónica, requieren las mismas vacunas que la población general, además de que los pacientes con hepatopatía crónica que contraen hepatitis viral, tienen un mayor riesgo de sufrir enfermedades hepáticas descompensadas. Por este motivo, deberían recibir la vacuna contra la hepatitis A y B. Excepto si tienen pruebas documentadas de inmunidad (las serologías de esta paciente no indican inmunidad); por lo anterior, esta paciente debe recibir vacuna de Hepatitis A, hepatitis B, influenza, PPSV23 y Tdap.

En cuanto a la opción A, recordemos que la vacuna contra la influenza está indicada anualmente para todos los adultos. Es importante tener en cuenta que la vacuna intramuscular inactivada contra la influenza parece ser más efectiva que la vacuna viva.

Por otro lado, en cuanto a la opción B, se debe tener en cuenta que todos los adultos deben recibir la vacuna contra el tétanos cada 10 años (esta paciente la recibió hace más de 10 años), ya sea con la vacuna contra el tétanos y la difteria que es la vacuna toxoide (Td) o la vacuna reductora contra el tétanos, la difteria y la tos ferina acelular (Tdap). Para adultos que no han recibido Tdap, se les debe administrar al menos una dosis de Tdap.

Finalmente, la vacunación neumocócica con la vacuna antineumocócica de polisacáridos de 23 valencias (PPSV23) se recomienda para todos los adultos ≥65 años y para aquellos <65 años con ciertas comorbilidades que aumentan el riesgo de enfermedad neumocócica (cirrosis, DM2, Fumadores); por ende esta paciente debe recibir vacuna de PPSV23.

147
Q
  1. Masculino de 24 años consulta para evaluar resultados de pruebas de transmisión sexual luego de recibir tratamiento con ceftriaxona y azitromicina para una proctocolitis por transmisión sexual hace 3 días. Sus síntomas han mejorado y refiere no haber tenido ITS previas. Las pruebas de amplificación son positivas para clamidia y negativos para gonorrea y virus de herpes. La prueba de inmunoensayo enzimático para sífilis es positiva, con reagina plasmática rápida negativa y prueba de absorción de anticuerpos treponémicos fluorescente positiva. La serología de VIH es negativa. ¿Cuál es el manejo adecuado?
    A. Amoxicilina.
    B. Penicilina benzatina.
    C. Doxiciclina y penicilina benzatina.
    D. No es necesaria un tratamiento adicional.
A

Respuesta correcta: B

Tema e ítem: Manejo de la sífilis de duración desconocida.

Argumento: La penicilina benzatínica es el manejo adecuado para este paciente. Muchos laboratorios utilizan la estrategia de prueba serológica “inversa” para sífilis, iniciando con una prueba de inmunoensayo enzimático automatizado (EIA) seguido de una prueba no específica (reagina plasmática rápida [RPR] o prueba del Laboratorio de Investigación de Enfermedades Venéreas [VDRL]). Los pacientes con un resultado positivo de EIA pero un resultado negativo de la prueba RPR o VDRL deben someterse a una segunda prueba de anticuerpos treponémicos específicos para confirmar el resultado. A aquellos con un resultado positivo confirmado y sin antecedentes de tratamiento para la sífilis se les debe ofrecer manejo para sífilis de duración desconocida con penicilina benzatínica una vez por semana en tres dosis. En este paciente, las pruebas de amplificación de ácido nucleico confirmaron un diagnóstico de proctitis por clamidia, por lo que el tratamiento empírico proporcionado con ceftriaxona y azitromicina habría sido suficiente para tratar esa infección.

  • No se recomienda la amoxicilina oral para tratar la sífilis. En países donde no se disponía de penicilina benzatínica, los estudios observacionales han demostrado que el tratamiento de la sífilis es exitoso con amoxicilina oral en dosis altas y de larga duración combinada con probenecid. Este régimen oral prolongado es mucho menos deseable que tres inyecciones semanales de penicilina benzatínica.
  • Se agregaría un ciclo de doxiciclina a la penicilina benzatínica si este paciente no respondiera al tratamiento empírico proporcionado contra Clamydia trachomatis, pero no es este el caso.
148
Q
  1. Masculino de 64 años presenta dolor lumbar y dificultad para subir escaleras que ha empeorado en los últimos días. El dolor se irradia bilateralmente a las piernas. No ha orinado en las últimas 24 horas. Tiene antecedente de cáncer de próstata tratado con radioterapia externa. Signos vitales normales. Al examen físico los reflejos del tobillo y reflejos rotulianos están disminuidos bilateralmente. Hay debilidad a la flexión dorsal y plantar bilateralmente, al igual que debilidad en los principales grupos de músculos de piernas y muslos. ¿Cuál es el diagnostico más probable?
    A. Síndrome de cola de caballo.
    B. Síndrome piriforme.
    C. Fractura por insuficiencia pélvica inducida por radiación.
    D. Fractura por compresión vertebral.
A

Respuesta correcta: A

Tema e ítem: Radiculopatía Lumbar – Síndrome de cola de caballo.

Argumento: El cuadro clínico de este paciente consiste en un síndrome de cauda equina debido a cáncer de próstata metastásico. Está indicada la resonancia magnética urgente de la columna lumbosacra, junto con una evaluación neuroquirúrgica inmediata. El síndrome de cola de caballo se presenta clásicamente con dolor lumbar agudo que se irradia a las piernas. El hallazgo dominante es debilidad bilateral en las piernas, que puede estar asociada con anestesia en silla de montar, disfunción intestinal y vesical y disfunción eréctil. En el examen físico, se pueden observar reflejos distales disminuidos o ausentes y disminución del tono del esfínter anal junto con debilidad bilateral en las piernas. El síndrome de cauda equina es causado comúnmente por una hernia de disco, cambios degenerativos o espondilosis, pero también puede ser el resultado de un traumatismo, una infección o un cáncer (más comúnmente cáncer de próstata, linfoma o mieloma múltiple). La resonancia magnética de la columna lumbosacra puede visualizar el compromiso neurológico de la cola de caballo.

149
Q
  1. Masculino de 72 años se encuentra hospitalizado por insuficiencia cardíaca descompensada. El manejo inicial consistió en furosemida IV igual a la dosis oral ambulatoria (40 mg). La producción durante la noche fue de 250 ml, sin cambios en los síntomas. Tiene antecedente de hiperlipidemia e hipertensión arterial por lo que toma lisinopril, metoprolol succinato, furosemida y atorvastatina. Signos vitales: PA de 120/80 mmHg, FC de 80 lpm, FR de 26 rpm y SatO2 de 95% con oxigeno a 2L minuto. Se encuentra alerta y su piel es cálida. La presión venosa central esta aumentada y tiene edema en ambos miembros inferiores. Preclínicos: electrolitos normales, creatinina de 1.5mg/dl y péptido natriurético aumentado. ¿Cuál es el tratamiento adecuado?
    A. Añadir milrinona IV.
    B. Añadir nitroglicerina IV.
    C. Suspender el metoprolol.
    D. Aumentar la dosis de furosemida IV.
A

Respuesta correcta: D

Tema e ítem: Manejo de la insuficiencia cardiaca aguda.

Argumento: El tratamiento adecuado es aumentar la dosis de furosemida intravenosa. Este paciente presentó insuficiencia cardíaca aguda en el contexto de una sobrecarga de volumen y tiene varios signos de aumento del volumen intravascular- En este contexto, la dosis IV inicial de un diurético debe ser de 1 a 2,5 veces la dosis oral diaria total del paciente; si el paciente no responde a la dosis IV inicial, se debe aumentar entre un 50% y un 100%. También se puede considerar agregar un diurético tiazídico, pero estos agentes generalmente se asocian con una mayor incidencia de hiponatremia.
- Los inotrópicos, como la milrinona, no mejoran los resultados en pacientes con insuficiencia cardíaca aguda descompensada ni aumentan la producción de orina. Se pueden considerar los inotrópicos en pacientes con insuficiencia cardíaca de bajo gasto, pero no deben usarse en el tratamiento de rutina de pacientes ingresados con insuficiencia cardíaca.

  • Ocasionalmente se agrega terapia vasodilatadora intravenosa, como nitroglicerina, en pacientes hospitalizados con insuficiencia cardíaca aguda y podría considerarse para este paciente. Con la reducción de la resistencia vascular sistémica, a menudo se puede mejorar el volumen sistólico, con la consiguiente mejoría del paciente. Sin embargo, la terapia con diuréticos es el tratamiento principal para pacientes con insuficiencia cardíaca descompensada y sobrecarga de líquidos.
  • En pacientes hospitalizados con insuficiencia cardiaca cardíaca con un perfil húmedo y cálido que reciben terapia con bloqueadores beta al momento del ingreso deben continuar el tratamiento a menos que se presenten con hipotensión. Si la insuficiencia cardíaca sigue siendo refractaria a los diuréticos, la dosis debe reducirse a la mitad. La interrupción del metoprolol podría ser apropiada si este paciente es resistente a dosis altas de diuréticos intravenosos; De todos modos, el mejor paso inicial para este paciente sería aumentar la dosis de furosemida intravenosa.
150
Q
  1. Femenina de 49 años consulta por cefalea intensa y repentina desde hace 24 horas de duración. La cefalea alcanza su máxima intensidad en 1 minuto. Por lo demás se encuentra bien y no toma medicamentos. Sus signos vitales son normales. Al examen físico hay rigidez nucal. El estado mental y examen fundoscopico es normal. No hay déficits de pares craneales. La TAC de cráneo es normal. ¿Cuál es el manejo adecuado para este paciente?
    A. Angiografía por catéter cerebral.
    B. Realizar una punción lumbar.
    C. Realizar una venografia por resonancia magnética.
    D. Iniciar sumatriptán oral.
A

Respuesta correcta: B

Tema e ítem: Hemorragia subaracnoidea diagnóstico.

Argumento: El siguiente paso más apropiado en el abordaje de este paciente es la punción lumbar. El paciente es atendido después de la aparición repentina de una cefalea intenso (cefalea en trueno). La cefalea en trueno se define como un ataque severo de dolor de cabeza que se desarrolla abruptamente y alcanza la intensidad máxima en 1 minuto. Aunque en ocasiones es de origen primario, la cefalea en trueno es una emergencia médica que justifica una evaluación diagnóstica inmediata. La cefalea en trueno tiene un diagnóstico diferencial amplio; la más grave es la hemorragia subaracnoidea por aneurisma (HSA). Una pequeña proporción (menos del 5%) de los pacientes con HSA aneurismática tendrán una tomografía computarizada de la cabeza que resulta negativa para sangrado y requerirán una punción lumbar para descartar esta afección. En pacientes con HSA, los resultados de la punción lumbar realizada entre 12 y 24 horas después de la aparición de los síntomas revelarán recuentos elevados de eritrocitos y xantocromía (color amarillo causado por la degradación de los glóbulos rojos). Si la punción lumbar se realiza dentro de las 12 horas posteriores al inicio de los síntomas, puede resultar difícil distinguir un proceso traumático de la presencia aguda de eritrocitos secundarios a una hemorragia aneurismática porque es posible que aún no se haya desarrollado la xantocromía.

151
Q
  1. Masculino de 72 años presenta celulitis en extremidad inferior derecha. Niega antecedentes de importancia u otro problema médico. Signos vitales: Temp. de 38°C, PA de 150/80 mmHg, FC de 110 lpm y FR de 22 rpm. Hay eritema extenso, calor, edema y dolor que se extiende desde la pantorrilla hasta el maleolo medial y cara dorsomedial de piel derecho. La ecografía doppler del miembro inferior derecho no muestra evidencia de TVP ni acumulación de líquido. Se inicia cefazolina intravenosa. ¿Cuál de los siguientes iniciaria para prevenir tromboembolismo venoso?
    A. Iniciar un anticoagulante oral directo como el rivaroxaban.
    B. Realizar compresión neumatica intermitente.
    C. Iniciar un anticoagulante oral directo + heparina de bajo peso molecular.
    D. Iniciar heparina de bajo peso molecular.
A

Respuesta correcta: D

Tema e ítem: Prevención de la trombosis venosa profunda.

Argumento: El tratamiento más adecuado para prevenir la tromboembolia venosa (TEV) en este paciente hospitalizado es la heparina de bajo peso molecular (HBPM). Las tasas de TEV (tanto embolia pulmonar como trombosis venosa profunda) son elevadas en pacientes con enfermedades agudas. Los pacientes con enfermedades agudas son pacientes hospitalizados por una enfermedad médica. En pacientes con enfermedades agudas se sugiere la anticoagulación parenteral con HBPM, heparina no fraccionada o fondaparinux para prevenir el TEV.

  • Se prefieren los anticoagulantes orales directos a las HBPM para la profilaxis posoperatoria de TEV en artroplastia total de rodilla y cadera, no en pacientes hospitalizados sin estos antecedentes.
  • En pacientes con enfermedades agudas o críticas, sugiere el uso de profilaxis farmacológica de TEV en lugar de profilaxis mecánica como la compresión neumática intermitente (CNI) o el uso de medias. La CNI está indicada para pacientes con mayor riesgo de TEV y alto riesgo de hemorragia grave o que experimentan hemorragia mientras reciben anticoagulantes en dosis profilácticas. También es muy poco probable que este paciente con celulitis extensa tolere la incomodidad de la IPC o las medias degradadas.
152
Q
  1. Mujer de 51 años asiste a consulta rutinaria. Tiene deposiciones normales, sin hematoquecia, melena o pérdida de peso inesperada. Tiene HTA. Su madre desarrolló cáncer de colon a los 85 años. No consume tabaco ni alcohol. El examen físico es normal. Le realizan una colonoscopia en la cual observan y extirpan 2 pólipos hiperplásicos pequeños (8 mm). ¿Cuál es la conducta más adecuada?
    A. Realizar una prueba de sangre oculta en heces en 6 meses.
    B. Realizar una colonoscopia en 1 año.
    C. Realizar una colonoscopia en 10 años.
    D. Realizar una TAC de abdomen.
A

Respuesta correcta: C

Tema e ítem: Tamizaje de Cáncer de Colon - Manejo

Argumento: Esta paciente tiene pólipos hiperplásicos, una anomalía no neoplásica que suele detectarse en la colonoscopia de cribado. Los pólipos hiperplásicos son más frecuentes en el colon rectosigmoide. Los pólipos hiperplásicos pequeños (<1 cm) no aumentan el riesgo de cáncer de colon, por lo que los pacientes pueden continuar con el cribado colorrectal a los intervalos habituales: Repetir la colonoscopia a los 10 años para los individuos con un riesgo por lo demás medio; Repetir la colonoscopia en 5 años para las personas con antecedentes familiares de alto riesgo (familiar de primer grado con cáncer colorrectal a una edad <60 años), la madre de este paciente tenía más de 60 años en el momento del diagnóstico, lo que no aumenta apreciablemente el riesgo del paciente.

  • Un cribado más frecuente (<10 años) está indicado para pacientes con pólipos adenomatosos, y el intervalo específico depende del tamaño y el número. Los pólipos hiperplásicos no modifican la frecuencia normal de cribado.
  • La TAC abdominal es una medida de estadificación adecuada para los pacientes con un nuevo diagnóstico de cáncer colorrectal. No es necesaria en la evaluación de pólipos neoplásicos (adenoma) o no neoplásicos (hiperplásicos).
  • El análisis periódico de sangre oculta en heces es una modalidad de cribado adecuada para las personas con un riesgo medio de cáncer colorrectal. Normalmente se realiza a intervalos de 1 año (no de 6 meses). Este paciente acaba de someterse a una colonoscopia con resultados de bajo riesgo; no serán necesarias pruebas de cribado adicionales durante varios años.
153
Q
  1. Hombre, 22 años, sufre accidente de tránsito de alta energía. Iba sin cinturón. Tiene dolor torácico anterior y pleurítico. Examen físico: FC 110/min, los demás signos son normales. Está alerta, con presión venosa yugular normal y hematomas en tórax anterior. El resto del examen físico es normal. Rx de tórax, TAC de columna cervical y eFAST normales. El dolor mejora con analgésicos. ¿Cuál es la mejor conducta?
    A. Realizar un electrocardiograma.
    B. Dar de alta con analgésicos.
    C. Hospitalizar y realizar monitoreo seriado de enzimas cardiacas.
    D. Realizar un ecocardiograma transesofágico.
A

Respuesta correcta: A

Tema e ítem: Trauma precordial - Manejo

Argumento: Este paciente implicado en una colisión automovilística de alto riesgo presenta dolor en la pared torácica anterior, lo que sugiere un trauma torácico cerrado. Está estable, su rx de tórax y el e-FAST son normales. Se debe realizar un EKG para detectar una lesión cardiaca contusa (LCC). En el trauma torácico cerrado, el estado hemodinámico determina la estrategia de tratamiento. Los pacientes inestables se someten simultáneamente a reanimación y evaluación (rx de tórax, eFAST, EKG) para buscar lesiones potencialmente mortales. Se realizan intervenciones estabilizadoras para cualquier anomalía identificada (tubo torácico, etc). En los pacientes estabilizados, se realizan pruebas adicionales (TAC de tórax) para evaluar otras lesiones no observadas en la evaluación inicial (lesión aórtica). Los pacientes que permanecen inestables se llevan directamente a toracotomía. En los pacientes estables, el alcance de la evaluación la determina el mecanismo de la lesión y el examen físico inicial.
Los pacientes sin mecanismo de alto riesgo y sin lesiones graves en la exploración se someten a EKG y rx de tórax. Si hay hallazgos anormales se realizan intervenciones y se obtienen pruebas adicionales (TAC torácica), los pacientes sin anomalías pueden ser dados de alta de forma segura con analgesia y signos de alarma. Los pacientes estables pero con un mecanismo de alto riesgo (alta energía, sin cinturón de seguridad) o con lesiones graves deben ser tratados de forma similar a los pacientes inestables. Por lo tanto, además del eFAST y la radiografía de tórax, a este paciente se le debe realizar un EKG. El EKG puede ayudar a evaluar la presencia de LCC, que se produce debido a la desaceleración rápida y puede causar taponamiento, rotura de la pared o ser asintomático. Puede observarse taquicardia sinusal, arritmia o cambios en el ST. Un ECG anormal requiere una observación por aprox. 24h (riesgo de arritmias mortales), pruebas de enzimas cardiacas y ecocardiografía (descartar taponamiento).

  • La LCC causa taquicardia sinusal; sin embargo, los betabloqueantes no se administran de forma rutinaria porque la LCC también suele causar edema miocárdico con disminución de la contractilidad, llevando a hipotensión. La taquicardia sinusal suele ser una respuesta fisiológica que debe tratarse abordando la causa subyacente en lugar de con un bloqueo beta.
  • ecocardiografía transesofágica es útil para evaluar la lesión aórtica torácica en pacientes inestables sin tener que trasladarlos a TAC.
154
Q
  1. Hombre, 37 años. Tiene dolor abdominal agudo y emesis. Ha hecho 2 deposiciones desde que empezó el dolor. Es alcohólico. Lleva 5 días hospitalizado por endocarditis, con hemocultivos (+) para S. aureus y evidencia de vegetaciones en la válvula mitral, recibe vancomicina. Examen físico: T 37,5°C, PA 150/90 mmHg, FC 110/min. Tiene un soplo holosistólico 3/6 en el ápex. Con dolor leve difuso a la palpación abdominal, peristaltismo disminuido, sin irritación peritoneal. ¿Cuál es la causa del cuadro?
    A. Absceso intraabdominal.
    B. Síndrome de abstinencia alcohólica.
    C. Pancreatitis aguda.
    D. Isquemia mesentérica.
A

Respuesta correcta: D

Tema e ítem: Isquemia Mesentérica - Diagnóstico

Argumento: Este paciente con endocarditis infecciosa (EI) presenta dolor abdominal agudo e intenso y mínimos hallazgos en la exploración abdominal, lo que hace sospechar una isquemia mesentérica aguda (IMA). En la mayoría de los casos, la IMA se debe a una oclusión arterial por eventos embólicos cardíacos (fibrilación auricular, vegetación debida a EI, aneurisma cardiovascular) o trombosis aguda (enfermedad arterial periférica). En pacientes con EI, el mesenterio es el segundo lugar más frecuente de embolización sistémica (tras la neurovasculatura). La IMA se presenta típicamente con dolor abdominal medio de inicio súbito, intenso y mal localizado (visceral), acompañado de náuseas y vómitos. Al principio la exploración física suele ser anodina a pesar de la presencia de dolor intenso; este hallazgo se conoce clásicamente como “dolor desproporcionado con respecto a los hallazgos”. La isquemia local puede provocar deseo defecatorio, como se observa en esta paciente.

Si se produce un infarto intestinal, puede aparecer una sensibilidad abdominal más focal (debido a la inflamación/infarto local), signos peritoneales, hemorragia rectal y sepsis. La mayoría de los pacientes presentan leucocitosis, hemoconcentración, amilasa elevada y acidosis metabólica (láctica).

  • La pancreatitis aguda suele cursar con náuseas, vómitos y dolor epigástrico irradiado a la espalda. Aunque la ingesta importante de alcohol desencadena con frecuencia la pancreatitis, la mayoría de los pacientes presentan una intensa sensibilidad mediopigástrica en la exploración. Además, la presencia de EI hace más probable la embolización séptica con IMA.
  • El síndrome de abstinencia alcohólica suele cursar con inquietud, diaforesis y taquicardia. En casos de moderados a graves, también puede causar convulsiones, alucinaciones y alteraciones del estado mental. El dolor abdominal agudo e intenso es menos frecuente. Además, la mayoría de los casos aparecen a las 24 horas del cese (no 5 días después).
  • Los abscesos intraabdominales suelen presentarse con fiebre subaguda, sensibilidad abdominal focal y pérdida de peso.
155
Q
  1. Mujer, 55 años. Tiene dolor epigástrico agudo irradiado a espalda, asociado a emesis. Examen físico: T 37,1°C, PA 117/76 mmHg, FC 102/min, FR 16/min. Tiene dolor a la palpación profunda en epigastrio. Paraclínicos: Albúmina 4.2 g/dL, FA 148 U/L, AST 111 U/L, ALT 160 U/L, Amilasa 940 U/L, Lipasa 2155 U/L, Hb 12,8 g/dL, Plaq 220.000/mm3, GB 13.200/mm3. Eco: colelitiasis sin colecistitis. Recibe analgesia, LEV y 2 días después recupera la vía oral. Sus enzimas disminuyen. ¿Cuál es la conducta a seguir?
    A. Programar para CPRE.
    B. programar para colecistectomía.
    C. Realizar una nueva ecografía en 4 semanas.
    D. Realizar una gammagrafía con ácido iminodiacético hepatobiliar (HIDA).
A

Respuesta correcta: B

Tema e ítem: Pancreatitis aguda - Manejo

Argumento: El dolor epigástrico, las náuseas, los vómitos y la marcada elevación de enzimas pancreáticas sugieren una pancreatitis aguda. Como esta paciente no bebe alcohol y tiene elevadas las enzimas de la función hepática con colelitiasis, su pancreatitis es muy probablemente debida a cálculos biliares. Algunos estudios han demostrado que ALT >150 U/L tiene un valor predictivo positivo del 95% para diagnosticar pancreatitis por cálculos biliares. Una vez que los síntomas y los valores de laboratorio de esta paciente han mejorado, su pancreatitis aguda se considera leve y resuelto. Sin embargo, corre un mayor riesgo de sufrir recurrencias debido a los cálculos biliares. Se recomienda la colecistectomía en pacientes médicamente estables luego de una pancreatitis aguda y son candidatos quirúrgicos. La colecistectomía puede reducir notablemente el riesgo de pancreatitis recurrente por cálculos biliares.

  • La colangiopancreatografía retrógrada endoscópica (CPRE) se recomienda en pacientes con pancreatitis biliar que presentan colangitis, dilatación/obstrucción visible del conducto biliar común o niveles elevados de enzimas hepáticas.
  • La gammagrafía con ácido iminodiacético hepatobiliar (HIDA) utiliza un trazador nuclear que se excreta en la bilis. La no visualización del trazador en la vesícula biliar sugiere obstrucción. La HIDA, aunque poco realizada, puede utilizarse para evaluar la colecistitis en pacientes con hallazgos ecográficos indeterminados. Sin embargo, la ecografía de este paciente no mostró hallazgos de obstrucción biliar o engrosamiento de la pared de la vesícula biliar.
  • Se puede repetir la ecografía en 4 semanas en pacientes con síntomas de cólico biliar que no tienen cálculos biliares en la ecografía inicial. Sin embargo, este paciente tiene cálculos biliares complicados por pancreatitis aguda y requiere tratamiento en este momento.
156
Q
  1. Hombre, 24 años, consulta luego de que el todoterreno que conducía volcara y cayera sobre él. Examen físico: PA 82/50 mmHg, FC 128/min y FR 24/min. Abdomen distendido, con dolor difuso. Rx de tórax: fractura de 8°-9° costillas der, 4° izq, sin neumotórax. FAST: gran cantidad de líquido libre intraperitoneal. Sigue hipotenso tras reanimación y es llevado a laparotomía. ¿Cuál es la causa más probable del cuadro?
    A. Lesión de aorta abdominal.
    B. Laceración hepática.
    C. Hematoma esplénico.
    D. Hematoma duodenal.
A

Respuesta correcta: B

Tema e ítem: Laceración de hígado (LH) - Diagnóstico

Argumento: Este paciente con traumatismo abdominal cerrado (TCA) presenta signos de shock hipovolémico debido a hemorragia intraabdominal. Dadas las fracturas concomitantes de las costillas inferiores derechas de la paciente, que indican un probable traumatismo del cuadrante superior derecho (CSD), esta presentación es más consistente con una laceración hepática.
El hígado es uno de los órganos más frecuentemente lesionados en el TCA, con un espectro de lesiones que va desde el hematoma subcapsular a la disrupción del parénquima (laceración) que causa hemorragia en el peritoneo. Los traumatismos en el CSD, tórax inferior derecha o el flanco, deben hacer sospechar una lesión hepática.
El triaje de los pacientes con BAT sin peritonitis franca (p. ej., rigidez abdominal, sensibilidad de rebote) incluye la evaluación centrada en la cama con ecografía para traumatismos (FAST) para evaluar la presencia de líquido libre intraperitoneal, un marcador de lesión de órganos intraabdominales. Como en este caso, los pacientes hemodinámicamente inestables (p. ej., presión arterial sistólica <90 mm Hg) con un FAST positivo deben someterse a una laparotomía exploratoria.

  • Las lesiones aórticas abdominales por TCA son raras (<1%). Muchos pacientes con lesión aórtica traumática mueren antes de llegar al hospital. Los que sobreviven (porque la hemorragia se ha contenido en el retroperitoneo) no suelen tener una gran cantidad de líquido libre intraperitoneal en el FAST.
  • El TCA puede causar hematoma duodenal y esplénico por hemorragia intraluminal y subcapsular, respectivamente. Sin embargo, dado que la hemorragia está contenida dentro de la luz o la cápsula, la formación de hematomas suele taponar la hemorragia posterior, lo que hace menos frecuente un grado de shock tan grave. Además, ninguno de los dos tipos de hematoma causaría líquido libre intraperitoneal.
157
Q
  1. Masculino de 30 años, consultó por dolor hemiescrotal derecho progresivo, de 24 horas de evolución, que inicia posterior a levantamiento de pesas. El dolor mejoró con medidas locales e ibuprofeno; sin embargo, actualmente se acompaña de náuseas, emesis y dificultad para caminar. Antecedente de hernia umbilical a los 2 años con posterior herniorrafia. Signos vitales: T° de 37,7 °C y PA de 142/90 mm Hg. Al examen físico, edema, induración, eritema y reflejo cremastérico negativo del hemiescroto derecho. La ecografía informa que el testículo derecho es heterogéneo en textura, está aumentado de tamaño y tiene un pequeño hidrocele. ¿Cuál es la causa más probable de este cuadro clínico?
    A. Hernia inguinal
    B. Varicocele
    C. Neoplasia testicular
    D. Torsión testicular
A

Respuesta correcta: D

Tema e ítem: Torsión testicular-diagnóstico

Argumento: Este paciente que presentó un dolor hemiescrotal derecho de forma aguda, que se acompaña de edema y ausencia del reflejo cremasterico, junto a un reporte ecográfico que refiere aumento de tamaño, heterogeneidad en la textura y la presencia de un pequeño hidrocele, nos debe hacer pensar en una torsión testicular. Recordemos que esta condición, ocurre debido a una fijación insuficiente del testículo a la túnica vaginal, que puede provocar hipermovilidad testicular, torsión del cordón espermático, isquemia y necrosis. Aunque la torsión testicular ocurre comúnmente en adolescentes (debido al rápido crecimiento testicular durante la pubertad), puede presentarse en cualquier edad. Los pacientes suelen desarrollar un dolor escrotal agudo posterior a la actividad física o traumatismos testiculares leves. Además, las náuseas y los vómitos asociados son comunes. Dentro de los hallazgos al examen físico, el reflejo cremastérico ausente junto al cuadro clínico de este paciente, es suficiente para el diagnóstico clínico de esta entidad; no obstante, la ecografía Doppler del escroto se puede utilizar para confirmación o en caso de duda, realizar el descarte de las opciones diagnósticas (La ecografía puede demostrar torsión del cordón umbilical o flujo sanguíneo reducido, también puede haber hidrocele reactivo, así como una heterogeneidad en la textura esofágica, la cual es un hallazgo tardío que indica necrosis testicular, que se desarrolla después de >12 horas de isquemia).

En cuanto a la hernia inguinal, este puede causar una masa escrotal y presentarse posterior a situaciones que generen un aumento de la presión intraabdominal; sin embargo, el dolor generalmente se localiza en la ingle o el abdomen, en lugar del escroto, a su vez los testículos permanecen de tamaño normal y la ecografía demuestra asas intestinales dentro del escroto. Por otra parte, el varicocele se caracteriza por la dilatación del plexo pampiniforme dentro del cordón espermático. Los hallazgos al examen físico, se describen como una “bolsa de gusanos” en el escroto. Además, esta entidad suele ser asintomática, pero puede causar un dolor sordo que empeora con la bipedestación. Finalmente, las neoplasias testiculares generalmente se presentan de forma subaguda como una enfermedad indolora, con presencia de una masa testicular de crecimiento lento

158
Q
  1. Masculino de 43 años, consultó por 24 horas de dolor progresivo en abdomen, escroto y región perineal, junto a malestar general y náuseas. Antecedente de obesidad grado 2, DM2 e hipercolesterolemia, sin cirugías o traumas previos. Signos vitales Tº de 39,7 °C, PA de 80/60 mm/Hg y FC de 112 lpm. Al examen físico, la piel sobre la parte inferior del abdomen, el escroto y el perineo está tensa, dolorosa a la palpación, con eritema, edema y crepitación del tejido. Paraclínicos que reportan leucocitos 27.800/mm3, bandas 20%, creatinina en 1,7 mg/dl y glucosa en 280 mg/dL. Se obtienen hemocultivos e inician líquidos intravenosos. ¿Cuál es el manejo a seguir con este paciente?
    A. Solicitar una tomografía computarizada de abdomen y periné
    B. Realizar cirugía de urgencia
    C. Administrar un bolo de corticosteroides intravenosos
    D. Admisión a unidad de cuidados intensivos
A

Respuesta correcta: B

Tema e ítem: Gangrena de Fournier (fascitis necrotizante)- Manejo

Argumento: Este paciente que se presenta con síntomas de infección en tejidos blandos de región abdominal, genital y perianal (dolor, eritema, edema y crepitación), junto a síntomas sistémicos como hipotensión, fiebre, leucocitosis y falla renal aguda, probablemente indique una Gangrena de Fournier (GF); y por ende, va a requerir aparte del manejo inicial con antibióticos de amplio espectro y líquidos intravenoso, una exploración quirúrgica y desbridamiento urgentes para retirar el tejido necrosado. Recordemos que la GF es una fascitis necrotizante potencialmente mortal, que rápidamente progresa hasta la sepsis y la muerte si no se realiza una intervención oportuna. La mayoría de los casos de GF surgen en el contexto de laceraciones cutáneas en la región perianal o región genital, lo que permite una puerta de entrada para organismos polimicrobianos del colon o de la zona urogenital. Posteriormente, se propaga la infección a lo largo de la grasa subcutánea a través de planos fasciales, generando microtrombos de vasos cutáneos (produce gangrena). Además, condiciones como la DM2 mal controlada y la obesidad son factores de riesgo para la GF.

En cuanto a realizar una TAC, no se indica por qué va a retrasar la intervención quirúrgica. Además, las imágenes, generalmente no se recomiendan cuando la presentación clínica es altamente sospechosa de GF. Por otro lado, la administración de glucocorticoides se indica para pacientes con shock séptico refractario; que son pacientes que tienen PA sistolica <90 mm/Hg durante >1 hora a pesar de líquidos intravenosos y agentes vasopresores. Este paciente con una GF requiere cirugía y no cumple con los criterios de shock séptico refractario; por ende, los glucocorticoides no estarían indicados por ahora. Finalmente, este paciente probablemente recibiría atención en la unidad de cuidados intensivos después de la cirugía; pero primero requiere una intervención quirúrgica para eliminar el tejido necrótico.

159
Q
  1. Masculino de 70 años, consultó por cuadro de 3 días de tos productiva con esputo amarillo y disnea. En este último año estuvo hospitalizado 2 veces por cuadros similares. Antecedente de EPOC en manejo con budesonida, formoterol y bromuro de tiotropio inhalados diariamente. Signos vitales Tº de 37.6° C, FC de 76 lpm, FR de 26 rpm con SO2 de 86% al ambiente y PA de 142/80 mmHg. Al examen físico, cianosis perioral, sibilancias difusas a la auscultación pulmonar, sin otro hallazgo anormal. Por lo anterior, a este paciente se le inició manejo con salbutamol nebulizado. ¿Cuál es el paso a seguir en el manejo de este paciente?
    A. Iniciar ventilación mecánica invasiva.
    B. Administrar terapia antibiótica empírica.
    C. Administrar terapia con corticoides inhalados.
    D. Solicitar una espirometría.
A

Respuesta correcta: B

Tema e ítem: Exacerbación de la EPOC – Manejo

Argumento: Este paciente con antecedente de EPOC grado E por la GOLD 2023 (por # de hospitalizaciones previas), se presenta ahora con empeoramiento de la disnea, aumento de la frecuencia y severidad de la tos, junto a purulencia del esputo, lo que nos indican que el paciente está presentando una nueva exacerbación aguda de EPOC, por lo que requiere posterior al manejo con salbutamol nebulizado, una terapia antibiótica empírica durante 4 a 7 días, para mejorar la resolución de síntomas. Recordemos que la terapia antibiótica empírica está indicada si el paciente tiene 3 o más síntomas cardinales (2 síntomas son suficientes si uno de los síntomas es un aumento de la purulencia del esputo) o si el paciente requiere ventilación mecánica. Los antibióticos utilizados para el manejo de la neumonía adquirida en la comunidad, también se utilizan en la exacerbación de EPOC.

En cuanto al soporte ventilatorio en exacerbación aguda de EPOC, esta indicado si el paciente presenta acidosis respiratoria, hipoxemia persistente a pesar de oxígeno suplementario o disnea severa con signos de aumento de trabajo respiratorio y/o fatiga de los músculos respiratorios. Si bien este paciente tiene hipoxemia y dificultad para respirar, la ventilación con presión positiva no invasiva (VPPNI) es la estrategia de primera línea recomendada, por lo que la ventilación mecánica invasiva está indicada solo si falla la VPPNI, si esta se encuentra contraindicada o el paciente presenta inestabilidad hemodinámica. Por otra parte, falta evidencia empírica para apoyar el uso rutinario de corticosteroides inhalados en el tratamiento de la exacerbación aguda de EPOC. En cambio, la terapia con corticosteroides orales o intravenosos a corto plazo está indicada debido a sus beneficios clínicos comprobados en la disminución de la duración y la gravedad de la exacerbación. Finalmente, la espirometría es una parte integral del diagnóstico de la EPOC estable, no es una práctica para realizar en entornos de emergencia, y tiene un impacto menor en el tratamiento agudo de las exacerbaciones en comparación con otros parámetros diagnósticos y clínicos. Además, recordemos que unos de los principios de contraindicación relativa para realizar una espirometría, es sospecha de un paciente con síntomas de infección respiratoria activa.

160
Q
  1. Masculino de 30 años, consultó por mordedura de serpiente con patrón “RANA” y de ojo pequeño en la pierna derecha hace 1 hora. El paciente refiere parestesias y dolor urente en sitio de la mordedura junto a náuseas. Signos vitales T° 37°C, FC de 90 lpm, FR de 25 rpm y PA de 120/80 mmHg. Al examen físico, se evidencian dos heridas punzantes eritematosas en miembro inferior derecho, acompañadas de edema. No hay sangrado ni petequias. Además, el paciente presenta debilidad muscular generalizada, ptosis palpebral y salivación excesiva. ¿Cuál es el manejo apropiado inmediato para este paciente?
    A. Aplicar un torniquete en la región superior de la pierna
    B. Administrar 5 frascos de suero antiofídico polivalente
    C. Administrar 5 a 10 frascos de antiveneno anticoral polivalente
    D. No es necesario el uso de suero antiofídico
A

Respuesta correcta: C

Tema e ítem: Accidente ofídico por coral-Manejo

Argumento: Este paciente presenta una mordedura por serpiente de una coral, debido a que nos describen hallazgos de la “verdadera coral” como lo es el patrón “RANA” (Rojo, Amarillo/blanco, Negro, Amarillo-blanco), así como una serpiente de ojo pequeño. Además, la clínica del paciente, que presenta principalmente síntomas neurotóxicos, refuerza la idea de que se trata de una mordedura por una serpiente coral; y por ende, el paciente va a requerir 5 a 10 frascos de antiveneno anticoral polivalente (disponible en Colombia en el instituto nacional de salud) debido a que estas mordeduras siempre se clasifican graves sin importar la clínica del paciente. Recordemos que el veneno de esta familia de las corales (Micrurus mipartitus) es principalmente neurotóxico, tanto presináptico como postsináptico, debido a la presencia de neurotoxinas bloqueadoras de canales iónicos, de receptores y enzimas con actividad fosfolipasa “A” sobre sistema nervioso periférico y la unión neuromuscular; por ende, el daño tisular local significativo y la coagulopatía no son típicos del envenenamiento por serpientes de coral, y si lo son los síntomas neurotóxicos.

En cuanto a los torniquetes y vendajes constrictivos, no se recomiendan después de una mordedura de serpiente. Aunque la constricción teóricamente puede retrasar la propagación del veneno a través de los canales linfáticos, no hay evidencia de que esta intervención proporcione algún beneficio clínico. Además, los torniquetes pueden empeorar la isquemia local y el daño tisular. Por otra parte, el suero antiofídico polivalente, cuenta con inmunoglobulinas efectivas contra envenenamiento por serpientes del género Bothrops (Mapaná) y Crotalus (Cascabel). Las serpientes de los géneros previamente mencionados, no cursan con la característica del patrón RANA de este caso (y morfológicamente son diferentes a una coral en aspectos como color, presencia de cascabel, forma de la cabeza, entre otros hallazgos), y en el caso del Bothrops no genera neurotoxicidad, más bien cursa con cuadros hemorrágicos e hipotensivos. Finalmente, la mordedura de la falsa coral, debido a no ser venenosa, no requiere suero antiofídico; sin embargo, este paciente fue mordido por una coral verdadera (Las diferencias es que la coral verdadera cuenta con el patrón de color “RANA”, tiene ojo pequeño, hocico negro, no cuenta con escama Loreal y tiene una cola proporcionalmente más corta).

161
Q
  1. Masculino de 65 años, consultó por dolor urente y tumefacción en miembro inferior derecho (MID) desde hace 24 horas. Refiere que hace 48 horas sufrió una mordedura de su gato en el miembro afectado. Antecedente de DM2, y no comenta ninguna alergia. Signos vitales con Tº de 38.8°C, FC 145 lpm, PA 90/51 mm/Hg y FR 25 rpm. Al examen físico, en MID a nivel de la pierna, hay una herida cortante de 2 cm, con la piel perilesional eritematosa, inflamada, caliente al tacto y con exudado purulento. ¿Cuál es el tratamiento adecuado de este paciente?
    A. Administrar ceftriaxona
    B. Administrar clindamicina
    C. Administrar ampicilina/sulbactam
    D. Administrar metronidazol
A

Respuesta correcta: C

Tema e ítem: Mordedura animal- Tratamiento

Argumento: Las infecciones de piel y tejidos blandos tras la mordedura de un animal están causadas por patógenos que pertenecen a la microbiota oral del animal y la de la piel del paciente; por ende, la mayoría de las infecciones son polimicrobianas y los antibióticos de elección son las penicilinas de espectro extendido asociadas a un inhibidor de β-lactamasa como ampicilina/sulbactam, amoxicilina/ácido Clavulaico o piperacilina/tazobactam. En cuanto a la ceftriaxona, no tiene indicación, debido a que esta se utiliza como alternativa para los pacientes con antecedentes de reacción anafiláctica a la penicilina (este paciente no refiere tal antecedente). Por otra parte, el metronidazol y la clindamicina son efectivos para cubrir infecciones por anaerobios; no obstante, las infecciones por mordeduras de animales pueden ser polimicrobianas y debe considerarse una cobertura adicional para las infecciones aerobias.

162
Q
  1. Doña Diana de 72 años. Consulta por historial de 5 meses de evolución consistente en dolor en hemiabdomen superior: intermitente, exacerbado durante las comidas y asociado en ocasiones a distensión abdominal y deposiciones acuosas de bajo gasto. Además, comenta que ha perdido 10 kg de peso. Refiere haber consultado a un médico general el cual le prescribió omeprazol 20 mg/día durante ayunas por 4 semanas sin mejoría alguna. Antecedentes: HTA, dislipidemia mixta, tabaquismo desde hace 30 años y fue sometido a una intervención coronaria percutánea por SCA con ST de cara inferior hace 5 años. Actualmente se encuentra en manejo con ASA, enalapril, bisoprolol, empaglifozina y rosuvastatina. Examen físico: PA 140/90 mh, FC 80 lpm, abdomen blando, depresible, no doloroso, peristalsis (+).
    ¿Cuál de las siguientes es la conducta más apropiada?
    A. Solicitar colonoscopia
    B. Solicitar AngioTAC abdominal
    C. Realizar ecografía abdominal total
    D. Solicitar lipasa y TAC abdominal con contraste
A

Respuesta correcta: B

Tema e ítem: Isquemia Mesentérica Crónica - ¿Qué tan común es?

Argumento: Resulta indispensable iniciar el argumento definiendo la isquemia mesentérica como la reducción del flujo sanguíneo intestinal, la cual se clasifica como aguda (aparición repentina de síntomas/signos de hipoperfusión intestinal) o crónica dependiendo del curso temporal de los síntomas. La isquemia mesentérica crónica (IMC), también llamada angina intestinal, se refiere a la hipoperfusión episódica o continua del intestino delgado que ocurre típicamente en pacientes con estenosis u oclusión de múltiples vasos de la arteria mesentérica.

La IMC ocurre por la oclusión parcial y progresiva por una placa de ateroma a nivel de las arterias celíacas o mesentéricas superiores. Esta oclusión inhibe el aumento postprandial normal de la circulación esplácnica necesario para la digestión, lo que provoca dolor postprandial (angina intestinal). Aunque la mayoría de los pacientes con isquemia mesentérica crónica debido a enfermedad aterosclerótica no presentan síntomas debido a la extensa red colateral dentro de la vasculatura mesentérica que puede formarse para compensar el flujo reducido. Sin embargo, los pacientes que desarrollan síntomas suelen tener más de 60 años y tienen tres veces más probabilidades de ser mujeres que hombres. Además, la mayoría de los pacientes tienen antecedentes de tabaquismo y antecedente de enfermedad coronaria, enfermedad cerebrovascular o enfermedad arterial periférica.

La angio-TAC abdominal es la prueba diagnóstica de elección en IMC, ya que permite visualizar cualquier oclusión en la circulación esplácnica.

163
Q
  1. Mujer de 51 años, portadora de implantes mamarios desde hace 10 años, remitida por su médico de atención primaria por cuadro clínico de 1 semana de evolución consistente en tumefacción y dolor en mama izquierda. No tiene otros antecedentes relevantes. Al examen físico se evidencia tumefacción moderada y sensibilidad alrededor de la prótesis mamaria izquierda. No hay nódulos, eritema ni alteraciones cutáneas asociadas o ganglios axilares palpables. Analítica: Hb 12 g/dL, leucocitos 8.000, plaquetas 250.000. La ecografía reporta liquido periprotésico en mama izquierda sin signos que sugieran rotura protésica. La aspiración de líquido muestra liquido de aspecto seroso.
    ¿Cuál de los siguientes es el diagnóstico más probable?
    A. Linfoma anaplásico de células T
    B. Serositis autoinmune
    C. Cáncer de mama
    D. Infección del implante mamario
A

Respuesta correcta: A

Tema e ítem: Linfoma anaplásico de células T asociado a implantes mamarios

Argumento: Aunque es poco frecuente, cada vez hay más reportes de caso de linfoma no Hodgkin, normalmente linfoma anaplásico de células T, que aparece alrededor de los implantes mamarios. La principal asociación ha sido con los implantes mamarios texturizados, que ahora ya no se utilizan. Las pacientes con estos linfomas suelen presentar tumefacción y molestias con una mediana de 8 a 9 años tras la colocación de la prótesis. La evaluación inicial tras la exploración física se realiza mediante ecografía, la cual, en este contexto, suele ser superior a la mamografía, la TC o la RM para buscar una acumulación de líquido y aspirarlo si está presente. La citología puede confirmar células malignas compatibles con un linfoma anaplásico de células T. Como dato para los futuros residentes: estos linfomas anaplásicos de células T suelen ser CD30+ y ALK negativos.
Si la enfermedad se diagnostica precozmente, puede curarse sólo con cirugía con extracción del implante y resección completa de la cápsula circundante. Si se ha colocado un implante contralateral, también debe retirarse. En casos de enfermedad más avanzada con afectación ganglionar, diseminación más distante o recidiva, puede estar indicada la quimioterapia y/o la radioterapia. El tratamiento aún no está estandarizado, y las recomendaciones siguen evolucionando para esta entidad.

  • Aunque los implantes mamarios pueden infectarse, la ausencia de fiebre, leucocitosis y eritema y la presencia de líquido seroso no purulento no sugieren infección en esta paciente. Si bien es cierto que estas infecciones suelen producirse poco después de la colocación de la prótesis, también pueden desarrollarse años más tarde.
164
Q
  1. Doña Claudia de 51 años, sin antecedentes de interés, consulta al servicio de Urgencias por palpitaciones persistentes. El electrocardiograma realizado se muestra a continuación.
    ¿Cuál es el diagnóstico más probable?
    A. Fibrilación auricular
    B. Flúter auricular
    C. Taquicardia auricular
    D. Taquicardia sinusal
A

Respuesta correcta: B

Tema e ítem: Taquicardias supraventriculares – Flúter auricular

Argumento: El electrocardiograma de esta paciente con palpitaciones muestra un flúter auricular, el cual se caracteriza por una frecuencia auricular regular, a menudo cercana a 300/min, y con un patrón en diente de sierra. Las ondas de aleteo se observan más fácilmente en las derivaciones inferiores (II, III y aVF) y en la derivación V1. Lo más habitual es que exista una conducción 2:1 hacia los ventrículos, lo que da lugar a una frecuencia ventricular de aproximadamente 150/min, como se observa en la primera parte de este trazado electrocardiográfico.

165
Q
  1. Femenina de 27 años, G2P0C2, sin antecedentes relevantes, en décimo quinto día postcesarea sin complicaciones, consulta al servicio de Urgencias por cuadro clínico de 2 horas de evolución consistente en dolor torácico opresivo de intensidad 8/10. Al ingreso presenta PA 120/75 mh, FC 95 lpm regular, FR 25 rpm. A la auscultación cardiopulmonar se encuentra murmullo vesicular conservado y un S4 presente. Analítica: troponinas (+). EKG: elevación del ST de 1.7 mm en V3-V4. El ecocardiograma muestra hipocinesia de la pared anterior con FEVI del 40%.
    ¿Cuál de los siguientes es el diagnóstico más probable?
    A. Rotura de placa aterosclerótica - IAM tipo 1
    B. Miocardiopatía periparto
    C. Disección coronaria espontánea
    D. Miocardiopatía de takotsubo
A

Respuesta correcta: C

Tema e ítem: Emergencias cardiovasculares asociadas al embarazo – Disección espontanea de arteria coronaria (DEAC)

Argumento: La DEAC es la causa más frecuente de infarto de miocardio asociado al embarazo y suele producirse durante el embarazo o en el primer mes posparto. Esta afección implica el desarrollo de un hematoma intramural no traumático y no iatrogénico con o sin disección intimal con comunicación luminal. El agrandamiento del hematoma en la falsa luz comprime la verdadera luz de la arteria coronaria y, en posible combinación con la disección obstructiva, provoca dolor torácico, isquemia y/o infarto.

El diagnóstico requiere una AngioTAC coronaria o una angiografía coronaria para confirmar los rasgos de imagen característicos. Cuando se asocia a infarto de miocardio con elevación del segmento ST, la DEAC puede tratarse de forma invasiva; sin embargo, la intervención coronaria percutánea puede aplazarse con seguridad cuando el flujo coronario está preservado y los síntomas pueden controlarse y vigilarse estrechamente.

  • La fisiopatología del infarto de miocardio con elevación del segmento ST suele implicar la rotura de la placa dentro de una arteria coronaria. La rotura provoca adhesión, activación y agregación plaquetaria, lo que da lugar a la trombosis de la arteria coronaria y a la oclusión aguda del vaso. La pérdida repentina del flujo sanguíneo coronario provoca isquemia transmural del miocardio y la elevación del segmento ST en el electrocardiograma. EL SCA es una causa poco probable de infarto de miocardio que la DEAC en esta paciente que se encuentra en el postparto y no tiene factores de riesgo de enfermedad cardiovascular.
  • El cuadro clínico de esta paciente no concuerda con la miocardiopatía periparto, que es una disfunción sistólica del ventrículo izquierdo de reciente aparición en los meses posteriores al parto o hacia el final del embarazo en ausencia de otra causa identificable. Las pacientes con miocardiopatía periparto suelen presentar características de insuficiencia cardíaca. La ecocardiografía en la miocardiopatía periparto suele demostrar una reducción global de la función sistólica, no una hipocinesia focal.
  • La miocardiopatía inducida por estrés (takotsubo) se caracteriza por una disfunción cardiaca regional transitoria, que suele afectar a las porciones apical y media del ventrículo izquierdo. Suele precipitarse por un acontecimiento físico o emocional estresante. Se han descrito casos posparto de miocardiopatía inducida por estrés, especialmente tras un parto por cesárea. Los pacientes con miocardiopatía inducida por estrés presentan características que imitan un síndrome coronario agudo. El cuadro clínico de esta paciente es incompatible con una miocardiopatía inducida por estrés, dada la hipocinesia de la pared regional, que es más típica de la isquemia de la arteria coronaria.
166
Q
  1. Varón de 41 años, acude como cita prioritaria al programa de control. Hace 2 meses fue diagnostico de tuberculosis latente y actualmente se encuentra en manejo con isoniazida. Refiere historial de fatiga, malestar general y dificultad respiratoria de esfuerzo desde hace 1 mes. Al examen físico presenta signos vitales estables y palidez mucocutánea. Trae reporte de paraclínicos: Hb 7.2 g/dL, VCM 75, RR 0.7%, leucocitos 7100/m3, plaquetas 200.000/m3, Cr 0.9 mg/dL, hierro sérico 250 μg/dL, ferritina 450 ng/mL, capacidad de fijación de Hierro 220 μg/dL, glicemia central 85 mg/dL, Hb1Ac 5.8%.
    RR: recuento de reticulocitos
    ¿Cuál es el diagnóstico más probable?
    A. Anemia de enfermedad crónica
    B. Anemia por deficiencia de hierro
    C. α-talasemia
    D. Anemia inducida por isoniazida
A

Respuesta correcta: D

Tema e ítem: Anemia inducida por isoniazida - ¿Cuándo pensar en ella y cómo prevenirla?

Argumento: Estamos frente a una anemia microcítica (Hb disminuida y VCM bajo) como causa más probable del cuadro clínico del paciente (fatiga, malestar general y dificultad respiratoria).

La isoniazida es un fármaco de primera línea para el tratamiento de la tuberculosis latente y activa en todo el mundo. El efecto secundario más común es la hepatotoxicidad, pero en circunstancias muy raras, puede provocar aplasia pura de glóbulos rojos, anemia sideroblástica, anemia hemolítica y agranulocitosis. El aumento del hierro y la ferritina con disminución de la capacidad total de fijación del hierro orientan a pensar en anemia sideroblástica (AS) secundaria a la isoniazida.

La fisiopatología de la AS se explica a partir de la deficiencia de vitamina B6 (piridoxina) causada por el medicamento. Cuando la piridoxina es absorbida, esta es activada por algunas enzimas. La isoniazida compite por estas mismas enzimas, lo que reduce la cantidad de la forma activada de piridoxina en el organismo. Los metabolitos de isoniazida también reaccionan con la piridoxina para desactivarla.

La ALA sintasa es una enzima que requiere vitamina B6 como cofactor para un paso importante de la biosíntesis del hemo. En consecuencia, el hierro se acumula en las mitocondrias y no puede utilizarse en la síntesis del hemo. Si reportan el ancho de distribución eritrocitaria, probablemente estaría elevado, lo que refleja que los eritrocitos presenten variabilidad del tamaño celular (observando comúnmente en anemia sideroblástica). Un aspirado de médula ósea mostrará sideroblastos con anillos azules, ya que el hierro de las mitocondrias de los normoblastos absorbe la tinción con azul de Prusia. Para evitar efectos adversos en los pacientes que reciben tratamiento con isoniazida, debe administrarse también piridoxina de forma profiláctica.

167
Q

1.Usted se encuentra en un hospital de tercer nivel y llega un traslado primario desde Bolombolo con una paciente de 3 años con diagnóstico de dengue grave. La madre refiere que la paciente inició desde hace 4 días con fiebre objetiva de hasta 40°C, sin otra sintomatología. A los dos días, inició con vómito, dolor abdominal intenso y continuo, cefalea e hiporexia marcada, por lo cual al día siguiente la llevó al hospital local. Se realizó prueba de IgM para dengue la cual fue positiva, con AST y ALT elevadas. Al ingreso, hipotensa, taquicárdica, buena perfusión distal, petequias en extremidades. ¿Cuál es el manejo inicial en esta paciente?
A. Tomar función hepática completa para evaluar falla hepática
B. Hospitalizar e iniciar líquidos de mantenimiento por dengue con signos de alarma
C. Hospitalizar e iniciar líquidos intravenosos a 10 ml/kg/hora por dengue con signos de alarma
D. Iniciar bolo de 20 ml/kg y evaluar respuesta

A

Respuesta: D

Tema-Ítem: Dengue-Manejo del dengue grave

Argumentación: Esta paciente con dengue confirmada por IgM, se encuentra en la etapa crítica, cuando baja la fiebre y hay mayor riesgo de inestabilidad. Presentó inicialmente signos de alarma (dolor abdominal intenso y continuo) y ahora tiene ALT y AST elevadas, lo que indica compromiso hepático, una manifestación del dengue grave.

El dengue es una enfermedad viral aguda, endemo-epidémica, causada por un arbovirus de la familia Flaviviridae y transmitida por la picadura de hembras de mosquitos del género Aedes, principalmente el aegypti.

Es una enfermedad dinámica, a pesar de ser de corta duración (no más de una semana en casi el 90% de los casos). Puede agravarse de manera súbita; por lo cual se requiere seguimiento diario. Entre las formas graves, destacan:

♦️ Hepatitis
♦️ Insuficiencia hepática
♦️ Encefalopatía
♦️ Miocarditis
♦️ Hemorragias severas
♦️ Choque

Se caracteriza por 3 etapas: Febril, crítica y recuperación.

La etapa crítica (3 y 6 día para los niños) coincide con la defervescencia, es cuando hay extravasación de plasma. La manifestación más grave es el choque, que se evidencia con frialdad de la piel, pulso filiforme, taquicardia e hipotensión. Pueden presentarse: hemorragias digestivas, alteraciones hepáticas y quizás de otros órganos.

Para el tratamiento, clásicamente se dividen los pacientes en 3 grupos:

-A: Pacientes que pueden ser manejados ambulatoriamente en un primer nivel.

-B: Pacientes que deben ser hospitalizado para una estrecha observación: Se dividen a su ves en pacientes con y sin signos de alarma.

Los signos de alarma son: Dolor abdominal intenso y continuo, vómitos persistentes, diarrea, somnolencia y/o irritabilidad, hipotensión postural, hepatomegalia dolorosa > 2cms, disminución de la diuresis, caída de la temperatura, hemorragias en mucosas, caída abrupta de plaquetas (<100.000) asociada a hemoconcentración.

-C: Pacientes que requieren tratamiento de emergencia y cuidados intensivos por dengue grave

En estos pacientes con dengue grave, debemos evaluar inicialmente si está en choque (como en esta paciente que se encuentra con taquicardia e hipotensión) e iniciar resucitación con cristaloides a 20 cc/kg. En caso de que no haya mejoría, podmos pasar hasta dos bolos más. Si hay mejoría, debemos iniciar administración de líquidos de 5-7 cc/kg por dos horas, reevaluar y continuar 3-5 cc/kg en las siguientes 4 horas, reevaluar, continuar 2 cc/kg/h por dos horas. Después de esto, podemos iniciar líquidos de mantenimiento.

168
Q

2.Paciente de 5 días, llevada por su madre a la consulta. Nació a término por parto vaginal sin complicaciones, con buena adaptación neonatal, con peso de 3402 g. Se encuentra con lactancia materna a libre demanda. Diuresis y deposiciones normales. Al examen físico, peso de 3260 g, con descamación de manos y pies, al igual que acrocianosis. Al retirar el pañal, usted observa que está mojado y con algunas manchas rosadas. ¿Cuál de los siguientes es el siguiente paso más apropiado con este paciente?
A. Continuar lactancia materna a libre demanda
B. Uroanálisis
C. Lactancia materna y fórmula infantil para reforzar
D. Ionograma

A

Respuesta: A

Tema-Ítem: Examen físico normal del neonato

Argumentación: Este recién nacido a término tiene un examen físico normal. Se espera que la piel de las manos y los pies esté con descamación a medida que la piel se adapta al ambiente extrauterino que es seco. La aparición de “manchas rosadas” o “polvo de ladrillo” en los pañales del neonato representan cristales de ácido úrico y su excreción es especialmente alta al nacer y disminuye hacia la adolescencia, cuando llega a niveles normales en adultos. Los cristales de ácido úrico se observan comúnmente durante la primera semana, cuando baja la leche materna, o en meses posteriores, en la orina de la mañana, después de que el bebé comienza a dormir toda la noche.
Aunque el peso de este paciente ha disminuido aproximadamente 4% desde el nacimiento, es normal que los recién nacidos sanos normalmente pierdan hasta un 7% de su peso al nacer en los primeros cinco días de vida debido a la eliminación del exceso de líquido adquirido en la vida intrauterina y durante el parto. La pérdida de peso es más pronunciada en los lactantes amamantados exclusivamente a medida que el suministro de leche materna aumenta gradualmente para satisfacer las demandas del lactante. Se debe fomentar la lactancia materna frecuente y se debe brindar educación sobre la deshidratación.
Como regla general, la cantidad de pañales mojados debe ser igual a la edad en días durante la primera semana de vida. Por ejemplo, un recién nacido de 4 días debe mojar >4 pañales por día. Después de la primera semana, los bebés deben mojar más de 6 pañales por día. El peso al nacer debe recuperarse entre los 10 y 14 días de edad (segunda semana).
Los electrolitos séricos, particularmente el sodio, se pueden utilizar como una medida de la deshidratación. Los lactantes que pierden >7% del peso al nacer o que parecen deshidratados en el examen físico corren el riesgo de sufrir una deshidratación hipernatrémica potencialmente mortal. Sin embargo en este paciente que está bien hidratado, no es necesario hacer pruebas adicionales.
Se debe asegurar a los padres que los cristales de ácido úrico en la orina rara vez representan un trastorno del metabolismo de las purinas (p. ej., síndrome de Lesch-Nyhan). No es necesario ningún estudio si el bebé se alimenta, crece y orina normalmente.

169
Q
  1. Luis, 7 años. Llevado a consulta por su madre debido a aparición de acné en la cara que ha aparecido en los últimos meses. Fue automedicado con peróxido de benzoilo tópico y jabón con ácido salicílico, sin mejoría. También refiere que notó un aumento significativo en la estatura y ahora es mucho más alto que su hermano gemelo. Al examen físico, con altura y peso en +3 desviaciones estándar. Al examen físico con acné quístico en cara hombros; se observa vello púbico y axilar grueso, con volumen testicular de 2 ml. La edad ósea es 2 desviaciones estándar mayor que la edad cronológica. En paraclínicos, con niveles iniciales de LH bajos y que no aumentan después de la estimulación con un agonista de GnRH. ¿Cuál de los siguientes es el diagnóstico más probable en este paciente?
    A. Hiperplasia suprarrenal congénita no clásica
    B. Pubertad precoz idiopática
    C. Microadenoma hipofisario
    D. Síndrome de Klinefelter
A

Respuesta: A

Tema-Ítem: Pubertad precoz

Argumentación: La pubertad precoz es la aparición de características sexuales secundarias (p. ej., vello púbico/axilar, acné quístico, crecimiento acelerado) en niñas <8 años y niños <9 años. Estos pacientes se evalúan inicialmente obteniendo una edad ósea, que diferencia la verdadera pubertad precoz (edad ósea avanzada) de una telarquia/adrenarca precoz aislada (edad ósea normal). En pacientes con edad ósea avanzada (>2 desviaciones estándar por encima de la edad cronológica), los niveles de LH diferencian entre pubertad precoz periférica y central. Los pacientes con pubertad precoz periférica (independiente de gonadotropinas) tienen un nivel de LH bajo/normal debido a la inhibición del hipotálamo por niveles elevados de andrógenos circulantes provenientes de las glándulas suprarrenales o los testículos.
La aparición temprana de características sexuales secundarias en este paciente, la edad ósea avanzada y el nivel bajo de LH sugieren pubertad precoz periférica, probablemente hiperplasia suprarrenal congénita (CAH) no clásica secundaria a deficiencia de 21-hidroxilasa (CYP21A2). De manera similar a la CAH clásica, la deficiencia de 21-hidroxilasa altera la conversión de 17-hidroxiprogesterona en 11-desoxicortisol; la 17-hidroxiprogesterona se desvía hacia la sobreproducción de andrógenos suprarrenales (es decir, la pubertad precoz). Sin embargo, en pacientes con CAH no clásica, se mantienen niveles suficientes de glucocorticoides y mineralocorticoides; por lo tanto, los pacientes tienen electrolitos normales (sin excreción de sal).
La pubertad precoz idiopática (activación prematura del eje hipotalámico-pituitario-gonadal) y los microadenomas hipofisarios causan pubertad precoz (central) dependiente de gonadotropina. Debido a la activación continua del hipotálamo, los pacientes con pubertad precoz central tienen un nivel de LH basal alto o un nivel de LH que aumenta con la estimulación con GnRH.
El síndrome de Klinefelter (47,XXY) causa hipogonadismo primario, que se caracteriza por niveles bajos de testosterona y niveles elevados de LH. Los pacientes tienen una estatura alta; sin embargo, la pubertad suele retrasarse.

170
Q
  1. Paciente de 1 mes, llevado a urgencias porque en las últimas 2 semanas durante la alimentación se observa agitado, con taquipnea, diaforesis y palidez. Al examen físico, con signos vitales normales, leves retracciones, soplo holosistólico de 4/6 en el borde esternal inferior izquierdo con un frémito palpable y crépitos en ambas bases pulmonares. El hígado es palpable 3 cm por debajo del margen costal derecho. ¿Cuál de las siguientes es la causa más probable de la presentación clínica de este paciente?
    A. Aumento del flujo sanguíneo pulmonar
    B. Derivación intracardíaca de derecha a izquierda
    C. Aumento de la poscarga del ventrículo izquierdo
    D. Disminución de la contractilidad del ventrículo izquierdo
A

Respuesta: B

Tema-Ítem: Cardiopatías Congénitas

Argumentación: En este paciente encontramos signos de insuficiencia cardíaca y un soplo holosistólico en el borde esternal inferior izquierdo, hallazgos característicos de un defecto del tabique ventricular (comunicación interventricular). En el útero, esta comunicación anormal entre los ventrículos derecho e izquierdo no tiene consecuencias hemodinámicas debido a presiones cardíacas casi equivalentes en los lados derecho e izquierdo (es decir, resistencia vascular pulmonar (RVP) alta). Al nacer, los pulmones reemplazan a la placenta como lugar de oxigenación, y la RVP y las presiones del lado derecho caen, creando una derivación de izquierda a derecha de sangre oxigenada a través de la CIV.
Con una CIV grande, el ventrículo derecho se sobrecarga de volumen con flujo tanto de la aurícula derecha como del ventrículo izquierdo. A su vez, aumentan el flujo sanguíneo pulmonar y el retorno sanguíneo del lado izquierdo. Para mantener el flujo sanguíneo sistémico, el gasto del ventrículo izquierdo (VI) aumenta pero no puede seguir el ritmo del mayor retorno sanguíneo, lo que provoca insuficiencia cardíaca de gasto alto.
Los síntomas tanto de la insuficiencia del VI de alto gasto como de la sobrecirculación pulmonar incluyen aumento del trabajo respiratorio (p. ej., taquipnea, retracciones), taquicardia, diaforesis y palidez, en particular durante momentos de esfuerzo (p. ej., alimentación en lactantes). Pueden presentarse crépitos bilaterales por aumento del trasudado extravascular en los pulmones y un frémito palpable debido al flujo intracardíaco turbulento. También son comunes los signos de insuficiencia cardíaca derecha (p. ej., hepatomegalia).
-La insuficiencia cardíaca por CIV se debe a la incapacidad del ventrículo izquierdo para mantener el ritmo de la sobrecarga de volumen del lado izquierdo a pesar de un gasto normal o aumentado. La contractilidad del VI sigue siendo normal.
-La alteración de la relajación diastólica del VI puede ocurrir con una hipertrofia concéntrica del VI que ocurre con la sobrecarga crónica de presión del VI (p. ej., hipertensión sistémica). Por el contrario, la CIV implica una sobrecarga de volumen (en lugar de presión) del VI con hipertrofia excéntrica compensatoria y un aumento de la capacidad de llenado diastólico.
La poscarga del VI disminuye y no aumenta con la CIV, porque la conexión con el ventrículo derecho proporciona una vía de resistencia relativamente baja para el flujo de salida del VI.
Una CIV no tratada puede causar enfermedad vascular pulmonar debido al aumento prolongado del flujo sanguíneo pulmonar. El aumento de la RVP finalmente conduce a una inversión de la derivación intracardíaca hacia el flujo de derecha a izquierda (síndrome de Eisenmenger), lo que causa cianosis, que no se observa en este paciente.

171
Q
  1. Santiago tiene 5 años y llega a urgencias por 3 días de fiebre. Durante los últimos 2 días también ha presentado odinofagia e hiporexia. Al examen físico febril, taquicárdico, en orofaringe se observan varias vesículas dispersas de 1 mm en los pilares palatinos anteriores. También se observan varias úlceras grises y poco profundas en el paladar blando y la úvula, sin lesiones en otros sitios de la mucosa oral. ¿Cuál de los siguientes es el patógeno más probable asociado con la enfermedad de este paciente?
    A. Virus Coxsackie
    B. Virus del herpes simple
    C. Streptococcus pyogenes
    D. Adenovirus
A

Respuesta: A

Tema-Ítem: Herpangina-Enfermedades infecciosas en pediatría

Argumentación:
La herpangina y la gingivoestomatitis herpética son infecciones orales vesiculares comunes en los niños. Se presentan de manera similar con fiebre y faringitis; sin embargo, la característica distintiva principal es la ubicación de las lesiones orales.
La herpangina es causada por el virus Coxsackie y generalmente afecta a niños más pequeños. Se observa vesículas grises que progresan hasta convertirse en úlceras recubiertas de fibrina. El enantema oral se encuentra en la orofaringe, en el paladar blando posterior, los pilares palatinos anteriores, las amígdalas y la úvula. A diferencia de la enfermedad mano-pie-boca, otra afección causada por el virus Coxsackie, la herpangina no se asocia con una erupción.
Por el contrario, la gingivoestomatitis herpética es una infección por el virus del herpes simple caracterizada por grupos de vesículas que generalmente se localizan en la cavidad bucal anterior (mucosa bucal, lengua, encía, paladar duro) y los labios.
El adenovirus es una causa común de amigdalitis en niños y se presenta con fiebre, tos y dolor de garganta. Puede haber exudados amigdalinos, pero no vesículas o úlceras orofaríngeas.
El Streptococcus pyogenes es la causa principal de faringitis estreptocócica, que se presenta con fiebre, dolor de garganta, linfadenopatía cervical anterior dolorosa y exudados amigdalinos. La ausencia de exudados y la presencia de vesículas en este paciente hacen menos probable el diagnóstico de Streptococcus.

172
Q
  1. Masculino de 58 años, consultó por múltiples episodios de hemoptisis abundantes en las últimas 2 horas, junto a disnea progresiva de esfuerzo y tos productiva con pintas de snagre desde hace 6 meses. Antecedente de IPA de 30. Signos vitales: PA de 96/62 mm\Hg, FC de 118 lpm, FR 22 rpm y SO2 de 89% al ambiente. Al examen físico, paciente caquéctico, con disminución bilateral de ruidos respiratorios. Sin adenopatías. Paraclínicos: Hb de 11,5 g/dL, leucocitos de 8.800 y plaquetas en 160.000. Radiografía de tórax reporta hiperinsuflación pulmonar y un nódulo de 2 cm en el lóbulo inferior derecho. Se intuba al paciente para asegurar vía aérea, se inician líquidos endovenosos y se ubica al paciente en decúbito lateral derecho.
    ¿Cuál es el siguiente paso en el manejo del paciente?
    A. Realizar una broncoscopia
    B. Solicitar una arteriografía
    C. Realizar cirugía para resección pulmonar
    D. Solicitar una tomografía computarizada de tórax
A

Respuesta correcta. A

Tema e ítem: Hemoptisis-Manejo

Argumento: Este paciente con antecedentes importantes de tabaquismo, junto al hallazgo radiológico de un nódulo pulmonar en combinación con síntomas pulmonares (disnea, apoptosis, necesidad de asegurar vía aérea) y síntomas sistémicos como la caquexia, sugieren una hemorragia pulmonar debido a un carcinoma broncogénico; por ende, una vez realizado el tratamiento de la hemoptisis masiva potencialmente mortal, que consiste en el posicionamiento correcto del paciente (con el pulmón sangrante en posición dependiente), vía aérea asegurada y soporte cardiovascular (líquidos), se debe realizar una broncoscopia para identificar el origen del sangrado y detenerlo (Un taponamiento con balón, lavado con solución salina a baja temperatura, epinefrina tópica o terapia con láser).

En cuanto a la arteriografía, se indica en pacientes con hemoptisis persistente, posterior al fracaso de manejo con la primera línea (broncoscopia). La arteriografía se utiliza para identificar el sitio de hemorragia intrapulmonar y detener el sangrado mediante embolización. Por otra parte, la cirugía para resección pulmonar se debe considerar en pacientes con sangrado unilateral no controlable con uso de estrategias de primera y segunda línea (broncoscopia y arteriografía, respectivamente), por lo que este paciente aún tiene opciones de tratamiento menos invasivas. Además, la resección, particularmente en pacientes con hemorragia pulmonar activa, se asocia con alta morbilidad y mortalidad, por lo que requiere una evaluación de la capacidad del paciente para someterse a cirugía, y este paciente probablemente por su IPA elevado, sea descalificado para la cirugía de resección pulmonar. Finalmente, la tomografía computarizada de tórax está indicada en pacientes con una lesión de novo detectada en una radiografía de tórax o para identificar el sitio de sangrado en pacientes con hemoptisis (debe realizarse dentro de las primeras 24 horas después de la manifestación sintomatica), sin embargo, este paciente con sangrado activo e inestabilidad hemodinámica, se beneficia de otro manejo como la broncoscopia.

173
Q
  1. Masculino de 30 años, consultó por episodios recurrentes de disnea, junto a tos no productiva durante los últimos 8 meses. Refiere que el cuadro empeora en las noches y con el ejercicio, con una frecuencia de 2 veces por mes. Signos vitales FC de 73 lpm, FR de 15 rpm, PA de 122/70 mm/Hg, SO2 del 98%. Al examen físico sin hallazgos anormales. Se realizó una espirometría que presentó una CVF del 95%, una relación VEF1/CVF de 0.65, un VEF1 del 78% y un valor del VEF1 postbroncodilatador que aumentó 18% y 320ml con respecto al prebroncodilatador.
    ¿Cuál es el manejo adecuado para este paciente?
    A. Baja dosis de corticosteroide inhalado + formoterol a necesidad
    B. Baja dosis de corticosteroide inhalado + formoterol para mantenimiento
    C. Baja dosis de corticosteroide inhalado para mantenimiento
    D. Manejo exclusivo con antagonista de los receptores de leucotrienos
A

Respuesta correcta: B

Tema e ítem: Asma-manejo

Argumento: Este paciente presenta en la espirometría un patrón obstructivo que revierte al broncodilatador, junto a una clínica de disnea asociada al ejercicio, tos no productiva, y cuadros respiratorios que empeoran 2 veces por mes en la noche, lo que nos sugiere un cuadro de asma del adulto; que a su vez, dado los síntomas del paciente sé clásica según la guía GINA 2023 como un STEP 3, y por ende la recomendación de tratamiento preferida es el uso de bajas dosis de corticosteroide inhalado + formoterol para mantenimiento, debido a que los agonistas β2 de acción corta son el tratamiento de elección para los síntomas de asma.

En cuanto al uso de un corticosteroide inhalado (CI), se puede utilizar como controlador del asma en el tratamiento del asma leve, moderada y persistente. Con el tiempo, reduce la frecuencia de los síntomas, así como la gravedad de cada exacerbación posterior; sin embargo, el tratamiento con ICS solo, no es eficaz para aliviar los síntomas agudos del asma debido a su efecto tardío inicio de acción; por ende, se administra en combinación con medicamentos de acción corta o de acción prolongada como un agonista β2-adrenérgicos, antagonistas de los receptores de leucotrienos o antagonistas muscarínicos. Por otra parte, el uso de bajas dosis de corticosteroide inhalado + formoterol a necesidad, se recomienda para pacientes en la escala STEP 1 o 2 de la guía GINA 2023. Estos pacientes se caracterizan por tener síntomas 2 veces al mes o más, pero menos de 4 a 5 días por semana, sin síntomas nocturnos. Por lo que este paciente, al clasificar como un STEP 3, requiere un manejo no a necesidad, sino de mantenimiento. Finalmente, los antagonistas de los receptores de leucotrienos (Moontelukast), se pueden utilizar para tratar y controlar el asma; sin embargo, el uso solo de este medicamento es ineficaz para el manejo adecuado de este paciente, y no se consideraría en este paciente con síntomas persistentes antes que otros medicamentos de control preferidos (El uso de CI y un SAMA o LABA)

174
Q
  1. Masculino de 68 años, consulta por disnea y tos productiva durante el último año. Refiere que puede subir las escaleras sin detenerse. Antecedente de hipertensión arterial, IPA de 15. Signos vitales con FC de 98 lpm, PA de 134/88 mm/Hg, SO2 del 93% al ambiente. Al examen físico presenta sibilancias espiratorias y roncus en ambos campos pulmonares. La espirometría muestra una relación FEV1:FVC del 63% y un FEV1 del 65% del predicho. Su capacidad de difusión de monóxido de carbono (CDMC) es del 40% de lo previsto.
    ¿Cuál es la causa más probable de los hallazgos de este paciente?
    A. Asma
    B. Bronquitis crónica
    C. Enfisema pulmonar
    D. Hipertensión pulmonar
A

Respuesta correcta: C

Tema e ítem: Espirometría-Diagnóstico

Argumento: Este paciente, la edad que tiene, los síntomas (disnea, tos productiva, sibilancias espiratorias), y los antecedentes de asma y consumo de tabaco en combinación con signos de obstrucción de las vías respiratorias en las pruebas de función pulmonar son altamente sugestivas de enfermedad pulmonar obstructiva crónica (EPOC). La EPOC puede manifestarse con enfisema por la destrucción de las paredes alveolares, debido a la obstrucción respiratoria e infecciones recurrentes. La disminución de la CDMC es un hallazgo típico de enfisema, que refleja la reducción del intercambio gaseoso debido a una pérdida del área de difusión.

En cuanto al asma, los síntomas respiratorios y los hallazgos de la espirometría de este paciente pueden sugerir esta entidad; sin embargo, la CDMC está reducida en lugar de normal o elevada, como se esperaría en pacientes con asma (La CDMC está elevada en pacientes con asma, debido a un aumento de perfusión en los ápices pulmonares). Además, sus síntomas han progresado durante más de un año, y no suceden en forma de “ataques” agudos de asma. Por otra parte, la bronquitis crónica es un diagnóstico diferencial importante en pacientes con antecedentes de fumar. Además, este grupo de pacientes suele presentar disnea (de esfuerzo), tos productiva y sibilancias al final de la espiración que duran al menos 3 meses en 2 años consecutivos, junto a un patrón pulmonar obstructivo; sin embargo, la CDMC suele ser normal en pacientes con bronquitis crónica, en contraste con los niveles disminuidos observados en este paciente, ya que no hay cambios estructurales en los alvéolos que afectan la capacidad de difusión. Finalmente, la hipertensión pulmonar suele ser asintomática en las primeras etapas, pero puede manifestarse con disnea y tos, para posteriormente cursar con síntomas de falla del corazón derecho. Además, la CDMC también suele verse afectada en los individuos con hipertensión pulmonar, y la espirometría puede mostrar un patrón pulmonar obstructivo (Cuando la EPOC es la causa subyacente, generalmente muestra resultados normales en pacientes con hipertensión pulmonar)

175
Q
  1. Femenina de 54 años, consultó por 6 meses de dolor sordo, unilateral en pierna derecha de miembro inferior (MI), acompañado de edema. El dolor empeora al final del día y se alivia caminando o elevando los pies. Antecedente de trombosis venosa profunda en la pantorrilla derecha hace 6 meses, manejada con anticoagulantes. Al examen físico, edema grado 2+ con fóvea en la pierna derecha. La piel alrededor del tobillo derecho muestra una coloración marrón rojiza y múltiples telangiectasias, junto a venas varicosas dilatadas en la pierna derecha.
    ¿Qué examen se debe solicitar para esclarecer el diagnóstico?
    A. Tomografía computarizada con contraste del MI derecho
    B. Índice tobillo-brazo
    C. Ecografía Doppler
    D. Electromiografía
A

Respuesta correcta: C

Tema e ítem: Insuficiencia venosa crónica- diagnóstico

Argumento: En esta paciente, el dolor sordo crónico unilateral, junto al edema, las venas dilatadas en las extremidades inferiores y los antecedentes de trombosis venosa profunda (TVP) sugieren una insuficiencia venosa crónica (IVC), donde la ecografía es la primera línea de abordaje en estos pacientes con características sugestivas de IVCI, como se ve aquí, debido a que la ecografía confirma el diagnóstico, y ayuda a determinar el curso del tratamiento, identificando qué válvulas son incompetentes y evaluando la permeabilidad de las venas profundas previamente trombosadas.

En cuanto a la tomografía computarizada con contraste de las extremidades inferiores, se indica para evaluar a los pacientes en busca de infecciones profundas (abscesos), neoplasias malignas o lesiones en la parte inferior de las extremidades. No se utiliza para evaluar IVC. Por otra parte, el índice de presión tobillo-brazo (IPTB) se utiliza para detectar enfermedad arterial periférica (EAP) en pacientes que presentan dolor en las extremidades inferiores, que sugieran dolor por claudicación (es decir, dolor al caminar que se alivia con el reposo). El dolor de este paciente mejora al caminar, haciendo que la EAP sea una causa poco probable de sus síntomas. Sus síntomas son consistentes con IVC, la cual no se puede evaluar con IPTB. Finalmente, los estudios de conducción nerviosa se utilizan para evaluar la función del sistema periférico. Son particularmente críticos para diagnosticar polineuropatías (por ejemplo, debido a diabetes mellitus crónica). Aunque la polineuropatía puede manifestarse con asimetría o dolor unilateral, que se observa en este paciente, el dolor generalmente cursa como parestesias, no como un dolor sordo. Además, la polineuropatía no causaría el edema y telangiectasias observadas en esta paciente.

176
Q
  1. Femenina de 24 años, consulta junto a su pareja por cuadro súbito de diaforesis, palidez y pérdida del conocimiento hace dos horas mientras estaba sentada en una sesión de tatuajes. La pareja de la paciente refiere “ella presentó durante 5 segundos movimientos espásticos de brazos y piernas”, posteriormente recuperó el conocimiento al cabo de medio minuto y ya estaba completamente orientada y alerta. Signos vitales FC de 68 lpm, FR 16 rpm y la PA de 120/72 mm/Hg. Al examen físico no hay hallazgos anormales. Un electrocardiograma reporta un bloqueo de rama derecha incompleto.
    ¿Cuál es el diagnóstico más probable?
    A. Convulsión tónico-clónica generalizada
    B. Bradiarritmia
    C. Ataque de pánico
    D. Síncope vasovagal
A

Respuesta correcta: D

Tema e ítem: Síncope- diagnóstico

Argumento: Esta paciente que presentó una pérdida del estado de conciencia de corto tiempo (30 segundos), junto a un periodo corto de recuperación del estado de alerta y conciencia, en el contexto de una situación dolorosa o estresante (sesión de tatuaje), se le debe sospechar un síncope vasovagal (SV). Recordemos que el SV, es un tipo de síncope mediado neuralmente, y causado por hiperactividad parasimpática y/o hipoactividad simpática, que provoca bradicardia y/o vasodilatación; que a su vez, conduce a disminución de la presión arterial y perfusión cerebral. Los pacientes experimentan un pródromo presincopal, así como sacudidas mioclónicas que duran menos de 15 segundos.

En cuanto a una convulsión tónico-clónica generalizada, está implicaría una pérdida del conocimiento y la presencia de contracciones tónico-clónicas. Aunque se describió que esta paciente tenía “movimientos espaticos” durante el evento, los movimientos tónico-clónicos durante las convulsiones suelen durar más de 30 segundos. Además, los síntomas prodrómicos, preceden a un ataque epiléptico (olores inusuales, automatismos como la masticación y/o alucinaciones auditivas o visuales). Adicionalmente, esta paciente recuperó el conocimiento y estuvo completamente alerta y orientada al cabo de medio minuto, mientras que los pacientes experimentan confusión postictal o amnesia después de un ataque epiléptico. Por otra parte, una bradiarritmia (síndrome del seno enfermo, bloqueo auriculoventricular) y las taquiarritmias ventriculares, pueden provocar síncope al disminuir la fracción de eyección; por ende, la perfusión cerebral; sin embargo, los hallazgos del EKG de esta paciente de bloqueo incompleto de rama derecha, pueden ser normales en adultos jóvenes y no suele asociarse con síncope en reposo. Además, los pacientes con síncope debido a arritmias cardíacas no experimentan estados prodrómicos antes del síncope, y pierden la conciencia repentinamente durante el esfuerzo (aumenta las demandas de oxígeno del cuerpo). Finalmente, los ataques de pánico pueden causar episodios similares a síncopes debido a la hiperventilación inducida, que lleva a hipocapnia y posterior vasoconstricción cerebral; sin embargo, la verdadera pérdida del conocimiento debido al ataque de pánico es poco común, a menos que la hiperventilación sea bastante grave.

177
Q
  1. Mujer de 35 años, G4P3, 34 sem de gestación. Asiste a control prenatal. No nota contracciones ni flujo vaginal. Su último hijo nació a las 35 sem por parto espontáneo, los demás a término sin complicaciones. Útero compatible con 34 sem. Eco: feto podálico, FCF 148/min. ¿Cuál es la opción más adecuada?
    A. Debe continuar con observación.
    B. Se debe administrar sulfato de magnesio intravenoso.
    C. Debe ser programada para cesárea.
    D. Debe aplicarse dexametasona intramuscular.
A

Respuesta correcta: A

Tema e ítem: Presentación podálica - Manejo.

Argumento: La mayoría de las presentaciones podálicas se resuelven antes de las 37 sem de gestación, ya que el feto es móvil con mayores volúmenes de líquido amniótico. Dado que la paciente tiene 34 sem de gestación, no está en trabajo de parto y no tiene complicaciones fetales, todavía existe la posibilidad de conversión espontánea a la posición cefálica: no se requiere ninguna intervención en este momento y la observación es suficiente. Si la presentación de nalgas no se resuelve a las 37 sem de gestación, la paciente tiene varias opciones: versión cefálica externa (VCE) antes del parto espontáneo con un intento de parto vaginal, seguido de conversión a cesárea si no tiene éxito; parto por cesárea programado sin intento de VCE; o un intento de parto vaginal sin VCE. Sin embargo, los partos vaginales podálicos se asocian a un mayor riesgo de lesión hipóxica y traumatismo en el parto debido al atrapamiento del hombro o la cabeza.

  • El sulfato de magnesio IV tiene varios usos en el embarazo, incluyendo la prevención/tratamiento de convulsiones en pacientes con preeclampsia/eclampsia, neuroprotección para fetos con < 32 sem de gestación y tocolisis en partos prematuros para permitir la administración de corticosteroides prenatales. Esta paciente no cumple los criterios para ninguna de estas indicaciones.
  • El parto por cesárea puede ser apropiado para pacientes con ≥ 37 sem de gestación con un feto de nalgas que no son buenas candidatas para la versión cefálica externa o el parto vaginal podálico, o para aquellas que prefieren el parto por cesárea a un intento de parto vaginal. Esta paciente no ha llegado a término ni está de parto, y no hay indicios de sufrimiento fetal que justifiquen una cesárea. Una vez que esta paciente esté en ≥ 37 sem de gestación, puede elegir un parto por cesárea electiva si el feto permanece podálico.
  • La dexametasona intramuscular mejora la madurez pulmonar del feto y, por lo tanto, debe administrarse a mujeres embarazadas que se encuentren entre las 23 semanas’ y las 33 semanas + 6 días’ de gestación y que presenten un mayor riesgo de parto prematuro en los próximos 7 días. Esta paciente ya tiene 34 semanas de gestación; la dexametasona no mejoraría aún más los resultados pulmonares fetales.
178
Q
  1. Mujer, 36 años, G1P0, 22 sem de gestación. Fumó cigarrillo por 15 años, ganó peso al dejar de fumar (hace 6 años) e inició su propia dieta. Examen físico: T 37°C, FC 88/min, PA 115/72 mmHg e IMC 18 kg/m2. Fondo uterino entre la sínfisis y el ombligo. Peso fetal estimado en percentil (ptl) 9 (cabeza ptl 20, abdomen ptl 9), ILA bajo. ¿Qué factor se asocia más a los hallazgos ecográficos?
    A. Trastorno de la conducta alimentaria.
    B. Edad materna avanzada.
    C. Antecedente de tabaquismo.
    D. Posible aneuploidía.
A

Respuesta correcta: A

Tema e ítem: RCIU - Diagnóstico.

Argumento: Las pacientes embarazadas con antecedentes de anorexia nerviosa o anorexia actual tienen un mayor riesgo de RCIU debido a la malnutrición crónica y al bajo peso durante el embarazo. Este riesgo persiste incluso años después del tratamiento de un trastorno alimentario, muy probablemente debido a los síntomas residuales.

El retraso del crecimiento intrauterino asimétrico suele producirse a finales del segundo o principios del tercer trimestre y afecta al crecimiento fetal más que a la formación de órganos. Normalmente, las dimensiones de la cabeza son normales debido a la derivación de la sangre a los órganos fetales vitales (por ejemplo, cerebro, corazón), mientras que los órganos menos vitales (por ejemplo, hígado, músculo) se omiten, dando lugar a un tronco y extremidades pequeñas.

Hoy en día se sigue clasificando el crecimiento fetal restringido en simétrico y asimétrico, pues da cuenta de los fenómenos fisiopatológicos implicados de cada caso, aunque las guías más recientes no lo consideran necesario desde el punto de vista asistencial.

  • Hay poca evidencia que sugiera que la edad materna avanzada (35-40 años de edad) por sí sola sea un factor de riesgo aumentado de RCIU. En cambio, la edad materna avanzada debe considerarse en el contexto de otros factores de riesgo (por ejemplo, hipertensión, diabetes).
  • El tabaquismo actual es un factor de riesgo significativo para el RCIU, probablemente debido a las anomalías placentarias que induce el tabaquismo. Esta paciente, sin embargo, dejó de fumar hace 6 años, lo que significa que el consumo de nicotina no explica los hallazgos.
  • La aneuploidía fetal es una causa potencial de RCIU simétrico, en lugar de asimétrico.
179
Q
  1. Mujer, 37 años, G2C1, 35 sem de gestación. Presenta sangrado vaginal continuo, oscuro y dolor en hipogastrio por 1h. Tiene HTA crónica, mala adherencia al manejo. Examen físico: FC 90/min, FR 16/min y PA 145/90 mmHg. Hay hipertonía uterina, sangre en vulva e introito. El feto es podálico. Monitoreo: FCF 180/min con desaceleraciones recurrentes. ¿Cuál es la causa del compromiso fetal?
    A. Ruptura uterina.
    B. Desprendimiento de la placenta.
    C. Posición distócica.
    D. Ruptura de vasos fetales aberrantes.
A

Respuesta correcta: B

Tema e ítem: desprendimiento de placenta - diagnóstico.

Argumento: El desprendimiento de la placenta provoca la rotura de los vasos placentarios (principalmente maternos). La hemorragia resultante puede disecarse entre las membranas y la pared miometrial hacia la vagina causando el característico sangrado vaginal rojo oscuro o disecarse en la pared miometrial, resultando en un útero tenso y sensible y contracciones hipertónicas o ambos. El segmento desprendido es incapaz de intercambiar gases y nutrientes, lo que puede provocar sufrimiento fetal. Dado que hay signos de sufrimiento fetal, debe ser llevada a cesárea de urgencia.

  • Un parto por cesárea previo es un factor de riesgo importante para la rotura uterina, que también puede manifestarse con hemorragia anteparto, dolor y sensibilidad abdominal (debido a la irritación peritoneal por la sangre) y estado fetal no tranquilizador. Sin embargo, las pacientes con rotura uterina son hemodinámicamente inestables y la exploración mostraría una ausencia característica de contracciones uterinas y pérdida de estación de la parte fetal que se presenta. La presencia de contracciones uterinas hipertónicas y la presentación fetal evidente hacen que la rotura uterina sea una causa poco probable.
  • La presentación podálica puede asociarse a sufrimiento fetal, debido al prolapso/compresión del cordón. Es más frecuente en pacientes con placenta previa, que a su vez puede presentar hemorragia preparto. Sin embargo, la hemorragia sería entonces característicamente rutilante e indolora. La sangre oscura y dolor abdominal hacen que la placenta previa sea una causa poco probable de un estado fetal no tranquilizador en este caso.
  • La rotura de vasos fetales aberrantes (que recubren el orificio cervical) se produce en la vasa previa cuando se rompen las membranas y se caracteriza por una hemorragia vaginal indolora de color rojo brillante. Dado que la hemorragia es principalmente fetal, los vasa previa provocan rápidamente sufrimiento fetal. Sin embargo, la presencia de sangrado vaginal rojo oscuro y sensibilidad uterina en esta paciente, especialmente con antecedentes de hipertensión mal controlada, hace más probable otro diagnóstico.
180
Q
  1. Mujer de 28 años, G2P1, 14 sem de gestación. Lleva 3 días con flujo vaginal anormal. Sin fiebre ni dolor abdominal. Es sexualmente activa, no usa métodos de barrera. Se observa un cérvix friable con secreción mucopurulenta y maloliente, sin sensibilidad uterina ni cervical. pH vaginal: 4,3. ¿Cuál es el paso más apropiado para el diagnóstico?
    A. Preparación en húmedo.
    B. Prueba de aminas.
    C. Análisis de orina y cultivo.
    D. Prueba de amplificación de ácidos nucleicos.
A

Respuesta correcta: D

Tema e ítem: Clamidia y gonorrea en cervicitis - Pruebas diagnósticas

Argumento: La prueba de amplificación de ácidos nucleicos (NAAT) es la modalidad preferida para diagnosticar infecciones por Chlamydia trachomatis y Neisseria gonorrhoeae, que son las causas más comunes de cervicitis. La NAAT también puede diagnosticar ciertas causas infrecuentes de cervicitis aguda, como la tricomoniasis. Las pacientes con gonorrea, clamidia o tricomoniasis confirmadas también deben someterse a pruebas de detección del VIH y la sífilis. Las pruebas de clamidia y gonorrea están indicadas durante la visita prenatal inicial (aproximadamente a las 10 semanas de gestación) en todas las pacientes < 25 años de edad y en pacientes ≥ 25 años de edad con factores de riesgo de infecciones de transmisión sexual.

  • La preparación en fresco del frotis vaginal puede utilizarse para detectar protozoos flagelados característicos de la tricomoniasis y células clave características de la vaginosis bacteriana (VB). Estas afecciones pueden causar flujo vaginal maloliente, pero suelen dar lugar a un pH vaginal elevado (pH > 4,5). Además, la cervicitis aislada no es una manifestación típica de ninguna de estas afecciones. Una preparación de montaje húmedo no es fiable para diagnosticar las causas infecciosas más comunes de cervicitis porque estas infecciones sólo causan el hallazgo inespecífico de un número elevado de leucocitos en el montaje húmedo.
  • La prueba de las aminas (whiff test) se utiliza para diagnosticar la vaginosis bacteriana (VB), que puede causar un flujo vaginal maloliente. Sin embargo, la VB se manifiesta típicamente con un pH vaginal elevado (pH > 4,5) y un flujo gris o lechoso en lugar de mucopurulento. La cervicitis aislada hace que la VB sea muy poco probable. El test de aminas no evalúa las causas infecciosas más comunes de cervicitis.
  • El análisis y cultivo de orina sería el paso inicial más apropiado si se sospechara una infección del tracto urinario (ITU). Esta paciente presenta cervicitis y no refiere características de una ITU (por ejemplo, disuria, tenesmo vesical, polaquiuria, dolor suprapúbico).
181
Q
  1. Mujer de 25 años, G1P0, 12 sem de gestación. Tiene una prueba positiva que confirma cervicitis por Chlamydia trachomatis, fue negativa para Neisseria gonorrhoeae. ¿Cuál es la farmacoterapia más adecuada?
    A. Doxiciclina 100 mg c/12h VO por 7 días
    B. Azitromicina 1 g VO DU
    C. Ceftriaxona 500 mg IM DU
    D. Levofloxacino 500 mg c/día VO por 7 días
A

Respuesta correcta: B

Tema e ítem: Cervicitis – manejo

Argumento: La doxiciclina es el tratamiento preferido para la infección por clamidia en pacientes no embarazadas, pero está contraindicada durante el embarazo porque puede provocar lesiones óseas y manchas en los dientes del niño. Una dosis única de azitromicina oral es el tratamiento de primera línea recomendado para la infección por clamidia en pacientes embarazadas. También se recomienda acelerar las pruebas y el tratamiento de la pareja para evitar la reinfección. Además, las pacientes y sus parejas sexuales deben abstenerse de mantener relaciones sexuales durante los siete días siguientes al inicio del tratamiento.

  • La doxiciclina se utiliza en caso de infección por clamidia, pero la administración de doxiciclina durante el embarazo está contraindicada porque su ingesta puede provocar lesiones óseas y depósitos en los dientes (amarillamiento de los dientes primarios) de la descendencia. Aunque la cefoxitina es eficaz contra N. gonorrhoeae, esta paciente ha dado negativo en las pruebas de N. gonorrhoeae y, por tanto, no necesita cefoxitina. Dada la alta sensibilidad (∼ 99%) de las pruebas de amplificación de ácidos nucleicos para la detección de la gonorrea, un resultado negativo prácticamente descarta la infección gonocócica.
  • La ceftriaxona intramuscular es el tratamiento de elección para la infección por N. gonorrhea. Sin embargo, este paciente ha dado negativo para N. gonorrhea y positivo para C. trachomatis; la ceftriaxona no es la terapia de primera línea para C. trachomatis.
  • La levofloxacina es muy eficaz contra C. trachomatis pero la administración de fluoroquinolonas está contraindicada durante el embarazo y la lactancia debido a su potencial para causar daños óseos y cartilaginosos en el feto.
182
Q
  1. Hombre, 23 años, sufre accidente de tránsito. Tuvo rotura esplénica y laceración hepática. En cirugía transfundieron 6 UGR. 18h postoperatorio: FC 140/min, PA 80/50 mmHg, presión venosa central 19 cm H2O. Hay distensión yugular, incisión sin signos de ISO, abdomen distendido. Orina en últimas 6h: 90 ml, nitrógeno ureico 90 mg/dL, Cr 3 mg/dL, HCO3 29 mEq/L. ¿Cuál es la mejor conducta?
    A. Administración de plasma fresco congelado.
    B. Reintervenir y dejar abdomen abierto.
    C. Administración de epinefrina IM.
    D. Administración de antibióticos intravenosos.
A

Respuesta correcta: B

Tema e ítem: Síndrome compartimental abdominal - manejo.

Argumento: La condición de este paciente a menudo es secundaria a una reanimación con volumen masivo (postoperatorios, shock hipovolémico, quemaduras graves). La descompresión abdominal por laparotomía con cierre temporal alivia la presión intraabdominal (PIA) del síndrome compartimental abdominal con signos de insuficiencia orgánica progresiva (lesión renal aguda). El manejo médico (paracentesis, control del dolor, descompresión por sonda nasogástrica) y controles de la PIA siguen a la descompresión quirúrgica y deben continuar hasta PIA <12 mmHg. Una cirugía prolongada que requiere abundantes LEV y transfusiones producen un aumento de la PIA secundario a la acumulación de líquido tisular. El aumento de la PIA provoca un abdomen distendido y conduce al síndrome compartimental abdominal si la presión continúa aumentando (≥ 20 mmHg). Esto baja la precarga, reduciendo el gasto cardíaco (hipotensión, taquicardia, lesión prerrenal). En algunos casos, la PIA elevada se extiende hacia el compartimento torácico, lo que provoca distensión venosa yugular.

  • El plasma fresco congelado se utiliza en coagulopatía grave con hemorragia significativa. Este paciente tiene riesgo de coagulopatía (trauma, transfusión masiva), está inestable, con tensión abdominal, lo que podría resultar de un sangrado. Sin embargo, la hemorragia causa depleción del volumen (presión venosa yugular baja en lugar de distensión).
  • La epinefrina intramuscular es el tratamiento médico de emergencia más importante para la anafilaxia, que generalmente surge muy poco después de la administración del agente causante (minutos a horas) y se asocia con urticaria y/o angioedema. El shock anafiláctico causa vasodilatación sistémica que conduce a una presión venosa yugular baja. Además, la anafilaxia no explicaría el abdomen distendido de este paciente.
  • Los antibióticos están indicados para la infección bacteriana (sepsis), para la cual este paciente tiene un alto riesgo. La infección normalmente requiere días para progresar a sepsis, en lugar de horas. La hipotensión en la sepsis se debe a la vasodilatación inducida por toxinas bacterianas, que causaría una presión venosa yugular baja.
183
Q
  1. Hombre, 34 años, sufre accidente ofídico en pierna derecha. Hay edema, equimosis, ampollas y dolor a la dorsiflexión del pie; recibe antídoto y realizan fasciotomía. A las 2 sem: dificultad al caminar, pérdida de sensibilidad infero-lateral de pierna derecha y dorso del pie, eversión del pie 1/5. Sin debilidad en la dorsiflexión. ¿Cuál es el nervio lesionado?
    A. Nervio tibial.
    B. Nervio peroneo profundo.
    C. Nervio peroneo superficial.
    D. Nervio sural.
A

Respuesta correcta: C

Tema e ítem: Lesión del nervio peroneo superficial - diagnóstico.

Argumento: Los síntomas de este paciente probablemente se deben a una lesión intraoperatoria del nervio que inerva los músculos peroneo largo y corto. El nervio peroneo superficial (NPS)suministra inervación motora al compartimento lateral de la pierna, cuyas funciones principales son la eversión del pie y la flexión plantar. La lesión de este nervio compromete la eversión del pie pero no afecta la dorsiflexión, producida por el compartimento anterior de la pierna (inervado por el peroneo profundo). El NPS inerva sensorialmente la parte distal de la cara anterolateral de la pierna y al dorso del pie, por tanto, su lesión lleva a pérdida de la sensibilidad local de esa zona. Los factores de riesgo del síndrome compartimental incluyen accidentes ofídicos, quemaduras circunferenciales, fracturas de huesos largos y lesiones por aplastamiento.

  • Una lesión del nervio tibial provoca un deterioro de la función motora de los músculos flexores del pie (incapacidad para invertir el pie o caminar en puntas), nada de lo cual se ve aquí. Los pacientes también experimentan pérdida sensorial en la planta del pie, no en el dorso del pie. Además, es poco probable que se produzcan lesiones durante una fasciotomía lateral porque el nervio tibial se encuentra en el compartimento posterior profundo de la pierna.
  • El nervio sural es una rama puramente sensorial del nervio tibial, su lesión provoca alteración de la sensación en el lado posterolateral de la parte inferior de la pierna, el tobillo, el talón y el pie, pero no en el lado anterolateral de la pierna y el dorso del pie. Además, los pacientes no presentan hallazgos motores. Finalmente, es poco probable que se produzca una lesión de este nervio durante la fasciotomía lateral dada su ubicación en la cara posterior de la pierna.
  • Una lesión profunda del nervio peroneo provoca un deterioro de la función motora del pie y de los extensores de los dedos, así como una disminución de la sensación en el espacio interdigital entre el hallux y el segundo dedo del pie. Los pacientes presentan dificultades con la dorsiflexión y la eversión (marcha en Steppage). El paciente no tiene debilidad en la dorsiflexión. Además, es poco probable que se produzcan lesiones durante una fasciotomía lateral porque el nervio peroneo profundo está en el compartimento anterior de la pierna.
184
Q
  1. Masculino de 8 años es llevado a urgencias porque fue mordido por el perro de su vecino hace 3 horas. El niño perseguía al perro con un palo cuando este lo atacó. El paciente ha alimentado al perro antes y nunca lo había mordido. Su padre vio al perro 2 horas después del incidente y su comportamiento era normal. Nunca ha recibido vacuna antirrábica. Al examen físico hay una herida de 2 cm en pantorrilla izquierda con eritema mínimo alrededor. El resto del examen físico es normal. La herida se irriga y se lava con solución salina y clorhexidina. ¿Cuál es el manejo adecuado en este caso?
    A. Observar al perro durante 10 días.
    B. Realizar eutanasia al perro y realizar pruebas para rabia.
    C. Administrar vacuna antirrábica e inmunoglobulina.
    D. Administrar vacuna contra la rabia.
A

Respuesta correcta: A

Tema e ítem: Clasificar y manejar la mordedura por animales potencialmente transmisores de rabia.

Argumento: Posterior a la mordedura de un animal se debe evaluar la necesidad de profilaxis posexposición contra la rabia. Después de la mordedura provocada por un animal doméstico no inmunizado que no muestre síntomas de rabia, un periodo de observación del animal de 10 días es el mejor paso a seguir para evaluar el riesgo de rabia. Debido a que la rabia tiene un periodo de incubación prolongado (Generalmente de 1 a 3 meses) y los animales que pueden transmitir la rabia desarrollan los síntomas dentro de 5 a 10 días, es seguro posponer la profilaxis posexposición para observar al animal. Si el perro mantiene un comportamiento normal durante la observación, no es necesaria la profilaxis. Si el perro desarrolla síntomas de rabia se debe sacrificar, se deben realizar pruebas en el animal e iniciar manejo en el paciente. Si las pruebas en el perro son negativas se suspende la profilaxis. Una excepción importante es una herida por mordedura en cabeza o cuello ya que hay informes en los que la infección por rabia se produje de los 4 días posteriores a la exposición (Por su cercanía al SNC)

  • La eutanasia al perro y realización de pruebas de rabia solo se encuentran indicadas en caso de que el animal muestre signos de rabia.
  • La vacuna y la inmunoglobulina antirrábica son el método usado para personas que no se han vacunado previamente. Sin embargo, en este paciente no vacunado que ha sido mordido por un animal observable que se encuentra disponible para observación no se encuentra indicado dicho manejo.
  • La administración de vacuna contra la rabia se puede considerar en pacientes previamente vacunados con un alto riesgo de exposición. Este no es el caso.
185
Q
  1. Masculino de 37 años consulta por sensación de ardor en el pecho desde hace 1 mes de evolución. La sensación es más prominente cuando se encuentra acostado en la cama y posterior a comidas copiosas. Afirma un aliento desagradable en las mañanas. Su padre tuvo CA de colon y su madre tiene HTA. Su IMC es de 27 kg/m2. Signos vitales son normales. El examen físico es normal. Un hemograma es normal. ¿Cuál es el manejo adecuado?
    A. Realizar una endoscopia de vías digestivas altas.
    B. Realizar una prueba de aliento con urea.
    C. Iniciar terapia con inhibidores de bomba de protones.
    D. Realizar una prueba de trago de bario.
A

Respuesta correcta: C

Tema e ítem: Manejo de la enfermedad por reflujo gastroesofágico.

Argumento: El cuadro clínico de este paciente sugieren una enfermedad por reflujo gastroesofágico. En este caso el manejo empirico con IBP es el manejo inicial más adecuado para la enfermedad por reflujo gastroesofágico de aparición reciente sin síntomas de alarma: Disfagia, odinofagia, anemia, hemorragia gastrointestinal, perdida de peso involuntaria, vómitos persistentes, neumonía por aspiración; y pocos o ningún factor para esófago de Barret. De todos los medicamentos que se usan para la enfermedad por reflujo gastroesofágico, los inhibidores de la bomba de protones se asocian con la mayor reducción de síntomas y tasas de recaída. Además de la prueba de IBP durante 8 semanas se deben realizar modificaciones en el estilo de vida: dejar de fumar, perder peso, elevar la cabecera de la cama mientras duerme, evitar comidas nocturnas.

  • El examen endoscópico se realiza con el objetivo de descartar adenocarcinoma en pacientes con enfermedad por reflujo gastroesofágico y signos de alarma. También en casos para detectar esófago de Barret en paciente con más de 5 años de síntomas o más de 50 años de edad, obesidad, antecedente de tabaquismo o familiares de esófago de Barret o adenocarcinoma de esófago.
  • La prueba de infección por H. pylori mediante la prueba de urea en aliento no se encuentra indicada en pacientes con enfermedad por reflujo gastroesofágico porque su prevalencia de infección por H. pylori no es mayor que la población general. La detección se encuentra indicada en casos de úlcera péptica, gastritis atrófica o linfomas MALT.
  • La prueba de bario se usa para confirmar diverticulosis esofágica que puede presentar síntomas similares. Sin embargo, esta condición se presenta en pacientes mayores de 60 años. Además de que los pacientes presentan disfagia y regurgitación de alimentos no digeridos.
186
Q
  1. Masculino de 35 años es llevado a urgencias 40 minutos luego de derramar aceite caliente sobre él en un accidente de concina. Al examen físico hay múltiples ampollas tensas sobre el abdomen, la parte anterior del tórax y toda la pierna derecha. Al quitar el recubrimiento de las ampollas se evidencia una piel subyacente sensible y moteada que no palidece con la presión. Al examen físico del muslo izquierdo la piel es sensible y eritematosa con llenado capilar rápido a la digitopresión. ¿Cuál es la estimación más cercana de superficie corporal total afectada por quemaduras de segundo grado en este paciente?:
    A. 9%
    B. 18%
    C. 45%
    D. 36%
A

Respuesta correcta: D

Tema e ítem: Quemaduras – calcular el porcentaje de superficie corporal quemada.

Argumento: La regla del 9 divide el cuerpo en 11 regiones: cabeza, parte anterior del tórax, abdomen, parte superior de la espalda, parte inferior de la espalda, brazo derecho, brazo izquierdo, muslo derecho, muslo izquierdo, parte inferior de la pierna derecha, parte inferior de la pierna izquierda. Cada una de las cuales se estima ser el 9% de la superficie corporal total. Este paciente tiene quemaduras profundas de segundo grado (Caracterizadas por ampollas y piel subyacente que no palidece) en pecho, abdomen y toda la pierna derecha, lo que corresponde a un 36% de la superficie corporal total. También presenta una quemadura de primer grado en muslo izquierdo.

187
Q

1.Paciente de 10 años, con antecedentes de anemia de células falciformes, en seguimiento con hematología. Hace un año, requirió colecistectomía, sin complicaciones. En casa, ha presentado crisis de dolor que se han controlado de forma adecuada con dipirona. Hace dos años no acudía a urgencias por este motivo hasta hoy, que consulta por seis horas de evolución de dolor en región proximal de muslos y en hombros de forma bilateral. Al examen físico, taquicárdico, con resto de signos vitales normales, sin signos inflamatorios ni dolor a la palpación en extremidades. En paraclínicos: Hb 11.5, GB 17500, N 13000, LDH 700, BT 2, BD 0.6. ¿Cuál considera que es el tratamiento más adecuado en este paciente?
A. Iniciar hiperhidratación a 1.5-2 veces el mantenimiento y manejo del dolor con opioides
B. Iniciar 3 bolos de 20 cc/kg y luego continuar con el mantenimiento y manejo del dolor
C. Tomar PCR, hemocultivar, hiperhidratar y manejar el dolor
D. Iniciar antimicrobiano, hiperhidratar y manejar el dolor

A

Respuesta correcta: A

Tema-Ítem: Anemia de células falciformes – Crisis vasooclusiva

Argumentación: Este paciente tiene anemia de células falciformes y presenta una crisis vasooclusiva o dolorosa, que no mejoró con dipirona en casa. Está taquicárdico muy seguramente por el dolor, no reporta otra sintomatología que nos haga sospechar una causa infecciosa. La leucocitosis posiblemente sea debido a inflamación y no secundario a alguna infección.

La anemia de células falciformes (ACF) es una enfermedad que se establece a partir de una mutación en el gen de la beta globina secundario a un cambio durante la transcripción genética, que genera la codificación de valina en lugar de ácido glutámico. El cambio conformacional de la HbS, hace que en casos de estrés oxidativo (desoxigenación, acidosis (pH bajo), deshidratación, altas concentraciones de HbS, entre otros) se genere una polimerización de la hemoglobina que da lugar a glóbulos rojos con forma drepanocítica (forma de hoz), los cuales son menos deformables y tienen tendencia a la agregación. Los cambios en la estructura de la membrana, asociados a un aumento de la viscosidad sanguínea, adherencia al endotelio vascular y una mayor activación de leucocitos y plaquetas, genera fenómenos vaso oclusivos y la consecuente isquemia, lo que clínicamente se traduce en las diferentes manifestaciones del paciente.

Las crisis vaso oclusivas son la causa más frecuente de consulta al servicio de urgencias en esta población; puede ocurrir a cualquier edad y suele presentarse como el debut de la enfermedad. Aproximadamente el 33% de los pacientes con enfermedad falciforme ha presentado por lo menos una crisis dolorosa al año de edad, 66% a los 2 años y hasta el 90% a los 6 años. La mayoría de los episodios no tienen un desencadenante claro. Pueden afectar cualquier parte del cuerpo, aunque las regiones más frecuentemente comprometidas son el pecho, la espalda y las extremidades. Pueden presentar o no cambios como eritema, dolor o edema.

Es fundamental la hiperhidratación, calculada para 1.5 veces el mantenimiento basal y sin excederlo en más de 2 veces. En las crisis dolorosas, lo más importante es la analgesia adecuada que se debe ajustar según la gravedad del episodio doloroso. En caso de no haber recibido manejo para el dolor en casa y si este es leve o moderado, se puede iniciar manejo con AINES (ibuprofeno, naproxeno, diclofenaco) o con opioides orales en urgencias; para crisis dolorosas graves se pueden adicionar opioides como oxicodona, morfina e hidromorfona al manejo establecido. Los corticoides se reservan para casos graves; se propone la administración de metilprednisolona a dosis de 15 mg/kg/día por 2 a 3 días con un desmonte lento.

188
Q
  1. Luis tiene 6 años. Es llevado a urgencias por su papá debido a cuadro clínico de cinco días de evolución de edema facial. Hace dos semanas tuvo odinofagia que se resolvió espontáneamente. Al examen físico, taquicárdico y afebril, con edema grado II con fóvea en extremidades y edema periorbitario. El abdomen está levemente distendido. En paraclínicos: Hb 13.1, Albúmina 2.1, CT 290, TG 210. Uroanálisis con proteínas ++++. ¿Cuál de los siguientes hallazgos es más probable que muestre una biopsia renal de este paciente?
    A. Borramiento de los procesos podocitarios en microscopía óptica.
    B. Depósitos subepiteliales granulares de IgG, IgM y C3 en inmunofluorescencia.
    C. Hallazgos normales en microscopía óptica.
    D. Proliferación mesangial en microscopía óptica.
A

Respuesta correcta: C

Tema – Ítem: Urgencias nefrológicas en pediatría - Síndrome nefrótico

Argumentación: Este paciente tiene seis años. La proteinuria elevada, la hipoalbuminemia, el edema y la hiperlipidemia son indicativos de síndrome nefrótico.

Lo más probable es que este paciente tenga una enfermedad de cambios mínimos (ECM), que se manifiesta sin cambios visibles en el microscopio óptico. En esta condición, las citocinas circulantes pueden provocar el borramiento de los podocitos, lo que sólo es visible con microscopía electrónica. Esta es la causa más común de síndrome nefrótico en niños (especialmente <10 años de edad). Puede ser desencadenado por una infección reciente (como se ve en este caso) o por una vacuna. La ECM responde muy bien a los corticoides y tiene buen pronóstico, por lo que la biopsia renal generalmente no está indicada para el diagnóstico.
La proteinuria, el edema y la hipoalbuminemia sugieren síndrome nefrótico, siendo la enfermedad de cambios mínimos la causa más probable en los niños. Si bien la enfermedad de cambios mínimos se caracteriza por el borramiento de las apófisis de los podocitos del pie , esto sólo puede observarse mediante microscopía electrónica . El borramiento de los procesos de los podocitos del pie también se observa en la glomeruloesclerosis focal y segmentaria , que puede causar síndrome nefrótico , pero es poco común en los niños.
La proliferación mesangial se observa con microscopía óptica en la nefropatía por IgA, que es la causa más común de glomerulonefritis en adultos. Los hallazgos adicionales incluyen depósitos mesangiales de IgA en la inmunofluorescencia y de complejos inmunes en la microscopía electrónica. Generalmente se manifiesta con episodios únicos o recurrentes de hematuria y sedimento nefrítico en lugar de nefrótico, durante o inmediatamente después de una infección.

En la glomerulonefritis postestreptocócica (GNPE) se observan depósitos granulares subepiteliales de IgG, IgM y C3 en la inmunofluorescencia. Aunque la GNPE puede manifestarse con edema entre 10 y 30 días después de una infección por S. pyogenes (la causa potencial del dolor de garganta de este paciente), generalmente se manifiesta con sedimento nefrítico en lugar de sedimento nefrótico.

189
Q
  1. Paciente de 4 años. Es llevada por su abuela al servicio de urgencias del primer nivel de atención donde usted labora, por quemadura accidental con líquido caliente hace aproximadamente 10 horas. Refiere que se encontraba una olla con agua hirviendo en el fogón de la casa, sintió un estruendo y seguido de esto la paciente empezó a gritar y a llorar; al asomarse a la cocina la encuentra mojada y la olla a los pies de ella, vacía. Inmediatamente la llevó a la ducha con agua fría y le aplicó café, pero empezó a verle ampollas, por lo que decide traerla. Al examen físico taquicárdica, irritable, con quemaduras grado II en tronco anterior, extremidades superiores, muslos y cuello. ¿Cuál es la conducta a seguir con esta paciente?
    A. Remisión a un tercer nivel de complejidad
    B. Iniciar reanimación con bolos de líquidos de cristaloides
    C. Limpieza de herida con agua y jabón, quitar el tejido desvitalizado con gasa.
    D. Empezar reanimación con líquidos, antibiótico profiláctico y remisión a tercer nivel
A

Respuesta correcta: A.

Tema-Ítem: Manejo del paciente pediátrico quemado

Argumentación: Esta paciente de 4 años, presentó al parecer un trauma accidental con líquido caliente. Tiene quemaduras grado II con compromiso de >50% de la superficie corporal, por lo que requiere traslado primario a un centro especializado, previo a esto con una reanimación adecuada de líquidos.

Las quemaduras son una lesión traumática sobre la piel u otros tejidos por la acción de agentes físicos, químicos o biológicos, que producen alteraciones que varían desde el eritema hasta la destrucción de tejidos profundos. Más del 80% son prevenibles, el 90% son por accidentes caseros. Las zonas más afectadas son miembros superiores, cabeza y cuello. Para iniciar el manejo debemos determinar el área de superficie corporal afectada, esto se usa solo para quemaduras de segundo y tercer grado. Tenemos varios métodos como: Regla de los nueve (Wallace), método palmar y la tabla de Lund y Browder. En los niños, la más fiable es esta última porque tiene en cuenta tamaños corporales.
Si el paciente cumple con alguno de los siguientes criterios, debo remitir a un centro especializado:
○ Segundo grado >10% ASCq (área superficie corporal quemada)
○ Tercer grado >5% ASCq
○ Áreas especiales: cara, mano, pies, genitales, periné, articulaciones principales. Las quemaduras en áreas especiales siempre requieren remisión y hospitalización
○ Eléctricas o de alto voltaje (rayos y torres eléctricas)
○ Lesión por inhalación
○ Trauma asociado
○ Comorbilidades
● Circulares
● Quemaduras químicas
● Menores de 2 años
● Patología de base que puede complicar el manejo, prolongar la recuperación o afectar la mortalidad (DM, inmunosupresión)
● Sospecha de maltrato infantil
● Beneficios claros de manejo intrahospitalario (ej. Pobre acceso a servicios de salud, cuidadores poco comprometidos, etc.)

Si el paciente tiene un ASCq >15% o no tolera la vía oral, debemos iniciar manejo con líquidos IV. Los bolos de 10 a 20 cc/kg de Hartman o SS 0,9%, solo deben usarse si el paciente está en choque. Si se pueden evitar mejor, porque aumentan el edema extravascular.
La fórmula de Parkland es la más común pero la menos recomendada porque toma a todos los niños como si fueran iguales. Existen otras fórmulas recomendadas en niños, como Galveston o Cincinnati.
Recordemos que los antimicrobianos profilácticos NO están recomendados en el paciente quemado, solo iniciamos antimicrobiano cuando hay signos o síntomas compatibles con infección.

190
Q
  1. Paciente de 14 años, previamente sano, llega a urgencias por debilidad rápidamente progresiva en extremidades inferiores, acompañado de incapacidad para caminar desde esta mañana. Informa que ayer sintió un hormigueo en los pies antes de que comenzara la debilidad. Hace 2 semanas presentó fiebre, rinorrea hialina y tos, lo cual ya se resolvió. Al examen físico, se evidencia hipoestesia simétrica en piernas y ausencia de reflejos aquilianos y rotulianos. ¿Cuál de los siguientes hallazgos es más probable que muestre el citoquímico de líquido cefalorraquídeo en este paciente?
    A. Presión de apertura 120 mm H2O, Glucosa 50 mg/dL, Proteínas 50 mg/dL y GB 400/mm3
    B. Presión de apertura 120 mm H2O, Glucosa 50 mg/dL, Proteínas 550 mg/dL y GB 3/mm3
    C. Presión de apertura 350 mm H2O, Glucosa 20 mg/dL, Proteínas 50 mg/dL y GB 25/mm3
    D. Presión de apertura 350 mm H2O, Glucosa 50 mg/dL, Proteínas 550 mg/dL y GB 20/mm3
A

Respuesta correcta: B.

Tema-Ítem: Polineuropatías – Síndrome de Guillain Barré

Argumentación: El desarrollo de parálisis rápidamente progresiva con parestesias periféricas simétricas y ausencia de reflejos musculares 2 semanas después de una infección del tracto respiratorio superior es altamente sugestivo de síndrome del Guillain-Barré.

Este análisis del LCR muestra un gran aumento de proteínas sin un aumento proporcional en el recuento de leucocitos (<10), una condición llamada disociación albuminocitológica, que resulta de una disfunción de la barrera hematoencefálica y es un hallazgo característico del SGB. Los síntomas de este paciente son causados por una reacción autoinmune postinfecciosa que genera anticuerpos de reacción cruzada contra gangliósidos (p. ej., anticuerpos anti-GM1) u otros antígenos desconocidos de las células de Schwann periféricas, lo que conduce a una desmielinización segmentaria y degeneración axonal.
El síndrome de Guillain-Barré (SGB) es una polineuropatía aguda inmunomediada que típicamente se manifiesta con parálisis flácida ascendente bilateral y afectación sensorial, por ejemplo, parestesias (el hormigueo referido por el paciente). La patogénesis del SGB implica autoanticuerpos contra antígenos de la vaina de mielina, otros antígenos de las células de Schwann y las membranas de los axones. Aproximadamente el 65% de los pacientes tienen una infección del tracto respiratorio superior o gastrointestinal antes de la aparición de los síntomas. El diagnóstico es principalmente clínico. Se deben considerar diagnósticos alternativos, especialmente en pacientes con presentaciones atípicas. El análisis del líquido cefalorraquídeo (LCR) que muestra la disociación albuminocitológica y los hallazgos del estudio electrodiagnóstico ayudan a respaldar el diagnóstico. El tratamiento médico depende de la gravedad de los síntomas: Las opciones de tratamiento disponibles son inmunoglobulina intravenosa (IgIV) y plasmaféresis. Se requiere una estrecha vigilancia de todos los pacientes, pues pueden ocurrir complicaciones como insuficiencia respiratoria, embolia pulmonar y/o paro cardíaco aumentan la mortalidad. Hasta el 20% de los pacientes quedan gravemente discapacitados y aproximadamente el 5% de los pacientes mueren a pesar del tratamiento médico. Hasta el 70% de los pacientes tienen un buen pronóstico y los síntomas se resuelven en el orden inverso a su desarrollo.

191
Q
  1. Paciente de 13 años, llevado a su consulta por la madre por aparición de brote indoloro y no pruriginoso en la cara hace 5 días. Su abuela materna, de 62 años, tiene penfigoide ampolloso. No tiene antecedentes patológicos. Al examen físico, afebril, sin alteraciones en signos vitales, con evidencia de costras de color miel en el mentón. Sin otras lesiones. El resto del examen no muestra anomalías. ¿Cuál sería el manejo más adecuado para este paciente?
    A. Clindamicina oral
    B. Cefalexina oral
    C. Mupirocina tópica
    D. Confirmar con biopsia de piel
A

Respuesta correcta: C.

Tema – Ítem: Infecciones en piel – Impétigo

Argumentación: Este paciente con costras indoloras, no pruriginosas y de color miel o melicéricas (sin vesículas) ubicadas en el mentón son consistentes con impétigo no ampolloso, que generalmente se debe a una infección por Staphylococcus aureus y, a veces, por Estreptococos del grupo A.
El impétigo no ampolloso es el tipo más común de impétigo y se manifiesta con lesiones papulares y/o pustulosas con supuración serosa que se secan y forman costras características de color miel. Aunque estas lesiones suelen aparecer en la cara, también pueden afectar las extremidades. El impétigo no ampolloso suele ser un diagnóstico clínico. Puede ser necesario un cultivo de piel en pacientes con infección persistente y recurrente o si el diagnóstico no es concluyente. Se utilizan antibióticos tópicos como mupirocina en casos leves. En pacientes con múltiples lesiones o ectima, pueden ser necesarios antibióticos sistémicos (cefalexina, dicloxacilina). Los pacientes con ectima presentan costras secas y amontonadas que están muy adheridas a la piel en lugar de costras llenas de pus de color miel. En el caso del ectima, las extremidades se ven afectadas con mayor frecuencia que la cara.
Algunos factores predisponentes a esta infección, son:
-Clima cálido y húmedo
-Condiciones de vida hacinadas e insalubres
-Mala higiene personal
-Condiciones médicas
-Lesiones cutáneas preexistentes ( p. ej., dermatitis atópica, sarna, picaduras de insectos, abrasiones, eccema)
-Diabetes mellitus
-Inmunodeficiencia (p. ej., VIH , postrasplante de órganos, corticosteroides sistémicos)

192
Q

Doña Ana de 51 años, consulta al servicio de Urgencias por cuadro clínico de 2 días de evolución dado por dolor en hipocondrio derecho, fiebre e ictericia. Refiere náuseas y vómitos. Al ingreso se encuentra en regular estado general, T 38.5°C, FC 95 lpm, FR 19 rpm, PA 110/85 mhg, ictericia conjuntival, dolor a la palpación en cuadrante superior derecho, sin signos de irritación peritoneal. Analítica: Hb 12 g/dL, leucocitos 11.800, plaquetas 250.000, BUN 30 mg/dL, albumina 3.7 Cr 1.5 mg/dL, ALT/AST: 200/205 U/L, FA 450 U/L, BT 3.5 mg/dL, BD 2.5 mg/dL. La ecografía de hígado y vías biliares muestra dilatación de los conductos biliares intra y extrahepáticos y múltiples cálculos en vesícula biliar.

  1. ¿Cuál debió ser la conducta terapéutica inicial adecuada en esta paciente?
    A. Suspender la vía oral, administrar líquidos endovenosos, analgesia e iniciar ceftriaxona + metronidazol
    B. Ordenar analgésicos, antibioticos orales y dar de alta con instrucciones de consultar ante presencia de signos de alarma
    C. Dar de alta y manejo ambulatorio con antibiótico oral y analgésicos
    D. Suspender la vía oral, administrar líquidos endovenosos, analgesia, piperacilina/tazobactam y programar para colecistectomía en las siguientes 12 horas
A

(A)

Estamos indudablemente frente a un caso de colangitis aguda (CA), caracterizado por la triada de Charcot: fiebre, ictericia y dolor abdominal en hipocondrio derecho. Los marcadores de colestasis como la fosfatasa alcalina/hiperbilirrubinemia a expensas de la directa y los leucocitos refuerzan el diagnostico. Además, la ecografía muestra una dilatación biliar intra y extrahepática que sugiere indudablemente una colangitis ascendente aguda.

El abordaje de los pacientes con CA inicia con una evaluación de la gravedad de la enfermedad. Este paciente presenta actualmente un cuadro leve ya que no tiene criterios de disfunción orgánica que lo clasifiquen como grave o criterios moderados (leucocitos >12.000/mm3 o <4.000/mm, fiebre 39°C, edad ≥75 años, bilirrubina total ≥5 mg/dL, hipoalbuminemia).

En casos leves, la mayoría de los pacientes responden al tratamiento médico. El manejo inicial en el servicio de Urgencias implica evaluar el ABC, monitoreo cardíaco y oximetría de pulso, obtener un acceso intravenoso, iniciar líquidos endovenosos guiados por metas, corrección de alteraciones electrolíticas según corresponda, y cuidados de apoyo. Además, se requiere el inicio empírico de antibióticos intravenosos que se sabe que alcanzan altas concentraciones biliares siendo la piperacilina/tazobactam en monoterapia o el biconjugado ceftriaxona + metronidazol los esquemas recomendados por las directrices. La colangiopancreatografía retrógrada endoscópica (CPRE) es un procedimiento electivo que se realiza 24-48 horas después de la mejoría clínica.

En los pacientes que no responden al manejo inicial como es el caso de doña Ana que presenta un deterioro a las 24 horas o pacientes que al ingreso presentan criterios de colangitis grave, la CPRE debe realizarse de forma emergente. Si la CPRE no es posible en esta paciente, el siguiente paso sería el drenaje biliar transhepático percutáneo, ya que drenaría de forma más fiable todo el árbol biliar.

193
Q

Doña Ana de 51 años, consulta al servicio de Urgencias por cuadro clínico de 2 días de evolución dado por dolor en hipocondrio derecho, fiebre e ictericia. Refiere náuseas y vómitos. Al ingreso se encuentra en regular estado general, T 38.5°C, FC 95 lpm, FR 19 rpm, PA 110/85 mhg, ictericia conjuntival, dolor a la palpación en cuadrante superior derecho, sin signos de irritación peritoneal. Analítica: Hb 12 g/dL, leucocitos 11.800, plaquetas 250.000, BUN 30 mg/dL, albumina 3.7 Cr 1.5 mg/dL, ALT/AST: 200/205 U/L, FA 450 U/L, BT 3.5 mg/dL, BD 2.5 mg/dL. La ecografía de hígado y vías biliares muestra dilatación de los conductos biliares intra y extrahepáticos y múltiples cálculos en vesícula biliar.

  1. Veinte cuatro horas más tarde, doña Ana presenta en su evolución un aspecto tóxico y se encuentra desorientada en las tres esferas. Al examen físico presenta T 39.2°C, FC 120 lpm, FR 25 rpm, PA 95/78 mh.
    ¿Cuál sería la conducta adecuada a seguir?
    A. Realizar TAC simple de abdomen
    B. Realizar laparotomía en forma emergente
    C. Realizar CPRE en forma emergente
    D. Realizar colecistectomía percutánea emergente
A

(C)

Estamos indudablemente frente a un caso de colangitis aguda (CA), caracterizado por la triada de Charcot: fiebre, ictericia y dolor abdominal en hipocondrio derecho. Los marcadores de colestasis como la fosfatasa alcalina/hiperbilirrubinemia a expensas de la directa y los leucocitos refuerzan el diagnostico. Además, la ecografía muestra una dilatación biliar intra y extrahepática que sugiere indudablemente una colangitis ascendente aguda.

El abordaje de los pacientes con CA inicia con una evaluación de la gravedad de la enfermedad. Este paciente presenta actualmente un cuadro leve ya que no tiene criterios de disfunción orgánica que lo clasifiquen como grave o criterios moderados (leucocitos >12.000/mm3 o <4.000/mm, fiebre 39°C, edad ≥75 años, bilirrubina total ≥5 mg/dL, hipoalbuminemia).

En casos leves, la mayoría de los pacientes responden al tratamiento médico. El manejo inicial en el servicio de Urgencias implica evaluar el ABC, monitoreo cardíaco y oximetría de pulso, obtener un acceso intravenoso, iniciar líquidos endovenosos guiados por metas, corrección de alteraciones electrolíticas según corresponda, y cuidados de apoyo. Además, se requiere el inicio empírico de antibióticos intravenosos que se sabe que alcanzan altas concentraciones biliares siendo la piperacilina/tazobactam en monoterapia o el biconjugado ceftriaxona + metronidazol los esquemas recomendados por las directrices. La colangiopancreatografía retrógrada endoscópica (CPRE) es un procedimiento electivo que se realiza 24-48 horas después de la mejoría clínica.

En los pacientes que no responden al manejo inicial como es el caso de doña Ana que presenta un deterioro a las 24 horas o pacientes que al ingreso presentan criterios de colangitis grave, la CPRE debe realizarse de forma emergente. Si la CPRE no es posible en esta paciente, el siguiente paso sería el drenaje biliar transhepático percutáneo, ya que drenaría de forma más fiable todo el árbol biliar.

194
Q
  1. Niño de 3 años, previamente sano, residente en el área de los Montes de María, es llevado al servicio de Urgencias por cuadro clínico de 6 días de fiebre, siendo el mayor valor cuantificado en 39.5°C. Al examen físico se encuentra estable hemodinámicamente, T 39.4°C, FC 115 lpm, FR 20 rpm, rash maculopapular eritematoso generalizado, lengua aframbuesada e hiperemia conjuntival no purulenta bilateral, sin otros hallazgos a la exploración.
    ¿Cuál sería la conducta a seguir en este paciente?
    A. Solicitar PCR/VSG, hemograma con recuento de plaquetas, albumina, ALT, fosfatasa alcalina, uroanálisis y ecocardiograma e iniciar inmunoglobulina IV 2 g/kg + ácido acetilsalicílico 80 mg/kg
    B. Iniciar inmunoglobulina IV 2 g/kg + ácido acetilsalicílico 80 mg/kg
    C. Solicitar PCR/VSG, hemograma con recuento de plaquetas, albumina, ALT, uroanálisis y ecocardiograma
    D. Iniciar inmunoglobulina IV 2 g/kg + ácido acetilsalicílico 80 mg/kg + metilprednisolona IV 30 mg/kg
A

Respuesta correcta: C

Tema e ítem: Enfermedad de Kawasaki – Presentación clínica incompleta/incompleta

Argumento: Primero que todo hay que tener en cuenta que la enfermedad de Kawasaki (EK) puede tener una presentación incompleta y que los hallazgos clínicos NO siempre son concurrentes en el tiempo. Se denomina EK incompleta cuando se cumple el criterio principal de fiebre persistente, junto con otros criterios clínicos que apoyan el diagnóstico, habiendo excluido otras enfermedades que pueden simular dicho proceso. Este paciente con fiebre de más de 5 de duración tiene 3 de los 5 criterios mayores para EK:
(1) Exantema polimorfo (rash maculopapular eritematoso)
(2) Cambios en la mucosa oral (lengua aframbuesada)
(3) Hiperemia conjuntival bilateral no exudativa

Estos hallazgos sugieren fuertemente la variante de la enfermedad llamada EK incompleta. Para esta situación, el colegio americano de Cardiología (AHA) ha desarrollado un flujograma que incluye estudios de laboratorio y un ecocardiograma para hacer el diagnóstico.

Los estudios de laboratorio recomendados por la AHA incluyen: PCR-VSG, hemograma con recuento de plaquetas, albumina sérica, enzimas hepáticas (específicamente ALT) y uroanálisis. Se realiza el diagnostico de EK incompleta si se encuentra una PCR ≥ 3mg/dL y/o VSG ≥ 40 mm/hr con ≥ 3 de los siguientes hallazgos: anemia para la edad del paciente, plaquetas ≥ 450.000 después del día 7 de fiebre, albumina ≤ 3 g/dL, ALT elevada, leucocitosis ≥ 15.000/mm, leucocituria ≥ 10 por campo o si el ecocardiograma es patológico y se inicia de forma inmediata tratamiento con inmunoglobulina IV + ácido acetilsalicílico para reducir el riesgo de desarrollar anomalías coronarias.

195
Q
  1. Varón de 31 años, Procedente de Chalán-Sucre, consulta al servicio de urgencias de segundo nivel por cuadro clínico de 3 días de evolución consistente en fiebre alta, escalofríos, artralgias, mialgias y cefalea global. Al examen físico presenta T 39.2°C y exantema maculopapular difuso. Analítica: Hb 12 g/dL, leucocitos 3.100/m3, plaquetas 98.000/m3, ALT 105 U/L. La gota gruesa fue negativa y el extendido de sangre periférico no reporto hallazgos anormales.
    ¿Cuál de las siguientes es la causa más probable de su cuadro clínico?
    A. Virus Chikungunya
    B. Virus Zika
    C. Virus Dengue
    D. Virus Epstein-Barr
A

Respuesta correcta: C

Tema e ítem: Arbovirosis – Infección por virus Dengue

Argumento: Se trata indudablemente de un cuadro clínico compatible con dengue: fiebre alta, escalofríos, artromialgias, paraclínicos que reportan leucopenia y trombocitopenia y residencia en un área endémica como lo es Chalán-Sucre.

El dengue es una enfermedad febril causada por uno de los cuatro virus dengue (DENV) transmitidos por el mosquito Aedes aegypti. Los cuatro serotipos circulan en el país. Las infecciones son asintomáticas hasta en el 75% de los casos. El diagnóstico de dengue debe sospecharse en individuos con fiebre y manifestaciones clínicas típicas (fiebre, cefalea global/retroorbitaria, mialgia, artralgia, exantema, manifestaciones, prueba de torniquete positiva, leucopenia) y nexo epidemiológico importante.

Durante la primera semana de la enfermedad, el diagnóstico se puede establecer mediante aislamiento viral donde la muestra debe ser recolectada hasta el quinto día de inicio de síntomas o mediante la detección del antígeno de proteínas no estructurales del virus dengue (NS1). La sensibilidad de la detección de NS1 puede exceder el 90% y la antigenemia puede persistir durante varios días después de la resolución de la fiebre. Por otro lado, la serología es utilizada para la detección de anticuerpos antidengue (IgM) y debe ser solicitada a partir del sexto día de inicio de síntomas (ELISA).

196
Q
  1. Un estudio epidemiológico intenta establecer la relación causal entre el consumo de metimazol durante el segundo trimestre del embarazo y el riesgo de desarrollar cierre atresia de coanas (AC). En el estudio se seleccionaron madres de recién nacidos con AC y se compararon con madres de recién nacidos sanos, en cuanto a los antecedentes de consumo de metimazol.
    ¿De qué estudio epidemiológico se trata?
  2. Cohortes prospectivo
  3. Cohortes retrospectivo
  4. Casos y controles
  5. Transversal
A

Respuesta correcta: C

Tema e ítem: Identificación de estudios epidemiológicos

Argumento: Indudablemente se trata de un estudio observacional, en el que se desea evaluar la relación/riesgo de desarrollar atresia de coanas en el feto durante el embarazo según la exposición previa al metimazol. En este caso se seleccionan directamente pacientes que han sufrido este evento adverso con antecedente de consumo del medicamento y se compara con grupo de personas sanas que no tuvieron relación alguna. Es decir, el estudio se direcciona desde la enfermedad al factor de riesgo (retrospectivo), y de las opciones de respuesta, solo los estudios de casos y controles cumplen con estas características.

197
Q

Paciente femenina de 30 años presentó una lesión cortopunzante en brazo derecho. Refiere haber tenido sangrado abundante, sin embargo, actualmente no se encuentra sangrando. Se encuentra estable. Al examen físico no hay hematomas, no se ausculta soplo ni se palpa thrill. La extremidad no se encuentra pálida ni fría. La paciente niega parestesias, pero sus pulsos distales se encuentran disminuidos. ¿Cuál es el manejo correcto?
A. Exploración vascular urgente.
B. Observación clínica.
C. Realizar angioTAC.
D. Suturar herida y alta médica.

A

Respuesta correcta: C

Tema e ítem: Trauma vascular de extremidades – Manejo.

Argumento: Este tema ha sido ampliamente preguntado en los exámenes de residencia, fue preguntado en el examen UdeA 2023, y su enfoque suele basarse en el reconocimiento de signos de certeza y de duda de trauma vascular y cual es el manejo a seguir con respecto a este. Por ello, tenga en cuenta que todo paciente inestable hemodinámicamente debe ser llevado a quirófano. Aquellos pacientes estables, como este caso, debemos basarnos en el la anamnesis y examen físico para determinar signos de certeza y signos de duda, que permitan establecer conductas apropiadas.

Los signos de certeza de lesión vascular: Sangrado activo, Hematoma grande, creciente o pulsátil, ausencia de pulsos distales, soplos o frémitos sobre el área de trauma y signos o síntomas de isquemia regional; determinan de forma confiable una lesión vascular y tales pacientes deben ser llevados inmediatamente a quirófano para realizar exploración quirúrgica sin ser sometidos a ayudas diagnosticas. La excepción a esta regla son aquellos pacientes con lesiones en diferentes niveles de la extremidad o aquellos con enfermedades que disminuyen la distensibilidad del vaso sanguíneo; en estos casos, se hace necesario la realización de imágenes para planificación quirúrgica.

Los signos de duda: sangrado previo importante que cedió (Como esta paciente), hematoma pequeño no expansivo sin frémito o soplo, lesión de nervio, pulsos distales disminuidos (Como esta paciente), hipotensión de causa no clara y en trayecto de vasos sanguíneos; son más subjetivos y no tienen el mismo riesgo de lesión vascular, por lo tanto, se encuentran indicadas las ayudas diagnosticas sin indicación de ser llevado de forma inmediata a cirugía. Dentro de las imágenes diagnósticas se tienen las siguientes opciones: ecografía Doppler, angiotomografía, resonancia magnética y arteriografía. La angiotomografía es la principal herramienta para la valoración de lesiones vasculares y debe ser la primera elección siempre que se encuentra disponible.

198
Q

Femenina de 37 años, acude consulta para un examen ginecológico de rutina. Refiere sentirse bien y sus menstruaciones son regulares y de duración normal. Es sexualmente activa con una sola pareja y usa preservativos como método anticonceptivo. Trae reporte de citología de rutina con AGC-neo.
¿Cuál es el manejo apropiado para esta paciente?
A. Repetir citología cervical en 6 meses.
B. Colposcopia + biopsia cervical + cepillado endocervical + biopsia endometrial.
C. Realizar escisión electroquirurgica con asa.
D. Control citólogico en 12 meses

A

Respuesta correcta: B

Tema e ítem: Lesiones preinvasivas de cérvix

Comentario: El cáncer de cuello uterino es el tercero más común en población femenina en Colombia luego del de mama y colon. Este el único cáncer prevenible casi en su totalidad bien sea a través de medidas de prevención primaria con vacunas altamente efectivas o de prevención secundaria con pruebas de tamización como las pruebas de biología molecular con detección ADN de virus de papiloma humano (ADN VPH) en cepillado cervical, citología convencional o en base líquida y pruebas de inspección visual con ácido acético y Lugol (VIA-VILI).

En este caso, se trata de una paciente que trae reporte de citología convencional con AGC-neo (células glandulares atípicas que sugieren neoplasia). Las pacientes con este hallazgo tienen un riesgo de cursar con patología neoplásica o preneoplásica de hasta el 30%. Por lo anterior, la conducta adecuada a seguir es realizar los más pronto una colposcopia con toma de muestra cervical y endocervical para descartar lesiones intraepiteliales cervicales y adenocarcinoma cervical in situ. Además, debemos tener en cuenta que la edad ≥35 años y/o la presencia de factores de riesgo para neoplasia endometrial (Sangrado genital inexplicable, obesidad, anovulación crónica) obligan también a descartar una patología en endometrio, por lo que se debe realizar una biopsia endometrial.

199
Q

Femenina de 40 años G5C4 con embarazo de 35 semanas consulta por dolor abdominal difuso y sangrado vaginal. Previamente presentó varios días de actividad uterina aumentada. Tiene antecedente de miomectomía múltiple hace un año, quedando con al menos 2 fibromas que no fueron resecables. Signos vitales: PA de 90/60 mmHg, FC de 100 lpm, T° de 37°C, FCF de 90 lpm. Al examen físico hay dolor intenso a la palpación abdominal, el útero está atónico, sin actividad uterina detectable. Hay abundante sangrado vaginal. Al tacto vaginal, dilatación de 3 cm sin palparse presentación fetal. ¿Cuál es el diagnostico más probable?
A. Abruptio de placenta.
B. Amenaza de parto pretérmino.
C. Ruptura uterina.
D. Mioma con necrosis roja del embarazo.

A

Respuesta correcta: C
Tema e ítem: Hemorragia obstétrica del segundo y tercer trimestre: Enfoque etiológico.
Argumento: La hemorragia obstétrica es causante del 22% de las muertes maternas en Latinoamérica y en el caribe, siendo la segunda causa de muerte materna directa en Colombia. La mayoría de defunciones eran prevenibles, y es por esto que, desde las diferentes entidades mundiales, se ha hecho un llamado a implementar estrategias para disminuirlas. El primer paso es realizar una anamnesis ágil y completa que ayude a discernir una posible etiología y su gravedad, teniendo en cuenta antecedentes y desencadenantes. Los reportes de ecografía anteriores pueden orientar hacia anomalías de inserción placentaria o del cordón, hematomas retrocoriales y signos de abruptio crónico. En el examen físico se evaluarán signos vitales y estado de conciencia, tono uterino, fetocardia, valorar la dilatación cervical y evaluar el canal vaginal en busca de lesiones (Recuerde que en caso de sospechar placenta previa se encuentra contraindicado el tacto vaginal).

La ruptura uterina es una complicación potencialmente mortal tanto para la madre como para el feto, que puede ocurrir antes o durante el parto y ser parcial o total. Frecuentemente está relacionado con antecedentes quirúrgicos como cesárea, miomectomía o corrección de ruptura uterina anterior. Se debe sospechar en mujeres que presentaron aparente taquisistolia (Esta paciente presentó varios días de actividad uterina aumentada), dolor abdominal agudo que no es igual a las contracciones que venía sintiendo en el trabajo de parto, dolor que no mejora con anestesia epidural, cambio súbito del patrón de contracciones, bradicardia fetal o ascenso de la presentación.

Dentro de las opciones de respuesta, el abruptio de placenta es el principal diferencial. Los principales factores de riesgo para esta incluyen: trauma reciente, hipertensión materna, malformaciones uterinas, consumo de cigarrillo o cocaína durante el embarazo y antecedente de abruptio en embarazos previos. El diagnóstico es netamente clínico y se confirma con la inspección placentaria posterior al parto. Puede tener una clínica similar: Debe sospechar en una paciente con hipertonía (Esta paciente tiene atonía uterina) o taquisistolia uterina asociado a sangrado vaginal de color rojo oscuro variable, y disminución de movimientos o bradicardia fetal, no siempre se cumplen todos los signos y síntomas lo cual dificulta el diagnóstico.

200
Q

Doña Ana de 75 años es traída al servicio de Urgencias por dolor torácico retroesternal de inicio súbito asociado a dificultad respiratoria. Refiere que los síntomas inician luego de enterarse que su hijo tuvo un accidente en motocicleta y su pronostico es reservado. Tiene antecedente de HTA en manejo con enalapril. Al ingreso presenta PA 140/85 mh, FC 101 lpm, SatO2 99% a aire ambiente. A la auscultación cardiaca se detecta un cuarto ruido (S4). Electrocardiograma: elevación del segmento ST en derivaciones precordiales anteriores. Analítica: troponinas (+). El ecocardiograma reporta función ventricular izquierda basal conservada con hipocinesia apical-ventricular media con FEVI 36%. La angiografía coronaria muestra arterias coronarias normales.
¿Cuál de los siguientes es el diagnóstico más probable?
A. Disección aórtica aguda
B. Disección de arteria coronaria
C. Embolia coronaria
D. Miocardiopatía de takotsubo

A

Respuesta correcta: D

Tema e ítem: Miocardiopatía de Takotsubo/síndrome del corazón roto/cardiomiopatía inducida por estrés – Criterios diagnósticos

Argumento: La miocardiopatía de takotsubo es un síndrome caracterizado por disfunción ventricular sistólica regional transitoria que simula un infarto de miocardio:
* Fracción de eyección reducid
* Troponinas positivas
* Signos de isquemia en el electrocardiograma) en ausencia de evidencia angiográfica de enfermedad coronaria obstructiva.

En la mayoría de los casos de miocardiopatía de takotsubo, la anomalía regional del movimiento de la pared se extiende más allá del territorio perfundido por una única arteria coronaria epicárdica. El síndrome suele precipitarse por un estrés emocional extremo, como lo que está ocurriendo con su hijo. Se desconoce la patogénesis de la miocardiopatía de takotsubo, pero se postula que es el resultado de la toxicidad miocárdica reversible inducida por niveles muy elevados de catecolaminas. Las mujeres se ven afectadas con más frecuencia que los hombres. El tratamiento suele ser de apoyo y similar al de la insuficiencia cardiaca por otras causas; la mayoría de los pacientes recuperan la función cardiaca en varios meses.

  • La disección aórtica aguda se presenta clásicamente como un dolor torácico o lumbar intenso y súbito de tipo desgarro. Otras manifestaciones asociadas pueden ser la hipertensión, síncope, soplo de regurgitación aórtica e insuficiencia cardiaca. La disección aguda de la aorta ascendente puede causar una oclusión coronaria aguda; sin embargo, las anomalías del movimiento de la pared afectarían con mayor probabilidad a una única distribución arteria coronaria, y la obstrucción se detectaría mediante angiografía. Además, puede visualizarse en el ecocardiograma.
  • La disección de arteria coronaria suele identificarse mediante angiografía coronaria, aunque los hallazgos pueden ser sutiles. Una disección única causa anomalías regionales del movimiento de la pared en un patrón de distribución de una sola arteria coronaria en lugar de una disfunción ventricular media y apical global.
  • La embolia coronaria es una causa potencial de infarto agudo de miocardio, que da lugar a una función ventricular anormal en un paciente sin enfermedad coronaria obstructiva; sin embargo, una embolia de la arteria coronaria generalmente causa anomalías regionales del movimiento de la pared en una única distribución de la arteria coronaria en lugar de una disfunción ventricular media y apical global.
201
Q

Masculino de 6 años de edad se presenta con dolor abdominal umbilical agudo intermitente y varios episodios eméticos durante 4 horas. El dolor se irradia hace la región inferior derecha del abdomen y ocurre cada 15 a 30 minutos. Durante estos periodos de dolor el niño se coloca en posición fetal. Tuvo dos episodios similares en los últimos 6 meses. Al examen físico hay dolor periumbilical sin palpar masas. La ecografía de abdomen evidencia anillos concéntricos del intestino. Su Hb es de 10.2 g/dL. ¿Cuál es el diagnostico más probable en este caso?
A. Malrotación con vólvulo.
B. Adherencias intestinales.
C. Apendicitis aguda.
D. Divertículo de Meckel.

A

Respuesta correcta: D

Tema e ítem: Intususcepción y divertículo de Meckel.

Argumento: Este niño de 6 años se presenta con dolor abdominal cíclico, agudo que lo obliga a colocarse en posición fetal, vómitos no biliosos y hallazgos ecográficos descritos que sugieren una intususcepción. Además, la recurrencia del episodio y la presencia de anemia sugieren un divertículo de Meckel. El divertículo de Meckel es el punto que causa intususcepción. Siempre se debe sospechar esta condición en niños >5 años de edad que presentan episodios recurrentes de dolor, especialmente si los síntomas ocurren dentro de los 6 meses posteriores al ultimo episodios. El divertículo de Meckel puede causar sangrado si contiene células ectópicas de la mucosa gástrica, ya que el ácido puede irritar el intestino delgado hasta el punto de ulcerarlo causando sangrado y posterior anemia.

  • La malrotación con vólvulo se presenta con inicio agudo de dolor abdominal y heces con sangre. En la ecografía abdominal el vólvulo se presenta con un signo de remolino que indica que el mesenterio se encuentra torcido. Aunque las características clínicas pueden ser similares, la malrotación del vólvulo es poco común en niños de esta edad.
  • Las adherencias intestinales pueden provocar obstrucción intestinal o actuar como un punto patológico para el desarrollo de intususcepción. Suelen formarse posterior a una cirugía abdominal, inflamación intraabdominal, radioterapia abdominal, ninguna de las cuales se menciona en este caso.
  • La apendicitis aguda se manifiesta con dolor abdominal continuo y difuso que migra de región periumbilical al cuadrante inferior derecho, vómitos y signo de diana en la ecografía, lo que la convierte en un diagnostico diferencial importante para los síntomas de este paciente. Sin embargo, los ataques repetidos de dolor y la presencia de anemia sugieren otro diagnóstico.